Download as pdf or txt
Download as pdf or txt
You are on page 1of 192

EXAME NACIONAL DO ENSINO MÉDIO

PROVA DE CIÊNCIAS HUMANAS E SUAS TECNOLOGIAS


PROVA DE CIÊNCIAS DA NATUREZA E SUAS TECNOLOGIAS

1º DIA
CADERNO

3
2014 BRANCO
2ª APLICAÇÃO
A COR DA CAPA DO SEU CADERNO DE QUESTÕES É BRANCO. MARQUE-A EM SEU CARTÃO-RESPOSTA.

LEIA ATENTAMENTE AS INSTRUÇÕES SEGUINTES: 8 5HVHUYHRVPLQXWRV¿QDLVSDUDPDUFDUVHX&$57­25(63267$


2VUDVFXQKRVHDVPDUFDo}HVDVVLQDODGDVQR&$'(512'(
1 9HUL¿TXH QR &$57­25(63267$ VH RV VHXV GDGRV HVWmR 48(67®(6QmRVHUmRFRQVLGHUDGRVQDDYDOLDomR
UHJLVWUDGRV FRUUHWDPHQWH &DVR KDMD DOJXPD GLYHUJrQFLD
FRPXQLTXHDLPHGLDWDPHQWHDRDSOLFDGRUGDVDOD 9 4XDQGR WHUPLQDU DV SURYDV DFHQH SDUD FKDPDU R
DSOLFDGRUHHQWUHJXHHVWH&$'(512'(48(67®(6HR
2 ATENÇÃO DSyV D FRQIHUrQFLD HVFUHYD H DVVLQH VHX QRPH &$57­25(63267$
QRV HVSDoRV SUySULRV GR &$57­25(63267$ FRP FDQHWD
HVIHURJUi¿FDGHWLQWDSUHWD 10 9RFrSRGHUiGHL[DURORFDOGHSURYDVRPHQWHDSyVGHFRUULGDV
3 ATENÇÃO WUDQVFUHYD QR HVSDoR DSURSULDGR GR VHX GXDV KRUDV GR LQtFLR GD DSOLFDomR H SRGHUi OHYDU VHX
&$57­25(63267$FRPVXDFDOLJUD¿DXVXDOFRQVLGHUDQGR &$'(512'(48(67®(6DRGHL[DUHPGH¿QLWLYRDVDODGH
DVOHWUDVPDL~VFXODVHPLQ~VFXODVDVHJXLQWHIUDVH SURYDQRVPLQXWRVTXHDQWHFHGHPRWpUPLQRGDVSURYDV

11 9RFrVHUiHOLPLQDGRGR([DPHDTXDOTXHUWHPSRQRFDVRGH
Agora todos os caminhos vêm.
D SUHVWDUHPTXDOTXHUGRFXPHQWRGHFODUDomRIDOVDRXLQH[DWD
4 (VWH &$'(512 '( 48(67®(6 FRQWpP  TXHVW}HV E SHUWXUEDUGHTXDOTXHUPRGRDRUGHPQRORFDOGHDSOLFDomR
QXPHUDGDVGHDGLVSRVWDVGDVHJXLQWHPDQHLUD GDV SURYDV LQFRUUHQGR HP FRPSRUWDPHQWR LQGHYLGR
D DV TXHVW}HV GH Q~PHUR  D  VmR UHODWLYDV j iUHD GH GXUDQWHDUHDOL]DomRGR([DPH
&LrQFLDV+XPDQDVHVXDV7HFQRORJLDV
F VH FRPXQLFDU GXUDQWH DV SURYDV FRP RXWUR SDUWLFLSDQWH
E DV TXHVW}HV GH Q~PHUR  D  VmR UHODWLYDV j iUHD GH YHUEDOPHQWHSRUHVFULWRRXSRUTXDOTXHURXWUDIRUPD
&LrQFLDVGD1DWXUH]DHVXDV7HFQRORJLDV
G SRUWDU TXDOTXHU WLSR GH HTXLSDPHQWR HOHWU{QLFR H GH
5 &RQ¿UD VH R VHX &$'(512 '( 48(67®(6 FRQWpP D FRPXQLFDomRDSyVLQJUHVVDUQDVDODGHSURYDV
TXDQWLGDGH GH TXHVW}HV H VH HVVDV TXHVW}HV HVWmR QD RUGHP
PHQFLRQDGD QD LQVWUXomR DQWHULRU &DVR R FDGHUQR HVWHMD H XWLOL]DU RX WHQWDU XWLOL]DU PHLR IUDXGXOHQWR HP EHQHItFLR
LQFRPSOHWR WHQKD TXDOTXHU GHIHLWR RX DSUHVHQWH GLYHUJrQFLD SUySULRRXGHWHUFHLURVHPTXDOTXHUHWDSDGR([DPH
FRPXQLTXH DR DSOLFDGRU GD VDOD SDUD TXH HOH WRPH DV I XWLOL]DU OLYURV QRWDV RX LPSUHVVRV GXUDQWH D UHDOL]DomR
SURYLGrQFLDVFDEtYHLV GR([DPH
6 3DUD FDGD XPD GDV TXHVW}HV REMHWLYDV VmR DSUHVHQWDGDV J VH DXVHQWDU GD VDOD GH SURYDV OHYDQGR FRQVLJR R
RSo}HV$SHQDVXPDUHVSRQGHFRUUHWDPHQWHjTXHVWmR &$'(512'(48(67®(6DQWHVGRSUD]RHVWDEHOHFLGR
HRXR&$57­25(63267$DTXDOTXHUWHPSR
7 2 WHPSR GLVSRQtYHO SDUD HVWDV SURYDV p GH quatro horas e
trinta minutos K QmRFXPSULUFRPRGLVSRVWRQRHGLWDOGR([DPH

*bran75SAB1*
*BRAN75SAB2* 2014

CIÊNCIAS HUMANAS E SUAS QUESTÃO 03


TECNOLOGIAS FIGURA 1
Questões de 1 a 45
QUESTÃO 01
Maria da Penha
Você não vai ter sossego na vida, seu moço
Se me der um tapa
Da dona “Maria da Penha”
Você não escapa
O bicho pegou, não tem mais a banca
De dar cesta básica, amor
Vacilou, tá na tranca
5HVSHLWRD¿QDOpERPHHXJRVWR
[...]
3ULQFHVD$OH[DQGUD'LVSRQtYHOHPZZZGHPRFUDFLDIDVKLRQFRPEU$FHVVRHPDJR
Não vem que eu não sou
0XOKHUGH¿FDUHVFXWDQGRHVFXODFKR FIGURA 2
$TXLREXUDFRpPDLVHPEDL[R
$QRVVDSDL[mRMiIRLWDUGH
[...]
Se quer um conselho, não venha
&RPHVVDDUURJkQFLDIHUUHQKD
Vai dar com a cara
Bem na mão da “Maria da Penha”
ALCIONE. De tudo o que eu gosto. Rio de Janeiro: Indie; Warner, 2007.

$OHWUDGDFDQomRID]UHIHUrQFLDDXPDLQLFLDWLYDGHVWLQDGD
D FRPEDWHU XP WLSR GH GHVUHVSHLWR H H[FOXVmR VRFLDO
associado, principalmente, à(s)
A mudanças decorrentes da entrada da mulher no
mercado de trabalho.
B IRUPDV GH DPHDoD GRPpVWLFD TXH VH UHVWULQJHP j
YLROrQFLDItVLFD
C UHODo}HVGHJrQHURVRFLDOPHQWHFRQVWUXtGDVDRORQJR
da história. 'XTXHVDGH&DPEULGJH.DWH0LGGOHWRQ'LVSRQtYHOHPKWWSURFNDQGJODPRXUEORJVSRWFRP

D YLROrQFLD GRPpVWLFD FRQWUD D PXOKHU UHODFLRQDGD $FHVVRHPDJR

jSREUH]D $V¿JXUDVLQGLFDPPXGDQoDVQRXQLYHUVRIHPLQLQRFRPRD
E LQJHVWmRH[FHVVLYDGHiOFRROSHORVKRPHQV
A decadência da Monarquia, revelada pela aparição
QUESTÃO 02 VROLWiULDHLQIRUPDOGDVQREUHV
Sempre teceremos panos de seda B UHGXomR QD HVFRODULGDGH VLPEROL]DGD SHOD YLGD
E nem por isso vestiremos melhor dinâmica e sem dedicação à leitura.
Seremos sempre pobres e nuas C ampliação do status, FRQIHULGD SHOD SDVVDJHP GR
(WHUHPRVVHPSUHIRPHHVHGH ORFDOU~VWLFRSDUDRVMDUGLQVGRSDOiFLR
1XQFDVHUHPRVFDSD]HVGHJDQKDUWDQWR
Que possamos ter melhor comida. D LQFOXVmR QD SROtWLFD UHSUHVHQWDGD SHOD GLIHUHQoD
CHRÉTIEN DE TROYES. Yvain ou le chevalier au lion (1177-1181). Apud MACEDO, J. R.
entre o espaço privado e o espaço público.
A mulher na Idade Média6mR3DXOR&RQWH[WR DGDSWDGR  E YDORUL]DomRGRFRUSRVDOLHQWDGDSHORXVRGHURXSDV
2WHPDGRWUDEDOKRIHPLQLQRYHPVHQGRDERUGDGRSHORV PDLVFXUWDVHSHODSRVWXUDPDLVUHOD[DGD
HVWXGRV KLVWyULFRV PDLV UHFHQWHV $OJXPDV IRQWHV VmR
importantes para essa abordagem, tal como o poema
apresentado, que alude à
A inserção das mulheres em atividades tradicionalmente
masculinas.
B ambição das mulheres em ocupar lugar preponderante
na sociedade.
C possibilidade de mobilidade social das mulheres na
LQG~VWULDWr[WLOPHGLHYDO
D H[SORUDomRGDVPXOKHUHVQDVPDQXIDWXUDVWr[WHLVQR
mundo urbano medieval.
E VHUYLGmRIHPLQLQDFRPRWLSRGHPmRGHREUDYLJHQWH
nas tecelagens europeias.

CH - 1º dia | Caderno 3 - BRANCO - Página 2


2014 *BRAN75SAB3*
QUESTÃO 04 QUESTÃO 05
Quem acompanhasse os debates na Câmara dos 2SUySULRPRYLPHQWRRSHUiULRQmRSRGHVHUUHGX]LGR
DepXWDGRV HP  SRGHULD RXYLU D OHLWXUD GH XPD DXPFRQÀLWRGHLQWHUHVVHVHFRQ{PLFRVRXDXPDUHDomR
PRomRGHID]HQGHLURVGR5LRGH-DQHLUR³1LQJXpPQR FRQWUDDSUROHWDUL]DomR(OHpDQLPDGRSRUXPDLPDJHP
Brasil sustenta a escravidão pela escravidão, mas não GH³FLYLOL]DomR´LQGXVWULDOSHODLGHLDGHXPSURJUHVVRGDV
há um só brasileiro que não se oponha aos perigos da IRUoDVGHSURGXomRXWLOL]DGRSDUDREHPGHWRGRV2TXH
GHVRUJDQL]DomR GR DWXDO VLVWHPD GH WUDEDOKR´ /LYUHV p EHP GLIHUHQWH GD XWRSLD LJXDOLWDULVWD VLPSOHV SRXFR
os negros, as cidades seriam invadidas por “turbas preocupada com as condições de crescimento.
LJQDUDV´ ³JHQWH UHIUDWiULD DR WUDEDOKR H iYLGD GH TOURAINE, A. Os movimentos sociais. In: FORRACHI, M. M.; MARTINS, J. S. (Org.).
Sociologia e sociedade5LRGH-DQHLUR/LYURV7pFQLFRVH&LHQWt¿FRV
RFLRVLGDGH´$SURGXomRVHULDGHVWUXtGDHDVHJXUDQoD
GDV IDPtOLDV HVWDULD DPHDoDGD 9HLR D $EROLomR R &RQVLGHUDQGRDFDUDFWHUL]DomRDSUHVHQWDGDSHORWH[WRD
$SRFDOLSVH ¿FRX SDUD GHSRLV H R %UDVLO PHOKRURX RX busca pela igualdade pressupõe o(a)
VHUi TXH DOJXpP GXYLGD"  3DVVDGRV GH] DQRV GR A HVWtPXORGDOXWDSROtWLFD
LQtFLR GR GHEDWH HP WRUQR GDV Do}HV D¿UPDWLYDV H GR
B DGRomRGDLGHRORJLDPDU[LVWD
UHFXUVRjVFRWDVSDUDIDFLOLWDURDFHVVRGRVQHJURVjV
XQLYHUVLGDGHVS~EOLFDVEUDVLOHLUDVIHOL]PHQWHpSRVVtYHO C FROHWLYL]DomRGRVPHLRVGHSURGXomR
FRQIHULU D FRQVLVWrQFLD GRV DUJXPHQWRV DSUHVHQWDGRV D DSURIXQGDPHQWRGRVFRQÀLWRVVRFLDLV
FRQWUD HVVD LQLFLDWLYD 'H VDtGD YHLR D DGYHUWrQFLD E LQWHQVL¿FDomRGRFUHVFLPHQWRHFRQ{PLFR
GH TXH DV FRWDV H[DFHUEDULDP D TXHVWmR UDFLDO (VVD
ameaça vai completar 18 anos e não se registraram QUESTÃO 06
FDVRVVLJQL¿FDWLYRVGHH[DFHUEDomR $RIDODUGRFDUiWHUGHXPKRPHPQmRGL]HPRVTXH
GASPARI, E. As cotas e a urucubaca. Folha de S. PauloMXQ HOHpViELRRXTXHSRVVXLHQWHQGLPHQWRPDVTXHpFDOPR
RX WHPSHUDQWH 1R HQWDQWR ORXYDPRV WDPEpP R ViELR
O argumento elaborado pelo autor sugere que as
UHIHULQGRVH DR KiELWR H DRV KiELWRV GLJQRV GH ORXYRU
censuras às cotas raciais são chamamos virtude.
A politicamente ignoradas. ARISTÓTELES. Ética a Nicômaco6mR3DXOR1RYD&XOWXUDO

B VRFLDOPHQWHMXVWL¿FDGDV (P$ULVWyWHOHVRFRQFHLWRGHYLUWXGHpWLFDH[SUHVVDD
C FXOWXUDOPHQWHTXDOL¿FDGDV A H[FHOrQFLDGHDWLYLGDGHVSUDWLFDGDVHPFRQVRQkQFLD
D historicamente equivocadas. com o bem comum.
E HFRQRPLFDPHQWHIXQGDPHQWDGDV B FRQFUHWL]DomR XWLOLWiULD GH Do}HV TXH UHYHODP D
PDQLIHVWDomRGHSURSyVLWRVSULYDGRV
C concordância das ações humanas aos preceitos
emanados da divindade.
D UHDOL]DomRGHDo}HVTXHSHUPLWHPDFRQ¿JXUDomRGD
SD]LQWHULRU
E PDQLIHVWDomRGHDo}HVHVWpWLFDVFRURDGDVGHDGRUQR
HEHOH]D

CH - 1º dia | Caderno 3 - BRANCO - Página 3


*BRAN75SAB4* 2014

QUESTÃO 07 QUESTÃO 09
No sistema democrático de Schumpeter, os únicos TEXTO I
SDUWLFLSDQWHV SOHQRV VmR RV PHPEURV GH HOLWHV SROtWLFDV
em partidos e em instituições públicas. O papel dos 'HSXWDGR GH¿QLomRGRVpFXOR;9,,, 
FLGDGmRVRUGLQiULRVpQmRDSHQDVDOWDPHQWHOLPLWDGRPDV Substant. Aquele a quem se deu alguma comissão
IUHTXHQWHPHQWHUHWUDWDGRFRPRXPDLQWUXVmRLQGHVHMDGD GH MXULVGLomR RX FRQKHFLPHQWR 0DQGDGR GD SDUWH GH
QR IXQFLRQDPHQWR WUDQTXLOR GR SURFHVVR ³S~EOLFR´ GH alguma República, ou soberano. O que tem comissão do
tomada de decisões. ministro próprio.
HELD, D. Modelos de democracia%HOR+RUL]RQWH3DLGHLD
SILVA, A. M. Diccionario da lingua portugueza. Lisboa:
2I¿FLQDGH6LPmR7KDGGHR)HUUHLUD DGDSWDGR 
2PRGHORGHVLVWHPDGHPRFUiWLFRDSUHVHQWDGRSHORWH[WR
pressupõe a TEXTO II
A consolidação da racionalidade comunicativa. 'HSXWDGR GH¿QLomRGRVpFXOR;;, 
B DGRomRGRVLQVWLWXWRVGRSOHELVFLWRHGRUHIHUHQGR
[...]
C condução de debates entre cidadãos iguais e o
Estado. $TXHOHTXHUHSUHVHQWDRVLQWHUHVVHVGHRXWUHPHP
D VXEVWLWXLomR GD GLQkPLFD UHSUHVHQWDWLYD SHOD FtYLFR UHXQL}HVHGHFLV}HVR¿FLDLV
participativa. $TXHOHTXHpHOHLWRSDUDOHJLVODUHUHSUHVHQWDURV
E GHOLEHUDomR GRV OtGHUHV SROtWLFRV FRP UHVWULomR GD interesses dos cidadãos.
participação das massas.
$TXHOHTXHpFRPLVVLRQDGRSDUDWUDWDUGRVQHJyFLRV
QUESTÃO 08 alheios.
AULETE, C. Minidicionário contemporâneo da língua portuguesa.
As relações do Estado brasileiro com o movimento 6mR3DXOR/H[LNRQ DGDSWDGR 
RSHUiULR H VLQGLFDO EHP FRPR DV SROtWLFDV S~EOLFDV
$ PXGDQoD PDLV VLJQL¿FDWLYD QR VHQWLGR GD SDODYUD
voltadas para as questões sociais durante o primeiro
³GHSXWDGR´HQWUHRVpFXOR;9,,,HRVGLDVGHKRMHGiVH
JRYHUQR GD (UD 9DUJDV   VmR WHPDV pelo(a)
amplamente estudados pela academia brasileira em seus
YiULRVDVSHFWRV6mRWDPEpPRVWHPDVPDLVOHPEUDGRV A DXPHQWRQDLPSRUWkQFLDFRPRUHSUHVHQWDomRSROtWLFD
pela sociedade quando se pensa no legado varguista. dos cidadãos.
'¶$5$Ò-20&(VWDGRFODVVHWUDEDOKDGRUDHSROtWLFDVVRFLDLV,Q)(55(,5$-'(/*$'2 B FUHVFHQWHSDUWLFLSDomRGRVIXQFLRQiULRVQRSRGHUGR
L. A. (Org.).O tempo do nacional-estatismoGRLQtFLRDRDSRJHXGR(VWDGR1RYR Estado.
5LRGH-DQHLUR&LYLOL]DomR%UDVLOHLUD
C incentivo à intermediação dos interesses de
'XUDQWHRJRYHUQRGH*HW~OLR9DUJDVIRUDPGHVHQYROYLGDV particulares.
ações de cunho social, dentre as quais se destaca a D criação de diversas pequenas cidades-repúblicas.
A GLVVHPLQDomR GH RUJDQL]Do}HV SDUDPLOLWDUHV E diminuição do poder das assembleias.
LQVSLUDGDVQRVUHJLPHVIDVFLVWDVHXURSHXV
B aprovação de normas que buscavam garantir a posse
das terras aos pequenos agricultores.
C FULDomRGHXPFRQMXQWRGHOHLVWUDEDOKLVWDVDVVRFLDGDV
ao controle das representações sindicais.
D implementação de um sistema de previdência e
seguridade para atender aos trabalhadores rurais.
E LPSODQWDomRGHDVVRFLDo}HVFLYLVFRPRXPDHVWUDWpJLD
SDUDDSUR[LPDUDVFODVVHVPpGLDVHRJRYHUQR

&+žGLD_&DGHUQR%5$1&23iJLQD
2014 *BRAN75SAB5*
QUESTÃO 10 QUESTÃO 12
'HVGH  R ,QVWLWXWR GR 3DWULP{QLR +LVWyULFR H 6HQGRRVKRPHQVSRUQDWXUH]DWRGRVOLYUHVLJXDLV
$UWtVWLFR 1DFLRQDO ,SKDQ  WHP UHJLVWUDGR FHUWRV EHQV H LQGHSHQGHQWHV QLQJXpP SRGH VHU H[SXOVR GH VXD
LPDWHULDLV FRPR SDWULP{QLR FXOWXUDO GR SDtV (QWUH DV SURSULHGDGH H VXEPHWLGR DR SRGHU SROtWLFR GH RXWUHP
PDQLIHVWDo}HVTXHMiJDQKDUDPHVVHstatusHVWiRRItFLR sem dar consentimento. A maneira única em virtude da
GDVEDLDQDVGRDFDUDMp(QIDWL]HVHRRItFLRGDVEDLDQDV qual uma pessoa qualquer renuncia à liberdade natural
QmR D UHFHLWD GR DFDUDMp 4XDQGR XPD EDLDQD SUHSDUD e se reveste dos laços da sociedade civil consiste em
R DFDUDMp Ki XPD VpULH GH FyGLJRV LPSHUFHSWtYHLV SDUD FRQFRUGDU FRP RXWUDV SHVVRDV HP MXQWDUVH H XQLUVH
TXHPROKDGHIRUD$FRUGDURXSDDDPDUUDGRVSDQRV HP FRPXQLGDGH SDUD YLYHUHP FRP VHJXUDQoD FRQIRUWR
e os adereços mudam de acordo com o santo e com H SD] XPDV FRP DV RXWUDV JR]DQGR JDUDQWLGDPHQWH
D KLHUDUTXLD GHOD QR FDQGRPEOp 2 ,SKDQ FRQWD TXH GDV SURSULHGDGHV TXH WLYHUHP H GHVIUXWDQGR GH PDLRU
UHJLVWUDQGR R RItFLR ³HVVH H RXWURV PXQGRV OLJDGRV DR SURWHomRFRQWUDTXHPTXHUTXHQmRIDoDSDUWHGHOD
SUHSDURGRDFDUDMpSRGHPVHUGHVFRUWLQDGRV´
LOCKE, J. Segundo tratado sobre o governo civil. Os pensadores6mR3DXOR1RYD&XOWXUDO
.$=5$GLIHUHQoDHQWUHRDFDUDMpHRVDQGXtFKHGH%DXUXRevista de História da
Biblioteca Nacional, n. 13, out. 2006 (adaptado). 6HJXQGRD7HRULDGD)RUPDomRGR(VWDGRGH-RKQ/RFNH
De acordo com o autor, o Iphan evidencia a necessidade para viver em sociedade, cada cidadão deve
GH VH SURWHJHUHP FHUWDV PDQLIHVWDo}HV KLVWyULFDV SDUD A PDQWHU D OLEHUGDGH GR HVWDGR GH QDWXUH]D GLUHLWR
TXHFRQWLQXHPH[LVWLQGRGHVWDFDQGRVHQHVVHFDVRD inalienável.
A PLVWXUD GH WUDGLo}HV DIULFDQDV LQGtJHQDV H B abrir mão de seus direitos individuais em prol do bem
portuguesas no preparo do alimento por parte das comum.
FR]LQKHLUDVEDLDQDV
C abdicar de sua propriedade e submeter-se ao poder
B relação com o sagrado no ato de preparar o alimento, GRPDLVIRUWH
VREUHVVDLQGRVHRXVRGHVtPERORVHLQVtJQLDVSHODV
D concordar com as normas estabelecidas para a vida
FR]LQKHLUDV
em sociedade.
C XWLOL]DomR GH FHUWRV LQJUHGLHQWHV TXH VH PRVWUDP
FDGDYH]PDLVUDURVGHHQFRQWUDUFRPDVPXGDQoDV E UHQXQFLDUjSRVVHMXUtGLFDGHVHXVEHQVPDVQmRj
nos hábitos alimentares. sua independência.
D necessidade de preservação dos locais tradicionais QUESTÃO 13
GH SUHSDUR GR DFDUDMp DPHDoDGRV FRP DV
WUDQVIRUPDo}HVXUEDQDVQRSDtV $ LQWURGXomR GD RUJDQL]DomR FLHQWt¿FD WD\ORULVWD
E LPSRUWkQFLD GH VH WUHLQDUHP DV FR]LQKHLUDV EDLDQDV GR WUDEDOKR H VXD IXVmR FRP R IRUGLVPR DFDEDUDP SRU
D ¿P GH UHVJDWDU R PRGR WUDGLFLRQDO GH SUHSDUR GR UHSUHVHQWDU D IRUPD PDLV DYDQoDGD GD UDFLRQDOL]DomR
DFDUDMpTXHUHPRQWDjHVFUDYLGmR capitalista do processo de trabalho ao longo de várias
GpFDGDVGRVpFXOR;;
QUESTÃO 11 ANTUNES, R. Os sentidos do trabalhoHQVDLRVREUHDD¿UPDomRHDQHJDomRGRWUDEDOKR
6mR3DXOR%RLWHPSR DGDSWDGR 
$MXVWLoDpDSULPHLUDYLUWXGHGDVLQVWLWXLo}HVVRFLDLV
FRPRDYHUGDGHRpGRVVLVWHPDVGHSHQVDPHQWR&DGD 2 REMHWLYR GHVVH PRGHOR GH RUJDQL]DomR GR WUDEDOKR p
SHVVRDSRVVXLXPDLQYLRODELOLGDGHIXQGDGDQDMXVWLoDTXH R DOFDQFH GD H¿FLrQFLD Pi[LPD QR SURFHVVR SURGXWLYR
nem mesmo o bem-estar da sociedade como um todo industrial que, para tanto,
SRGHLJQRUDU3RUHVVDUD]mRDMXVWLoDQHJDTXHDSHUGD
GH OLEHUGDGH GH DOJXQV VH MXVWL¿TXH SRU XP EHP PDLRU A DGRWD HVWUXWXUDV GH SURGXomR KRUL]RQWDOL]DGDV
partilhado por todos. SULYLOHJLDQGRDVWHUFHLUL]Do}HV
HAWLS, J. 8PDWHRULDGDMXVWLoD. São Paulo: Martins Fontes, 2000 (adaptado). B UHTXHU WUDEDOKDGRUHV TXDOL¿FDGRV SROLYDOHQWHV H
aptos para as oscilações da demanda.
2 ¿OyVRIR D¿UPD TXH D LGHLD GH MXVWLoD DWXD FRPR XP
LPSRUWDQWH IXQGDPHQWR GD RUJDQL]DomR VRFLDO H DSRQWD C procede à produção em pequena escala, mantendo
FRPRVHXHOHPHQWRGHDomRHIXQFLRQDPHQWRR RVHVWRTXHVEDL[RVHDGHPDQGDFUHVFHQWH
D GHFRPS}H D SURGXomR HP WDUHIDV IUDJPHQWDGDV
A povo.
e repetitivas, complementares na construção do
B Estado. produto.
C governo. E RXWRUJDDRVWUDEDOKDGRUHVDH[WHQVmRGDMRUQDGDGH
D LQGLYtGXR WUDEDOKRSDUDTXHHOHVGH¿QDPRULWPRGHH[HFXomR
E magistrado. GHVXDVWDUHIDV

&+žGLD_&DGHUQR%5$1&23iJLQD
*BRAN75SAB6* 2014

QUESTÃO 14 QUESTÃO 15
Número de imigrantes
Perfil do comércio Brasil-China internacionais do Brasil
Em 2010 268 000

Vendas do Brasil para a China 143 000

Básicos Semimanufaturados
83,7% 11,8%
Entre 1995 e 2000 Entre 2005 e 2010
US$
30,785 ,%*('LVSRQtYHOHPZZZLEJHJRYEU$FHVVRHPDJR
bilhões
A variação do número de imigrantes internacionais no
4,5% %UDVLO YHUL¿FDGD SHOD DQiOLVH GR JUi¿FR p UHVXOWDGR
Manufaturados direto da
A VLWXDomRLQWHUQDFLRQDOGHFULVHHFRQ{PLFD
B limitação europeia à entrada de estrangeiros.
Vendas da China para o Brasil C DWUDomRH[HUFLGDSHODVEHODVSDLVDJHQVQDWXUDLV
D OHJLVODomRIDFLOLWDGRUDGDHQWUDGDGHHVWUDQJHLURV
Manufaturados Básicos
E HVFROKD GR SDtV FRPR VHGH GH JUDQGHV HYHQWRV
97,5% 2,1% esportivos.

US$ QUESTÃO 16
25,593
bilhões
Participação das regiões no PIB do país, em %
0,4%
Semimanufaturados

Fonte: Ministério do Desenvolvimento, Norte


Indústria e Comércio Exterior.
4,7 5,3
$/9$5(1*$''LVSRQtYHOHPKWWSJJORERFRP$FHVVRHPGH] IUDJPHQWR  Nordeste
1DV ~OWLPDV GpFDGDV WHP VH REVHUYDGR XP LQFUHPHQWR 13 13,5
QRFRPpUFLRHQWUHR%UDVLOHD&KLQD$FRPSDUDomRHQWUH
RVJUi¿FRVGHPRQVWUDD
Centro-Oeste
A SRVLomR GR %UDVLO FRPR JUDQGH H[SRUWDGRU GH 8,8 9,3
commodities. Sudeste
B IDOWD GH FRPSOHPHQWDULGDGH SURGXWLYD HQWUH RV GRLV
SDtVHV 2002 56,7 55,4
C vantagem competitiva da China no setor de produção 2010
DJUtFROD Sul
D proporcionalidade entre as trocas de bens de alto
16,9 16,5
valor agregado.
E restrita participação de bens de alta tecnologia no
Fonte: IBGE.
FRPpUFLRELODWHUDO
'LVSRQtYHOHPZZZDFHUYRIROKDFRPEU$FHVVRHPGH] IUDJPHQWR 

$V WUDMHWyULDV GH SDUWLFLSDomR GDV UHJL}HV 6XGHVWH H


Nordeste, apresentadas no mapa, estão, respectivamente,
associadas ao(à)
A UHGXomR GD UHQGD PpGLD H j ÀH[LELOL]DomR GDV OHLV
ambientais.
B HQFDUHFLPHQWR GD PmR GH REUD H j SROtWLFD GH
LQFHQWLYRV¿VFDLV
C H[SDQVmR GD PDOKD URGRYLiULD H j LQVWDODomR GH
JUDQGHVSURMHWRVKLGUHOpWULFRV
D diminuição da participação do setor de serviços e à
ampliação do agronegócio.
E DXPHQWR GR ÀX[R PLJUDWyULR LQWHUUHJLRQDO H j
GHVFREHUWDGHQRYDVMD]LGDVGHFDUYmRPLQHUDO

CH - 1º dia | Caderno 3 - BRANCO - Página 6


2014 *BRAN75SAB7*
QUESTÃO 17 QUESTÃO 18

TEXTO I $ PLWRORJLD FRPSDUDGD VXUJH QR VpFXOR ;9,,, (VVD


WHQGrQFLDLQÀXHQFLRXRHVFULWRUFHDUHQVH-RVpGH$OHQFDU
TXHLQVSLUDGRSHORHVWLORGDHSRSHLDKRPpULFDQDIlíada,
propõe em Iracema XPD HVSpFLH GH PLWR IXQGDGRU GR
povo brasileiro. Assim como a Ilíada vincula a constituição
GR SRYR KHOrQLFR j *XHUUD GH 7URLD GHÀDJUDGD SHOR
romance proibido de Helena e Páris, Iracema vincula a
IRUPDomR GR SRYR EUDVLOHLUR DRV FRQÀLWRV HQWUH tQGLRV H
FRORQL]DGRUHV DWUDYHVVDGRV SHOR DPRU SURLELGR HQWUH
XPDtQGLD²,UDFHPD²HRFRORQL]DGRUSRUWXJXrV0DUWLP
Soares Moreno.
DETIENNE, M. $LQYHQomRGDPLWRORJLD5LRGH-DQHLUR-RVp2O\PSLR DGDSWDGR 

A comparação estabelecida entre a Ilíada e Iracema


demonstra que essas obras
A FRPELQDP IROFORUH H FXOWXUD HUXGLWD HP VHXV HVWLORV
HVWpWLFRV
B articulam resistência e opressão em seus gêneros
literários.
C associam história e mito em suas construções
$EDSRUX'LVSRQtYHOHPZZZIROKDXROFRPEU$FHVVRHPDJR identitárias.
TEXTO II D UHÀHWHP SDFL¿VPR H EHOLFLVPR HP VXDV HVFROKDV
(PMDQHLURGH7DUVLODTXHULDGDUXPSUHVHQWHGH ideológicas.
DQLYHUViULRHVSHFLDODRVHXPDULGR2VZDOGGH$QGUDGH E WUDGX]HP UHYROWD H FRQIRUPLVPR HP VHXV SDGU}HV
Pintou o Abaporu(OHVDFKDUDPTXHSDUHFLDXPD¿JXUD alegóricos.
LQGtJHQDDQWURSyIDJDH7DUVLODOHPEURXVHGRGLFLRQiULR
WXSLJXDUDQLGHVHXSDL%DWL]RXVHRTXDGURGHAbaporu, QUESTÃO 19
TXH VLJQL¿FD KRPHP TXH FRPH FDUQH KXPDQD R
DQWURSyIDJR(2VZDOGHVFUHYHXR0DQLIHVWR$QWURSyIDJR ÈUHDVHPHVWDEHOHFLPHQWRGHDWLYLGDGHVHFRQ{PLFDV
HIXQGDUDPR0RYLPHQWR$QWURSRIiJLFR VHPSUHVHFRORFDUDPFRPRJUDQGHFKDPDUL])RLDVVLP
QR OLWRUDO QRUGHVWLQR QR LQtFLR GD FRORQL]DomR FRP R
'LVSRQtYHOHPZZZWDUVLODGRDPDUDOFRPEU$FHVVRHPDJR DGDSWDGR 
SDXEUDVLO D FDQDGHDo~FDU R IXPR DV SURGXo}HV
O movimento originado da obra Abaporu pretendia GH DOLPHQWRV H R FRPpUFLR 2 HQULTXHFLPHQWR UiSLGR
se apropriar H[DFHUERXRHVStULWRGHDYHQWXUDGRKRPHPPRGHUQR
FARIA, S. C. A Colônia em movimento5LRGH-DQHLUR1RYD)URQWHLUD DGDSWDGR 
A da cultura europeia, para originar algo brasileiro.
B GDDUWHFOiVVLFDSDUDFRSLDURVHXLGHDOGHEHOH]D 2SURFHVVRGHVFULWRQRWH[WRWURX[HFRPRHIHLWRR D
C do ideário republicano, para celebrar a modernidade. A DFXPXODomR GH FDSLWDLV QD &RO{QLD SURSLFLDQGR D
D GDV WpFQLFDV DUWtVWLFDV QDWLYDV SDUD FRQVDJUDU VXD FULDomRGHXPDPELHQWHLQWHOHFWXDOHIHUYHVFHQWH
tradição. B surgimento de grandes cidades coloniais, voltadas
E da herança colonial brasileira, para preservar sua SDUD R FRPpUFLR H FRP JUDQGH FRQFHQWUDomR
identidade. monetária.
C concentração da população na região litorânea, pela
IDFLOLGDGHGHHVFRDPHQWRGDSURGXomR
D IDYRUHFLPHQWRGRVQDWXUDLVGD&RO{QLDQDFRQFHVVmR
GHWtWXORVGHQREUH]DH¿GDOJXLDSHOD0RQDUTXLD
E construção de relações de trabalho menos
desiguais que as da Metrópole, inspiradas pelo
empreendedorismo.

CH - 1º dia | Caderno 3 - BRANCO - Página 7


*BRAN75SAB8* 2014

QUESTÃO 20 QUESTÃO 22
(PGH]HPEURGHFRPHoRXXPDJUHYHGHGRLV 3DVVDGD D IHVWD GD DEROLomR RV H[HVFUDYRV
PHVHV QR SULQFLSDO SRUWR GD ÈIULFD 2FLGHQWDO )UDQFHVD procuraram distanciar-se do passado de escravidão,
Dacar. As autoridades só conseguiram levar os grevistas negando-se a se comportar como antigos cativos.
de volta ao trabalho com grandes aumentos de salário e, Em diversos engenhos do Nordeste, negaram-se a
RTXHpDLQGDPDLVLPSRUWDQWHSRQGRHPSUiWLFDWRGRR receber a ração diária e a trabalhar sem remuneração.
DSDUDWR GH UHODo}HV LQGXVWULDLV XVDGR QD )UDQoD ² HP 4XDQGR GHFLGLUDP ILFDU LVVR QmR VLJQLILFRX TXH
UHVXPRDJLQGRFRPRVHRVJUHYLVWDVIRVVHPPRGHUQRV concordassem em se submeter às mesmas condições
operários industriais. de trabalho do regime anterior.
COOPER, F.; HOLT, T.; SCOTT, R. $OpPGDHVFUDYLGmR.
FRAGA, W.; ALBUQUERQUE, W. R. Uma história da cultura afro-brasileira.
5LRGH-DQHLUR&LYLOL]DomR%UDVLOHLUD DGDSWDGR 
6mR3DXOR0RGHUQD DGDSWDGR 

'XUDQWHRQHRFRORQLDOLVPRRWUDEDOKRIRUoDGR²TXHQmR 6HJXQGR R WH[WR RV SULPHLURV DQRV DSyV D DEROLomR GD


VHFRQIXQGHFRPDHVFUDYLGmR²IRLXPDFRQVWDQWHHP HVFUDYLGmRQR%UDVLOWLYHUDPFRPRFDUDFWHUtVWLFDR D
GLYHUVDVUHJL}HVGRFRQWLQHQWHDIULFDQRDWpRVpFXOR;;
'HDFRUGRFRPRWH[WRVXDVXSHUDomRGHULYDGD A FDUiWHURUJDQL]DWLYRGRPRYLPHQWRQHJUR
A FUtWLFDPRUDOGDLQWHOHFWXDOLGDGHPHWURSROLWDQD B equiparação racial no mercado de trabalho.
B pressão articulada dos organismos multilaterais. C EXVFDSHORUHFRQKHFLPHQWRGRH[HUFtFLRGDFLGDGDQLD
C UHVLVWrQFLDRUJDQL]DGDGRVWUDEDOKDGRUHVQDWLYRV D HVWDEHOHFLPHQWR GR VDOiULR PtQLPR SRU SURMHWR
D concessão pessoal dos empresários imperialistas. legislativo.
E EDL[DOXFUDWLYLGDGHGRVHPSUHHQGLPHQWRVFDSLWDOLVWDV E HQWXVLDVPRFRPDH[WLQomRGDVSpVVLPDVFRQGLo}HV
de trabalho.
QUESTÃO 21
QUESTÃO 23
Os escravos, obviamente, dispunham de poucos
UHFXUVRV SROtWLFRV PDV QmR GHVFRQKHFLDP R TXH VH 9HQH]D HPHUJLQGR REVFXUDPHQWH DR ORQJR GR
passava no mundo dos poderosos. Aproveitaram-se LQtFLR GD ,GDGH 0pGLD GDV iJXDV jV TXDLV GHYLD VXD
das divisões entre estes, selecionaram temas que lhes imunidade a ataques, era nominalmente submetida
LQWHUHVVDYDPGRLGHiULROLEHUDOHDQWLFRORQLDOWUDGX]LUDPH DR ,PSpULR %L]DQWLQR PDV QD SUiWLFD HUD XPD FLGDGH
HPSUHVWDUDPVLJQL¿FDGRVSUySULRVjVUHIRUPDVRSHUDGDV HVWDGR LQGHSHQGHQWH QD DOWXUD GR VpFXOR ; 9HQH]D
QRHVFUDYLVPREUDVLOHLURDRORQJRGRVpFXOR;,; era única na cristandade por ser uma comunidade
REIS, J. J. Nos achamos em campo a tratar da liberdade: a resistência negra no Brasil comercial: “Essa gente não lavra, semeia ou colhe uvas”,
oitocentista. In: MOTA, C. G. (Org.). 9LDJHPLQFRPSOHWDDH[SHULrQFLDEUDVLOHLUD FRPR XP VXUSUHVR REVHUYDGRU GR VpFXOR ;, FRQVWDWRX
  6mR3DXOR6HQDF
&RPHUFLDQWHV YHQH]LDQRV SXGHUDP QHJRFLDU WHUPRV
$R ORQJR GR VpFXOR ;,; RV QHJURV HVFUDYL]DGRV IDYRUiYHLV SDUD FRPHUFLDU FRP &RQVWDQWLQRSOD PDV
FRQVWUXtUDPYDULDGDVIRUPDVSDUDUHVLVWLUjHVFUDYLGmRQR WDPEpPVHUHODFLRQDUDPFRPPHUFDGRUHVGRLVOm
%UDVLO$HVWUDWpJLDGHOXWDFLWDGDQRWH[WREDVHDYDVHQR FLETCHER, R. A cruz e o crescenteFULVWLDQLVPRHLVOmGH0DRPpj5HIRUPD
aproveitamento das 5LRGH-DQHLUR1RYD)URQWHLUD

A estruturas urbanas como ambiente para escapar do $H[SDQVmRGDVDWLYLGDGHVGHWURFDVQD%DL[D,GDGH


cativeiro. 0pGLD GLQDPL]DGDV SRU FHQWURV FRPR 9HQH]D
B GLPHQV}HVWHUULWRULDLVFRPRHOHPHQWRSDUDIDFLOLWDUDV UHIOHWHR D
IXJDV A importância das cidades comerciais.
C OLPLWDo}HV HFRQ{PLFDV FRPR SUHVVmR SDUD R ¿P GR B integração entre a cidade e o campo.
escravismo.
C GLQDPLVPRHFRQ{PLFRGD,JUHMDFULVWm
D FRQWUDGLo}HVSROtWLFDVFRPREUHFKDSDUDDFRQTXLVWD
da liberdade. D FRQWUROHGDDWLYLGDGHFRPHUFLDOSHODQREUH]DIHXGDO
E LGHRORJLDVRULJLQiULDVFRPRDUWLItFLRSDUDUHVJDWDUDV E ação reguladora dos imperadores durante as trocas
UDt]HVDIULFDQDV comerciais.

CH - 1º dia | Caderno 3 - BRANCO - Página 8


2014 *BRAN75SAB9*
QUESTÃO 24 QUESTÃO 25
TEXTO I 4XDQGR 'HXV FRQIXQGLX DV OtQJXDV QD WRUUH GH
2 SUtQFLSH ' -RmR 9, SRGLD WHU GHFLGLGR ¿FDU HP %DEHO SRQGHURX )LOR +HEUHX TXH WRGRV ¿FDUDP PXGRV
3RUWXJDO1HVVHFDVRR%UDVLOFRPFHUWH]DQmRH[LVWLULD H VXUGRV SRUTXH DLQGD TXH WRGRV IDODVVHP H WRGRV
$ &RO{QLD VH IUDJPHQWDULD FRPR VH IUDJPHQWRX D SDUWH ouvissem, nenhum entendia o outro. Na antiga Babel,
HVSDQKROD GD $PpULFD 7HUtDPRV HP YH] GR %UDVLO GH KRXYH VHWHQWD H GXDV OtQJXDV QD %DEHO GR ULR GDV
KRMH FLQFR RX VHLV SDtVHV GLVWLQWRV -RVp 0XULOR GH $PD]RQDVMiVHFRQKHFHPPDLVGHFHQWRHFLQTXHQWD
Carvalho) E assim, quando lá chegamos, todos nós somos mudos
e todos eles, surdos. Vede agora quanto estudo e quanto
TEXTO II WUDEDOKRVHUmRQHFHVViULRVSDUDTXHHVVHVPXGRVIDOHP
+i QR %UDVLO XPD LQVLVWrQFLD HP UHIRUoDU R OXJDU e esses surdos ouçam.
FRPXPVHJXQGRRTXDOIRL'-RmR9,RUHVSRQViYHOSHOD VIEIRA, A. Sermões pregados no Brasil. In: RODRIGUES, J. H. História viva.
6mR3DXOR*OREDO DGDSWDGR 
XQLGDGHGRSDtV,VVRQmRpYHUGDGH$XQLGDGHGR%UDVLO
IRLFRQVWUXtGDDRORQJRGRWHPSRHpDQWHVGHWXGRXPD 1R GHFRUUHU GD FRORQL]DomR SRUWXJXHVD QD$PpULFD DV
IDEULFDomRGD&RURD$LGHLDGHTXHHUDSUHFLVRIRUWDOHFHU tentativas de resolução do problema apontado pelo padre
XP,PSpULRFRPRVWHUULWyULRVGH3RUWXJDOH%UDVLOFRPHoRX $QW{QLR9LHLUDUHVXOWDUDPQD
MiQRVpFXOR;9,,, (YDOGR&DEUDOGH0HOOR
A ampliação da violência nas guerras intertribais.
1808 – O primeiro ano do resto de nossas vidas. Folha de S. PauloQRY DGDSWDGR 
B GHVLVWrQFLDGDHYDQJHOL]DomRGRVSRYRVQDWLYRV
(P  IRL FRPHPRUDGR R ELFHQWHQiULR GD FKHJDGD
C LQGLIHUHQoDGRVMHVXtWDVHPUHODomRjGLYHUVLGDGHGH
GD IDPtOLD UHDO SRUWXJXHVD DR %UDVLO 1RV WH[WRV GRLV
OtQJXDVDPHULFDQDV
importantes historiadores brasileiros se posicionam
GLDQWHGHXPGRVSRVVtYHLVOHJDGRVGHVVHHSLVyGLRSDUD D pressão da Metrópole pelo abandono da catequese
D KLVWyULD GR SDtV 2 OHJDGR GLVFXWLGR H XP DUJXPHQWR QDVUHJL}HVGHGLItFLODFHVVR
TXHVXVWHQWDDGLIHUHQoDGRSULPHLURSRQWRGHYLVWDSDUD E VLVWHPDWL]DomR GDV OtQJXDV QDWLYDV QXPD HVWUXWXUD
o segundo estão associados, respectivamente, em: JUDPDWLFDOIDFLOLWDGRUDGDFDWHTXHVH
A ,QWHJULGDGH WHUULWRULDO ² &HQWUDOL]DomR GD
DGPLQLVWUDomRUpJLDQD&RUWH QUESTÃO 26
B 'HVLJXDOGDGHVRFLDO²&RQFHQWUDomRGDSURSULHGDGH (P EXVFD GH PDWpULDVSULPDV H GH PHUFDGRV SRU
IXQGLiULDQRFDPSR
FDXVD GD DFHOHUDGD LQGXVWULDOL]DomR RV HXURSHXV
C +RPRJHQHLGDGH LQWHOHFWXDO ² 'LIXVmR GDV LGHLDV
liberais nas universidades. UHWDOKDUDP HQWUH VL D ÈIULFD 0DLV GR TXH DOHJDo}HV
HFRQ{PLFDV KDYLD MXVWL¿FDWLYDV SROtWLFDV FLHQWt¿FDV
D 8QLIRUPLGDGHFXOWXUDO²0DQXWHQomRGDPHQWDOLGDGH
HVFUDYLVWDQDVID]HQGDV LGHROyJLFDV H DWp ¿ODQWUySLFDV 2 UHL EHOJD /HRSROGR ,,
E &RQWLQXLGDGHHVSDFLDO²&RRSWDomRGRVPRYLPHQWRV GHIHQGLDRWUDEDOKRPLVVLRQiULRHDFLYLOL]DomRGRVQDWLYRV
VHSDUDWLVWDVQDVSURYtQFLDV do Congo, argumento desmascarado pelas atrocidades
praticadas contra a população.
NASCIMENTO, C. PartilhDGDÈIULFDRDVVRPEURGRFRQWLQHQWHPXWLODGRRevista de
História da Biblioteca NacionalDQRQGH] DGDSWDGR 

$ DWXDomR GRV SDtVHV HXURSHXV FRQWULEXLX SDUD TXH D


ÈIULFD²HQWUHH²VHWUDQVIRUPDVVHHPXPD
HVSpFLHGHJUDQGH³FROFKDGHUHWDOKRV´(VVHSURFHVVRIRL
motivado pelo(a)
A EXVFD GH DFHVVR j LQIUDHVWUXWXUD HQHUJpWLFD GRV
SDtVHVDIULFDQRV
B tentativa de regulação da atividade comercial com os
SDtVHVDIULFDQRV
C UHVJDWH KXPDQLWiULR GDV SRSXODo}HV DIULFDQDV HP
VLWXDomRGHH[WUHPDSREUH]D
D GRPtQLRVREUHRVUHFXUVRVFRQVLGHUDGRVHVWUDWpJLFRV
SDUDRIRUWDOHFLPHQWRGDVQDo}HVHXURSHLDV
E QHFHVVLGDGH GH H[SDQGLU DV IURQWHLUDV FXOWXUDLV GD
(XURSDSHORFRQWDWRFRPRXWUDVFLYLOL]Do}HV

&+žGLD_&DGHUQR%5$1&23iJLQD
*BRAN75SAB10* 2014

QUESTÃO 27 QUESTÃO 29
Os holandeses desembarcaram em Pernambuco O enclave supõe a presença de “muros sociais”
no ano de 1630, em nome da Companhia das Índias internos que separam e distanciam populações e grupos
2FLGHQWDLV :,& HIRUDPDRVSRXFRVRFXSDQGRDFRVWD de um mesmo lugar. Tais muros revelam as grandes
TXHLDGDIR]GR5LR6mR)UDQFLVFRDR0DUDQKmRQRDWXDO
contradições e discrepâncias presentes nas cidades
Nordeste brasileiro. Eles chegaram ao ponto de destruir
Olinda, antiga sede da capitania de Duarte Coelho, para brasileiras. É aqui que o território merece ser considerado
HUJXHUQR5HFLIHXPDSHTXHQD$PVWHUGm XP QRYR HOHPHQWR QDV SROtWLFDV S~EOLFDV HQTXDQWR
1$6&,0(1725/;$WRTXHGHFDL[DVRevista de História da Biblioteca Nacional,
XP VXMHLWR FDWDOLVDGRU GH SRWrQFLDV QR SURFHVVR GH
DQRQMXO UHIXQGDomRGRVRFLDO
'RSRQWRGHYLVWDHFRQ{PLFRDVUD]}HVTXHOHYDUDPRV KOGA, D. Medidas de cidades: entre territórios de vida e territórios vividos.
KRODQGHVHVDLQYDGLUHPRQRUGHVWHGD&RO{QLDGHFRUULDP 6mR3DXOR&RUWH]

GRIDWRGHTXHHVVDUHJLmR 1R FRQWH[WR DWXDO GDV P~OWLSODV WHUULWRULDOL]Do}HV


A era a mais importante área produtora de açúcar na DSRQWDGDV QR IUDJPHQWR D IRUPDomR GH HQFODYHV
$PpULFDSRUWXJXHVD IRUWL¿FDGRVQRHVSDoRXUEDQRpUHVXOWDGRGD
B SRVVXtD DV PDLV ULFDV PDWDV GH SDXEUDVLO QR OLWRUDO A autossegregação elitista em prol de garantia de
GDV$PpULFDV
segurança.
C FRQWDYD FRP R SRUWR PDLV HVWUDWpJLFR SDUD D
navegação no Atlântico Sul. B VHJPHQWDomRVRFLDOGDVSROtWLFDVS~EOLFDVSRUQtYHLV
de carência.
D representava o principal entreposto de escravos
DIULFDQRVSDUDDV$PpULFDV C LQÀXrQFLD GH JUXSRV SROtWLFRV JOREDLV HP UHGH QR
E FRQVWLWXtDXPUHGXWRGHULFRVFRPHUFLDQWHVGHDo~FDU cotidiano urbano.
GHRULJHPMXGDLFD D ampliação dos territórios móveis nas áreas
residenciais tradicionais.
QUESTÃO 28
E necessidade da população em associar
2VPRYLPHQWRVVRFLDLVGRVpFXOR;;,Do}HVFROHWLYDV espacialmente trabalho e moradia.
GHOLEHUDGDV TXH YLVDP j WUDQVIRUPDomR GH YDORUHV H
LQVWLWXLo}HV GD VRFLHGDGH PDQLIHVWDPVH QD H SHOD QUESTÃO 30
internet. O mesmo pode ser dito do movimento ambiental,
o movimento das mulheres, vários movimentos pelos $ SULQFLSDO IRUPD GH UHODomR HQWUH R KRPHP H D
GLUHLWRV KXPDQRV PRYLPHQWRV GH LGHQWLGDGH pWQLFD QDWXUH]D RX PHOKRU HQWUH R KRPHP H R PHLR p GDGD
movimentos religiosos, movimentos nacionalistas e SHOD WpFQLFD ² XP FRQMXQWR GH PHLRV LQVWUXPHQWDLV H
GRV GHIHQVRUHVSURSRQHQWHV GH XPD OLVWD LQ¿QGiYHO GH VRFLDLVFRPRVTXDLVRKRPHPUHDOL]DVXDYLGDSURGX]
SURMHWRVFXOWXUDLVHFDXVDVSROtWLFDV e, ao mesmo tempo, cria espaço.
CASTELLS, M. A galáxia da internetUHÀH[}HVVREUHDLQWHUQHWRVQHJyFLRVHDVRFLHGDGH SANTOS, M. $QDWXUH]DGRHVSDoR. São Paulo: Edusp, 2002 (adaptado).
Rio de Janeiro: Jorge Zahar, 2003.

'H DFRUGR FRP R WH[WR D SRSXODomR HQJDMDGD HP $ UHODomR HVWDEHOHFLGD QR WH[WR DVVRFLDGD D XPD
SURFHVVRV SROtWLFRV SRGH XWLOL]DU D UHGH PXQGLDO GH SURIXQGDGHJUDGDomRDPELHQWDOpYHUL¿FDGDQD
FRPSXWDGRUHV FRPR UHFXUVR SDUD PRELOL]DomR SRLV D A UDFLRQDOL]DomRGRXVRGHUHFXUVRVKtGULFRVSDUD¿QV
LQWHUQHWFDUDFWHUL]DVHSRU de abastecimento residencial.
A diminuir a insegurança do sistema eleitoral. B DSURSULDomRGHUHVHUYDVH[WUDWLYLVWDVSDUDDWHQGHUj
B UHIRUoDU D SRVVLELOLGDGH GH PDLRU SDUWLFLSDomR demanda de subsistência.
TXDOL¿FDGD
C retirada da cobertura vegetal com o intuito de
C JDUDQWLU R FRQWUROH GDV LQIRUPDo}HV JHUDGDV QDV desenvolver a agricultura intensiva.
PRELOL]Do}HV
D ampliação da produção de alimentos orgânicos
D LQFUHPHQWDU R HQJDMDPHQWR FtYLFR SDUD DOpP GDV
IURQWHLUDVORFDLV SDUDPLQLPL]DUSUREOHPDVGDIRPH
E DPSOLDU D SDUWLFLSDomR SHOD VROXomR GD HVFDVVH] GH E UHRUGHQDomR GR HVSDoR UXUDO SDUD IDYRUHFHU R
tempo dos cidadãos. desenvolvimento do ecoturismo.

CH - 1º dia | Caderno 3 - BRANCO - Página 10


2014 *BRAN75SAB11*
QUESTÃO 31

QUINO. Toda Mafalda6mR3DXOR0DUWLQV)RQWHV DGDSWDGR 

1RVTXDGULQKRVID]VHUHIHrência a um evento que correspondia a um dos grandes medos da população mundial no


SHUtRGRGD*XHUUD)ULD'XUDQWHHVVHSHUtRGRDSRVVLELOLGDGHGHRFRUUrQFLDGHVVHHYHQWRHUDJUDQGHHPIXQomRGR D
A acirramento da rivalidade Norte-Sul.
B LQWHQVL¿FDomRGDFRUULGDDUPDPHQWLVWD
C RFRUUrQFLDGHFULVHVHFRQ{PLFDVJOREDLV
D emergência de novas potências mundiais.
E DSURIXQGDPHQWRGHGHVLJXDOGDGHVVRFLDLV

QUESTÃO 32
'HPRGRJHUDORVORJUDGRXURVGH)RUWDOH]DDWpPHDGRVGRVpFXOR;,;HUDPFRQKHFLGRVSRUGHVLJQDo}HVVXUJLGDV
GDWUDGLomRRXGHIXQo}HVHHGL¿FDo}HV TXH OKHV FDUDFWHUL]DYDP$VVLP FKDPDYDVH7UDYHVVD GD 0XQLFLSDOLGDGH
DWXDO*XLOKHUPH5RFKD SRUODGHDURSUpGLRGD,QWHQGrQFLD0XQLFLSDO6%HUQDUGR KRMH3HGUR3HUHLUD SRUFRQWD
GHLJUHMDKRP{QLPD5XDGR&DMXHLUR DWXDO3HGUR%RUJHV SRUDEULJDUXPDGDVPDLVDQWLJDVHSRSXODUHViUYRUHVGD
FDSLWDO-iD3UDoD-RVpGH$OHQFDUQDGpFDGDGHHUDSRSXODUPHQWHGHVLJQDGDSRU3UDoDGR3DWURFtQLRSRLVHP
VHXODGRQRUWHVHHQFRQWUDYDXPDLJUHMDKRP{QLPD
SILVA FILHO, A. L. M. FortalezaLPDJHQVGDFLGDGH)RUWDOH]D0XVHXGR&HDUi6HFXOW&( DGDSWDGR 

Os atos de nomeação dos logradouros, analisados de uma perspectiva histórica, constituem


A IRUPDVGHSURPRYHURVQRPHVGDVDXWRULGDGHVLPSHULDLV
B PRGRVR¿FLDLVHSRSXODUHVGHSURGXomRGDPHPyULDQDVFLGDGHV
C UHFXUVRVDUTXLWHW{QLFRVIXQFLRQDLVjUDFLRQDOL]DomRGRHVSDoRXUEDQR
D PDQHLUDVGHKLHUDUTXL]DUHVWUDWRVVRFLDLVHGLYLGLUDVSRSXODo}HVXUEDQDV
E PHFDQLVPRVGHLPSRVLomRGRVLWLQHUiULRVVRFLDLVHÀX[RVHFRQ{PLFRVQDFLGDGH

QUESTÃO 33
1DSULPHLUDGpFDGDGRVpFXOR;;UHIRUPDUDFLGDGHGR5LRGH-DQHLURSDVVRXDVHURVLQDOPDLVHYLGHQWHGD
PRGHUQL]DomRTXHVHGHVHMDYDSURPRYHUQR%UDVLO2SRQWRFXOPLQDQWHGRHVIRUoRGHPRGHUQL]DomRVHGHXQDJHVWmR
GRSUHIHLWR3HUHLUD3DVVRVHQWUHH³25LRFLYLOL]DYDVH´HUDIUDVHFpOHEUHjpSRFDHFRQGHQVDYDRHVIRUoR
para iluminar as vielas escuras e esburacadas, controlar as epidemias, destruir os cortiços e remover as camadas
populares do centro da cidade.
OLIVEIRA, L. L. Sinais de modernidade na Era Vargas: vida literária, cinema e rádio. In: FERREIRA, J.; DELGADO, L. A. (Org.). O tempo do nacional-estatismoGRLQtFLRDRDSRJHXGR
(VWDGR1RYR5LRGH-DQHLUR&LYLOL]DomR%UDVLOHLUD

2SURFHVVRGHPRGHUQL]DomRPHQFLRQDGRQRWH[WRWUD]LDXPSDUDGR[RTXHVHH[SUHVVDYDQR D
A substituição de vielas por amplas avenidas.
B impossibilidade de se combaterem as doenças tropicais.
C LGHDOGHFLYLOL]DomRDFRPSDQKDGRGHPDUJLQDOL]DomR
D VREUHSRVLomRGHSDGU}HVDUTXLWHW{QLFRVLQFRPSDWtYHLV
E SURMHWRGHFLGDGHLQFRPSDWtYHOFRPDUXJRsidade do relevo.

CH - 1º dia | Caderno 3 - BRANCO - Página 11


*BRAN75SAB12* 2014

QUESTÃO 34 QUESTÃO 36

(QTXDQWRDVUHEHOL}HVDJLWDYDPRSDtVDVWHQGrQFLDV
SROtWLFDVQRFHQWURGLULJHQWHLDPVHGH¿QLQGR$SDUHFLDP
HP JHUPH RV GRLV JUDQGHV SDUWLGRV LPSHULDLV ² R
Conservador e o Liberal. Os conservadores reuniam
magistrados, burocratas, uma parte dos proprietários
rurais, especialmente do Rio de Janeiro, Bahia e
Pernambuco, e os grandes comerciantes, entre os quais
muitos portugueses. Os liberais agrupavam a pequena
FODVVHPpGLDXUEDQDDOJXQVSDGUHVHSURSULHWiULRVUXUDLV
de áreas menos tradicionais, sobretudo de São Paulo,
Minas e Rio Grande do Sul.
FAUSTO, B. História do Brasil6mR3DXOR(GXVS

1R WH[WR R DXWRU FRPSDUD D FRPSRVLomR GDV IRUoDV


SROtWLFDVTXHDWXDUDPQR6HJXQGR5HLQDGR  
'RLVDVSHFWRVTXHFDUDFWHUL]DPRVSDUWLGRV&RQVHUYDGRU $ (VWiWXD GR /DoDGRU WRPEDGD FRPR SDWULP{QLR
e Liberal estão indicados, respectivamente, em: HP  p XP PRQXPHQWR GH 3RUWR $OHJUH56 TXH
UHSUHVHQWDRJD~FKR HPWUDMHVWtSLFRV 
A $EROLomR GD HVFUDYLGmR ² $GRomR GR WUDEDOKR
'LVSRQtYHOHPZZZSRUWRDOHJUHWXUEU$FHVVRHPDJR DGDSWDGR 
assalariado.
B 'LIXVmR GD LQGXVWULDOL]DomR ² &RQVHUYDomR GR 2PRQXPHQWRLGHQWL¿FDXP D
ODWLI~QGLRPRQRFXOWRU A H[HPSORGHEHPLPDWHULDO
C 3URPRomR GR SURWHFLRQLVPR ² 5HPRomR GDV B IRUPDGHH[SRVLomRGDLQGLYLGXDOLGDGH
EDUUHLUDVDOIDQGHJiULDV C modo de enaltecer os ideais de liberdadade.
D 3UHVHUYDomR GR XQLWDULVPR ² $PSOLDomR GD D PDQLIHVWDomRKLVWyULFRFXOWXUDOGHXPDSRSXODomR
GHVFHQWUDOL]DomRSURYLQFLDO
E maneira de propor mudanças nos costumes.
E ,PSOHPHQWDomR GR UHSXEOLFDQLVPR ² &RQWLQXDomR
da monarquia constitucional. QUESTÃO 37

QUESTÃO 35 Canto dos lavradores de Goiás


7HPID]HQGDHID]HQGD
4XHpJUDQGHSHUIHLWDPHQWH
Sobe serra desce serra
Salta muita água corrente
6HPODYRXUDHVHPQLQJXpP
O dono mora ausente.
Lá só tem caçambeiro
Tira onda de valente
,VVRpTXHpJUDQGHEDUUHLUD
4XHHVWiHPQRVVDIUHQWH
Tem muita gente sem terra
'LVSRQtYHOHPZZZLQGLDQDHGX!$FHVVRHPDJR DGDSWDGR  Tem muita terra sem gente.
As redes sociais tornaram-se espaços importantes de MARTINS, J. S. Cativeiro da terra6mR3DXOR&LrQFLDV+XPDQDV

relacionamento e comunicação. A charge apresenta o


No canto registraGRSHODFXOWXUDSRSXODUDFDUDFWHUtVWLFD
LPSDFWR GD LQWHUQHW QD YLGD GRV LQGLYtGXRV TXDQGR ID]
UHIHUrQFLDj GRPXQGRUXUDOEUDVLOHLURQRVpFXOR;;GHVWDFDGDpD

A DPSOLDomRGRSRGHUGRVFOpULJRVQRFRQWUROHGRV¿pLV A atuação da bancada ruralista.


B adequação dos ritos sacramentais ao cotidiano. B H[SDQVmRGDIURQWHLUDDJUtFROD
C perda de privacidade em ambiente virtual. C YDORUL]DomRGDDJULFXOWXUDIDPLOLDU
D reinterpretação da noção de pecado. D PDQXWHQomRGDFRQFHQWUDomRIXQGLiULD
E PRGHUQL]DomRGDVLQVWLWXLo}HVUHOLJLRVDV E implementação dDPRGHUQL]DomRFRQVHUYDGRUD

CH - 1º dia | Caderno 3 - BRANCO - Página 12


2014 *BRAN75SAB13*
QUESTÃO 38
2XVRLQWHQVRGDViJXDVVXEWHUUkQHDVVHPSODQHMDPHQWRWHPFDXVDGRVpULRVSUHMXt]RVjVRFLHGDGHDRXVXiULR
HDRPHLRDPELHQWH(PYiULDVSDUWHVGRPXQGRSHUFHEHVHTXHDH[SORUDomRGHIRUPDLQFRUUHWDWHPOHYDGRD
SHUGDVGRSUySULRDTXtIHUR
7(,;(,5$:HWDODecifrando a Terra6mR3DXOR&LD(GLWRUD1DFLRQDO DGDSWDGR 

1RWH[WRDSRQWDPVHGL¿FXOGDGHVDVVRFLDGDVDRXVRGHXPLPSRUWDQWHUHFXUVRQDWXUDO8PSUREOHPDGHULYDGRGH
VXDXWLOL]DomRHXPDUHVSHFWLYDFDXVDSDUDVXDRFRUUrQFLDVmR
A &RQWDPLQDomRGRDTXtIHUR²&RQWHQomRLPSUySULDGRLQJUHVVRGLUHWRGHiJXDVXSHU¿FLDO
B ,QWUXVmRVDOLQD²([WUDomRUHGX]LGDGDiJXDGRFHGRVXEVROR
C 6XSHUH[SORUDomRGHSRoRV²&RQVWUXomRLQH¿FD]GHFDSWDo}HVVXEVXSHU¿FLDLV
D 5HEDL[DPHQWRGRQtYHOGDiJXD²%RPEHDPHQWRGRSRoRHTXLYDOHQWHjUHSRVLomRQDWXUDO
E (QFDUHFLPHQWRGDH[SORUDomRVXVWHQWiYHO²&RQVHUYDomRGDFREHUWXUDYHJHWDOORFDO

QUESTÃO 39

Escoamento das águas das chuvas


Precipitação
100%
Evapotranspiração

40%
Escoamento Precipitação
Superficial
10%

Água 50%
Subterrânea

Precipitação Escoamento
100% de telhados
Evapotranspiração
15%
25%

30%

Vazão
Esgoto
45%
Pluvial

Água 30%
Subterrânea

'LVSRQtYHOHPZZZHVVHQWLDHGLWRUDLIIHGXEU$FHVVRHPMXQ

Comparando o escoamento natural das águas de chuva com o escoamento em áreas urbanas, nota-se que a
XUEDQL]DomRSURPRYHPDLRU
A YD]mRKtGULFDQDVHVWUXWXUDVDUWL¿FLDLVFRQVWUXtGDVSHODVDWLYLGDGHVKXPDQDV
B DUPD]HQDJHPVXEWHUUkQHDXPDYH]TXHQDViUHDVXUEDQL]DGDVRFLFORKLGUROyJLFRpDOWHUDGRSHODVDWLYLGDGHV
antrópicas.
C evapotranspiração, pois, nas áreas urbanas, a diminuição da cobertura vegetal promove aumento no processo de
transpiração.
D WUDQVIHUrQFLDGHGHVFDUJDVXEWHUUkQHDSRLVDRDXPHQWDUDLPSHUPHDELOL]DomRWUD]VHFRPRFRQVHTXrQFLDPDLRU
DOLPHQWDomRGROHQoROIUHiWLFR
E LQ¿OWUDomRSRLVDRDXPHQWDUDLPSHUPHDELOL]DomRHVWDEHOHFHVHXPDUHODomRGLUHWDPHQWHSURSRUFLRQDOGHVVHV
elementos na composição do ciclo hidrológico.
CH - 1º dia | Caderno 3 - BRANCO - Página 13
*BRAN75SAB14* 2014

QUESTÃO 40 QUESTÃO 42
8PD FLGDGH TXH UHGX] HPLVV}HV HOHWUL¿FD FRP 2V GHVHTXLOtEULRV TXH VH UHJLVWUDP QDV HQFRVWDV
HQHUJLD VRODU VHXV HVWiGLRV PDV GHL[D EDLUURV VHP RFRUUHPQDPDLRULDGDVYH]HVHPIXQomRGDSDUWLFLSDomR
VDQHDPHQWR EiVLFR VHP DVVLVWrQFLD PpGLFD H VHP GR FOLPD H GH DOJXQV DVSHFWRV GDV FDUDFWHUtVWLFDV GDV
escola de qualidade nunca será sustentável. A mudança HQFRVWDV TXH LQFOXHP D WRSRJUD¿D JHRORJLD JUDX GH
GRUHJLPHGHFKXYDVTXHMiRFRUUHSRUFDXVDGDPXGDQoD intemperismo, solo e tipo de ocupação.
FOLPiWLFDID]FRPTXHLQXQGDo}HVHPiUHDVFRPHVJRWR CUNHA, S. B.; GUERRA, A. J. T. Degradação ambiental. In: GUERRA, A. J. T.; CUNHA, S.
H OL[}HV D FpX DEHUWR SURSDJXHP GRHQoDV GDV TXDLV R B. (Org.). Geomorfologia e meio ambiente5LRGH-DQHLUR%HUWUDQG%UDVLO
sistema de saúde não cuidará apropriadamente.
2V GHVHTXLOtEULRV UHVXOWDQWHV GD DWXDomR KXPDQD
$%5$1&+(66$VXVWHQWDELOLGDGHpKXPDQDHHFROyJLFD'LVSRQtYHOHPZZZHFRSROLWLFD MXQWRjVYHUWHQWHVtQJUHPHVGRUHOHYRVmRIRUWHPHQWH
FRPEU$FHVVRHPMXO DGDSWDGR 
ligados ao(à)
3UREOHPDWL]DQGR D QRomR GH VXVWHQWDELOLGDGH R
A aumento da atividade industrial.
DUJXPHQWRDSUHVHQWDGRQRWH[WRVXJHUHTXHR D
B crescimento populacional urbano desordenado.
A WHFQRORJLD YHUGH p QHFHVViULD DR SODQHMDPHQWR
C desconcentração das atividades comerciais e dos
urbano.
serviços.
B PXGDQoDFOLPiWLFDpSURYRFDGDSHORFUHVFLPHQWRGDV
D LQVWDODomR GH HTXLSDPHQWRV XUEDQRV QD SHULIHULD GD
cidades.
cidade.
C FRQVXPR FRQVFLHQWH p FDUDFWHUtVWLFR GH FLGDGHV
E FRQVWUXomR GH SURMHWRV KDELWDFLRQDLV YROWDGRV j
sustentáveis.
SRSXODomRGHEDL[DUHQGD
D GHVHQYROYLPHQWR XUEDQR p LQFRPSDWtYHO FRP D
preservação ambiental. QUESTÃO 43
E GHVHQYROYLPHQWR VRFLDO p FRQGLomR SDUD R
desenvolvimento sustentável.

QUESTÃO 41
$ YRDGHLUD FDQRD GH DOXPtQLR FRP PRWRU GH SRSD
XVDGD FRPR PHLR GH WUDQVSRUWH ÀXYLDO SHORV ULEHLULQKRV
GD$PD]{QLDJDQKRXXPDYHUVmRPRYLGDDHQHUJLDVRODU
HPYH]GHFRPEXVWtYHO
%5$6,/.9RDGHLUDPRYLGDDHQHUJLDVRODUpRSomRSDUDRWUDQVSRUWHÀXYLDOQD$PD]{QLD Madeira para
Folha de S. Paulo, 12 maio 2012. celulose e papel

1R WH[WR HVWi GHVFULWD XPD VLWXDomR GH PXGDQoD QD Quantidade produzida em 2006
(metros cúbicos)
WHFQRORJLDGRWUDQVSRUWHÀXYLDOQD$PD]{QLD&RQ¿JXUDVH
2 624 058
como uma consequência ambiental derivada da mudança
apresentada a redução 1 500 000

A da área de mata ciliar. 650 000

B da erosão dos solos aluviais. 170 000

C GHGHVFDUJDVHOpWULFDVQDViJXDV 2

D GRDVVRUHDPHQWRGRVFXUVRVÀXYLDLV 'LVSRQtYHOHPZZZIFWXQHVSEU$FHVVRHPDJR

E GDHPLVVmRGHSROXHQWHVDWPRVIpULFRV A distribuição espacial de madeira para papel e celulose


QR%UDVLOSRVVXLXPDHVWUDWpJLDORJtVWLFDTXHUHVXOWDQD
A UHJLmRSURGXWLYDFRQWtQXDGHSHU¿OOLWRUkQHR
B LQWHJUDomRLQWHUPRGDOHQWUH6XO6XGHVWHH1RUWHGRSDtV
C FRQVWUXomR GH HL[RV URGRYLiULRV HQWUH DV ]RQDV
produtoras.
D RUJDQL]DomR GD SURGXomR SUy[LPD jV iUHDV GH
escoamento.
E ORFDOL]DomRGRVHWRUQRVOLPLWHVGDVXQLGDGHVSROtWLFR
administrativas.

&+žGLD_&DGHUQR%5$1&23iJLQD
2014 *BRAN75SAB15*
QUESTÃO 44
8PDPDLRUGLVSRQLELOLGDGHGHFRPEXVWtYHOIyVVLOFRPRDFRQWHFHFRPDVFUHVFHQWHVSRVVLELOLGDGHVEUDVLOHLUDV
p IRQWH GH LPSRUWDQWHV SHUVSHFWLYDV HFRQ{PLFDV SDUD R SDtV$R PHVPR WHPSR SRUpP QXPD pSRFD GH SUHVVmR
PXQGLDO SRU DOLPHQWRV H ELRFRPEXVWtYHLV DV UHVHUYDV QDFLRQDLV GH iJXD GRFH R FOLPD IDYRUiYHO H R GRPtQLR GH
WHFQRORJLDVGHSRQWDQRVHWRUFRQIHUHPjPDWUL]HQHUJpWLFDEUDVLOHLUDXPSDSHOFKDYHQDPXGDQoDGRSDUDGLJPD
HQHUJpWLFRSURGXWLYR
SODRÉ, M. 5HLQYHQWDQGRDHGXFDomRGLYHUVLGDGHGHVFRORQL]DomRHUHGHV3HWUySROLV9R]HV

1RWH[WRpUHVVDOWDGDDLPSRUWkQFLDGDPDWUL]HQHUJpWLFDEUDVLOHLUDHQTXDQWRUHIHUrQFLDGHFDUiWHUPDLVVXVWHQWiYHO
(VVDLPSRUWkQFLDpGHULYDGDGD
A FRQTXLVWDGDDXWRVVX¿FLrQFLDSHWUROtIHUDSHODGHVFREHUWDGHQRYDVMD]LGDV
B H[SDQVmRGDIURQWHLUDDJUtFRODLQWHQVLYDSDUDSURGXomRGHELRFRPEXVWtYHLV
C superação do uso de energia não renovável no setor de transporte de cargas.
D DSURSULDomRGDVFRQGLo}HVQDWXUDLVGRWHUULWyULRSDUDGLYHUVL¿FDomRGDVIRQWHV
E UHGXomRGRLPSDFWRVRFLDODGYLQGRGDVXEVWLWXLomRGHWHUPHOpWULFDVSRUKLGUHOpWULFDV

QUESTÃO 45

Território e territorialização da produção de soja

LEGENDA

Território da soja

Territorialização da produção
de soja

Área plantada de soja em 2006

597 858

330 000 (hectares)


150 000 Atlas da questão agrária brasileira
38 000 100 0 200 400 600 km

1
Dados: IBGE - Produção Agrícola Municipal

GIRARDI, E. P. $WODVGDTXHVWmRDJUiULDEUDVLOHLUD'LVSRQtYHOHPZZZIFWXQHVSEU$FHVVRHPDJR DGDSWDGR 

$IRUPDomRGRWHUULWyULRGDVRMDQR%UDVLOUHÀHWLXDVHJXLQWHFDUDFWHUtVWLFDHVSDFLDO
A Inclusão de regiões com elevadas concentrações populacionais.
B ,QFRUSRUDomRGHHVSDoRVFRPEDL[DIHUWLOLGDGHQDWXUDOGRVVRORV
C ,QWHJUDomRFRPHVSDoRVGHFRQVROLGDomRGHUHVHUYDVH[WUDWLYLVWDV
D 1HFHVVLGDGHGHSUR[LPLGDGHItVLFDFRPRVSULQFLSDLVSRUWRVGRSDtV
E 5HXWLOL]DomRGHiUHDVSURGXWLYDVGHFDGHQWHVGDWUDGLFLRQDOFXOWXUDFDQDYLHLUD

&+žGLD_&DGHUQR%5$1&23iJLQD
*BRAN75SAB16* 2014

CIÊNCIAS DA NATUREZA QUESTÃO 48

E SUAS TECNOLOGIAS 8P SHVTXLVDGRU DYDOLRX R HIHLWR GD WHPSHUDWXUD GR


PRWRU HPYHORFLGDGHFRQVWDQWH HGDYHORFLGDGHPpGLD
Questões de 46 a 90 GH XP YHtFXOR FRP WHPSHUDWXUD GR PRWRU FRQVWDQWH 
VREUHDHPLVVmRGHPRQy[LGRGHFDUERQR &2 HPGRLV
QUESTÃO 46 tipos de percurso, aclive e declive, com iguais distâncias
percorridas em linha reta. Os resultados são apresentados
Se por um lado a Revolução Industrial instituiu QDVGXDV¿JXUDV
um novo patamar de tecnologia e, com isso, uma
melhoria na qualidade de vida da população, por outro 40
ODGR RV UHVtGXRV GHFRUUHQWHV GHVVH SURFHVVR SRGHP
se acumular no ar, no solo e na água, causando

Emissão de CO (g/km)
GHVHTXLOtEULRVQRDPELHQWH 30
2 DF~PXOR GRV UHVtGXRV SURYHQLHQWHV GRV SURFHVVRV
LQGXVWULDLV TXH XWLOL]DP FRPEXVWtYHLV IyVVHLV WUD] FRPR
consequência o(a) 20 Aclive
A HXWUR¿]DomR GRV FRUSRVG¶iJXD DXPHQWDQGR D
Declive
produtividade dos sistemas aquáticos.
B SUHFLSLWDomRGHFKXYDViFLGDVGDQL¿FDQGRÀRUHVWDV 10
ecossistemas aquáticos e construções.
C mudança na salinidade dos mares, provocando a
PRUWDOLGDGHGHSHL[HVHGHPDLVVHUHVDTXiWLFRV 0
30 40 50 60 70
D acúmulo de detritos, causando entupimento de
bueiros e alagamento das ruas. Temperatura do motor (°C)

E presença de mosquitos, levando à disseminação de


doenças bacterianas e virais. 40

QUESTÃO 47
Emissão de CO (g/km)

30
$VPLUDJHQVH[LVWHPHSRGHPLQGX]LUjSHUFHSomRGH
TXHKiiJXDRQGHQmRH[LVWH(ODVVmRDPDQLIHVWDomRGH
XPIHQ{PHQRySWLFRTXHRFRUUHQDDWPRVIHUD 20 Aclive
'LVSRQtYHOHPZZZLQYLYR¿RFUX]EU$FHVVRHPIHY
Declive
(VVHIHQ{PHQRySWLFRpFRQVHTXrQFLDGD
10
A UHIUDomR GD OX] QDV FDPDGDV GH DU SUy[LPDV GR
chão quente.
B UHÀH[mRGDOX]DRLQFLGLUQRVRORTXHQWH 0
C UHÀH[mRGLIXVDGDOX]QDVXSHUItFLHUXJRVD 40 50 60 70 80
D GLVSHUVmR GD OX] QDV FDPDGDV GH DU SUy[LPDV GR Velocidade média (km/h)
chão quente.
'LVSRQtYHOHPZZZSURGXFDRXIUJVEU$FHVVRHPDJR DGDSWDGR 
E GLIUDomR GD OX] QDV FDPDGDV GH DU SUy[LPDV GR
chão quente. A partir dos resultados, a situação em que ocorre maior
HPLVVmR GH SROXHQWHV p DTXHOD QD TXDO R SHUFXUVR p
IHLWRFRPRPRWRU
A DTXHFLGR HP PHQRUHV YHORFLGDGHV PpGLDV H HP
pista em declive.
B DTXHFLGRHPPDLRUHVYHORFLGDGHVPpGLDVHHPSLVWD
em aclive.
C IULR HP PHQRUHV YHORFLGDGHV PpGLDV H HP SLVWD
em declive.
D IULR HP PHQRUHV YHORFLGDGHV PpGLDV H HP SLVWD
em aclive.
E IULR HP PDLRUHV YHORFLGDGHV PpGLDV H HP SLVWD
em aclive.

CN - 1º dia | Caderno 3 - BRANCO - Página 16


2014 *BRAN75SAB17*
QUESTÃO 49
$FODVVL¿FDomRGRVVHUHVYLYRVSHUPLWHDFRPSUHHQVmRGDVUHODo}HVHYROXWLYDVHQWUHHOHV2HVTXHPDUHSUHVHQWD
a história evolutiva de um grupo.
peixes sem peixes peixes
mandíbulas cartilaginosos ósseos anfíbios répteis aves mamíferos

Tempo (milhões de anos atrás)


100

200

répteis primitivos
(extintos)
300

anfíbios
primitivos (extintos)
400

placodermos acantodianos
500 (extintos) (extintos)

ostracodermos
(extintos)

ancestral dos
cordados

'LVSRQtYHOHPZZZVRELRORJLDFRPEU$FHVVRHPMDQ DGDSWDGR 

2VDQLPDLVUHSUHVHQWDGRVQHVVHHVTXHPDSHUWHQFHPDR¿ORGRVFRUGDGRVSRUTXH
A SRVVXHPDQFHVWUDLVTXHMiIRUDPH[WLQWRV
B VXUJLUDPKiPDLVGHPLOK}HVGHDQRV
C HYROXtUDPDSDUWLUGHXPDQFHVWUDOFRPXP
D GHUDPRULJHPDRVJUXSRVGHPDPtIHURVDWXDLV
E YLYHPQRDPELHQWHDTXiWLFRHPDOJXPDIDVHGDYLGD
QUESTÃO 50
(P PXVHXV GH FLrQFLDV p FRPXP HQFRQWUDUHPVH PiTXLQDV TXH HOHWUL]DP PDWHULDLV H JHUDP LQWHQVDV
GHVFDUJDV HOpWULFDV 2 JHUDGRU GH 9DQ GH *UDDII )LJXUD   p XP H[HPSOR FRPR DWHVWDP DV IDtVFDV )LJXUD  
TXHHOHSURGX]2H[SHULPHQWR¿FDPDLVLQWHUHVVDQWHTXDQGRVHDSUR[LPDGRJHUDGRUHPIXQFLRQDPHQWRFRPD
PmRXPDOkPSDGDÀXRUHVFHQWH )LJXUD 4XDQGRDGHVFDUJDDWLQJHDOkPSDGDPHVPRGHVFRQHFWDGDGDUHGH
HOpWULFDHODEULOKDSRUEUHYHVLQVWDQWHV0XLWDVSHVVRDVSHQVDPTXHpRIDWRGHDGHVFDUJDDWLQJLUDOkPSDGDTXH
D ID] EULOKDU &RQWXGR VH D OkPSDGD IRU DSUR[LPDGD GRV FRUSRV GD VLWXDomR )LJXUD   QR PRPHQWR HP TXH D
GHVFDUJDRFRUUHUHQWUHHOHVDOkPSDGDWDPEpPEULOKDUiDSHVDUGHQmRUHFHEHUQHQKXPDGHVFDUJDHOpWULFD
Figura 1 Figura 2 Figura 3

*HUDGRUGH9DQGH*UDDII 'HVFDUJDHOpWULFDQRJHUDGRU /kPSDGDÀXRUHVFHQWH


'LVSRQtYHOHPKWWSQDYHDVWURFRP$FHVVRHPDJR

$ JUDQGH]D ItVLFD DVVRFLDGD DR EULOKR LQVWDQWkQHR GD OkPSDGD ÀXRUHVFHQWH SRU HVWDU SUy[LPD D XPD GHVFDUJD
HOpWULFDpR D
A FDUJDHOpWULFD
B FDPSRHOpWULFR
C FRUUHQWHHOpWULFD
D FDSDFLWkQFLDHOpWULFD
E FRQGXWLYLGDGHHOpWULFD
CN - 1º dia | Caderno 3 - BRANCO - Página 17
*BRAN75SAB18* 2014

QUESTÃO 51 QUESTÃO 53
3DUWtFXODV EHWD DR DWUDYHVVDUHP D PDWpULD YLYD 2VRQDUpXPHTXLSDPHQWRHOHWU{QLFRTXHSHUPLWHD
FROLGHPFRPXPDSHTXHQDSRUFHQWDJHPGHPROpFXODVH ORFDOL]DomRGHREMHWRVHDPHGLGDGHGLVWkQFLDVQRIXQGR
GHL[DPDWUiVGHVLXPUDVWURDOHDWRULDPHQWHSRQWLOKDGRGH GR PDU SHOD HPLVVmR GH VLQDLV V{QLFRV H XOWUDVV{QLFRV
UDGLFDLVOLYUHVHtRQVTXLPLFDPHQWHDWLYRV(VVDVHVSpFLHV HDUHFHSomRGRVUHVSHFWLYRVHFRV2IHQ{PHQRGRHFR
podem romper ainda outras ligações moleculares, FRUUHVSRQGH j UHÀH[mR GH XPD RQGD VRQRUD SRU XP
causando danos celulares. REMHWR D TXDO YROWD DR UHFHSWRU SRXFR WHPSR GHSRLV GH
HEWITT, P. G. Física conceitual3RUWR$OHJUH%RRNPDQ DGDSWDGR  o som ser emitido. No caso do ser humano, o ouvido
$FDSDFLGDGHGHJHUDURVHIHLWRVGHVFULWRVGiVHSRUTXH pFDSD]GHGLVWLQJXLUVRQVVHSDUDGRVSRUQRPtQLPR
WDOSDUWtFXODpXP 0,1 segundo.
A HOpWURQHSRUSRVVXLUPDVVDUHODWLYDGHVSUH]tYHOWHP Considerando uma condição em que a velocidade do som
HOHYDGDHQHUJLDFLQpWLFDWUDQVODFLRQDO QR DU p  PV TXDO p D GLVWkQFLD PtQLPD D TXH XPD
B QrXWURQ H SRU QmR SRVVXLU FDUJD HOpWULFD WHP DOWD SHVVRD GHYH HVWDU GH XP DQWHSDUR UHÀHWRU SDUD TXH VH
FDSDFLGDGHGHSURGX]LUUHDo}HVQXFOHDUHV SRVVDGLVWLQJXLURHFRGRVRPHPLWLGR"
C Q~FOHRGRiWRPRGHKpOLR +H HSRUSRVVXLUPDVVD A 17 m
elevada, tem grande poder de penetração.
B P
D IyWRQHSRUQmRSRVVXLUPDVVDWHPJUDQGHIDFLOLGDGH
GHLQGX]LUDIRUPDomRGHUDGLFDLVOLYUHV C 68 m
E núcleo do átomo de hidrogênio (H) e, por possuir D 1 700 m
FDUJDSRVLWLYDWHPDOWDUHDWLYLGDGHTXtPLFD E P
QUESTÃO 52
1RDQRGHUHJLVWURXVHXPVXUWRJOREDOGHJULSH
FDXVDGDSRUXPYDULDQWHGRYtUXV,QÀXHQ]D$, designada
+1$2UJDQL]DomR0XQGLDOGH6D~GH 206 VROLFLWRX
TXH RV SDtVHV LQWHQVL¿FDVVHP VHXV SURJUDPDV GH
prevenção para que não houvesse uma propagação da
doença. Uma das ações mais importantes recomendadas
SHOD 206 HUD D KLJLHQL]DomR DGHTXDGD GDV PmRV
especialmente após tossir e espirrar.
$DomRUHFRPHQGDGDSHOD206WLQKDFRPRREMHWLYR
A UHGX]LUDUHSURGXomRYLUDO
B LPSHGLUDSHQHWUDomRGRYtUXVSHODSHOH
C UHGX]LURSURFHVVRGHDXWRLQIHFomRYLUDO
D UHGX]LUDWUDQVPLVVmRGRYtUXVQRDPELHQWH
E LPSHGLUDVHOHomRQDWXUDOGHYtUXVUHVLVWHQWHV

CN - 1º dia | Caderno 3 - BRANCO - Página 18


2014 *BRAN75SAB19*
QUESTÃO 54

2V EDI{PHWURV HWLO{PHWURV  LQGLFDP D TXDQWLGDGH GH iOFRRO &2H6O (etanol), presente no organismo de uma
SHVVRDDWUDYpVGRDUH[SLUDGRSRUHOD(VVHVGLVSRVLWLYRVXWLOL]DPFpOXODVDFRPEXVWtYHOTXHIXQFLRQDPGHDFRUGRFRP
DVUHDo}HVTXtPLFDVUHSUHVHQWDGDV

(I) C2H6O J ĺ&2HO (g) + 2 H+ (aq) + 2 eï

(II) 1
O2 (g) + 2 H+ (aq) + 2 eïĺ+2O (l)
2
%5$$7+(13&+iOLWRFXOSDGRRSULQFtSLRTXtPLFRGREDI{PHWURQuímica Nova na EscolaQPDLR DGDSWDGR 

1DUHDomRJOREDOGHIXQFLRQDPHQWRGREDI{PHWURRVUHDJHQWHVHRVSURGXWRVGHVVHWLSRGHFpOXODVmR
A RiOFRROH[SLUDGRFRPRUHDJHQWHiJXDHOpWURQVH++ como produtos.
B RR[LJrQLRGRDUH++FRPRUHDJHQWHViJXDHHOpWURQVFRPRSURGXWRV
C DSHQDVRR[LJrQLRGRDUFRPRUHDJHQWHDSHQDVRVHOpWURQVFRPRSURGXWR
D DSHQDVRiOFRROH[SLUDGRFRPRUHDJHQWHiJXD&2HO e H+ como produtos.
E RR[LJrQLRGRDUHRiOFRROH[SLUDGRFRPRUHDJHQWHViJXDH&2HO como produtos.

QUESTÃO 55

2VJrPHRVVHPSUHH[HUFHUDPXPIDVFtQLRSDUDDPDLRULDGDVSHVVRDVSULQFLSDOPHQWHRVPRQR]LJyWLFRVRX
LGrQWLFRV3DUWHGHVVHLQWHUHVVHHVWiUHODFLRQDGDDRIDWRGHTXHHVVHVLQGLYtGXRVUHSUHVHQWDPDPDQLIHVWDomR
QDWXUDOTXHPDLVVHDSUR[LPDGDFORQDJHPQDHVSpFLHKXPDQD
2PHFDQLVPRTXHHVWiDVVRFLDGRFRPDIRUPDomRGRVLQGLYtGXRVFLWDGRVpD
A GLYLVmRGRIHWRHPJHVWDomRHPGRLVLQGLYtGXRVVHSDUDGRV
B divisão do embrião em dois grupos celulares independentes.
C IHFXQGDomRGHXPyYXORSRUGRLVHVSHUPDWR]RLGHVGLIHUHQWHV
D RFRUUrQFLDGHGXDVIHFXQGDo}HVVLPXOWkQHDVQR~WHURPDWHUQR
E IHUWLOL]DomRVXFHVVLYDGHGRLVyYXORVSRUDSHQDVXPHVSHUPDWR]RLGH

QUESTÃO 56

8PHQJHQKHLURGHFLGLXLQVWDODUXPDTXHFHGRUVRODUHPVXDFDVDFRQIRUPHPRVWUDRHVTXHPD

CAIXA
FRIA
COLETOR
SOLAR
BOILER

'HDFRUGRFRPDVLQVWUXo}HVGHPRQWDJHPSDUDVHWHUXPDSURYHLWDPHQWRPi[LPRGDLQFLGrQFLDVRODUDVSODFDV
do coletor solar devem ser instaladas com um ângulo de inclinação determinado.
2SDUkPHWURTXHGH¿QHRYDORUGRkQJXORGHLQFOLQDomRGHVVDVSODFDVFROHWRUDVpD
A altitude.
B latitude.
C longitude.
D nebulosidade.
E umidade relativa do ar.
&1žGLD_&DGHUQR%5$1&23iJLQD
*BRAN75SAB20* 2014

QUESTÃO 57 QUESTÃO 59

$ iJXD SRWiYHO SUHFLVD VHU OtPSLGD RX VHMD QmR Para a proteção contra curtos-circuitos em residências
GHYH FRQWHU SDUWtFXODV HP VXVSHQVmR WDLV FRPR WHUUD VmR XWLOL]DGRV GLVMXQWRUHV FRPSRVWRV SRU GXDV OkPLQDV
GHPHWDLVGLIHUHQWHVFRPVXDVVXSHUItFLHVVROGDGDVXPD
ou restos de plantas, comuns nas águas de rios e
j RXWUD RX VHMD XPD OkPLQD ELPHWiOLFD (VVD OkPLQD
ODJRDV$ UHPRomR GDV SDUWtFXODV p IHLWD HP HVWDo}HV
WRFDRFRQWDWRHOpWULFRIHFKDQGRRFLUFXLWRHGHL[DQGRD
de tratamento, onde Ca(OH)2 HP H[FHVVR H$O2(SO)3
FRUUHQWH HOpWULFD SDVVDU 4XDQGR GD SDVVDJHP GH XPD
VmRDGLFLRQDGRVHPXPWDQTXHSDUDIRUPDUVXOIDWRGH corrente superior à estipulada (limite), a lâmina se curva
FiOFLR H KLGUy[LGR GH DOXPtQLR (VVH ~OWLPR VH IRUPD SDUD XP GRV ODGRV DIDVWDQGRVH GR FRQWDWR HOpWULFR H
FRPR ÀRFRV JHODWLQRVRV LQVRO~YHLV HP iJXD TXH assim, interrompendo o circuito. Isso ocorre porque os
VmR FDSD]HV GH DJUHJDU SDUWtFXODV HP VXVSHQVmR PHWDLVGDOkPLQDSRVVXHPXPDFDUDFWHUtVWLFDItVLFDFXMD
Em uma estação de tratamento, cada 10 gramas de UHVSRVWDpGLIHUHQWHSDUDDPHVPDFRUUHQWHHOpWULFDTXH
KLGUy[LGRGHDOXPtQLRpFDSD]GHFDUUHJDUJUDPDVGH passa no circuito.
SDUWtFXODV$SyVGHFDQWDomRH¿OWUDomRDiJXDOtPSLGD $ FDUDFWHUtVWLFD ItVLFD TXH GHYH VHU REVHUYDGD SDUD D
pWUDWDGDFRPFORURHGLVWULEXtGDSDUDDVUHVLGrQFLDV$V HVFROKD GRV GRLV PHWDLV GHVVD OkPLQD ELPHWiOLFD p R
massas molares dos elementos H, O, Al, S e Ca são, FRH¿FLHQWHGH
UHVSHFWLYDPHQWHJPROJPROJPROJPRO
A GXUH]D
HJPRO
B elasticidade.
Considerando que 1 000 litros da água de um rio possuem C GLODWDomRWpUPLFD
 JUDPDV GH SDUWtFXODV HP VXVSHQVmR D TXDQWLGDGH
D compressibilidade.
PtQLPDGH$O2(SO)3TXHGHYHVHUXWLOL]DGDQDHVWDomRGH
E FRQGXWLYLGDGHHOpWULFD
WUDWDPHQWRGHiJXDFDSD]GHWUDWDUOLWURVGHiJXD
GHXPDVyYH]SDUDJDUDQWLUTXHWRGDVDVSDUWtFXODVHP
VXVSHQVmRVHMDPSUHFLSLWDGDVpPDLVSUy[LPDGH
A J
B J
C J
D J
E J

QUESTÃO 58

1D WpFQLFD GH SODQWLR FRQKHFLGD SRU KLGURSRQLD RV


vegetais são cultivados em uma solução de nutrientes no
lugar do solo, rica em nitrato e ureia.
1HVVH FDVR DR IRUQHFHU HVVHV QXWULHQWHV QD IRUPD
DSURYHLWiYHOSHODSODQWDDWpFQLFDGLVSHQVDRWUDEDOKRGDV
EDFWpULDV¿[DGRUDVGRVRORTXHQDQDWXUH]DSDUWLFLSDP
do ciclo do(a)
A água.
B carbono.
C nitrogênio.
D R[LJrQLR
E IyVIRUR

CN - 1º dia | Caderno 3 - BRANCO - Página 20


2014 *BRAN75SAB21*
QUESTÃO 60 QUESTÃO 61
)ROKDV GH SDSHO FRPR DV XWLOL]DGDV SDUD D $R VH FDUDFWHUL]DUHP RV DVSHFWRV DPELHQWDLV GR
impressão de documentos, são opacas e permeáveis VHWRUVXFURDOFRROHLURpSUHFLVRDQDOLVDUGRLVVHWRUHVR
DRVOtTXLGRV(VVHPDWHULDOpFRQVWLWXtGRGHPLFUR¿EUDV VHWRUDJUtFRODTXHVHUHIHUHjVDWLYLGDGHVGHVHQYROYLGDV
entrelaçadas de celulose, que são transparentes à na área que a cultura da cana-de-açúcar ocupa, e o setor
OX] 4XDQGR VREUH HODV VH GHUUDPD JOLFHULQD HODV VH LQGXVWULDOTXHHVWiUHODFLRQDGRjIDEULFDomR GHDo~FDU
WRUQDPWUDQVO~FLGDV8PDLPDJHPGDVXSHUItFLHGHXPD e álcool.
IROKDGHSDSHODPSOLDGDSRUXPPLFURVFySLRHOHWU{QLFR ALVARENGA, R. P.; QUEIROZ, T. R. 3URGXomRPDLVOLPSDHDVSHFWRVDPELHQWDLVQD
GH YDUUHGXUD SRGH VHU YLVWD QD ¿JXUD 1R TXDGUR p indústria sucroalcooleira'LVSRQtYHOHPZZZDGYDQFHVLQFOHDQHUSURGXFWLRQQHW
Acesso em: 3 ago. 2012 (adaptado).
DSUHVHQWDGD D UD]mR n  HQWUH D YHORFLGDGH GD OX] QR
vácuo e no respectivo material (celulose, glicerina ou ar). Para essa atividade produtiva, como impacto ambiental
causado pelo setor industrial, tem-se o(a)
A compactação do solo.
B assoreamento dos rios.
C desmatamento de áreas.
D queima da cana-de-açúcar.
E JHUDomRGHUHVtGXRVSROXLGRUHV

QUESTÃO 62
2VFRUDLVIXQFLRQDPFRPRWHUP{PHWURVFDSD]HVGH
indicar, mudando de coloração, pequenas alterações na
temperatura da água dos oceanos. Mas, um alerta, eles
HVWmR ¿FDQGR EUDQFRV 2 VHX FODUHDPHQWR SURJUHVVLYR
acontece pela perda de minúsculas algas, chamadas
]RR[DQWHODV TXH YLYHP GHQWUR GH VHXV WHFLGRV QXPD
relação de mutualismo.
'LVSRQtYHOHPKWWSVXSHUDEULOFRPEU$FHVVRHPGH] DGDSWDGR 

2GHVHTXLOtEULRGHVVDUHODomRID]FRPTXHRVSyOLSRVTXH
IRUPDPRVFRUDLVWHQKDPGL¿FXOGDGHHP
A SURGX]LURSUySULRDOLPHQWR
B obter compostos nitrogenados.
Material n C UHDOL]DUDUHSURGXomRVH[XDGD
celulose  D DEVRUYHURR[LJrQLRGLVVROYLGRQDiJXD
E DGTXLULUQXWULHQWHVGHULYDGRVGDIRWRVVtQWHVH
glicerina 
ar 1,00

Nessa situação, o papel se tornou translúcido porque a


OX]p
A PDLVUHÀHWLGD
B mais absorvida.
C mais espalhada.
D PHQRVUHIUDWDGD
E menos transmitida.

CN - 1º dia | Caderno 3 - BRANCO - Página 21


*BRAN75SAB22* 2014

QUESTÃO 63 QUESTÃO 65
(P XP FDVR GH DQHPLD D TXDQWLGDGH GH VXOIDWR Ao assistir a uma apresentação musical, um músico
GH IHUUR ,,  )H62 PDVVD PRODU LJXDO D  JPRO  que estava na plateia percebeu que conseguia ouvir
UHFRPHQGDGDFRPRVXSOHPHQWRGHIHUURIRLGHPJGLD TXDVH SHUIHLWDPHQWH R VRP GD EDQGD SHUGHQGR XP
Acima desse valor, a mucosa intestinal atua como barreira, SRXFRGHQLWLGH]QDVQRWDVPDLVDJXGDV(OHYHULILFRX
LPSHGLQGR D DEVRUomR GH IHUUR )RUDP DQDOLVDGRV FLQFR
TXHKDYLDPXLWDVSHVVRDVEHPPDLVDOWDVjVXDIUHQWH
IUDVFRV GH VXSOHPHQWR FRQWHQGR VROXomR DTXRVD GH
bloqueando a visão direta do palco e o acesso aos
FeSOFXMRVUHVXOWDGRVHQFRQWUDPVHQRTXDGUR
DOWRIDODQWHV6DEHVHTXHDYHORFLGDGHGRVRPQRDU
&RQFHQWUDomRGHVXOIDWRGHIHUUR ,,  p  PV H TXH D UHJLmR GH IUHTXrQFLDV GDV QRWDV
Frasco HPLWLGDVpGHDSUR[LPDGDPHQWH+]D+]
PRO/
1 0,02 4XDOIHQ{PHQRRQGXODWyULRpRSULQFLSDOUHVSRQViYHO
2 0,20 SDUD TXH R P~VLFR SHUFHEHVVH HVVD GLIHUHQFLDomR
GRVRP"
3 0,30
  A 'LIUDomR
  B 5HÀH[mR
C 5HIUDomR
6HIRULQJHULGDXPDFROKHU P/ SRUGLDGRPHGLFDPHQWR
para anemia, a amostra que conterá a concentração de D Atenuação.
VXOIDWRGHIHUUR ,, PDLVSUy[LPDGDUHFRPHQGDGDpDGR E ,QWHUIHUrQFLD
IUDVFRGHQ~PHUR
A 1. QUESTÃO 66
B 2. 2iOFRROFRPHUFLDO VROXomRGHHWDQRO pYHQGLGRQD
C 3. FRQFHQWUDomRGHHPYROXPH(QWUHWDQWRSDUDTXH
D  SRVVDVHUXWLOL]DGRFRPRGHVLQIHWDQWHGHYHVHXVDUXPD
E  VROXomR DOFRyOLFD QD FRQFHQWUDomR GH  HP YROXPH
Suponha que um hospital recebeu como doação um lote
QUESTÃO 64 GHOLWURVGHiOFRROFRPHUFLDODHPYROXPHH
SUHWHQGHWURFiORSRUXPORWHGHiOFRROGHVLQIHWDQWH
Estranha neve:
espuma, espuma apenas 3DUDTXHDTXDQWLGDGHWRWDOGHHWDQROVHMDDPHVPDQRV
que o vento espalha, bolha em baile no ar, GRLVORWHVRYROXPHGHiOFRRODIRUQHFLGRQDWURFD
GHYHVHUPDLVSUy[LPRGH
vinda do Tietê alvoroçado ao abrir de comportas,
HVSXPDGHGRGHFLOEHQ]HQRLUUHGXWtYHO A /
HPHUJLQGRGDViJXDVSURIDQDGDVGRULREDQGHLUDQWH B 1 371 L.
KRMHULRGHVSHMR C /
GHPLOLPXQGtFLHVGRSURJUHVVR D 1 632 L.
ANDRADE, C. D. Poesia e prosa5LRGH-DQHLUR1RYD$JXLODU IUDJPHQWR  E 1 700 L.
1HVVHSRHPDRDXWRUID]UHIHUrQFLDj
A disseminação de doenças nas áreas atingidas por
inundações.
B FRQWDPLQDomR GR OHQoRO IUHiWLFR SHOD HOLPLQDomR GH
OL[RQRVULRV
C ocorrência de enchente causada pela
LPSHUPHDELOL]DomRGRVVRORV
D SUHVHQoD GH GHWHUJHQWHV VLQWpWLFRV FRPR DJHQWHV
poluentes de águas.
E GHVWUXLomR GH IDXQD H ÀRUD SHOD FRQWDPLQDomR GH
EDFLDVKLGURJUi¿FDV

CN - 1º dia | Caderno 3 - BRANCO - Página 22


2014 *BRAN75SAB23*
QUESTÃO 67 QUESTÃO 69
Na Antiguidade, algumas pessoas acreditavam que, O movimento pelo saneamento do Brasil,
QR ODQoDPHQWR REOtTXR GH XP REMHWR D UHVXOWDQWH GDV desencadeado durante a Primeira República, colocou
IRUoDVTXHDWXDYDPVREUHHOHWLQKDRPHVPRVHQWLGRGD em evidência as precárias condições de saúde das
velocidade em todos os instantes do movimento. Isso SRSXODo}HVUXUDLV$RULJHPHWUDMHWyULDGHVVHPRYLPHQWR
QmR HVWi GH DFRUGR FRP DV LQWHUSUHWDo}HV FLHQWt¿FDV estiveram diretamente relacionadas à história da doença
DWXDOPHQWHXWLOL]DGDVSDUDH[SOLFDUHVVHIHQ{PHQR de Chagas.
'HVSUH]DQGR D UHVLVWrQFLD GR DU TXDO p D GLUHomR H R .523)63/,0$17'LVSRQtYHOHPZZZ¿RFUX]EU$FHVVRHPDJR DGDSWDGR 
VHQWLGRGRYHWRUIRUoDUHVXOWDQWHTXHDWXDVREUHRREMHWR
QRSRQWRPDLVDOWRGDWUDMHWyULD" $LQWHUYHQomRDPELHQWDOFRQVLGHUDGDIXQGDPHQWDOSDUDD
SUHYHQomRGHVVDGRHQoDpD
A ,QGH¿QLGRSRLVHOHpQXORDVVLPFRPRDYHORFLGDGH
vertical nesse ponto. A OLPSH]DGHWHUUHQRVEDOGLRVFRPDUHWLUDGDGHPDWpULD
B 9HUWLFDOSDUDEDL[RSRLVVRPHQWHRSHVRHVWiSUHVHQWH orgânica em decomposição.
durante o movimento. B construção de unidades de saúde, com atendimento
C +RUL]RQWDO QR VHQWLGR GR PRYLPHQWR SRLV GHYLGR j PDLVH¿FLHQWHDRVLQGLYtGXRVLQIHFWDGRV
LQpUFLDRREMHWRPDQWpPVHXPRYLPHQWR C melhoria das condições de habitação, com redução
D ,QFOLQDGR QD GLUHomR GR ODQoDPHQWR SRLV D IRUoD de insetos no ambiente domiciliar e peridomiciliar.
LQLFLDOTXHDWXDVREUHRREMHWRpFRQVWDQWH D construção de estradas e rodovias, com garantias
E ,QFOLQDGRSDUDEDL[RHQRVHQWLGRGRPRYLPHQWRSRLV de melhor acesso da população rural ao sistema
DSRQWDSDUDRSRQWRRQGHRREMHWRFDLUi de saúde.
QUESTÃO 68 E OLPSH]D GR DPELHQWH GRPLFLOLDU H SHULGRPLFLOLDU
com retirada de entulhos e recipientes que possam
$QWHV GH WpFQLFDV PRGHUQDV GH GHWHUPLQDomR acumular água.
GH SDWHUQLGDGH SRU H[DPH GH '1$ R VLVWHPD GH
GHWHUPLQDomR VDQJXtQHD $%2 IRL DPSODPHQWH XWLOL]DGR
FRPR IHUUDPHQWD SDUD H[FOXLU SRVVtYHLV SDLV (PERUD
UHVWULWR j DQiOLVH IHQRWtSLFD HUD SRVVtYHO FRQFOXLU D
H[FOXVmR GH JHQyWLSRV WDPEpP &RQVLGHUH TXH XPD
PXOKHU WHYH XP ¿OKR FXMD SDWHUQLGDGH HVWDYD VHQGR
contestada. A análise do sangue revelou que ela era tipo
VDQJXtQHR$%HR¿OKRWLSRVDQJXtQHR%
2 JHQyWLSR GR KRPHP SHOR VLVWHPD$%2 TXH H[FOXL D
SRVVLELOLGDGHGHSDWHUQLGDGHGHVVH¿OKRp
A IAIA.
B IAi.
C IBIB.
D IBi.
E ii.

CN - 1º dia | Caderno 3 - BRANCO - Página 23


*BRAN75SAB24* 2014

QUESTÃO 70 QUESTÃO 71

ÈJXD GXUD p DTXHOD TXH FRQWpP FRQFHQWUDo}HV O Brasil tem investido em inovações tecnológicas para
UHODWLYDPHQWH DOWDV GH tRQV &D2+ e Mg2+ dissolvidos. D SURGXomR H FRPHUFLDOL]DomR GH PDomV 8P H[HPSOR
p D DSOLFDomR GR FRPSRVWR YROiWLO PHWLOFLFORSURSHQR
$SHVDU GH HVVHV tRQV QmR UHSUHVHQWDUHP ULVFR SDUD D TXHFRPSHWHSHORVVtWLRVGHOLJDomRGRKRUP{QLRYHJHWDO
saúde, eles podem tornar a água imprópria para alguns HWLOHQRQDVFpOXODVGHVVHIUXWR
WLSRV GH FRQVXPR GRPpVWLFR RX LQGXVWULDO 2EMHWLYDQGR 'LVSRQtYHOHPKWWSUHYLVWDVHOHWURQLFDVSXFUVEU$FHVVRHPDJR DGDSWDGR 
UHGX]LU D FRQFHQWUDomR GH tRQV &D2+ e Mg2+ de uma
&RP EDVH QRV FRQKHFLPHQWRV VREUH R HIHLWR GHVVH
DPRVWUDGHiJXDGXUDDRPtQLPRSRVVtYHOXPWpFQLFRHP KRUP{QLRRPHWLOFLFORSURSHQRDJHUHWDUGDQGRR D 
TXtPLFDWHVWRXRVVHJXLQWHVSURFHGLPHQWRVQRODERUDWyULR
A IRUPDomRGRIUXWR
I – Decantação da amostra de água. B FUHVFLPHQWRGRIUXWR
II – Filtração da amostra de água. C DPDGXUHFLPHQWRGRIUXWR
D germinação das sementes.
III – Aquecimento da amostra de água.
E IRUPDomRGHVHPHQWHVQRIUXWR
IV – Adição do solvente orgânico CCl à amostra de
QUESTÃO 72
água.
6XUWVH\ p XPD LOKD YXOFkQLFD VLWXDGD SHUWR GD FRVWD
V – Adição de CaO e Na2CO3 à amostra de água.
sul da Islândia. A erupção vulcânica que lhe deu origem
BROWN, T. L. et al. Química, a ciência central6mR3DXOR3HDUVRQ3UHQWLFH+DOO DGDSWDGR  RFRUUHXQDGpFDGDGHRTXHID]GHODVHJXUDPHQWHD
LOKDPDLVQRYDGR2FHDQR$WOkQWLFR$VSULPHLUDVHVSpFLHV
2 PpWRGR FRQVLGHUDGR YLiYHO SDUD WUDWDU D iJXD GXUD H TXHDtVH¿[DUDPIRUDPPXVJRVHOLTXHQV¬PHGLGDTXH
DXPHQWDUVHXSRWHQFLDOGHXWLOL]DomRpR D DV DYHV IRUDP ¿[DQGRVH QD LOKD DV FRQGLo}HV GR VROR
IRUDP PHOKRUDQGR H HVSpFLHV YHJHWDLV PDLV FRPSOH[DV
A GHFDQWDomRSRLVSHUPLWHTXHHVVHVtRQVVHGHSRVLWHP SXGHUDP LQLFLDU D FRORQL]DomR GR WHUULWyULR (P  IRL
QRIXQGRGRUHFLSLHQWH observada a presença do primeiro arbusto.
B ¿OWUDomRSRLVDVVLPRVtRQV&D2+ e Mg2+ são retidos 'LVSRQtYHOHPZZZQDFRSDGDVDUYRUHVEORJVSRWFRPEU$FHVVRHPPDLR IUDJPHQWR 
QR¿OWURHVHSDUDGRVGDiJXD 2FRQMXQWRGDVDOWHUDo}HVRFRUULGDVQRDPELHQWHGHVFULWR
C aquecimento da amostra de água, para que esses pH[HPSORGH
tRQVVHMDPHYDSRUDGRVHVHSDUDGRV A nicho ecológico.
D adição do solvente orgânico CCl à amostra, para B H¿FLrQFLDHFROyJLFD
VROXELOL]DUHVVHVtRQVHVHSDUiORVGDiJXD C sucessão ecológica.
E UHDomR TXtPLFD FRP &D2 H 1D2CO3, para precipitar D irradiação adaptativa.
HVVHVtRQVQDIRUPDGHFRPSRVWRVLQVRO~YHLV E resistência ambiental.

&1žGLD_&DGHUQR%5$1&23iJLQD
2014 *BRAN75SAB25*
QUESTÃO 73 QUESTÃO 74
5HFHQWHPHQWH IRUDP REWLGRV RV ¿RV GH FREUH 2VPDQXDLVGRVIRUQRVPLFURRQGDVGHVDFRQVHOKDP
PDLV ¿QRV SRVVtYHLV FRQWHQGR DSHQDV XP iWRPR GH VRESHQDGHSHUGDGDJDUDQWLDTXHHOHVVHMDPOLJDGRVHP
HVSHVVXUD TXH SRGHP IXWXUDPHQWH VHU XWLOL]DGRV HP SDUDOHORMXQWDPHQWHDRXWURVDSDUHOKRVHOHWURGRPpVWLFRV
PLFURSURFHVVDGRUHV2FKDPDGRQDQR¿RUHSUHVHQWDGR por meio de tomadas múltiplas, popularmente conhecidas
QD¿JXUDSRGHVHUDSUR[LPDGRSRUXPSHTXHQRFLOLQGUR FRPR³EHQMDPLQV´RX³WrV´GHYLGRDRDOWRULVFRGHLQFrQGLR
GH FRPSULPHQWR  QP  QP  ï m). A seção reta e derretimento dessas tomadas, bem como daquelas dos
GH XP iWRPR GH FREUH p  QP2 e a resistividade próprios aparelhos.
GR FREUH p  ȍ˜nm. Um engenheiro precisa estimar Os riscos citados são decorrentes da
VH VHULD SRVVtYHO LQWURGX]LU HVVHV QDQR¿RV QRV
A UHVLVWLYLGDGH GD FRQH[mR TXH GLPLQXL GHYLGR j
microprocessadores atuais.
variação de temperatura do circuito.
B FRUUHQWH HOpWULFD VXSHULRU DR Pi[LPR TXH D WRPDGD
múltipla pode suportar.
C UHVLVWrQFLD HOpWULFD HOHYDGD QD FRQH[mR VLPXOWkQHD
GHDSDUHOKRVHOHWURGRPpVWLFRV
D WHQVmR LQVX¿FLHQWH SDUD PDQWHU WRGRV RV DSDUHOKRV
HOHWURGRPpVWLFRVHPIXQFLRQDPHQWR
E LQWHQVLGDGHGRFDPSRHOpWULFRHOHYDGDTXHFDXVDR
URPSLPHQWRGDULJLGH]GLHOpWULFDGDWRPDGDP~OWLSOD
$025,0(306,/9$(=$ELQLWLRVWXG\RIOLQHDUDWRPLFFKDLQVLQFRSSHUQDQRZLUHV
Physical Review B, v. 81, 2010 (adaptado). QUESTÃO 75
8PQDQR¿RXWLOL]DQGRDVDSUR[LPDo}HVSURSRVWDVSRVVXL 8PPpWRGRSDUDGHWHUPLQDomRGRWHRUGHHWDQROQD
UHVLVWrQFLDHOpWULFDGH gasolina consiste em misturar volumes conhecidos de
A 170 nȍ. iJXDHGHJDVROLQDHPXPIUDVFRHVSHFt¿FR$SyVDJLWDU
RIUDVFRHDJXDUGDUXPSHUtRGRGHWHPSRPHGHPVHRV
B 0,17 ȍ. YROXPHVGDVGXDVIDVHVLPLVFtYHLVTXHVmRREWLGDVXPD
C 1,7 ȍ. RUJkQLFDHRXWUDDTXRVD2HWDQRODQWHVPLVFtYHOFRPD
D 17 ȍ. JDVROLQDHQFRQWUDVHDJRUDPLVFtYHOFRPDiJXD
E 170 ȍ. 3DUDH[SOLFDURFRPSRUWDPHQWRGRHWDQRODQWHVHGHSRLV
GDDGLomRGHiJXDpQHFHVViULRFRQKHFHU
A DGHQVLGDGHGRVOtTXLGRV
B RWDPDQKRGDVPROpFXODV
C RSRQWRGHHEXOLomRGRVOtTXLGRV
D RViWRPRVSUHVHQWHVQDVPROpFXODV
E RWLSRGHLQWHUDomRHQWUHDVPROpFXODV

&1žGLD_&DGHUQR%5$1&23iJLQD
*BRAN75SAB26* 2014

QUESTÃO 76 QUESTÃO 78
No Brasil e no mundo têm surgido movimentos A tabela lista os valores de pH de algumas bebidas
e leis para banir o uso de sacolas plásticas, em consumidas pela população.
VXSHUPHUFDGRVIHLWDVGHSROLHWLOHQR2EWLGDDSDUWLUGR
SHWUyOHRDPDWpULDSULPDGRSROLHWLOHQRpRJiVHWLOHQR Bebida pH
TXH GHSRLV GH SROLPHUL]DGR Gi RULJHP DR SOiVWLFR 5HIULJHUDQWH 
FRPSRVWR HVVHQFLDOPHQWH IRUPDGR SHOD UHSHWLomR GH &DIp 3,0
JUXSRV ²&+2² 2 SULQFLSDO PRWLYR GR EDQLPHQWR p D
poluição, pois se estima que as sacolas levam cerca Vinho 
de 300 anos para se degradarem no meio ambiente, Suco de limão 
VHQGR UHVLVWHQWHV D DWDTXHV TXtPLFRV j UDGLDomR H D
Chá 6,0
microrganismos.
O motivo pelo qual essas sacolas demoram muito tempo 2HVPDOWHGRVGHQWHVpFRQVWLWXtGRGHKLGUR[LDSDWLWD
SDUDVHGHJUDGDUHPpTXHVXDVPROpFXODV (Ca 
(PO)32+  XP PLQHUDO TXH VRIUH GHVPLQHUDOL]DomR
HPPHLRiFLGRGHDFRUGRFRPDHTXDomRTXtPLFD
A apresentam muitas insaturações.
B contêm carbono em sua composição.
Ca(PO)3OH (s) &D2+ (aq) + 3 POí (aq) + OHí (aq)
C VmRIRUPDGDVSRUHOHPHQWRVGHDOWDPDVVDDW{PLFD
D VmR PXLWR ORQJDV H IRUPDGDV SRU OLJDo}HV TXtPLFDV Das bebidas listadas na tabela, aquela com menor
IRUWHV
SRWHQFLDOGHGHVPLQHUDOL]DomRGRVGHQWHVpR
E WrPRULJHPQRSHWUyOHRTXHpXPDPDWpULDSULPDQmR
renovável. A chá.
B FDIp
QUESTÃO 77
C vinho.
)XVtYHLVVmRGLVSRVLWLYRVGHSURWHomRGHXPFLUFXLWR D UHIULJHUDQWH
HOpWULFR VHQVtYHLV DR H[FHVVR GH FRUUHQWH HOpWULFD
2V PRGHORV PDLV VLPSOHV FRQVLVWHP GH XP ¿ODPHQWR E suco de limão.
PHWiOLFRGHEDL[RSRQWRGHIXVmRTXHVHIXQGHTXDQGR
a corrente ultrapassa determinado valor, evitando que
DVGHPDLVSDUWHVGRFLUFXLWRVHMDPGDQL¿FDGDV$¿JXUD
PRVWUD XP GLDJUDPD GH XP FLUFXLWR HP TXH R IXVtYHO F
protege um resistor RGHȍXPDOkPSDGDL de 6 W e
XPDOWRIDODQWHTXHFRQGX]$
F
12 V
R 12 Ω L 6W 1A

6DEHQGR TXH HVVH IXVtYHO IRL SURMHWDGR SDUD WUDEDOKDU


FRPXPDFRUUHQWHDWpPDLRUTXHDFRUUHQWHQRPLQDO
TXHDWUDYHVVDHVVHFLUFXLWRTXDOpRYDORUHPDPSqUHV
GDFRUUHQWHPi[LPDTXHRIXVtYHOFSHUPLWHSDVVDU"
A 1,0
B 
C 2,0
D 
E 3,0

CN - 1º dia | Caderno 3 - BRANCO - Página 26


2014 *BRAN75SAB27*
QUESTÃO 79 QUESTÃO 80
Durante um reparo na estação espacial internacional, 2 DUUR]GRXUDGR p XPD SODQWD WUDQVJrQLFD FDSD]
XPFRVPRQDXWDGHPDVVDNJVXEVWLWXLXPDERPED GHSURGX]LUTXDQWLGDGHVVLJQL¿FDWLYDVGHEHWDFDURWHQR
GRVLVWHPDGHUHIULJHUDomRGHPDVVDNJTXHHVWDYD TXHpDXVHQWHQDYDULHGDGHEUDQFD$SUHVHQoD GHVVD
GDQL¿FDGD ,QLFLDOPHQWH R FRVPRQDXWD H D ERPED VXEVWkQFLD WRUQD RV JUmRV DPDUHODGRV R TXH MXVWL¿FD
estão em repouso em relação à estação. Quando ele seu nome.
HPSXUUD D ERPED SDUD R HVSDoR HOH p HPSXUUDGR QR
$ LQJHVWmR GHVVD YDULHGDGH JHQHWLFDPHQWH PRGL¿FDGD
sentido oposto. Nesse processo, a bomba adquire uma
está relacionada à redução da incidência de
YHORFLGDGHGHPVHPUHODomRjHVWDomR
A IUDJLOLGDGHyVVHD
4XDO p R YDORU GD YHORFLGDGH HVFDODU DGTXLULGD SHOR
FRVPRQDXWDHPUHODomRjHVWDomRDSyVRHPSXUUmR" B IUDTXH]DPXVFXODU
C problemas de visão.
A PV
D alterações na tireoide.
B PV
E sangramento gengival.
C PV
D PV QUESTÃO 81
E PV
Determinado bioma brasileiro apresenta vegetação
FRQKHFLGDSRUSHUGHUDVIROKDVH¿FDUDSHQDVFRPJDOKRV
HVEUDQTXLoDGRVDRSDVVDUSRUDWpQRYHPHVHVGHVHFD
$V SODQWDV SRGHP DFXPXODU iJXD QR FDXOH H QD UDL]
DOpPGHDSUHVHQWDUHPIROKDVSHTXHQDVTXHHPDOJXPDV
HVSpFLHVDVVXPHPDIRUPDGHHVSLQKRV
4XDO UHJLmR ¿WRJHRJUi¿FD EUDVLOHLUD DSUHVHQWD SODQWDV
FRPHVVDVFDUDFWHUtVWLFDV"
A Cerrado.
B Pantanal.
C Caatinga.
D Mata Atlântica.
E )ORUHVWD$PD]{QLFD

CN - 1º dia | Caderno 3 - BRANCO - Página 27


*BRAN75SAB28* 2014

QUESTÃO 82
$¿JXUDDSUHVHQWDDFRPSDUDomRGRVJDVWRVGHWUrVWLSRVGHOkPSDGDVUHVLGHQFLDLVGHPHVPREULOKRGXUDQWH
FLQFRDQRV&RQVLGHUDVHDXWLOL]DomRPpGLDGHYLQWHSRQWRVGHOX]XWLOL]DQGRHPPpGLDGH]OkPSDGDVDFHVDVGXUDQWH
KRUDVDRFXVWRGH5SDUDFDGDN:KFRQVXPLGR

Incandescente Fluorescente LED


compacta
Investimento na compra R$ 60,00 R$ 360,00 R$ 2 800,00
Potência média de cada lâmpada 60 W 16 W 8W
Consumo de energia 6 480 kWh 1 728 kWh 864 kWh
Lâmpadas trocadas 110 20 Zero
Gasto com energia R$ 1 944,00 R$ 518,40 R$ 259,20
Gasto com lâmpadas trocadas R$ 330,00 R$ 360,00 Zero

Ano-base = 360 dias


'LVSRQtYHOHPKWWSSODQHWDVXVWHQWDYHODEULOFRPEU$FHVVRHPMXO DGDSWDGR 

&RPEDVHQDVLQIRUPDo}HVDOkPSDGDHQHUJHWLFDPHQWHPDLVH¿FLHQWHDPDLVYLiYHOHFRQRPLFDPHQWHHDGHPDLRU
vida útil são, respectivamente
A ÀXRUHVFHQWHFRPSDFWD/('/('
B /('ÀXRUHVFHQWHFRPSDFWD/('
C ÀXRUHVFHQWHFRPSDFWDLQFDQGHVFHQWH/('
D /('LQFDQGHVFHQWHÀXRUHVFHQWHFRPSDFWD
E ÀXRUHVFHQWHFRPSDFWDÀXRUHVFHQWHFRPSDFWD/('
QUESTÃO 83
2ELVIHQRO$pXPFRPSRVWRTXHVHUYHGHPDWpULDSULPDSDUDDIDEULFDomRGHSROtPHURVXWLOL]DGRVHPHPEDODJHQV
plásticas de alimentos, em mamadeiras e no revestimento interno de latas. Esse composto está sendo banido em
GLYHUVRVSDtVHVLQFOXLQGRR%UDVLOSULQFLSDOPHQWHSRUVHUXPPLPHWL]DGRUGHHVWUyJHQRV KRUP{QLRV TXHDWXDQGR
FRPR WDO QR RUJDQLVPR SRGH FDXVDU LQIHUWLOLGDGH QD YLGD DGXOWD 2 ELVIHQRO$ PDVVD PRODU LJXDO D  JPRO  p
SUHSDUDGRSHODFRQGHQVDomRGDSURSDQRQD PDVVDPRODULJXDODJPRO FRPIHQRO PDVVDPRODULJXDODJPRO 
HPPHLRiFLGRFRQIRUPHDSUHVHQWDGRQDHTXDomRTXtPLFD
H3C CH3
H 3C CH3 H+
2 + + H 2O
HO O HO OH
3$6725(0$QYLVDSURtEHPDPDGHLUDVFRPELVIHQRO$QR%UDVLOFolha de S. PauloVHW DGDSWDGR 

&RQVLGHUDQGRTXHDRUHDJLUJGHSURSDQRQDFRPJGHIHQROREWHYHVHNJGHELVIHQRO$GHDFRUGR
FRPDUHDomRGHVFULWDRUHQGLPHQWRUHDOGRSURFHVVRIRLGH
A 
B 
C 
D 
E 

CN - 1º dia | Caderno 3 - BRANCO - Página 28


2014 *BRAN75SAB29*
QUESTÃO 84

2FREUHPXLWRXWLOL]DGRHP¿RVGDUHGHHOpWULFDHFRPFRQVLGHUiYHOYDORUGHPHUFDGRSRGHVHUHQFRQWUDGRQDQDWXUH]D
QDIRUPDGHFDOFRFLWD&X26 V GHPDVVDPRODUJPRO3RUPHLRGDUHDomR&X2S (s) + O2 J ĺ&X V 622 J p
SRVVtYHOREWrORQDIRUPDPHWiOLFD

$ TXDQWLGDGH GH PDWpULD GH FREUH PHWiOLFR SURGX]LGD D SDUWLU GH XPD WRQHODGD GH FDOFRFLWD FRP  PP  GH
SXUH]Dp
A 1,0 × 103 mol.
B î2 mol.
C 1,0 × 100 mol.
D îí mol.
E îí mol.

QUESTÃO 85

$IRUPDomRGHHVWDODFWLWHVGHSHQGHGDUHYHUVLELOLGDGHGHXPDUHDomRTXtPLFD2FDUERQDWRGHFiOFLR &D&23)
p HQFRQWUDGR HP GHSyVLWRV VXEWHUUkQHRV QD IRUPD GH SHGUD FDOFiULD 4XDQGR XP YROXPH GH iJXD ULFD HP &22
GLVVROYLGRLQ¿OWUDVHQRFDOFiULRRPLQpULRGLVVROYHVHIRUPDQGRtRQV&D2+ e HCO3í. Numa segunda etapa, a solução
DTXRVDGHVVHVtRQVFKHJDDXPDFDYHUQDHRFRUUHDUHDomRLQYHUVDSURPRYHQGRDOLEHUDomRGH&22 e a deposição
de CaCO3, de acordo com a equação apresentada.

Ca2+ (aq) + 2 HCO3í (aq) CaCO3 (s) + CO2 (g) + H2O (l) ǻ+ N-PRO
KOTZ, J. C.; TREICHEL, P. L.; WEAVER, G. C. 4XtPLFDJHUDOHUHDo}HVTXtPLFDV. São Paulo: Cengage Learning, 2010 (adaptado).

&RQVLGHUDQGRRHTXLOtEULRTXHRFRUUHQDVHJXQGDHWDSDDIRUPDomRGHFDUERQDWRVHUiIDYRUHFLGDSHOR D
A GLPLQXLomRGDFRQFHQWUDomRGHtRQV2+í no meio.
B aumento da pressão do ar no interior da caverna.
C diminuição da concentração de HCO3í no meio.
D aumento da temperatura no interior da caverna.
E aumento da concentração de CO2 dissolvido.

QUESTÃO 86
(P XP H[SHULPHQWR IRUDP VHSDUDGRV WUrV UHFLSLHQWHV$ % H & FRQWHQGR  P/ GH OtTXLGRV GLVWLQWRV R
UHFLSLHQWH$FRQWLQKDiJXDFRPGHQVLGDGHGHJP/RUHFLSLHQWH%iOFRROHWtOLFRFRPGHQVLGDGHGHJP/
HRUHFLSLHQWH&FORURIyUPLRFRPGHQVLGDGHGHJP/(PFDGDXPGHVVHVUHFLSLHQWHVIRLDGLFLRQDGDXPD
SHGUDGHJHORFRPGHQVLGDGHSUy[LPDDJP/
1RH[SHULPHQWRDSUHVHQWDGRREVHUYRXVHTXHDSHGUDGHJHOR
A ÀXWXRXHP$ÀXWXRXHP%HÀXWXRXHP&
B ÀXWXRXHP$DIXQGRXHP%HÀXWXRXHP&
C DIXQGRXHP$DIXQGRXHP%HÀXWXRXHP&
D DIXQGRXHP$ÀXWXRXHP%HDIXQGRXHP&
E ÀXWXRXHP$DIXQGRXHP%HDIXQGRXHP&

&1žGLD_&DGHUQR%5$1&23iJLQD
*BRAN75SAB30* 2014

QUESTÃO 87
2V HIHLWRV GR H[HUFtFLR ItVLFR QD UHGXomR GH GRHQoDV FDUGLRYDVFXODUHV VmR EHP FRQKHFLGRV DXPHQWDQGR SRU
H[HPSOR D WROHUkQFLD D LQIDUWRV HP FRPSDUDomR FRP LQGLYtGXRV VHGHQWiULRV 9LVDQGR JDQKR GH IRUoD GH PDVVD
PXVFXODUHSHUGDGHJRUGXUDYHUL¿FDVHRXVRGHDQDEROL]DQWHVSRUDOJXQVHVSRUWLVWDV(PXPDSHVTXLVDFRPUDWRV
FRQ¿UPRXVHDPHOKRUDGDFRQGLomRFDUGtDFDHPUHVSRVWDDRH[HUFtFLRPDVYHUL¿FRXVHTXHRVHIHLWRVEHQp¿FRVGR
H[HUFtFLRItVLFRVmRSUHMXGLFDGRVSHORXVRGHDQDEROL]DQWHVFRPRRGHFDQRDWRGHQDQGURORQDDXPHQWDQGRDiUHD
FDUGtDFDDIHWDGDSHORLQIDUWR
&+$9(6($HWDO&DUGLRSURWHomRLQGX]LGDSHORH[HUFtFLRpSUHMXGLFDGDSHORWUDWDPHQWRFRPDQDEROL]DQWHGHFDQRDWRGHQDQGURORQDBrazilian Journal of Biomotricity, v. 1, n. 3, 2007 (adaptado).

4XDOJUi¿FRUHSUHVHQWDRVUHVXOWDGRVGHVVHHVWXGR"
60 60
A D
50 50
Área de infarto (%)

Área de infarto (%)


40 40

30 30

20 20

10 10

0 0
Sedentário Sedentário Exercício Exercício Sedentário Sedentário Exercício Exercício
sem com sem com sem com sem com
anabolizante anabolizante anabolizante anabolizante anabolizante anabolizante anabolizante anabolizante

60 60
B E
50 50
Área de infarto (%)

Área de infarto (%)

40 40

30 30

20 20

10 10

0 0
Sedentário Sedentário Exercício Exercício Sedentário Sedentário Exercício Exercício
sem com sem com sem com sem com
anabolizante anabolizante anabolizante anabolizante anabolizante anabolizante anabolizante anabolizante

60
C
50
Área de infarto (%)

40

30

20

10

0
Sedentário Sedentário Exercício Exercício
sem com sem com
anabolizante anabolizante anabolizante anabolizante

CN - 1º dia | Caderno 3 - BRANCO - Página 30


2014 *BRAN75SAB31*
QUESTÃO 88 QUESTÃO 89
A escolha de uma determinada substância )HUWLOL]DQWHV TXtPLFRV PLVWRV GHQRPLQDGRV 13.
SDUD VHU XWLOL]DGD FRPR FRPEXVWtYHO SDVVD SHOD VmR XWLOL]DGRV SDUD DXPHQWDU D SURGXWLYLGDGH DJUtFROD
análise da poluição que ela causa ao ambiente SRU IRUQHFHUHP RV QXWULHQWHV QLWURJrQLR IyVIRUR H
e pela quantidade de energia liberada em sua potássio, necessários para o desenvolvimento das
combustão completa. O quadro apresenta a entalpia plantas. A quantidade de cada nutriente varia de acordo
de combustão de algumas substâncias. As massas FRPD¿QDOLGDGHGRDGXER8PGHWHUPLQDGRDGXER13.
molares dos elementos H, C e O são, respectivamente,
possui, em sua composição, as seguintes substâncias:
LJXDLVDJPROJPROHJPRO
QLWUDWRGHDP{QLR 1+NO3), ureia (CO(NH2)2), nitrato de
Entalpia de potássio (KNO3 IRVIDWRGHVyGLR 1D3PO) e cloreto de
Substância Fórmula FRPEXVWmR potássio (KCl).
N-PRO A adição do adubo descrito provocou diminuição no pH
Acetileno C 2H 2 í GH XP VROR &RQVLGHUDQGR R FDUiWHU iFLGREiVLFR GDV
substâncias constituintes desse adubo, a diminuição do
Etano C 2H 6 í S+GRVRORGHYHVHUDWULEXtGDjSUHVHQoDQRDGXERGH
XPDTXDQWLGDGHVLJQL¿FDWLYDGH
Etanol C2HOH í
A ureia.
Hidrogênio H2 í B IRVIDWRGHVyGLR
C QLWUDWRGHDP{QLR
Metanol CH3OH í D nitrato de potássio.
/HYDQGRVH HP FRQWD VRPHQWH R DVSHFWR HQHUJpWLFR D E cloreto de potássio.
VXEVWkQFLDPDLVH¿FLHQWHSDUDDREWHQomRGHHQHUJLDQD
FRPEXVWmRGHNJGHFRPEXVWtYHOpR QUESTÃO 90
A etano. $ HXWUR¿]DomR p XP GRV IHQ{PHQRV UHVSRQViYHLV
B etanol. SHOD PRUWDOLGDGH GH SDUWH GDV HVSpFLHV DTXiWLFDV H
C metanol. HP UHJL}HV SUy[LPDV D FHQWURV XUEDQRV SHOD SHUGD GD
TXDOLGDGHGHYLGDGDSRSXODomR8PH[HPSORpD/DJRD
D acetileno.
GD3DPSXOKDXPGRVPDLVFRQKHFLGRVSRQWRVWXUtVWLFRV
E hidrogênio. da capital de Minas Gerais, onde as atividades de pesca
e nado não são mais permitidas.
3DUDHYLWDUDRFRUUrQFLDGHVVHIHQ{PHQRHPODJRVGHYHVH
A manter inalterado seu volume de água.
B DXPHQWDUDSRSXODomRGHDOJDVSODQFW{QLFDV
C GLPLQXLURWHRUGHQXWULHQWHVGHVSHMDGRVQDViJXDV
D LPSHGLUDIRWRVVtQWHVHGDVDOJDVDEDL[RGDVXSHUItFLH
E DXPHQWDUDSRSXODomRGHHVSpFLHVGRWRSRGDFDGHLD
alimentar.

CN - 1º dia | Caderno 3 - BRANCO - Página 31


*BRAN75SAB32* 2014

2014

CN - 1º dia | Caderno 3 - BRANCO - Página 32


EXAME NACIONAL DO ENSINO MÉDIO
PROVA DE REDAÇÃO E DE LINGUAGENS, CÓDIGOS E SUAS TECNOLOGIAS
PROVA DE MATEMÁTICA E SUAS TECNOLOGIAS

2º DIA
CADERNO

6
2014 CINZA
2ª APLICAÇÃO
A COR DA CAPA DO SEU CADERNO DE QUESTÕES É CINZA. MARQUE-A EM SEU CARTÃO-RESPOSTA.

LEIA ATENTAMENTE AS INSTRUÇÕES SEGUINTES: 7 2 WHPSR GLVSRQtYHO SDUD HVWDV SURYDV p GH cinco horas e
trinta minutos
1 9HUL¿TXHQR&$57­25(63267$HQD)2/+$'(5('$d­2 8 5HVHUYHRVPLQXWRV¿QDLVSDUDPDUFDUVHX&$57­25(63267$
TXH VH HQFRQWUD QR YHUVR GR &$57­25(63267$ VH RV 2VUDVFXQKRVHDVPDUFDo}HVDVVLQDODGDVQR&$'(512'(
VHXVGDGRVHVWmRUHJLVWUDGRVFRUUHWDPHQWH&DVRKDMDDOJXPD 48(67®(6QmRVHUmRFRQVLGHUDGRVQDDYDOLDomR
GLYHUJrQFLDFRPXQLTXHDLPHGLDWDPHQWHDRDSOLFDGRUGDVDOD
9 6RPHQWH VHUmR FRUULJLGDV DV UHGDo}HV WUDQVFULWDV QD )2/+$
2 ATENÇÃO DSyV D FRQIHUrQFLD HVFUHYD H DVVLQH VHX QRPH '(5('$d­2
QRV HVSDoRV SUySULRV GR &$57­25(63267$ H GD )2/+$
'(5('$d­2FRPFDQHWDHVIHURJUi¿FDGHWLQWDSUHWD 10 4XDQGR WHUPLQDU DV SURYDV DFHQH SDUD FKDPDU R
DSOLFDGRUHHQWUHJXHHVWH&$'(512'(48(67®(6HR
3 ATENÇÃO WUDQVFUHYD QR HVSDoR DSURSULDGR GR VHX &$57­25(63267$)2/+$'(5('$d­2
&$57­25(63267$FRPVXDFDOLJUD¿DXVXDOFRQVLGHUDQGR
DVOHWUDVPDL~VFXODVHPLQ~VFXODVDVHJXLQWHIUDVH 11 9RFrSRGHUiGHL[DURORFDOGHSURYDVRPHQWHDSyVGHFRUULGDV
GXDV KRUDV GR LQtFLR GD DSOLFDomR H SRGHUi OHYDU VHX
&$'(512'(48(67®(6DRGHL[DUHPGH¿QLWLYRDVDODGH
Tenho de fechar meus olhos para ver-te.
SURYDQRVPLQXWRVTXHDQWHFHGHPRWpUPLQRGDVSURYDV

4 (VWH &$'(512 '( 48(67®(6 FRQWpP D 3URSRVWD GH 12 9RFrVHUiHOLPLQDGRGR([DPHDTXDOTXHUWHPSRQRFDVRGH


5HGDomRHTXHVW}HVQXPHUDGDVGHDGLVSRVWDVGD D SUHVWDUHPTXDOTXHUGRFXPHQWRGHFODUDomRIDOVDRXLQH[DWD
VHJXLQWHPDQHLUD E SHUWXUEDUGHTXDOTXHUPRGRDRUGHPQRORFDOGHDSOLFDomR
D DVTXHVW}HVGHQ~PHURDVmRUHODWLYDVjiUHDGH GDV SURYDV LQFRUUHQGR HP FRPSRUWDPHQWR LQGHYLGR
/LQJXDJHQV&yGLJRVHVXDV7HFQRORJLDV GXUDQWHDUHDOL]DomRGR([DPH
E DVTXHVW}HVGHQ~PHURDVmRUHODWLYDVjiUHDGH F VH FRPXQLFDU GXUDQWH DV SURYDV FRP RXWUR SDUWLFLSDQWH
0DWHPiWLFDHVXDV7HFQRORJLDV YHUEDOPHQWHSRUHVFULWRRXSRUTXDOTXHURXWUDIRUPD
ATENÇÃO DV TXHVW}HV GH  D  VmR UHODWLYDV j OtQJXD G SRUWDU TXDOTXHU WLSR GH HTXLSDPHQWR HOHWU{QLFR H GH
HVWUDQJHLUD 9RFr GHYHUi UHVSRQGHU DSHQDV jV TXHVW}HV FRPXQLFDomRDSyVLQJUHVVDUQDVDODGHSURYDV
UHODWLYDVjOtQJXDHVWUDQJHLUD LQJOrVRXHVSDQKRO HVFROKLGDQR
DWRGHVXDLQVFULomR H XWLOL]DU RX WHQWDU XWLOL]DU PHLR IUDXGXOHQWR HP EHQHItFLR
SUySULRRXGHWHUFHLURVHPTXDOTXHUHWDSDGR([DPH
5 &RQ¿UD VH R VHX &$'(512 '( 48(67®(6 FRQWpP D
TXDQWLGDGH GH TXHVW}HV H VH HVVDV TXHVW}HV HVWmR QD RUGHP I XWLOL]DU OLYURV QRWDV RX LPSUHVVRV GXUDQWH D UHDOL]DomR
PHQFLRQDGD QD LQVWUXomR DQWHULRU &DVR R FDGHUQR HVWHMD GR([DPH
LQFRPSOHWR WHQKD TXDOTXHU GHIHLWR RX DSUHVHQWH GLYHUJrQFLD J VH DXVHQWDU GD VDOD GH SURYDV OHYDQGR FRQVLJR R
FRPXQLTXH DR DSOLFDGRU GD VDOD SDUD TXH HOH WRPH DV &$'(512'(48(67®(6DQWHVGRSUD]RHVWDEHOHFLGR
SURYLGrQFLDVFDEtYHLV HRX R &$57­25(63267$)2/+$ '( 5('$d­2 D
6 3DUD FDGD XPD GDV TXHVW}HV REMHWLYDV VmR DSUHVHQWDGDV TXDOTXHUWHPSR
RSo}HV$SHQDVXPDUHVSRQGHFRUUHWDPHQWHjTXHVWmR K QmRFXPSULUFRPRGLVSRVWRQRHGLWDOGR([DPH

*cinz25dom1*
*CINZ25DOM2* 2014

PROPOSTA DE REDAÇÃO
A partir da leitura dos textos motivadores seguintes e com base nos conhecimentos construídos ao longo de
sua formação, redija texto dissertativo-argumentativo em norma padrão da língua portuguesa sobre o tema
O que o fenômeno social dos “rolezinhos” representa?, apresentando proposta de intervenção, que respeite
os direitos humanos. Selecione, organize e relacione, de forma coerente e coesa, argumentos e fatos para defesa
de seu ponto de vista.

TEXTO I TEXTO II
Segundo o MC Daniel de Souza, a origem do
“rolezinho” remete aos chamados encontros de
admiradores, em que fãs dos cantores de funk iam aos
shoppings para encontrar os ídolos. “Antes do ‘rolezinho’
tinha o encontro de admiradores, que era com os
famosinhos das redes sociais, que faziam o seu encontro
e reuniam o povo no shopping”, declarou. “É o único
lugar que todo mundo conhece e é público”. O jovem
acredita que os encontros de admiradores cresceram e
se tornaram os “rolezinhos” de hoje, atraindo também
pessoas que aproveitam a situação para causar tumulto.

ANTONIO, T. Disponível em: http://memoria.ebc.com.br.


Acesso em: 16 jun. 2014 (adaptado).

Disponível em: http://altamiroborges.blogspot.com.br.


Acesso em: 16 jun. 2014.

TEXTO III
O funk da ostentação, surgido na Baixada Santista e Região Metropolitana de São Paulo nos últimos anos, evoca
o consumo, o luxo, o dinheiro e o prazer que tudo isso dá. Em seus clipes, os MCs aparecem com correntes e anéis
de ouro, vestidos com roupas de grife, em carros caros, cercado por mulheres. Diferentemente do núcleo duro do hip
hop paulista dos anos 80 e 90, que negava o sistema, e também do movimento de literatura periférica e marginal que,
no início dos anos 2000, defendia que, se é para consumir, que se comprem as marcas produzidas pela periferia, para
a periferia, o funk da ostentação coloca os jovens, ainda que para a maioria só pelo imaginário, em cenários até então
reservados para a juventude branca das classes média e alta. Esta, talvez, seja a sua transgressão. Em seus clipes,
os MCs têm vida de rico, com todos os signos dos ricos.
Esta exaltação do luxo e do consumo, interpretada como adesão ao sistema, tornou o funk da ostentação
desconfortável para uma parcela dos intelectuais brasileiros e mesmo para parte das lideranças culturais das periferias
de São Paulo. Agora, os rolezinhos – e a repressão que se seguiu a eles – deram a esta vertente do funk uma marca
de insurgência. Ao ocupar os shoppings, a juventude pobre e negra das periferias não estava apenas se apropriando
dos valores simbólicos, como já fazia pelas letras do funk da ostentação, mas também dos espaços físicos, o que
marca uma diferença.
BRUM, E. Disponível em: http://arquivo.geledes.org.br. Acesso em: 16 jun. 2014 (fragmento).

INSTRUÇÕES:
‡ O rascunho da redação deve ser feito no espaço apropriado.
‡ 2WH[WRGH¿QLWLYRGHYHVHUHVFULWRjWLQWDQDIROKDSUySULDHPDWpOLQKDV
‡ A redação que apresentar cópia dos textos da Proposta de Redação ou do Caderno de Questões terá o número de linhas
copiadas desconsiderado para efeito de correção.
Receberá nota zero, em qualquer das situações expressas a seguir, a redação que:
‡ WLYHUDWp VHWH OLQKDVHVFULWDVVHQGRFRQVLGHUDGD³LQVX¿FLHQWH´
‡ fugir ao tema ou que não atender ao tipo dissertativo-argumentativo.
‡ apresentar proposta de intervenção que desrespeite os direitos humanos.
‡ apresentar parte do texto deliberadamente desconectada com o tema proposto.

LC - 2º dia | Caderno 6 - CINZA - Página 2


2014 *CINZ25DOM3*
LINGUAGENS, CÓDIGOS E SUAS QUESTÃO 92

TECNOLOGIAS Languages and cultures use non-verbal communication


which conveys meaning. Although many gestures are
Questões de 91 a 135 VLPLODULQ7KDLDQG(QJOLVKVXFKDVQRGGLQJIRUDI¿UPDWLRQ
many others are not shared. A good example of this is
Questões de 91 a 95 (opção inglês) the ubiquitous “Thai smile”. The “smile” carries a far wider
range of meanings in Thai than it does in English culture.
This can sometimes lead to serious communication
QUESTÃO 91 breakdowns between Thais and English speakers.
An example from my own early experience in Thailand
illustrates the point. When confronting the Thai owner of a
language school with administrative problems, complaints
regarding student numbers in the class were met by a
beaming smile and little else. I took this to mean lack of
concern or an attempt to trivialise or ignore the problem.
I left the discussion upset and angry by what appeared to
be the owner’s offhand attitude to my problems.
It was only later when another native speaking English
teacher, with considerably more experience of Thailand,
explained that a smile meant an apology and the fact that
the following day all my complaints had been addressed,
that I fully understood the situation.
Disponível em: www.spring.org.uk. Acesso em: 11 jul. 2011 (fragmento).

Viver em um país estrangeiro pode ser uma experiência


HQULTXHFHGRUD HPERUD SRVVD WDPEpP VHU XP GHVD¿R
pelo choque cultural. A experiência relatada pelo autor
do texto revela diferentes atribuições de sentido a um
determinado comportamento, mostrando que naquela
situação o sorriso indicava um(a)
A forma educada de fazer uma reclamação.
B modo irônico de reagir a uma solicitação.
C jeito de reconhecer um erro e se desculpar.
D tentativa de minimizar um problema.
E estratégia para esconder a verdade.

QUESTÃO 93

Tennesse Mountain Properties


'LVSRQtYHOHPKWWSESEORJVSRWFRP$FHVVRHPMXO

Description
Implementar políticas adequadas de alimentação e
nutrição é uma meta prioritária em vários países do Own a renovated house for less than $290 per
mundo. A partir da campanha If you can´t read it, why PRQWK 1HZ ZLQGRZV VLGLQJ ÀRRULQJ ODPLQDWH
throughout and tile in entry way and bathroom), kitchen
eat it?, os leitores são alertados para o perigo de
cabinets, counter top, back door, fresh paint and laundry
A acessarem informações equivocadas sobre a RQPDLQÀRRU+HDWELOOVDUHYHU\ORZGXHWRDJRRGVROLG
formulação química de alimentos empacotados. KRXVHDQGDQHQHUJ\HI¿FLHQWIXUQDFH
'LVSRQtYHOHPZZZIUHHUHDOHVWDHDGVQHW$FHVVRHPQRY DGDSWDGR 
B consumirem alimentos industrializados sem o
interesse em conhecer a sua composição. Em jornais, há diversos anúncios que servem aos
leitores. O conteúdo do anúncio veiculado por este texto
C desenvolverem problemas de saúde pela falta de interessará a alguém que esteja procurando
conhecimento a respeito do teor dos alimentos.
A emprego no setor imobiliário.
D incentivarem crianças a ingerirem grande quantidade
B imóvel residencial para compra.
de alimentos processados e com conservantes.
C serviço de reparos em domicílio.
E ignorarem o aumento constante da obesidade D pessoa para trabalho doméstico.
causada pela má alimentação na fase de E curso de decorador de interiores.
desenvolvimento da criança.

/&žGLD_&DGHUQR&,1=$3iJLQD
*CINZ25DOM4* 2014

QUESTÃO 94 LINGUAGENS, CÓDIGOS E SUAS


, UHDG D VWXG\ WKDW PHDVXUHG WKH HI¿FLHQF\ RI TECNOLOGIAS
locomotion for various species on the planet. The condor
XVHGWKHOHDVWHQHUJ\WRPRYHDNLORPHWHU+XPDQVFDPH
in with a rather unimpressive showing about a third of the Questões de 91 a 135
way down the list ... That didn’t look so good, but then
someone at Scientific American had the insight to test Questões de 91 a 95 (opção espanhol)
the efficiency of locomotion for a man on a bicycle.
And a man on a bicycle blew the condor away. That’s what QUESTÃO 91
a computer is to me: the computer is the most remarkable
tool that we’ve ever come up with. It’s the equivalent of a El candombe es participación
bicycle for our minds.
Bastaría nombrar al tambor en nuestro país, y ya
JOBS, S. Disponível em: www.msnbc.msn.com. Acesso em: 28 fev. 2012 (adaptado).
estaría implícita la referencia al candombe, patrimonio
Ao abordar o deslocamento de várias espécies, com cultural uruguayo y, desde setiembre del 2009,
base em um estudo que leu, Steve Jobs apresenta o
computador como uma ferramenta que 3DWULPRQLR ,QPDWHULDO GH OD +XPDQLGDG UHFRQRFLGR SRU
las Naciones Unidas para la Educación, la Ciencia y la
A amplia a quantidade de energia gasta no planeta.
Cultura — Unesco. Proceso que se remonta a la época
B alcança a mesma velocidade de uma bicicleta.
colonial, cuando era tenido como un mero baile de negros
C altera a velocidade com a qual nos movemos.
HVFODYRVUXLGRVR\REVFHQRGH¿QLFLyQTXHWRPDQLQFOXVR
D WRUQDRVPHLRVGHWUDQVSRUWHPDLVH¿FLHQWHV algunos diccionarios antiguos y no tanto, viéndolo como
E aumenta o potencial de nossas mentes. divertimento de pésima categoría por su procedencia
QUESTÃO 95 racial. Esto fue cambiando merced al combate sin tregua
a los prejuicios, siempre en crecimiento su aceptación por
Turning Brown el conjunto de la sociedad, hasta llegar a nuestros días,
A four-year-old boy was eating an apple in the back cuando el Gobierno progresista en el 2006 promovió la
seat of the car, when he asked, “Daddy, why is my apple OH\FRQVDJUDQGRORVGHGLFLHPEUHFRPR'tDGHO
turning brown?” Candombe, la Cultura Afrouruguaya y la Equidad Racial,
“Because,” his dad explained, “after you ate the skin TXHHQVXDUWtFXORWR\¿QDOGLFH³'HFOiUHVHSDWULPRQLR
off, the meat of the apple came into contact with the air, cultural de la República Oriental del Uruguay al candombe,
which caused it to oxidize, thus changing the molecular
caracterizado por el toque de los tambores denominados
structure and turning it into a different color.”
chico, repique y piano, su danza y canto, creado por los
There was a long silence. Then the son asked softly, afrouruguayos a partir del legado ancestral africano, sus
“Daddy, are you talking to me?”
orígenes rituales y su contexto social como comunidad.”
Disponível em: http://hayspost.com. Acesso em: 10 nov. 2011.
ANDRADE, S. Disponível em: http://alainet.org. Acesso em: 22 fev. 2012 (adaptado).
Considerando os participantes da conversa nessa
piada, nota-se que o efeito de humor é obtido em funçãoO status atual do candombe, resultante de um conjunto
A GDGL¿FXOGDGHTXHRSDLHVWDYDHQIUHQWDQGRSDUDGDU de mudanças ocorridas no país, contrasta com um
XPDUHVSRVWDDR¿OKR passado marcado por preconceitos. Segundo o texto,
B de o pai dizer que a maçã tem carne e que muda de esse status VHGHYHj
cor em contato com o ar.
A GH¿QLomRGDGDSRUGLFLRQiULRVDWXDLV
C de um menino de quatro anos entender uma
H[SOLFDomRFLHQWt¿FDVREUHDR[LGDomR B receptividade crescente pela sociedade.
D do fato de a criança não saber por que a maçã que C FUtWLFDjVIHVWDVEDUXOKHQWDV
estava comendo era marrom.
D conservação da herança africana.
E da escolha inadequada do tipo de linguagem para se
conversar com uma criança. E visão da dança como obscena.

LC - 2º dia | Caderno 6 - CINZA - Página 4


2014 *CINZ25DOM5*
QUESTÃO 92 QUESTÃO 94

Disponível em: www.mec.es. Acesso em: 27 fev. 2012.

Gracias por no fumar é o título de um texto publicitário


produzido pelo Ministério da Educação e Cultura da
Espanha. Esse título, associado ao conteúdo do texto,
WHPFRPR¿QDOLGDGH
A proibir o consumo de cigarro em ambientes públicos.
B informar os sintomas do tabagismo.
C convencer os fumantes a abandonar o vício.
D questionar a qualidade de vida do fumante. 'LVSRQtYHOHPKWWSRIXHQDZRUGSUHVVFRP$FHVVRHPDJR
E DJUDGHFHUjVSHVVRDVTXHQmRIXPDP
A charge é um gênero textual que possui caráter
QUESTÃO 93 humorístico e crítico. Ao abordar o tema do uso da
tecnologia, essa charge critica o(a)
Al ingresar a la ciudad de Trevelín a través de un A postura das pessoas que não respeitam a opinião
gran valle cobijado por montañas elevadas y grandes dos outros.
VXSHU¿FLHV FRQ iUHDV FXOWLYDGDV OODPD OD DWHQFLyQ HO B tendência de algumas pessoas a interferir em
colorido de sus jardines y la huella que ha dejado la conversa alheia.
colonia galesa en cada rincón del pueblo y del área rural. C forma como a tecnologia ampliou a comunicação e a
Un ejemplo de ello es el primer molino harinero accionado interação entre as pessoas.
a caballo que se muestra en el museo regional junto a D hábito das pessoas que passam muitas horas
maquinarias, herramientas y vestimenta de la vida diaria conectadas.
en época de la colonia. E indivíduo cujo comportamento destoa de seu discurso.
Enclavada en un típico paisaje andino patagónico,
con sus vientos predominantes del sudoeste y su
naturaleza pródiga en vegetación y arroyos, Trevelín
sirve como punto de partida para hermosos paseos.
En la serenidad de sus calles, innumerables casas
de té ofrecen la clásica merienda galesa con sus tortas,
SDQHVFDVHURV\¿DPEUHVGHODUHJLyQSDUDDGHQWUDUVHHQ
esa gastronomía que no ha claudicado con el paso del
tiempo. Se aconseja probar su torta negra.
Disponível em: www.welcomeargentina.com. Acesso em: 25 jul. 2012 (adaptado).

Trevelín está localizada na Patagônia argentina. O destaque


das características da cidade, no texto, tem a função de
A narrar fatos do cotidiano da cidade e de seus
habitantes.
B despertar no leitor o desejo de conhecer a região.
C desenvolver o interesse pela gastronomia local.
D GLYXOJDULQIRUPDo}HVDFHUFDGDJHRJUD¿DGDUHJLmR
E FRPSDUWLOKDULPSUHVV}HVFROKLGDVHPXPDYLVLWDjFLGDGH

LC - 2º dia | Caderno 6 - CINZA - Página 5


*CINZ25DOM6* 2014

QUESTÃO 95 (VWDLQÀXrQFLDWDPEpPpGHYLGDDRIDWRGHDPHPyULD
humana, segundo estudos da psicologia cognitiva,
Retomando la inquietud propia y de tanta gente compreender e reter melhor as informações organizadas,
contraria a la denominación racista y xenófoba “Día de especialmente em diagramas e em mapas conceituais
la Raza” usada para el feriado 12 de octubre, donde se manipuláveis. Por isso, imagina-se que o hipertexto deva
recuerda el arribo de los primeros europeos a tierras favorecer o domínio mais rápido e fácil das informações,
posteriormente nombradas América, reforzamos la em contraponto a um audiovisual tradicional, por exemplo.
idea sumando agrupaciones e independientes de la Disponível em: vsites.unb.br. Acesso em: 1 ago. 2012.
militancia ciudadana motivados por lo mismo. Puede
parecer menor, pero un nombre dice mucho. Es un O texto informa como as interfaces são reaproveitadas
símbolo, una representación, un código que resume SHOR KLSHUWH[WR YLUWXDO LQÀXHQFLDQGR DV WHFQRORJLDV GH
LQ¿QLGDGGHFRVDVGHVGHORREMHWLYR\GHVGHORVXEMHWLYR informação e comunicação. De acordo com o texto, qual
Y lamentablemente no hubo “descubrimiento” sino pD¿QDOLGDGHGRXVRGRKLSHUWH[WRTXDQWRjDEVRUomRH
despojo y apropiación. No hubo “encuentro” sino saqueo manipulação das informações?
y masacre. La propuesta es que la sociedad uruguaya A Mesclar antigas interfaces com mecanismos virtuais.
logre una frase sustantiva que guarde memoria de los
hechos, apostando a un presente y futuro fraternal B Auxiliar os estudos de psicologia cognitiva com base
e igualitario, y a una convivencia sin hegemonías ni nos hipertextos.
predominios culturales aunque así haya sido el origen C Amparar a pesquisa de mapas e diagramas
de nuestra historia. UHODFLRQDGRVjFDUWRJUD¿D
ANDRADE, S. No más Día de la Raza. América Latina en movimiento. Disponível em: D Salientar a importância das tecnologias de informação
http://alainet.org. Acesso em: 22 fev. 2012 (adaptado). e comunicação.
Com a expressão Día de la Raza ID]VH UHIHUrQFLD j E Ajudar na apreensão das informações de modo mais
chegada dos primeiros europeus em território americano H¿FD]HIDFLOLWDGR
e denomina-se a comemoração desse dia. A autora do
WH[WR VXJHUH R ¿P GHVVD GHQRPLQDomR QR 8UXJXDL QUESTÃO 97
acreditando que
Liberada, judoca árabe faz história nos Jogos
A a nomenclatura adotada será esquecida, porque é de Olímpicos de Londres
conhecimento geral que não houve descoberta. Aos 16 anos de idade, a judoca Wojdan Ali Seraj
B a reivindicação convencerá outros grupos e adeptos, Shaherkani, da categoria pesado (acima de 78 kg), fez
porque muitos desconhecem esse nome. história nos Jogos Olímpicos de Londres. Ela se tornou a
C D VRFLHGDGH GHYH HQFRQWUDU XPD IUDVH VLJQL¿FDWLYD primeira mulher da Arábia Saudita a disputar uma Olimpíada.
para a preservação da lembrança dos fatos. Isso depois de superar não só o preconceito em seu país
como também o quase veto da Federação Internacional de
D o convívio permitirá o esquecimento dos massacres,
Judô (FIJ), que não queria permitir que a atleta competisse
porque não houve encontro no passado.
vestindo o hijab, o tradicional véu islâmico.
E o presente e o futuro são e serão fraternais e
igualitários para o estímulo do predomínio cultural.

QUESTÃO 96

Interfaces
Um dos mais importantes componentes do hipertexto
é a sua interface. As interfaces permitem a visualização
do conteúdo, determinam o tipo de interação que se
estabelece entre as pessoas e a informação, direcionando
sua escolha e o acesso ao conteúdo.
O hipertexto retoma e transforma antigas interfaces
GDHVFULWD DQRomRGHLQWHUIDFHQmRGHYHVHUOLPLWDGDjV
técnicas de comunicação contemporânea). Constitui-se,
na verdade, em uma poderosa rede de interfaces que se Disponível em: www.lancenet.com.br. Acesso em: 8 ago. 2012 (adaptado).

conectam a partir de princípios básicos e que permitem No âmbito do esporte de alto rendimento, o uso do véu
uma “interação amigável”. pela lutadora saudita durante os Jogos Olímpicos de
As particularidades do hipertexto virtual, como sua Londres 2012 representa o(a)
dinamicidade e seus aspectos multimidiáticos, devem- A GHVFXPSULPHQWRGDUHJUDR¿FLDOGRMXG{
VH DR VHX VXSRUWH yWLFR PDJQpWLFR GLJLWDO H j VXD B risco para a integridade física das atletas adversárias.
LQWHUIDFH DPLJiYHO $ LQÀXrQFLD GR KLSHUWH[WR p WDQWD
TXH DV UHSUHVHQWDo}HV GH WLSR FDUWRJUi¿FR JDQKDP C vantagem para a atleta saudita na competição de judô.
cada vez mais importância nas tecnologias intelectuais D LQÀXrQFLDGHDVSHFWRVFXOWXUDLVHUHOLJLRVRVQRHVSRUWH
de suporte informático. E GL¿FXOGDGHGDPXOKHULVOkPLFDSDUDYHQFHUSUHFRQFHLWRV
LC - 2º dia | Caderno 6 - CINZA - Página 6
2014 *CINZ25DOM7*
QUESTÃO 98 QUESTÃO 100

Soneto
Oh! Páginas da vida que eu amava,
Rompei-vos! nunca mais! tão desgraçado!...
Ardei, lembranças doces do passado!
Quero rir-me de tudo que eu amava!

E que doido que eu fui! como eu pensava


Em mãe, amor de irmã! em sossegado
Adormecer na vida acalentado
Pelos lábios que eu tímido beijava!

Embora — é meu destino. Em treva densa


'HQWURGRSHLWRDH[LVWrQFLD¿QGD
Pressinto a morte na fatal doença!

$PLPDVROLGmRGDQRLWHLQ¿QGD
Possa dormir o trovador sem crença. Giocondas gêmeas
Perdoa minha mãe — eu te amo ainda!
A existência de uma segunda pintura da Mona Lisa
AZEVEDO, A. Lira dos vinte anos. São Paulo: Martins Fontes, 1996.
— a Gioconda GH /HRQDUGR GD 9LQFL ² IRL FRQ¿UPDGD
A produção de Álvares de Azevedo situa-se na década pelo Museu do Prado, em Madri, em fevereiro. O quadro
de 1850, período conhecido na literatura brasileira como era conhecido desde o século XVIII, mas tido como uma
Ultrarromantismo. Nesse poema, a força expressiva da reprodução tardia do original. Um trabalho de restauração
H[DFHUEDomRURPkQWLFDLGHQWL¿FDVHFRPR D revelou que seu fundo de cor negra na verdade recobria
A amor materno, que surge como possibilidade de a reprodução de uma típica paisagem da Toscana,
salvação para o eu lírico. FRPR D SLQWDGD SRU 'D 9LQFL 5DGLRJUD¿DV PRVWUDUDP
B VDXGRVLVPR GD LQIkQFLD LQGLFDGR SHOD PHQomR jV que a tela é irmã gêmea do original, provavelmente
¿JXUDVGDPmHHGDLUPm pintada por discípulos do mestre, sob supervisão de
C construção de versos irônicos e sarcásticos, apenas 'D9LQFLQRVHXDWHOLrGH)ORUHQoDHQWUHH
com aparência melancólica. Os dois quadros serão, agora, expostos no Louvre.
D presença do tédio sentido pelo eu lírico, indicado pelo +i HQWUHWDQWR GLIHUHQoDV D ÀRUHQWLQD /LVD *KHUDUGLQL
seu desejo de dormir. (Mona Lisa), aparentemente na meia-idade, parece mais
moça na nova tela. O manto sobre o ombro esquerdo
E ¿[DomRGRHXOtULFRSHODLGHLDGDPRUWHRTXHROHYDD
do quadro original surge como um véu transparente, e o
sentir um tormento constante.
decote aparece com mais nitidez. A descoberta reforça
QUESTÃO 99 a tese de estudiosos, como o inglês Martin Kemp, de
que assistentes de Da Vinci ajudaram na composição de
Ave a raiva desta noite telas importantes do mestre.
A baita lasca fúria abrupta
Revista Planeta, ano 40, ed. 474, mar. 2012.
Louca besta vaca solta
Ruiva luz que contra o dia Para cumprir sua função social, o gênero notícia precisa
Tanto e tarde madrugada. divulgar informações novas. No texto Giocondas gêmeas,
LEMINSKI, P. Distraídos venceremos. São Paulo: Brasiliense, 2002 (fragmento). DOpPGHVHUFRQ¿UPDGDDH[LVWrQFLDGHXPDWHODJrPHD
de Mona Lisa e de serem destacadas as diferenças entre
No texto de Leminski, a linguagem produz efeitos sonoros
elas, o valor informativo do texto está centrado na
e jogos de imagens. Esses jogos caracterizam a função
poética da linguagem, pois A D¿UPDomRGHTXHDGioconda genuína estava na fase
da meia-idade.
A objetivam convencer o leitor a praticar uma
determinada ação. B revelação da identidade da mulher pintada por
B transmitem informações, visando levar o leitor a Da Vinci, a florentina Lisa Gherardini.
adotar um determinado comportamento. C consideração de que as produções artísticas de
C visam provocar ruídos para chamar a atenção do Da Vinci datam do período renascentista.
leitor. D descrição do fato de que a tela original mostra um
D apresentam uma discussão sobre a própria linguagem, manto sobre o ombro esquerdo da personagem.
explicando o sentido das palavras. E confirmação da hipótese de que Da Vinci teve
E representam um uso artístico da linguagem, com o assistentes que o auxiliaram em algumas de
objetivo de provocar prazer estético no leitor. suas obras.

LC - 2º dia | Caderno 6 - CINZA - Página 7


*CINZ25DOM8* 2014

QUESTÃO 101 QUESTÃO 103

Saiba impedir que os cavalos de troia abram a O veneno do bem


guarda de seu computador
Imagine que você cortou o rosto e, em vez de dar
A lenda da Guerra de Troia conta que gregos pontos, o seu médico passa uma supercola feita de
FRQVHJXLUDPHQWUDUQDFLGDGHFDPXÀDGRVHPXPFDYDOR sangue de boi e veneno de cascavel. Isso pode mesmo
e, então, abriram as portas da cidade para mais guerreiros acontecer. Mas não se assuste. A história moderna das
entrarem e vencerem a batalha. Silencioso, o cavalo de serpentes não tem nada a ver com o medo ancestral
troia é um programa malicioso que abre as portas do que inspiram. Para a ciência, elas guardam produtos
computador a um invasor, que pode utilizar como quiser o utilíssimos nas glândulas letais. O mais recente é uma
privilégio de estar dentro de uma máquina. Esse malware cola de pele genuinamente brasileira, que, segundo os
p LQVWDODGR HP XP FRPSXWDGRU GH IRUPD FDPXÀDGD testes já feitos, dá uma cicatrização perfeita.
sempre com o “consentimento” do usuário. A explicação
é que essa praga está dentro de um arquivo que parece $ GHVFREHUWD SHUWHQFH j HTXLSH GR SURIHVVRU
ser útil, como um programa ou proteção de tela — que, Benedito Barraviera, da Universidade Estadual
ao ser executado, abre caminho para o cavalo de troia. Paulista, em Botucatu. E não é a primeira feita no
A intenção da maioria dos cavalos de troia (trojans) não é Brasil. Nos anos 1960, o médico Sérgio Ferreira, atual
contaminar arquivos ou hardwares. Atualmente, o objetivo presidente da Sociedade Brasileira para o Progresso da
principal dos cavalos de troia é roubar informações de Ciência, descobriu na jararaca uma molécula que em
uma máquina. O programa destrói ou altera dados com 1977 virou remédio contra a hipertensão.
intenção maliciosa, causando problemas ao computador Disponível em: www.super.abril.com.br. Acesso em: 2 mar. 2012 (fragmento).
RXXWLOL]DQGRRSDUD¿QVFULPLQRVRVFRPRHQYLDUspams.
A primeira regra para evitar a entrada dos cavalos de troia Nos diferentes textos, pode-se inferir, entre outras
é: não abra arquivos de procedência duvidosa. informações, quais são os objetivos de seu produtor e
quem é seu público-alvo. No trecho, para aproximar-se
Disponível em: http://idgnow.uol.com.br. Acesso em: 14 ago. 2012 (adaptado).
do interlocutor, o autor
Cavalo de troia é considerado um malware que
invade computadores, com intenção maliciosa. Pelas A emprega uma linguagem técnica de domínio do leitor.
informações apresentadas no texto, depreende-se que a B enfatiza informações importantes para a vida do leitor.
¿QDOLGDGHGHVVHSURJUDPDp C introduz o tema antecipando possíveis reações do
A roubar informações ou alterar dados de arquivos de leitor.
procedência duvidosa. D explora um tema sobre o qual o leitor tem reconhecido
B inserir senhas para enviar spams, através de um interesse.
rastreamento no computador.
E apresenta ao leitor, de forma minuciosa, a descoberta
C rastrear e investigar dados do computador sem o dos médicos.
conhecimento do usuário.
D induzir o usuário a fazer uso criminoso e malicioso de QUESTÃO 104
seu computador.
E usurpar dados do computador, mediante sua 2V HVSRUWHV SRGHP VHU FODVVL¿FDGRV OHYDQGRVH
execução pelo usuário. em consideração diversos critérios, como a quantidade
de competidores, a relação com os companheiros de
QUESTÃO 102 equipe, a interação com o adversário, o ambiente, o
desempenho comparado e os objetivos táticos da ação.
Abrimos o Brasil a todo o mundo: mas queremos que
o Brasil seja Brasil! Queremos conservar a nossa raça, Os chamados esportes de invasão ou territoriais são
a nossa história, e, principalmente, a nossa língua, que aqueles nos quais os competidores entram no setor
é toda a nossa vida, o nosso sangue, a nossa alma, a defendido pelo adversário, objetivando atingir a meta
nossa religião. contrária para pontuar, além de se preocupar em proteger
BILAC, O. Últimas conferências e discursos. Rio de Janeiro: Francisco Alves, 1927. simultaneamente a sua própria meta.
GONZALEZ, F. J. Revista Digital, Buenos Aires, n. 71, abr. 2004 (adaptado).
Nesse trecho, Olavo Bilac manifesta seu engajamento
na constituição da identidade nacional e linguística, São exemplos de esportes de invasão ou territoriais:
ressaltando a
A +DQGHEROEDVTXHWHEROIXWHEROHYROHLERO
A transformação da cultura brasileira.
B Rúgbi, futsal, natação e futebol americano.
B religiosidade do povo brasileiro.
C abertura do Brasil para a democracia. C Tênis de mesa, vôlei de praia, badminton e futevôlei.
D importância comercial do Brasil. D Basquetebol, handebol, futebol e futsal.
E autorreferência do povo como brasileiro. E Ginástica olímpica, beisebol, judô e tae kwon do.

LC - 2º dia | Caderno 6 - CINZA - Página 8


2014 *CINZ25DOM9*
QUESTÃO 105 QUESTÃO 106

Futebol de rua
Pelada é o futebol de campinho, de terreno baldio.
Mas existe um tipo de futebol ainda mais rudimentar do
que a pelada. É o futebol de rua. Perto do futebol de
rua qualquer pelada é luxo e qualquer terreno baldio é o
Maracanã em jogo noturno. Se você é brasileiro e criado
1
Nada de pegar namorada do amigo.
No máximo, pegue uma carona com ela. em cidade, sabe do que eu estou falando. Futebol de rua
é tão humilde que chama pelada de senhora.

2
Já pegou uma ou duas
ou três cervejas? Não sei se alguém, algum dia, por farra ou nostalgia,
Então pegue um táxi.
botou num papel as regras do futebol de rua. Elas seriam
mais ou menos assim:
3 Pegar carona com a gatinha que não bebeu
pega muito bem.
'2&$032²2FDPSRSRGHVHUVyDWpR¿RGDFDOoDGD
calçada e rua, rua e a calçada do outro lado e — nos
4 Pegou no copo,
não pegue na direção.
clássicos — o quarteirão inteiro. O mais comum é jogar-se
só no meio da rua.

5 Pega muito mal exagerar na bebida. DA DURAÇÃO DO JOGO — Até a mãe chamar ou
escurecer, o que vier primeiro. Nos jogos noturnos, até
alguém da vizinhança ameaçar chamar a polícia.
6
Pro bicho não pegar, pegue táxi, ônibus
ou carona. Só não pegue no volante
depois de beber. DA FORMAÇÃO DOS TIMES — O número de jogadores
em cada equipe varia, de um a setenta para cada lado.

´ DO JUIZ — Não tem juiz.


DO INTERVALO PARA DESCANSO — Você deve estar
brincando.
VERISSIMO, L. F. In: Para gostar de ler: crônicas 6. São Paulo: Ática, 2002 (fragmento).
Disponível em: http://blog.planalto.gov.br. Acesso em: 29 fev. 2012.

Anúncios publicitários geralmente fazem uso de Nesse trecho de crônica, o autor estabelece a seguinte
elementos verbais e não verbais. Nessa peça publicitária, UHODomRHQWUHRIXWHEROGHUXDHRIXWHEROR¿FLDO
a imagem, que simula um manual, e o texto verbal, A As regras do futebol de rua descaracterizam o futebol
TXH ID] XVR GH XPD YDULHGDGH GH OtQJXD HVSHFt¿FD de campo, uma vez que entre as duas práticas não
combinados, pretendem há similaridades.
A fazer a gradação de comportamentos e de atitudes em B As condições materiais do futebol de rua impedem
termos da gravidade de efeitos da bebida alcoólica. o envolvimento das pessoas e o caráter prazeroso
B aconselhar o leitor da peça publicitária a não “pegar” desta prática.
a namorada do amigo para o “bicho não pegar”. C O futebol de rua expressa a possibilidade de autoria
C promover a mudança de comportamento dos jovens das pessoas para a prática de esporte e de lazer.
HPUHODomRDRFRQVXPRGRiOFRROHjGLUHomR D O futebol de rua é necessariamente um futebol de
D demonstrar que a viagem de ônibus ou de táxi é mais PHQRUYDORUHLPSRUWkQFLDHPUHODomRDRIXWHEROR¿FLDO
segura, independentemente do consumo de álcool. E A ausência de regras formalizadas no futebol de rua
E incentivar a prática da carona em carros de motoristas faz com que o jogo seja desonesto em comparação
do sexo feminino. FRPRIXWHEROR¿FLDO

LC - 2º dia | Caderno 6 - CINZA - Página 9


*CINZ25DOM10* 2014

QUESTÃO 107 No gênero crônica, Machado de Assis legou inestimável


contribuição para o conhecimento do contexto social
O termo Foco equivale ao ponto de concentração
de seu tempo e seus hábitos culturais. O fragmento
do ator. O nível de concentração é determinado pelo
destacado comprova que o escritor avalia o(a)
envolvimento com o problema a ser solucionado.
Tomemos o exemplo do jogo teatral Cabo de Guerra: A manipulação inconsequente dos remédios pela
o Foco desse jogo reside em dar realidade ao objeto, população.
que nesse caso é a corda imaginária. A dupla de B uso de animais em testes com remédios
jogadores no palco mobiliza toda sua atenção desconhecidos.
H HQHUJLD SDUD GDU UHDOLGDGH j FRUGD 4XDQGR D C IDWR GH DV GURJDV PDQLSXODGDV QmR WHUHP H¿FiFLD
concentração é plena, a dupla sai do jogo com toda garantida.
evidência de ter realmente jogado o Cabo de Guerra
— sem fôlego, com dor nos músculos do braço etc. D hábito coletivo de experimentar drogas com objetivos
A plateia observa em função do Foco. terapêuticos.
KOUDELA, I. D. Jogos teatrais. São Paulo: Perspectiva, 1990.
E ausência de normas jurídicas para regulamentar a
venda nas boticas.
De acordo com o texto, a autora argumenta que o uso
do foco da cena teatral permite QUESTÃO 109
A transformar um objeto imaginário em um objeto
Mães
concreto, produzindo sobre o espectador uma
VHQVDomR LJXDO j TXH HOH WHULD HP XP HVSHWiFXOR Triste, mas verdadeira, a constatação de Jairo
de mágica. Marques — colunista que tem um talento raro — em seu
B produzir sobre a plateia, por meio do envolvimento WH[WR³(DPmH¿FRXYHOKLQKD´ ³&RWLGLDQR´RQWHP 
dos atores, imagens e/ou situações capazes de ativar Aqueles que percebem que a mãe envelheceu
seu imaginário e seu conhecimento de mundo. sempre têm atitudes diversas. Ou não a procuram mais,
C provocar efeito físico no ator, o que lhe confere porque essa é uma forma de negar que um dia perderão
a certeza de que seu corpo foi trabalhado o amparo materno, ou resolvem estar ao lado dela o
adequadamente para a produção da cena. maior tempo possível, pois têm medo de perdê-la sem ter
D acionar no ator a atenção a múltiplas ações que retribuído plenamente o amor que receberam.
ocorrem concomitantemente, tornando-o mais Leonor Souza (São Paulo, SP) — Painel do Leitor. Folha de S. Paulo, 29 fev. 2012.

disponível para a atuação em cena. Os gêneros textuais desempenham uma função social
E determinar uma única leitura da ação proposta, HVSHFt¿FDHPGHWHUPLQDGDVVLWXDo}HVGHXVRGDOtQJXD
explicitando qual entendimento o espectador deve em que os envolvidos na interação verbal têm um objetivo
ter da cena. comunicativo. Considerando as características do gênero,
a análise do texto Mães revela que sua função é
QUESTÃO 108
A ensinar sobre os cuidados que se deve ter com as
O Jornal do Commércio deu um brado esta semana mães, especialmente na velhice.
contra as casas que vendem drogas para curar a gente, B LQÀXHQFLDURkQLPRGDVSHVVRDVOHYDQGRDVDTXHUHU
DFXVDQGRDV GH DV YHQGHU SDUD RXWURV ¿QV PHQRV agir segundo um modelo sugerido.
humanos. Citou os envenenamentos que tem havido
na cidade, mas esqueceu de dizer, ou não acentuou C informar sobre os idosos e sobre seus sentimentos e
bem, que são produzidos por engano das pessoas que necessidades.
manipulam os remédios. Um pouco mais de cuidado, D avaliar matéria publicada em edição anterior de jornal
um pouco menos de distração ou de ignorância, evitarão ou de revista.
males futuros. Mas todo ofício tem uma aprendizagem, e E apresentar nova publicação, visando divulgá-la para
não há benefício humano que não custe mais ou menos leitores de jornal.
duras agonias. Cães, coelhos e outros animais são
vítimas de estudos que lhes não aproveitam, e sim aos
homens; por que não serão alguns destes, vítimas do
que há de aproveitar aos contemporâneos e vindouros?
+i XP DUJXPHQWR TXH GHVID] HP SDUWH WRGRV HVVHV
DWDTXHVjVERWLFDVpTXHRKRPHPpHPVLPHVPRXP
laboratório. Que fundamento jurídico haverá para impedir
que eu manipule e venda duas drogas perigosas? Se elas
matarem, o prejudicado que exija de mim a indenização
que entender; se não matarem, nem curarem, é um
DFLGHQWHHXPERPDFLGHQWHSRUTXHDYLGD¿FD
ASSIS, M. Obra completa. Rio de Janeiro: Nova Aguilar, 1967 (fragmento).

LC - 2º dia | Caderno 6 - CINZA - Página 10


2014 *CINZ25DOM11*
QUESTÃO 110 QUESTÃO 111

TEXTO I A leitura nos tempos do e-book


Não é só nas bibliotecas e livrarias que se encerra
R FRQKHFLPHQWR $ LQWHUQHW SRU PHLR GH VHX LQ¿QLWR
conteúdo, e através de sites como Domínio Público e
muitos outros similares, demonstra as transformações
ocorridas na disponibilização de obras literárias ou
de todas as outras áreas. Sites, como o citado acima,
contêm arquivos com textos digitalizados dos mais
variados autores, dos clássicos aos contemporâneos.
Antes, esse conteúdo todo só seria passível de consulta
em suporte material. O suporte virtual, também conhecido
como e-book, é, digamos, semimaterial, pois nos põe em
contato com o texto através do computador, mas não nos
põe o livro nas mãos, a não ser que queiramos imprimir
o texto digital.
Nossa geração passa por um período de transição
lento que transformará profundamente o hábito da leitura.
Paradoxalmente, a alta velocidade com que se proliferam
as informações faz com que também seja aumentada a
nossa velocidade de captação dessas informações, ou
VHMD DRV SRXFRV H GH PRGR JHUDO D OHLWXUD YDL ¿FDQGR
BANKSY. Disponível em: www.banksy.co.uk. Acesso em: 4 ago. 2012.
FDGD YH] PDLV IUDJPHQWDGD ,VVR Mi DSUHVHQWD UHÀH[RV
TEXTO II no modo como lemos os diversos textos contidos em
Só Deus pode me julgar revistas, jornais ou internet, e igualmente na produção
Soldado da guerra a favor da justiça literária contemporânea.
,JXDOGDGHSRUDTXLpFRLVD¿FWtFLD Disponível em: www.tecnosapiens.com.br. Acesso em: 28 fev. 2012 (adaptado).
Você ri da minha roupa, ri do meu cabelo
Mas tenta me imitar se olhando no espelho A criação dos e-books oferece vantagens e facilidades
Preconceito sem conceito que apodrece a nação SDUD D OHLWXUD 1R WH[WR UHVVDOWDVH D LQÀXrQFLD GHVVH
Filhos do descaso mesmo pós-abolição meio virtual, sobretudo no contexto atual, pois
A as livrarias e bibliotecas estão se tornando lugares
MV BILL. Declaração de guerra. Manaus:BMG, 2002 (fragmento).

O trecho do rap H R JUD¿WH HYLGHQFLDP R SDSHO VRFLDO pouco atrativos para os leitores, uma vez que os livros
das manifestações artísticas e provocam a impressos estão em desuso.
B a semimaterialidade dos e-books garante maior
A consciência do público sobre as razões da interação entre o leitor e o texto.
desigualdade social.
C os e-books possibilitam maior difusão da leitura,
B UHMHLomRGRS~EOLFRDOYRjVLWXDomRUHSUHVHQWDGDQDV tendo em vista a velocidade e a dinamicidade da
obras. informação.
C UHÀH[mRFRQWUDDLQGLIHUHQoDQDVUHODo}HVVRFLDLVGH D DV REUDV FOiVVLFDV H FRQWHPSRUkQHDV ¿FDUDP
forma contundente. JUDWXLWDVGHYLGRjVGLJLWDOL]Do}HVSURSLFLDGDVFRPR
D ideia de que a igualdade é atingida por meio da surgimento da internet.
violência. E a velocidade de proliferação e captação de
E mobilização do público contra o preconceito racial em informações transforma a leitura fragmentada em
contextos diferentes. XPDVROXomRSDUDRDFHVVRjVREUDV

LC - 2º dia | Caderno 6 - CINZA - Página 11


*CINZ25DOM12* 2014

QUESTÃO 112 O uso da expressão “ainda assim” presente nesse texto


WHPFRPR¿QDOLGDGH
Evocação do Recife
A criticar o teor das informações fatuais até ali
A vida não me chegava pelos jornais nem pelos livros veiculadas.
Vinha da boca do povo na língua errada do povo B questionar a validade das ideias apresentadas
Língua certa do povo anteriormente.
Porque ele é que fala gostoso o português do Brasil C comprovar a veracidade das informações expressas
Ao passo que nós anteriormente.
O que fazemos D introduzir argumentos que reforçam o que foi dito
anteriormente.
É macaquear
E enfatizar o contrassenso entre o que é dito antes e o
A sintaxe lusíada… que vem em seguida.
BANDEIRA, M. Estrela da vida inteira.
Rio de Janeiro: Nova Fronteira, 2007. QUESTÃO 114
Segundo o poema de Manuel Bandeira, as variações Cena
linguísticas originárias das classes populares devem ser
O canivete voou
A satirizadas, pois as várias formas de se falar o
E o negro comprado na cadeia
português no Brasil ferem a língua portuguesa
autêntica. Estatelou de costas
E bateu coa cabeça na pedra
B questionadas, pois o povo brasileiro esquece a
ANDRADE, O. Pau-brasil. São Paulo: Globo, 2001.
sintaxe da língua portuguesa.
C subestimadas, pois o português “gostoso” de Portugal O Modernismo representou uma ruptura com os
deve ser a referência de correção linguística. padrões formais e temáticos até então vigentes na
literatura brasileira. Seguindo esses aspectos, o que
D reconhecidas, pois a formação cultural brasileira é
caracteriza o poema Cena como modernista é o(a)
garantida por meio da fala do povo.
E reelaboradas, pois o povo “macaqueia” a língua A construção linguística por meio de neologismo.
portuguesa original. B estabelecimento de um campo semântico inusitado.
C FRQ¿JXUDomR GH XP VHQWLPHQWDOLVPR FRQFLVR H
QUESTÃO 113 irônico.
D subversão de lugares-comuns tradicionais.
Miss Universo: “As pessoas racistas devem procurar
E uso da técnica de montagem de imagens justapostas.
ajuda”
SÃO PAULO — Leila Lopes, de 25 anos, não é a QUESTÃO 115
primeira negra a receber a faixa de Miss Universo. Se observarmos o maxixe brasileiro, a beguine da
A primazia coube a Janelle “Penny” Commissiong, de Martinica, o danzón de Santiago de Cuba e o ragtime
Trinidad e Tobago, vencedora do concurso em 1977. norte-americano, vemos que todos são adaptações
Depois dela vieram Chelsi Smith, dos Estados Unidos, da polca. A diferença de resultado se deve ao sotaque
LQHUHQWH j P~VLFD GH FDGD FRORQL]DGRU SRUWXJXrV
em 1995; Wendy Fitzwilliam, também de Trinidad e espanhol, francês e inglês) e, em alguns casos, a uma
Tobago, em 1998, e Mpule Kwelagobe, de Botswana, PDLRULQÀXrQFLDGDP~VLFDUHOLJLRVD
em 1999. Em 1986, a gaúcha Deise Nunes, que foi a &$=(6+ChoroGRTXLQWDODR0XQLFLSDO6mR3DXOR(GLWRUD DGDSWDGR 
SULPHLUD QHJUD D VH HOHJHU 0LVV %UDVLO ¿FRX HP VH[WR
$OpPGRVRWDTXHLQHUHQWHjP~VLFDGHFDGDFRORQL]DGRU
OXJDU QD FODVVL¿FDomR JHUDO $LQGD DVVLP D HVWXSLGH] H GD LQÀXrQFLD UHOLJLRVD TXH RXWUR HOHPHQWR DX[LOLRX D
humana faz com que, vez ou outra, surjam manifestações constituir os gêneros de música popular citados no texto?
preconceituosas como a de um site EUDVLOHLUR TXH jV A A região da África de origem dos escravos, trazendo
vésperas da competição, e se valendo do anonimato tradições musicais e religiosas de tribos distintas.
de quem o criou, emitiu opiniões do tipo “Como alguém B O relevo dos países, favorecendo o isolamento de
consegue achar uma preta bonita?”Após receber o título, comunidades, aumentando o número de gêneros
a mulher mais linda do mundo — que tem o português musicais surgidos.
FRPROtQJXDPDWHUQDHWDPEpPIDODÀXHQWHPHQWHRLQJOrV C O conjunto de portos, que favorecem o trânsito de
— disse o que pensa de atitudes como essa e também GLIHUHQWHVLQÀXrQFLDVPXVLFDLVHFUHGRVUHOLJLRVRV
sobre como sua conquista pode ajudar os necessitados D A agricultura das regiões, pois o que é plantado exerce
de Angola e de outros países. LQÀXrQFLDQDVFDQo}HVGHWUDEDOKRGXUDQWHRSODQWLR
E O clima dos países em questão, pois as temperaturas
COSTA, D. Disponível em: http://oglobo.globo.com. Acesso em: 10 set. 2011 (adaptado).
LQÀXHQFLDPQDFRPSRVLomRHYLYDFLGDGHGRVULWPRV
LC - 2º dia | Caderno 6 - CINZA - Página 12
2014 *CINZ25DOM13*
QUESTÃO 116 Quais as vantagens e desvantagens do ensino da
língua por meio das letras de música?
6HPÀHFKDQDULPD Não sou pedagogo ou educador, então só vejo
vantagens, porque as letras de música usam uma
2JUXSRGHUDS%U{0&VFULDGRQR¿QDOGHp
linguagem que é a do dia a dia, principalmente, dos jovens.
formado pelos pares de irmãos (daí o “bro”, de brother)
A música é algo que lhes dá prazer e, didaticamente,
Bruno/Clemerson e Kelvin/Charles, jovens que cresceram
pode fazer as vezes de algo que o aluno tem a noção de
ouvindo hip hop nas rádios da aldeia Jaguapiru Bororo, ser entediante — estudo da língua, sentar e abrir um livro.
em Dourados, Mato Grosso do Sul. Ao ouvir uma música, os exemplos surgem. É a grande
— Desde o começo a gente não queria impor uma vantagem e sempre foi a ideia do programa.
cultura estranha que invadisse a cultura indígena — Disponível em: http://revistalingua.uol.com.br. Acesso em: 8 ago. 2012 (fragmento).
D¿UPD R SURGXWRU FKDPDQGR D DWHQomR SDUD R JUDQGH 2V JrQHURV WH[WXDLV VmR GH¿QLGRV SRU PHLR GH VXD
destaque do Brô MCs: as letras em língua indígena. estrutura, função e contexto de uso.Tomando por base a
Expressar-se em língua originária e fazer com que os estrutura dessa entrevista, observa-se que
jovens indígenas percebam a vitalidade do idioma nativo
é uma das motivações do grupo. A a organização em turnos de fala reproduz o diálogo
que ocorre entre os interlocutores.
$GL¿FXOGDGHPDLRUYHPGRVFUtWLFRVTXHQmRDFHLWDP B R WHPD H R VXSRUWH RQGH IRL SXEOLFDGD MXVWL¿FDP D
o fato de que a cultura indígena é dinâmica e sempre ausência de traços da linguagem informal.
incorpora novidades.
C a ausência de referências sobre o entrevistado é uma
— “Mas índio cantando rap?”, tem gente que HVWUDWpJLDSDUDLQGX]LUjOHLWXUDGRWH[WRQDtQWHJUD
questiona. O rap é de quem canta, é de quem gosta, não D RXVRGRGHVWDTXHJUi¿FRpXPUHFXUVRGHHGLomRSDUD
é só dos americanos — avalia Dani [o vocal feminino]. ressaltar a importância do tema para o entrevistador.
BONFIM, E. Revista Língua Portuguesa, n. 81, jul. 2012 (adaptado). E o entrevistado é um especialista em abordagens
educacionais alternativas para o ensino da língua
Considerando-se as opiniões apresentadas no texto, a
portuguesa.
indagação “Mas índio cantando rap?” traduz um ponto de
vista que evidencia QUESTÃO 118
A GHVTXDOL¿FDomR GRV LQGtJHQDV FRPR P~VLFRV Fogo frio
desmerecendo sua capacidade musical devido a O Poeta
sua cultura. A névoa que sobe
B desvalorização da cultura rap HP FRQWUDSDUWLGD jV dos campos, das grotas, do fundo dos vales,
tradições musicais indígenas, motivo pelo qual os é o hálito quente da terra friorenta.
índios não devem cantar rap. O Lavrador
C preconceito por parte de quem não concebe que os Engana-se, amigo.
índios possam conhecer o rap e, menos ainda, cantar Aquilo é fumaça que sai da geada.
esse gênero musical.
O Poeta
D equívoco por desconsiderar as origens culturais do Fumaça, que eu saiba,
gênero musical, ligadas ao contexto urbano. somente de chama e brasa é que sai!
E entendimento do rap como um gênero ultrapassado O Lavrador
HPUHODomRjOLQJXDJHPPXVLFDOGRVLQGtJHQDV E, acaso, a geada não é
fogo branco caído do céu,
QUESTÃO 117
tostando tudinho, crestando tudinho, queimando tudinho,
Entrevista — Tony Bellotto sem pena, sem dó?
FORNARI, E. Trem da serra. Porto Alegre: Acadêmica, 1987.
A língua é rock
Neste diálogo poético, encena-se um embate de ideias
Guitarrista do Titãs e escritor completa dez anos à entre o Poeta e o Lavrador, em que
frente de programa televisivo em que discute a língua
A a vitória simbólica é dada ao discurso do lavrador
portuguesa por meio da música
e tem como efeito a renovação de uma linguagem
2TXHRDWUDLXQDSURSRVWDGH$¿QDQGRD/tQJXD" poética cristalizada.
No começo, em 1999, a ideia era fazer um programa B as duas visões têm a mesma importância e
que falasse de língua portuguesa usando a música como são equivalentes como experiência de vida e a
atrativo, principalmente, para os jovens. Com o passar capacidade de expressão.
do tempo, ele foi se transformando num programa sobre C o autor despreza a sabedoria popular e traça uma
a linguagem usada em letras de música, no jornalismo, caricatura do discurso do lavrador, simplório e repetitivo.
QD OLWHUDWXUD GH ¿FomR H QD SRHVLD &RPR QmR VRX XP D DV LPDJHQV FRQWUDGLWyULDV GH IULR H IRJR UHIHULGDV j
cara de TV, trago a experiência de escritor e músico, e geada compõem um paradoxo que o poema não é
sempre participo de forma mais ativa do que como um capaz de organizar.
mero apresentador. Estou nas reuniões de pauta e faço E R GLVFXUVR GR ODYUDGRU ID] XPD SHUVRQL¿FDomR
sugestões nos roteiros. Mas o conteúdo é feito pelo da natureza para explicar o fenômeno climático
pessoal do Futura. observado pelos personagens.

/&žGLD_&DGHUQR&,1=$3iJLQD
*CINZ25DOM14* 2014

QUESTÃO 119 QUESTÃO 121


Hipertextualidade
O papel do hipertexto é exatamente o de reunir, não
apenas os textos, mas também as redes de associações,
DQRWDo}HV H FRPHQWiULRV jV TXDLV HOHV VmR YLQFXODGRV
pelas pessoas. Ao mesmo tempo, a construção do senso
comum encontra-se exposta e como que materializada:
a elaboração coletiva de um hipertexto.
Trabalhar, viver, conversar fraternalmente com outros
'$+0(5$'LVSRQtYHOHPKWWSURXQG¿QDOEORJVSRWFRPEU VHUHV FUX]DU XP SRXFR SRU VXD KLVWyULD LVWR VLJQL¿FD
Acesso em: 14 dez. 2012.
entre outras coisas, construir uma bagagem de referências
Na tirinha, o autor utiliza estratégias para atingir sua HDVVRFLDo}HVFRPXQVXPDUHGHKLSHUWH[WXDOXQL¿FDGD
¿QDOLGDGH FRPXQLFDWLYD &RQVLGHUDQGR RV HOHPHQWRV um texto compartilhado, capaz de diminuir os riscos de
verbais e não verbais que constituem o texto, seu objetivo é incompreensão.
LEVY, P. As tecnologias da inteligência: o futuro do pensamento na era da informática.
A incentivar o uso da tecnologia na comunicação 6mR3DXOR(GLWRUD DGDSWDGR 
contemporânea. O texto evidencia uma relação entre o hipertexto e a
B mostrar o empenho do homem na resolução de sociedade em que essa tecnologia se insere. Constata-se
problemas sociais. que, nessa relação, há uma
C atrair a atenção do leitor para a generosidade das A estratégia para manutenção do senso comum.
pessoas.
B prioridade em sanar a incompreensão.
D chamar a atenção para o constante abandono de
C necessidade de publicidade das informações.
animais.
D forma de construção colaborativa de conhecimento.
E ID]HUXPDFUtWLFDjVLWXDomRVRFLDOFRQWHPSRUkQHD
E urgência em se estabelecer o diálogo entre pessoas.
QUESTÃO 120
QUESTÃO 122
Você se preocupa com sua família,
Mãos dadas
com seu trabalho e com sua casa.
Não serei o poeta de um mundo caduco.
E com você? Também não cantarei o mundo futuro.
(VWRXSUHVRjYLGDHROKRPHXVFRPSDQKHLURV
A mulher conquistou um espaço de destaque no
Estão taciturnos mas nutrem grandes esperanças.
DPELHQWH SUR¿VVLRQDO DOpP GH FXLGDU GD FDVD H GR Entre eles, considero a enorme realidade.
bem-estar da família. Acompanhada por essa mudança, O presente é tão grande, não nos afastemos.
também veio uma nova vida, com antigos hábitos Não nos afastemos muito, vamos de mãos dadas.
tipicamente masculinos, como o estresse, a falta de Não serei o cantor de uma mulher, de uma história.
tempo para se cuidar, o tabagismo e a maior incidência Não direi suspiros ao anoitecer, a paisagem vista na janela.
de obesidade e depressão. Isso aumentou muito os casos Não distribuirei entorpecentes ou cartas de suicida.
de infarto e doenças cardiovasculares. Elas já respondem 1mRIXJLUHLSDUDLOKDVQHPVHUHLUDSWDGRSRUVHUD¿QV
SRU  GR Q~PHUR WRWDO GRV FDVRV TXH PDWDP VHLV O tempo é a minha matéria, o tempo presente, os
vezes mais do que o câncer de mama. homens presentes,
a vida presente.
Cuide-se. Preocupe-se com sua saúde. Visite e
ANDRADE, C. D. Sentimento do mundo. São Paulo: Cia. das Letras, 2012.
incentive quem você gosta a visitar um cardiologista.
Escrito em 1940, o poema Mãos dadas revela um eu lírico
Cláudia,DQRQIHY DGDSWDGR 
marcado pelo contexto de opressão política no Brasil e
Esse texto, publicado em uma revista, inicialmente aponta da Segunda Guerra Mundial. Em face dessa realidade,
PRGL¿FDo}HVRFRUULGDVQDVRFLHGDGHHHPVHJXLGD o eu lírico
A descreve as diferentes atividades das mulheres hoje A considera que em sua época o mais importante é a
em dia. independência dos indivíduos.
B estimula as leitoras a buscar sua realização na vida B desvaloriza a importância dos planos pessoais na
SUR¿VVLRQDO vida em sociedade.
C alerta as mulheres para a possibilidade de problemas C UHFRQKHFH D WHQGrQFLD j DXWRGHVWUXLomR HP XPD
cardíacos. sociedade oprimida.
D informa as leitoras sobre mortes por câncer de mama D escolhe a realidade social e seu alcance individual
e por infarto. como matéria poética.
E valoriza as mulheres preocupadas com o bem-estar E critica o individualismo comum aos românticos e
da família. aos excêntricos.
LC - 2º dia | Caderno 6 - CINZA - Página 14
2014 *CINZ25DOM15*
QUESTÃO 123 O advento da escrita como tecnologia intelectual está
diretamente ligado a uma série de mudanças na forma
Sermão da Sexagésima de pensar e de construir o conhecimento nas sociedades.
A partir do texto, constata-se que, na elaboração do
Nunca na Igreja de Deus houve tantas pregações,
GLVFXUVRFLHQWt¿FRDHVFULWD
nem tantos pregadores como hoje. Pois se tanto se
semeia a palavra de Deus, como é tão pouco o fruto? A GHWHUPLQRX GH TXH PRGR D VRFLHGDGH FLHQWt¿FD
Não há um homem que em um sermão entre em si e se deveria se organizar para avançar.
resolva, não há um moço que se arrependa, não há um B possibilitou que os pesquisadores se distanciassem
velho que se desengane. Que é isto? Assim como Deus de informações presentes na memória.
não é hoje menos onipotente, assim a sua palavra não C permitiu que fossem documentados conceitos e
é hoje menos poderosa do que dantes era. Pois se a saberes advindos de experiências realizadas.
palavra de Deus é tão poderosa; se a palavra de Deus D IDFLOLWRX TXH DV LQIRUPDo}HV ¿FDVVHP DUPD]HQDGDV
tem hoje tantos pregadores, por que não vemos hoje igualmente na memória e no papel.
nenhum fruto da palavra de Deus? Esta, tão grande e E consentiu que a atenção dos homens se desviasse
tão importante dúvida, será a matéria do sermão. Quero para os saberes antigamente inalcançáveis.
começar pregando-me a mim. A mim será, e também
a vós; a mim, para aprender a pregar; a vós, que QUESTÃO 125
aprendais a ouvir.
VIEIRA, A. Sermões Escolhidos, v. 2. São Paulo: Edameris, 1965.
O mulato
No Sermão da sexagésima, padre Antônio Vieira Ana Rosa cresceu; aprendera de cor a gramática do
TXHVWLRQDDH¿FiFLDGDVSUHJDo}HV3DUDWDQWRDSUHVHQWD Sotero dos Reis; lera alguma coisa; sabia rudimentos
como estratégia discursiva sucessivas interrogações, as de francês e tocava modinhas sentimentais ao violão e
quais têm por objetivo principal ao piano. Não era estúpida; tinha a intuição perfeita da
virtude, um modo bonito, e por vezes lamentara não ser
A SURYRFDUDQHFHVVLGDGHHRLQWHUHVVHGRV¿pLVVREUH mais instruída. Conhecia muitos trabalhos de agulha;
o conteúdo que será abordado no sermão. bordava como poucas, e dispunha de uma gargantazinha
B FRQGX]LURLQWHUORFXWRUjVXDSUySULDUHÀH[mRVREUHRV de contralto que fazia gosto de ouvir.
temas abordados nas pregações. 8PD Vy SDODYUD ERLDYD j VXSHUItFLH GRV VHXV
C apresentar questionamentos para os quais a Igreja pensamentos: “Mulato”. E crescia, crescia, transformando-
não possui respostas. se em tenebrosa nuvem, que escondia todo o seu
passado. Ideia parasita, que estrangulava todas as outras
D LQVHULU DUJXPHQWRV j WHVH GHIHQGLGD SHOR SUHJDGRU
ideias.
VREUHDH¿FiFLDGDVSUHJDo}HV
E questionar a importância das pregações feitas pela — Mulato!
Igreja durante os sermões. Esta só palavra explicava-lhe agora todos os
mesquinhos escrúpulos, que a sociedade do Maranhão
QUESTÃO 124 usara para com ele. Explicava tudo: a frieza de certas
famílias a quem visitara; as reticências dos que lhe falavam
A escrita é uma tecnologia intelectual que vem
de seus antepassados; a reserva e a cautela dos que, em
auxiliar o trabalho biológico. É como uma nova memória, sua presença, discutiam questões de raça e de sangue.
situada fora do sujeito, e ilimitada. Com ela não é mais
AZEVEDO, A. O Mulato. São Paulo: Ática, 1996 (fragmento).
necessário reter todos os relatos – este auxiliar cognitivo
vem, portanto, relativizar a memória para que a mente O texto de Aluísio Azevedo é representativo do
humana possa desviar sua atenção consciente para 1DWXUDOLVPR YLJHQWH QR ¿QDO GR VpFXOR ;,; 1HVVH
outros recursos e faculdades. IUDJPHQWR R QDUUDGRU H[SUHVVD ¿GHOLGDGH DR GLVFXUVR
naturalista, pois
Se é arriscado associar diretamente o surgimento
A relaciona a posição social a padrões de comportamento
da ciência ao da escrita, podemos, de qualquer forma,
HjFRQGLomRGHUDoD
D¿UPDU TXH D HVFULWD GHX LPSXOVR H GHVHPSHQKRX XP
SDSHO IXQGDPHQWDO QD FRQVWUXomR GR GLVFXUVR FLHQWt¿FR B apresenta os homens e as mulheres melhores do que
eram no século XIX.
2GLVWDQFLDPHQWRSRVVLELOLWDGRSHODJUD¿DQRSDSHOWUD]R
registro das experiências e das hipóteses, o conhecimento C mostra a pouca cultura feminina e a distribuição de
especulativo, o documentário de comprovações, a saberes entre homens e mulheres.
compilação de teorias e de paradigmas em torno dos D ilustra os diferentes modos que um indivíduo tinha de
TXDLVDVFRPXQLGDGHVFLHQWt¿FDVYmRVHDJUXSDU ascender socialmente.
RAMAL, A. C. Educação na cibercultura: hipertextualidade, leitura, escrita e aprendizagem. E FULWLFD D HGXFDomR RIHUHFLGD jV PXOKHUHV H RV
Porto Alegre: Artmed, 2002. maus-tratos dispensados aos negros.
LC - 2º dia | Caderno 6 - CINZA - Página 15
*CINZ25DOM16* 2014

QUESTÃO 126 Nesse trecho de uma obra de Guimarães Rosa,


A internet amplia o que queremos e desejamos. depreende-se a predominância de uma das funções da
Pessoas alienadas se alienam mais na internet. Pessoas OLQJXDJHPLGHQWL¿FDGDFRPR
interessantes tornam a comunicação com a internet mais
A metalinguística, pois o trecho tem como propósito
interessante. Pessoas abertas utilizam a internet para
promover mais interação e compartilhamento. Pessoas essencial usar a língua portuguesa para explicar
individualistas se fecham mais ainda nos ambientes a própria língua, por isso a utilização de vários
GLJLWDLV3HVVRDVTXHWrPGL¿FXOGDGHVGHUHODFLRQDPHQWR VLQ{QLPRVHGH¿QLo}HV
na vida real muitas vezes procuram mil formas de fuga
B referencial, pois o trecho tem como principal objetivo
para o virtual. Aproveitaremos melhor as possibilidades
da internet, se equilibrarmos a qualidade das interações discorrer sobre um fato que não diz respeito ao
SUHVHQFLDLV²QDYLGDSHVVRDOSUR¿VVLRQDOHPRFLRQDO² escritor ou ao leitor, por isso o predomínio da terceira
com as interações digitais correspondentes. pessoa.
025$1-0'LVSRQtYHOHPZZZHFDXVSEU$FHVVRHPMXO DGDSWDGR 
C fática, pois o trecho apresenta clara tentativa de
O texto expressa um posicionamento a respeito do estabelecimento de conexão com o leitor, por isso
uso da internet e suas repercussões na vida cotidiana. o emprego dos termos “sabe-se lá” e “tome-se
Na opinião do autor, esse sistema de informação e hipotrélico”.
comunicação
D poética, pois o trecho trata da criação de palavras
A aumenta o número de pessoas alienadas.
novas, necessária para textos em prosa, por isso o
B resolve problemas de relacionamento.
emprego de “hipotrélico”.
C soluciona a questão do individualismo.
E expressiva, pois o trecho tem como meta mostrar a
D equilibra as interações presenciais.
subjetividade do autor, por isso o uso do advérbio de
E potencializa as características das pessoas.
dúvida “talvez”.
QUESTÃO 127
QUESTÃO 129
A tendência dos nomes
O nome é uma das primeiras coisas que não Eu vô transmiti po sinhô logo uma passage muito
escolhemos na vida. Estará inscrito nos registros: na importante, qu’ eu iscutei um velho de nome Ricardo
PDWHUQLGDGH QR 5* QR &3) QR RELWXiULR HWF (Q¿P Caetano Alves, que era neto do propietário da Fazenda
XPDHVFROKDTXHQmR¿]HPRVQRVDFRPSDQKDGREHUoR do Buraca. O pai dele, ele contava que o pai dele assistiu
ao túmulo, pois na lápide se dirá que ali jaz Fulano de Tal. uma cena muito importante aonde ele tava, do Jacarandá,
SILVA, D. Língua, n. 77, mar. 2012.
o chefe dos iscravo do Joaquim de Paula, com o chefe
Algumas palavras atuam no desenvolvimento de um texto dos iscravo do Vidigal, que chamava, era tratado Pai
FRQWULEXLQGR SDUD D VXD SURJUHVVmR $ SDODYUD ³HQ¿P´ Urubu. O Jacarandá era tratado Jacarandá purque ele
promove o encadeamento do texto, tendo sido utilizada
era um negro mais vermelho, tá intendeno com’ é que
com a intenção de
é, né? Intão é uma imitância de cerno de Jacarandá,
A explicar que os nomes das pessoas são escolhidos intão eles apilidaro ele de Pai Jacarandá. Agora, o Pai
no nascimento.
Urubu, diz que era o mais preto de todos os iscravo que
B UDWL¿FDUTXHRVQRPHVUHJLVWUDGRVQRQDVFLPHQWRVmR HUDFXQKLFLGRQHVVDpSRFD,QWmRHOH¿F{FRPRQRPH3DL
imutáveis.
8UXEX e TXHP GLULJLD GH WRGD FRQ¿DQoD GRV VLQKRUHV
C reiterar que os nomes recebidos são importantes até
Intão os sinhores cunhiciam eles como “pai”: Pai Urubu,
a morte.
Pai Jacarandá, Pai Francisco, que é o chefe da Fazenda
D concluir que os nomes acompanham os indivíduos
até a morte. das Abóbra, Pai Dumingo, que era da Fazenda do Buraca.
E acrescentar que ninguém pode escolher o próprio nome. SOUZA, J. Negros pelo vale%HOR+RUL]RQWH)DOH8)0*

QUESTÃO 128 O texto é uma transcrição da narrativa oral contada


+i R KLSRWUpOLFR 2 WHUPR p QRYR GH LPSHQVDGD por Pedro Braga, antigo morador do povoado Vau, de
RULJHPHDLQGDVHPGH¿QLomRTXHOKHDSDQKHHPWRGDV Diamantina (MG). Com base no registro da fala do
DV SpWDODV R VLJQL¿FDGR 6DEHVH Vy TXH YHP GR ERP narrador, entende-se que seu relato
português. Para a prática, tome-se hipotrélico querendo
dizer: antipodático, sengraçante imprizido; ou talvez, vice- A perpetua a memória e os saberes dos antepassados.
dito: indivíduo pedante, importuno agudo, falta de respeito B constrói uma voz dissonante da identidade nacional.
para com a opinião alheia. Sob mais que, tratando-se de
palavra inventada, e, como adiante se verá, embirrando C demonstra uma visão distanciada da cultura negra.
o hipotrélico em não tolerar neologismos, começa ele por D revela uma visão unilateral dos fazendeiros.
se negar nominalmente a própria existência. E transmite pouca experiência e sabedoria.
ROSA, G. Tutameia: terceiras estórias. Rio de Janeiro: Nova Fronteira, 2001 (fragmento).

LC - 2º dia | Caderno 6 - CINZA - Página 16


2014 *CINZ25DOM17*
QUESTÃO 130 A variação linguística afeta o processo de produção dos
sentidos no texto. No relato envolvendo Rui Barbosa, o
emprego das marcas de variação objetiva
A evidenciar a importância de marcas linguísticas
valorizadoras da linguagem coloquial.
B demonstrar incômodo com a variedade característica
de pessoas pouco escolarizadas.
C HVWDEHOHFHU XP MRJR GH SDODYUDV D ¿P GH SURGX]LU
efeito de humor.
D criticar a linguagem de pessoas originárias de fora
dos centros urbanos.
E HVWDEHOHFHUXPDSROtWLFDGHLQFHQWLYRjHVFULWDFRUUHWD
das palavras.

QUESTÃO 132

Reciclar é só parte da solução


O lixo é um grande problema da sustentabilidade.
Literalmente: todos os anos, cada brasileiro produz
NJGHUHVtGXRV²GiPLOK}HVGHWRQHODGDVQRWRWDO
O certo seria tentar diminuir ao máximo essa quantidade
de lixo. Ou seja, em vez de ter objetos recicláveis, o ideal
seria produzir sempre objetos reutilizáveis, o que diminui
os resíduos. Mas, enquanto isso não acontece, temos
que nos contentar com a reciclagem. E é aí que vem um
detalhe perigoso: reciclar o lixo também polui o ambiente
HJDVWDHQHUJLD5HFLFODUYLGURSRUH[HPSORpPDLV
caro do que produzi-lo a partir de matérias-primas virgens.
$¿QDO, é feito basicamente de areia, soda e calcário,
Disponível em: http://vicostudio.blogspot.com.br. Acesso em: 1 ago. 2012. que são abundantes na natureza. Então, nenhuma
Essa propaganda visa convencer as mães de que o empresa tem interesse em reciclá-lo. Já o alumínio é um
FDQDOGHWHOHYLVmRpDGHTXDGRDRVVHXV¿OKRV3DUDWDQWR supernegócio, porque economiza muita energia.
o locutor dirige-se ao interlocutor por meio de estratégias +257$0'LVSRQtYHOHPKWWSVXSHUDEULOFRPEU$FHVVRHPPDLR
argumentativas de
O emprego adequado dos elementos de coesão
A manipulação, ao detalhar os programas infantis que contribui para a construção de um texto argumentativo
compõem a grade da emissora. e para que os objetivos pretendidos pelo autor possam
B persuasão, ao evidenciar as características da ser alcançados. A análise desses elementos no texto
programação dirigida ao público infantil. mostra que o conectivo
C LQWLPLGDomR DR GLULJLUVH GLUHWDPHQWH jV PmHV SDUD A “ou seja” introduz um esclarecimento sobre a
FKDPiODVjUHÀH[mR diminuição da quantidade de lixo.
D comoção, ao tranquilizar as mães sobre a qualidade
B ³PDV´LQVWDXUDMXVWL¿FDWLYDVSDUDDFULDomRGHQRYRV
dos programas da emissora.
tipos de reciclagem.
E comparação, ao elencar os serviços oferecidos por
outras emissoras ao público infantil. C “também” antecede um argumento a favor da
reciclagem.
QUESTÃO 131 D ³D¿QDO´UHWRPDXPD¿QDOLGDGHSDUDRXVRGHPDWpULDV
primas.
Contam, numa anedota, que certo dia Rui Barbosa
VDLXjVUXDVGDFLGDGHHVHDVVXVWRXFRPDTXDQWLGDGH E “então” reforça a ideia de escassez de matérias-
de erros existentes nas placas das casas comerciais primas na natureza.
e que, diante disso, resolveu instituir um prêmio em
dinheiro para o comerciante que tivesse o nome de seu
estabelecimento grafado corretamente. Dias depois,
5XL %DUERVD VDLX j SURFXUD GR YHQFHGRU 6DWLVIHLWR
encontrou a placa vencedora: “Alfaiataria Águia de Ouro”.
No momento da entrega do prêmio, ao dizer o nome da
alfaiataria, Rui Barbosa foi interrompido pelo alfaiate
premiado, que disse:
— Sr. Rui, não é “águia de ouro”; é “aguia de ouro”!
O caráter político do ensino de língua portuguesa no Brasil. Disponível em:
http://rosabe.sites.uol.com.br. Acesso em: 2 ago. 2012.

LC - 2º dia | Caderno 6 - CINZA - Página 17


*CINZ25DOM18* 2014

QUESTÃO 133 QUESTÃO 135


Senhora E-mail no ambiente de trabalho
²0mHQRRRRVVVD(VVHVHXFDEHORQRYR¿FRXOLQGR T. C., consultor e palestrante de assuntos ligados
Parece que você é, tipo, mais jovem! ao mercado de trabalho, alerta que a objetividade, a
²-XUDPLQKD¿OKD"2EULJDGD RUJDQL]DomR GD PHQVDJHP VXD FRHUrQFLD H RUWRJUD¿D
são pontos de atenção fundamentais para uma
— Mas aí você vira de frente e aí a gente vê que, tipo,
FRPXQLFDomRYLUWXDOH¿FD]
não é, né?
E, para evitar que erros e falta de atenção resultem
— Coisa linda da mamãe! HPVDLDVMXVWDVHVLWXDo}HVFRQVWUDQJHGRUDVFRQ¿UDFLQFR
Esse diálogo é real. Claro que achei graça, mas o fato dicas para usar o e-mail com bom senso e organização:
de envelhecer já não é mais segredo para ninguém. 5HVSRQGD jV PHQVDJHQV LPHGLDWDPHQWH DSyV
Um belo dia, a vendedora da loja te pergunta: “A recebê-las.
senhora quer pagar como?” Senhora? Como assim? 2. Programe sua assinatura automática em todas as
Eu sempre fui a Marcinha! Agora eu sou a dona respostas e encaminhamentos.
Márcia! Sim, o porteiro, o motorista de táxi, o jornaleiro, $R¿QDOGRGLDH[FOXDDVPHQVDJHQVVHPLPSRUWkQFLD
o garçom, o mundo inteiro resolveu ter um respeito HDUTXLYHDVGHPDLVHPSDVWDVSUHYLDPHQWHGH¿QLGDV
comigo que eu não pedi! 8WLOL]H R UHFXUVR GH ³FRQ¿UPDomR GH OHLWXUD´ VRPHQWH
CABRITA, M. Disponível em: www.istoe.com.br. Acesso em: 11 ago. 2012 (fragmento). quando necessário.
5. Evite mensagens do tipo “corrente”.
A exploração de registros linguísticos é importante
'LVSRQtYHOHPKWWSQRWLFLDVXROFRPEU$FHVVRHPMXO IUDJPHQWR 
estratégia para o estabelecimento do efeito de sentido
pretendido em determinados textos. No texto, o recurso O texto apresenta algumas sugestões para o leitor.
a diferentes registros indica Esse caráter instrucional é atribuído, principalmente,
pelo emprego
A mudança na representação social do locutor.
B UHÀH[mRVREUHDLGHQWLGDGHSUR¿VVLRQDOGDPmH A do modo verbal imperativo, como em “responda” e
“programe”.
C referência ao tradicionalismo linguístico da autora do
texto. B GDV PDUFDV GH TXDOL¿FDomR GR HVSHFLDOLVWD FRPR
“consultor” e “palestrante”.
D HORJLR jV VLWXDo}HV YLYHQFLDGDV SHOD SHUVRQDJHP
mãe. C GHWHUPRVHVSHFt¿FRVGRGLVFXUVRQRPXQGRYLUWXDO
E compreensão do processo de envelhecimento como D GHDUJXPHQWRVIDYRUiYHLVjFRPXQLFDomRH¿FD]
algo prazeroso. E da palavra “dica” no desenvolvimento do texto.

QUESTÃO 134
História de assombração
Ah! Eu alembro uma história que aconteceu com
meu tii. Era dia de Sexta-Feira da Paixão, diz que eles
falava pra meu tii não num vai pescá não. Ele foi assim
mesmo, aí chegô lá, ele tá pescano... tá pescano... e
nada de pexe. Aí saiu um mundo véi de cobra em cima
dele, aí ele foi embora... Aí até ele memo contava isso e
falava É... nunca mais eu vou pescar no dia de Sexta-Feira
da Paixão...
COSTA, S. A. S. Narrativas tradicionais tapuias. Goiânia: UFG, 2011 (adaptado).

4XDQWR DR JrQHUR GR GLVFXUVR H j ¿QDOLGDGH VRFLDO GR


texto História de assombração, a organização textual e as
escolhas lexicais do locutor indicam que se trata de um(a)
A FULDomR OLWHUiULD HP SURVD TXH SURYRFD UHÀH[mR
acerca de problemas cotidianos.
B texto acadêmico, que valoriza o estudo da linguagem
regional e de suas variantes.
C relato oral, que objetiva a preservação da herança
cultural da comunidade.
D conversa particular, que favorece o compartilhar de
informações e experiências pessoais.
E anedota regional, que evidencia a fala e o vocabulário
exclusivo de um grupo social.

LC - 2º dia | Caderno 6 - CINZA - Página 18


2014 *CINZ25DOM19*
MATEMÁTICA E SUAS TECNOLOGIAS A vista superior da molécula, como representada na
¿JXUDp
Questões de 136 a 180
QUESTÃO 136
Em uma escola, cinco atletas disputam a medalha de
ouro em uma competição de salto em distância. Segundo
o regulamento dessa competição, a medalha de ouro será A
dada ao atleta mais regular em uma série de três saltos.
Os resultados e as informações dos saltos desses cinco
atletas estão no quadro.

Desvio
Atleta 1º salto 2º salto 3º salto Média Mediana padrão
I 2,9     0,25
II  2,8    0,40
III      0,17
IV      0,60 B
V   2,2   0,81

A medalha de ouro foi conquistada pelo atleta número


A I.
B II.
C III.
D IV.
E V.
C
QUESTÃO 137
$ ¿JXUD p XPD UHSUHVHQWDomR WULGLPHQVLRQDO GD
PROpFXOD GR KH[DÀXRUHWR GH HQ[RIUH TXH WHP D IRUPD
bipiramidal quadrada, na qual o átomo central de enxofre
HVWiFHUFDGRSRUVHLViWRPRVGHÀ~RUVLWXDGRVQRVVHLV
vértices de um octaedro. O ângulo entre qualquer par de
OLJDo}HVHQ[RIUHÀ~RUDGMDFHQWHVPHGHƒ

90°

Legenda:

Enxofre

Flúor

Ligação

'LVSRQtYHOHPZZZSRUWDOVDRIUDQFLVFRFRPEU$FHVVRHPPDU DGDSWDGR 

MT - 2º dia | Caderno 6 - CINZA - Página 19


*CINZ25DOM20* 2014

QUESTÃO 138 QUESTÃO 139


Corta-se um cubo ABCDEFGH por um plano
Após encerrar o período de vendas de 2012, uma
RUWRJRQDO jV IDFHV ABCD e EFGH que contém os
concessionária fez um levantamento das vendas de
pontos médios I e J das arestas CD e BC e elimina-se,
carros novos no último semestre desse ano. Os dados
em seguida, o prisma IJCLKG, obtendo-se o prisma
HVWmRH[SUHVVRVQRJUi¿FR
ABJIDEFKLH.
A B 40
J 35 35

I
D 30
C

Carros vendidos
25

20
14

10
E F
5 6

K 0
H G
Julho Agosto Setembro Outubro Novembro Dezembro
L

$ SODQL¿FDomR GD VXSHUItFLH GR SULVPD UHVXOWDQWH Ao fazer a apresentação dos dados aos funcionários,
ABJIDEFKLHFRUUHVSRQGHj¿JXUD o gerente estipulou como meta para o mês de janeiro
GH  XP YROXPH GH YHQGDV  VXSHULRU j PpGLD
mensal de vendas do semestre anterior.
Para atingir essa meta, a quantidade mínima de carros
A TXHGHYHULDPVHUYHQGLGRVHPMDQHLURGHVHULD
A 17.
B 20.
C 21.
D 24.
B E 

QUESTÃO 140
Uma loja decide premiar seus clientes. Cada cliente
receberá um dos seis possíveis brindes disponíveis,
conforme sua ordem de chegada na loja. Os brindes a
serem distribuídos são: uma bola, um chaveiro, uma
caneta, um refrigerante, um sorvete e um CD, nessa
C ordem. O primeiro cliente da loja recebe uma bola, o
segundo recebe um chaveiro, o terceiro recebe uma
caneta, o quarto recebe um refrigerante, o quinto
recebe um sorvete, o sexto recebe um CD, o sétimo
recebe uma bola, o oitavo recebe um chaveiro, e assim
sucessivamente, segundo a ordem dos brindes.
O milésimo cliente receberá de brinde um(a)
A bola.
D B caneta.
C refrigerante.
D sorvete.
E CD.

MT - 2º dia | Caderno 6 - CINZA - Página 20


2014 *CINZ25DOM21*
QUESTÃO 141
O ferro é um mineral fundamental para que as células mantenham seu bom funcionamento. Ele é essencial ao
WUDQVSRUWH GH R[LJrQLR VtQWHVH GH '1$ H PHWDEROLVPR HQHUJpWLFR e UHFRPHQGDGR SDUD PHQLQRV GH  D  DQRV
ingerirem, pelo menos, 8 mg de ferro por dia.
Pesquisadores elaboraram a tabela com alguns alimentos e as suas respectivas quantidades de ferro:

Alimento (100 g) Ferro (mg)


Coração de frango 6,5
Sardinha em conserva 
Amêndoa 
Caldo de cana 
Lentilha 1,5
Batata-doce 1,5
Feijão carioca 
Filé de frango (peito) 
A diretora de uma escola sabe que deve escolher para o almoço de seus alunos o máximo de cardápios possíveis
entre três cardápios existentes, no(s) qual(is) cada porção equivale a 100 g e cada copo a 50 g.

CARDÁPIO 1 CARDÁPIO 2 CARDÁPIO 3

2 porções de feijão carioca 2 copos de caldo de cana 2 porções de lentilha


1 porção de coração de frango 1 porção de sardinha em conserva SRUo}HVGH¿OpGHIUDQJR
1 porção de amêndoa 2 porções de feijão carioca 2 porções de batata doce
Disponível em: www.rgnutri.com.br. Acesso em: 2 ago. 2012 (adaptado).

Para ter certeza de que seus alunos estão ingerindo a quantidade mínima de ferro recomendada, a diretora deve
escolher o(s) cardápio(s)
A 1.
B 2.
C 
D 1 ou 2.
E RX

QUESTÃO 142
Um estudante se cadastrou numa rede social na internet que exibe o índice de popularidade do usuário.
(VVHtQGLFHpDUD]mRHQWUHRQ~PHURGHDGPLUDGRUHVGRXVXiULRHRQ~PHURGHSHVVRDVTXHYLVLWDPVHXSHU¿OQDUHGH
$RDFHVVDUVHXSHU¿OKRMHRHVWXGDQWHGHVFREULXTXHVHXtQGLFHGHSRSXODULGDGHp
2tQGLFHUHYHODTXHDVTXDQWLGDGHVUHODWLYDVGHDGPLUDGRUHVGRHVWXGDQWHHSHVVRDVTXHYLVLWDPVHXSHU¿OVmR
A HPFDGD
B HPFDGD
C HPFDGD
D HPFDGD
E HPFDGD

MT - 2º dia | Caderno 6 - CINZA - Página 21


*CINZ25DOM22* 2014

QUESTÃO 143 QUESTÃO 145

Uma fábrica de rapadura vende seus produtos Um lojista adquiriu novas embalagens para
empacotados em uma caixa com as seguintes presentes que serão distribuídas aos seus clientes.
dimensões: 25 cm de comprimento; 10 cm de altura e As embalagens foram entregues para serem montadas
15 cm de profundidade. O lote mínimo de rapaduras HWrPIRUPDGDGDSHOD¿JXUD
vendido pela fábrica é um agrupamento de 125 caixas
GLVSRVWDVFRQIRUPHD¿JXUD

Após montadas, as embalagens formarão um sólido com


quantas arestas?
A 10
Qual é o volume do lote mínimo comercializado pela B 12
fábrica de rapaduras?
C 14
A FP D 15
B 18 750 cm E 16
C FP
QUESTÃO 146
D 468 750 cm
E FP Um homem, determinado a melhorar sua saúde,
resolveu andar diariamente numa praça circular que há
QUESTÃO 144 HPIUHQWHjVXDFDVD7RGRVRVGLDVHOHGiH[DWDPHQWH
15 voltas em torno da praça, que tem 50 m de raio.
Uma concessionária de automóveis revende
8VHFRPRDSUR[LPDomRSDUDS.
atualmente três marcas de veículos, A, B e C, que são
UHVSRQViYHLV SRU   H  UHVSHFWLYDPHQWH Qual é a distância percorrida por esse homem em sua
de sua arrecadação. Atualmente, o faturamento médio caminhada diária?
mensal dessa empresa é de R$ 150 000,00. A direção A NP
dessa empresa estima que, após uma campanha
B 0,75 km
publicitária a ser realizada, ocorrerá uma elevação de
HQDDUUHFDGDomRFRPDVPDUFDV$%H& C 1,50 km
respectivamente. D 2,25 km
Se os resultados estimados na arrecadação forem E 4,50 km
alcançados, o faturamento médio mensal da empresa
passará a ser de
A R$ 180 000,00.
B R$ 181 500,00.
C R$ 187 500,00.
D R$ 240 000,00.
E R$ 257 400,00.
MT - 2º dia | Caderno 6 - CINZA - Página 22
2014 *CINZ25DOM23*
QUESTÃO 147 QUESTÃO 149
Enquanto as lâmpadas comuns têm 8 mil horas de A probabilidade de um empregado permanecer em
vida útil, as lâmpadas LED têm 50 mil horas.
uma dada empresa particular por 10 anos ou mais é de 1 .
MetroCuritiba, 18 ago. 2011 (adaptado). 6
De acordo com a informação e desprezando possíveis Um homem e uma mulher começam a trabalhar nessa
algarismos na parte decimal, a lâmpada LED tem uma companhia no mesmo dia. Suponha que não haja
durabilidade de nenhuma relação entre o trabalho dele e o dela, de
A 1 750 dias a mais que a lâmpada comum. PRGR TXH VHXV WHPSRV GH SHUPDQrQFLD QD ¿UPD VmR
independentes entre si.
B 2 000 dias a mais que a lâmpada comum.
C GLDVDPDLVTXHDOkPSDGDFRPXP A probabilidade de ambos, homem e mulher,
permanecerem nessa empresa por menos de 10 anos
D 42 000 dias a mais que a lâmpada comum.
é de
E 1 008 000 dias a mais que a lâmpada comum.

QUESTÃO 148 60
A

Um construtor pretende murar um terreno e, para
isso, precisa calcular o seu perímetro. O terreno está
UHSUHVHQWDGR QR SODQR FDUWHVLDQR FRQIRUPH D ¿JXUD 25
B
no qual foi usada a escala 1 : 500. Use 2,8 como 
aproximação para ¥ 8 .
24
cm C

6

12
D


1
E


1 QUESTÃO 150
O criador de uma espécie de peixe tem sete tanques,
1 9 cm sendo que cada tanque contém 14 600 litros de água.
Nesses tanques, existem em média cinco peixes para
De acordo com essas informações, o perímetro do cada metro cúbico (m) de água. Sabe-se que cada peixe
terreno, em metros, é consome 1 litro de ração por semana. O criador quer
construir um silo que armazenará a ração para alimentar
A 110. sua criação.
B 120.
Qual é a capacidade mínima do silo, em litros, para
C 124. armazenar a quantidade de ração que garantirá a
D  alimentação semanal dos peixes?
E 144. A 511
B 5 110
C 51 100
D 511 000
E 5 110 000

07žGLD_&DGHUQR&,1=$3iJLQD
*CINZ25DOM24* 2014

QUESTÃO 151 QUESTÃO 153

Um agricultor possui em sua fazenda um silo para A tabela apresenta uma estimativa da evolução
da população brasileira por faixa etária, em milhões de
armazenar sua produção de milho. O silo, que na época
SHVVRDVSDUDH
da colheita é utilizado em sua capacidade máxima,
tem a forma de um paralelepípedo retângulo reto, com Ano
os lados da base medindo L metros e altura igual a 2020 2030 2045
Faixa etária
h metros. O agricultor deseja duplicar a sua produção
para o próximo ano e, para isso, irá comprar um novo Até 14 anos 49 48 48
silo, no mesmo formato e com o dobro da capacidade De 15 a 49 anos 111 112 110
do atual. O fornecedor de silos enviou uma lista com os De 50 anos ou mais 50  78
tipos disponíveis e cujas dimensões são apresentadas Total 210 223 236
na tabela: STEFANO, F. Mais velho e mais rico: os ganhos da maturidade.
ExameHGDQRQQRY DGDSWDGR 
Lado Altura Com base na tabela, o valor que mais se aproxima da
Tipo de silo
(em metros) (em metros) média dos percentuais da população brasileira na faixa
I L 2h HWiULDDWpDQRVQRVDQRVGHHp
II 2L h A 21,5.
III 2L 2h B 21,7.
IV 4L h C 48,0.
D 
V L 4h
E 48,5.
3DUDDWHQGHUjVVXDVQHFHVVLGDGHVRDJULFXOWRUGHYHUi QUESTÃO 154
escolher o silo de tipo
8PDUWLVWDGHVHMDSLQWDUHPXPTXDGURXPD¿JXUDQD
A I. forma de triângulo equilátero ABC de lado 1 metro. Com o
objetivo de dar um efeito diferente em sua obra, o artista
B II.
traça segmentos que unem os pontos médios D, E e F
C III. dos lados BC, AC e AB, respectivamente, colorindo um
D IV. GRVTXDWURWULkQJXORVPHQRUHVFRPRPRVWUDD¿JXUD
E V.
C
QUESTÃO 152
E D
Um construtor precisa revestir o piso de uma sala
retangular. Para essa tarefa, ele dispõe de dois tipos de
A B
cerâmicas: F

a) cerâmica em forma de quadrado de lado 20 cm, que


custa R$ 8,00 por unidade; Qual é a medida da área pintada, em metros quadrados,
do triângulo DEF?
b) cerâmica em forma de triângulo retângulo isósceles
de catetos com 20 cm, que custa R$ 6,00 por unidade. 1
A
A sala tem largura de 5 m e comprimento de 6 m. 16
O construtor deseja gastar a menor quantia possível
com a compra de cerâmica. Sejam x o número de peças ¥
de cerâmica de forma quadrada e y o número de peças de B
16
cerâmica de forma triangular.
,VVR VLJQL¿FD HQWmR HQFRQWUDU YDORUHV SDUD x e y tais 1
C
que 0,04x + 0,02y•HTXHWRUQHPRPHQRUSRVVtYHO 8
valor de
A 8x + 6y. ¥
D
B 6x + 8y. 8
C x + 0,12y.
¥
D x + 0,02y. E
E 0,04x + 0,12y. 4

MT - 2º dia | Caderno 6 - CINZA - Página 24


2014 *CINZ25DOM25*
QUESTÃO 155 QUESTÃO 157
A caixa-d’água de uma casa tem a forma de um Um cliente fez um orçamento com uma cozinheira
paralelepípedo reto-retângulo e possui dimensões para comprar 10 centos de quibe e 15 centos de coxinha
externas (comprimento, largura e altura) de, HRYDORUWRWDOIRLGH5$R¿QDOL]DUDHQFRPHQGD
UHVSHFWLYDPHQWH  P  P H  P e QHFHVViULD decidiu aumentar as quantidades de salgados e acabou
a impermeabilização de todas as faces externas FRPSUDQGRFHQWRVGHTXLEHHFHQWRVGHFR[LQKD
dessa caixa, incluindo a tampa. O fornecedor do &RPLVVRHOHFRQVHJXLXXPGHVFRQWRGHQRSUHoR
impermeabilizante informou ao dono da casa que seu GRFHQWRGRTXLEHHGHQRSUHoRGRFHQWRGHFR[LQKD
produto é fornecido em galões, de capacidade igual a HRYDORUWRWDOGDFRPSUD¿FRXHP5
4,0 litros. Informou, ainda, que cada litro impermeabiliza
uma área de 17 700 cm2HVmRQHFHVViULDVGHPmRVGH De acordo com esses dados, qual foi o valor que o cliente
produto para garantir um bom resultado. pagou pelo cento da coxinha?

Com essas informações, para obter um bom resultado no A 5


trabalho de impermeabilização, o dono da casa precisará B 5
comprar um número mínimo de galões para a execução C R$ 24,90
desse serviço igual a
D 5
A 9. E 5
B 
C 19. QUESTÃO 158
D 25. Uma pessoa usa um programa de computador que
E 45. descreve o desenho da onda sonora correspondente a um
som escolhido. A equação da onda é dada, num sistema
QUESTÃO 156 de coordenadas cartesianas, por y = a ͽ sen[b(x + c)],
em que os parâmetros a, b, c são positivos. O programa
Em uma cidade, os impostos que incidem sobre o permite ao usuário provocar mudanças no som, ao fazer
FRQVXPR GH HQHUJLD HOpWULFD UHVLGHQFLDO VmR GH  alterações nos valores desses parâmetros. A pessoa
sobre o custo do consumo mensal. O valor total da conta deseja tornar o som mais agudo e, para isso, deve diminuir
a ser paga no mês é o valor cobrado pelo consumo
o período da onda.
acrescido dos impostos.
O(s) único(s) parâmetro(s) que necessita(m) ser
Considerando x o valor total da conta mensal de uma
alterado(s) é(são)
determinada residência e y o valor dos impostos, qual é a
expressão algébrica que relaciona x e y? A a.
B b.
x C c.
A y=
 D a e b.
E b e c.
B y x
QUESTÃO 159
x
C y= Os sistemas de cobrança dos serviços de táxi

nas cidades A e B são distintos. Uma corrida de táxi
QDFLGDGH$pFDOFXODGDSHORYDORU¿[RGDEDQGHLUDGD
x TXHpGH5PDLV5SRUTXLO{PHWURURGDGR
D y =
 1D FLGDGH % D FRUULGD p FDOFXODGD SHOR YDORU ¿[R
GD EDQGHLUDGD TXH p GH 5  PDLV 5  SRU
E y = 0,7 x quilômetro rodado.
Uma pessoa utilizou o serviço de táxi nas duas
cidades para percorrer a mesma distância de 6 km.
Qual o valor que mais se aproxima da diferença, em
reais, entre as médias do custo por quilômetro rodado ao
¿QDOGDVGXDVFRUULGDV"
A 0,75
B 0,45
C 
D 
E 

MT - 2º dia | Caderno 6 - CINZA - Página 25


*CINZ25DOM26* 2014

QUESTÃO 160 QUESTÃO 161


$ ¿JXUD PRVWUD RV SUHoRV GD JDVROLQD QR %UDVLO H Uma revista publicará os dados, apresentados
nos Estados Unidos (EUA), feita a conversão para reais, QR JUi¿FR VREUH FRPR RV WLSRV VDQJXtQHRV HVWmR
considerando o preço total de venda ao consumidor distribuídos entre a população brasileira. Contudo,
(abaixo dos nomes dos países) e os valores das parcelas R HGLWRU GHVVD UHYLVWD VROLFLWRX TXH HVVH JUi¿FR VHMD
FRUUHVSRQGHQWHVjUH¿QDULDDRVWULEXWRVHjGLVWULEXLomR publicado na forma de setores, em que cada grupo
e revenda. esteja representado por um setor circular.
Brasil Tipos sanguíneos

2,80 40
36
35 34

% da População brasileira
30

25
EUA Rh positivo
20
Refinaria 2,00
1,37 15
Rh negativo
(Petrobras)
Inclui a 10 8 9
6
mistura 5 3
de etanol 1,52 2 2
0
1,52 Refinaria Grupo A Grupo AB Grupo B Grupo O
Tributos 1,0
O ângulo do maior desses setores medirá, em graus,
0,26
Tributos A 108,0.
Distribuição
e revenda 0,43 0,22 Distribuição B 122,4.
e revenda
C 129,6.
Fontes: Petrobras, Agência Nacional do Petróleo (ANP)
D 151,2.
E
e Energy Information Administration (EIA).
154,8.
Note que, considerando apenas a parte QUESTÃO 162
FRUUHVSRQGHQWHjUH¿QDULDRSUHoRGDJDVROLQDYHQGLGD
no Brasil é inferior ao preço cobrado nos Estados A vazão de água (em m/h) em tubulações pode ser
Unidos, mas os tributos, a distribuição e a revenda medida pelo produto da área da seção transversal por onde
DXPHQWDPRSUHoR¿QDOGHYHQGDQRVSRVWRVEUDVLOHLURV passa a água (em m2) pela velocidade da água (em m/h).
Suponha que fosse tomada a decisão de se diminuir Uma companhia de saneamento abastece uma indústria
RSUHoR¿QDOGHYHQGDQRVSRVWRVEUDVLOHLURVVHPDOWHUDU utilizando uma tubulação cilíndrica de raio r, cuja vazão da
D SDUFHOD GR SUHoR GD JDVROLQD YHQGLGD QD UH¿QDULD GH água enche um reservatório em 4 horas. Para se adaptar
PRGR TXH R SUHoR ¿QDO VH LJXDODVVH DR FREUDGR QRV jV QRYDV QRUPDV WpFQLFDV D FRPSDQKLD GHYH GXSOLFDU
postos dos Estados Unidos. o raio da tubulação, mantendo a velocidade da água e
VejaHGDQRQIHY $GDSWDGR  mesmo material.
O percentual mais aproximado de redução dos valores Qual o tempo esperado para encher o mesmo
em tributos, distribuição e revenda seria UHVHUYDWyULRDSyVDDGDSWDomRjVQRYDs normas?
A 29. A 1 hora
B 44. B 2 horas
C 56. C 4 horas
D  D 8 horas
E 80. E 16 horas

MT - 2º dia | Caderno 6 - CINZA - Página 26


2014 *CINZ25DOM27*
QUESTÃO 163 QUESTÃO 166

Pesquisas indicam que o número de bactérias X Para fazer um pião, brinquedo muito apreciado
é duplicado a cada quarto de hora. Um aluno resolveu pelas crianças, um artesão utilizará o torno mecânico
ID]HUXPDREVHUYDomRSDUDYHUL¿FDUDYHUDFLGDGHGHVVD para trabalhar num pedaço de madeira em formato de
D¿UPDomR(OHXVRXXPDSRSXODomRLQLFLDOGH5 bactérias
cilindro reto, cujas medidas do diâmetro e da altura
;HHQFHUURXDREVHUYDomRDR¿QDOGHXPDKRUD estão ilustradas na Figura 1. A parte de cima desse
6XSRQKDTXHDREVHUYDomRGRDOXQRWHQKDFRQ¿UPDGR pião será uma semiesfera, e a parte de baixo, um cone
que o número de bactérias X se duplica a cada quarto de com altura 4 cm, conforme Figura 2. O vértice do cone
hora. deverá coincidir com o centro da base do cilindro.

Após uma hora do início do período de observação


6 cm
desse aluno, o número de bactérias X foi de
A 22 ͽ105
B 21 ͽ105
7 cm
C 22 ͽ105
4 cm
D 2 ͽ105
E 24 ͽ105

QUESTÃO 164

Os salários, em reais, dos funcionários de uma Figura 1 Figura 2


empresa são distribuídos conforme o quadro:
Valor do salário O artesão deseja fazer um pião com a maior altura que
622,00 1 244,00  6 220,00
(R$)
esse pedaço de madeira possa proporcionar e de modo
Número de
24 1 20  a minimizar a quantidade de madeira a ser descartada.
funcionários
A mediana dos valores dos salários dessa empresa é, Dados:
em reais, 4
O volume de uma esfera de raio r é ͽSͽr  ;

A 622,00.
O volume do cilindro de altura h e área da base S é Sͽh;
B 
1
C 1 244,00. O volume do cone de altura h e área da base S é ͽSͽh;

D 2 021,50. Por simplicidade, aproxime SSDUD
E 2 799,00.
A quantidade de madeira descartada, em centímetros
QUESTÃO 165 cúbicos, é

Um ciclista participará de uma competição e treinará A 45.


alguns dias da seguinte maneira: no primeiro dia, B 48.
pedalará 60 km; no segundo dia, a mesma distância do C 72.
primeiro mais r km; no terceiro dia, a mesma distância do
D 90.
segundo mais r km; e, assim, sucessivamente, sempre
pedalando a mesma distância do dia anterior mais r km. E 99.
No último dia, ele deverá percorrer 180 km, completando
o treinamento com um total de 1 560 km.
A distância r que o ciclista deverá pedalar a mais a cada
dia, em km, é
A 
B 7.
C 10.
D 
E 20.

MT - 2º dia | Caderno 6 - CINZA - Página 27


*CINZ25DOM28* 2014

QUESTÃO 167 QUESTÃO 169


Para as pessoas que não gostam de correr grandes (P  FHUFD GH  PLOK}HV GH SDVVDJHLURV
ULVFRVQRPHUFDGR¿QDQFHLURDDSOLFDomRHPFDGHUQHWD foram transportados entre os Estados Unidos e o Brasil,
de poupança é indicada, pois, conforme a tabela de acordo com dados divulgados pela Agência Nacional
(período 2005 até 2011), a rentabilidade apresentou GH$YLDomR &LYLO $QDF  2 JUi¿FR PRVWUD D GLVWULEXLomR
pequena variação. relativa do número de passageiros transportados entre
o Brasil e os cinco destinos mais procurados, dos quais
Ano Rentabilidade (%) apenas dois países são europeus: França e Portugal.
2005 7,0
2006 4,9
2007 6,4
Chile
8%
2008 6,2
França
2009 7,2
11% Estados
2010 6,8
Unidos
2011 7,0
Portugal 35%
Com base nos dados da tabela, a mediana dos percentuais 16%
de rentabilidade, no período observado, é igual a
A 6,2.
Argentina
B 6,5. 30%
C 6,6.
D 6,8.
E 7,0.

QUESTÃO 168
De acordo com esses dados, o valor mais aproximado
Uma pessoa comprou um aquário em forma de para a quantidade total de passageiros transportados
um paralelepípedo retângulo reto, com 40 cm de em 2010 entre o Brasil e os países europeus mostrados
comprimento, 15 cm de largura e 20 cm de altura. QRJUi¿FRp
Chegando em casa, colocou no aquário uma quantidade A 874 800.
GHiJXDLJXDOjPHWDGHGHVXDFDSDFLGDGH$VHJXLUSDUD
enfeitá-lo, irá colocar pedrinhas coloridas, de volume B 1 018 285.
igual a 50 cmFDGDTXH¿FDUmRWRWDOPHQWHVXEPHUVDV C 1 481 142.
no aquário. D 2 499 428.
Após a colocação das pedrinhas, o nível da água E 
GHYHUi¿FDUDFPGRWRSRGRDTXiULR
QUESTÃO 170
O número de pedrinhas a serem colocadas deve ser igual a
O Brasil desenvolveu técnicas próprias de plantio e
A 48. colheita de cana-de-açúcar, tornando-se o maior produtor
B 72. mundial. Cultivando novas variedades, foram produzidas,
C 84. na safra 2010/2011, 624 milhões de toneladas em
D 120.  PLOK}HV GH KHFWDUHV +RXYH XP VXEVWDQFLDO JDQKR
de produtividade (em toneladas por hectare) quando
E 168. se compara com a de décadas atrás, como a da safra
1974/1975, que foi de 47 toneladas por hectare.
Disponível em: www2.cead.ufv.br. Acesso em: 27 fev. 2011 (adaptado).

De acordo com dados apresentados, qual foi o valor mais


aproximado da taxa de crescimento da produtividade de
cana-de-açúcar, por hectare no Brasil, da safra 1974/1975
para a safra 2010/2011?
A 
B 
C 
D 
E 

MT - 2º dia | Caderno 6 - CINZA - Página 28


2014 *CINZ25DOM29*
QUESTÃO 171 QUESTÃO 174
O número de frutos de uma determinada espécie Alunos de um curso de engenharia desenvolveram
de planta se distribui de acordo com as probabilidades um robô “anfíbio” que executa saltos somente nas
apresentadas no quadro. direções norte, sul, leste e oeste. Um dos alunos
Número de representou a posição inicial desse robô, no plano
Probabilidade
frutos cartesiano, pela letra P, na ilustração.
0 0,65
1 0,15 y
5
2 
4 Direções
  3 N
4  P
2

5 ou mais 0,01 1
O L
-5 -4 -3 -2 -1 0 1 2 3 4 5 x
A probabilidade de que, em tal planta, existam, pelo -1
menos, dois frutos é igual a -2 S
A  -3

B  -4
-5
C 
D 
E  A direção norte-sul é a mesma do eixo y, sendo que o
QUESTÃO 172 sentido norte é o sentido de crescimento de y, e a direção
leste-oeste é a mesma do eixo x, sendo que o sentido
Um procedimento padrão para aumentar a
leste é o sentido de crescimento de x.
capacidade do número de senhas de banco é
acrescentar mais caracteres a essa senha. Essa prática, Em seguida, esse aluno deu os seguintes comandos
além de aumentar as possibilidades de senha, gera um GHPRYLPHQWDomRSDUDRURE{QRUWHOHVWHHVXOQRV
aumento na segurança. Deseja-se colocar dois novos TXDLV RV FRH¿FLHQWHV QXPpULFRV UHSUHVHQWDP R Q~PHUR
caracteres na senha de um banco, um no início e outro no
¿QDO'HFLGLXVHTXHHVVHVQRYRVFDUDFWHUHVGHYHPVHU de saltos do robô nas direções correspondentes, e cada
vogais e o sistema conseguirá diferenciar maiúsculas salto corresponde a uma unidade do plano cartesiano.
de minúsculas. Depois de realizar os comandos dados pelo aluno, a
&RPHVVDSUiWLFDRQ~PHURGHVHQKDVSRVVtYHLV¿FDUi posição do robô, no plano cartesiano, será
multiplicado por
A (0 ; 2).
A 100.
B  
B 90.
C 80. C (1 ; 2).
D 25. D (1 ; 4).
E 20. E (2 ; 1).

QUESTÃO 173
André, Carlos e Fábio estudam em uma mesma
escola e desejam saber quem mora mais perto da escola.
André mora a cinco vinte avos de um quilômetro da
escola. Carlos mora a seis quartos de um quilômetro da
escola. Já Fábio mora a quatro sextos de um quilômetro
da escola.
A ordenação dos estudantes de acordo com a ordem
GHFUHVFHQWHGDVGLVWkQFLDVGHVXDVUHVSHFWLYDVFDVDVj
escola é
A André, Carlos e Fábio.
B André, Fábio e Carlos.
C Carlos, André e Fábio.
D Carlos, Fábio e André.
E Fábio, Carlos e André.
MT - 2º dia | Caderno 6 - CINZA - Página 29
*CINZ25DOM30* 2014

QUESTÃO 175 QUESTÃO 177

O Brasil é o quarto produtor mundial de alimentos,


Um clube de futebol abriu inscrições para novos mas aproximadamente 64 toneladas de cada 100 toneladas
jogadores. Inscreveram-se 48 candidatos. Para realizar que se produz são perdidas ao longo da cadeia produtiva.
Em relação ao total de alimentos produzidos, a perda de
uma boa seleção, deverão ser escolhidos os que cumpram alimentos é distribuída da seguinte forma: 20 toneladas
algumas exigências: os jogadores deverão ter mais de na colheita, 8 toneladas no transporte e armazenamento,
15 toneladas na indústria de processamento, 1 tonelada
 DQRV HVWDWXUD LJXDO RX VXSHULRU j PtQLPD H[LJLGD H
no varejo e 20 toneladas no processamento culinário e
bom preparo físico. Entre os candidatos, 7 têm mais de hábitos alimentares.
8 Disponível em: www.bancodealimentos.org.br. Acesso em: 26 out. 2011 (adaptado).
14 anos e foram pré-selecionados. Dos pré-selecionados,
De acordo com os dados apresentados, os alimentos que
1  WrP HVWDWXUD LJXDO RX VXSHULRU j PtQLPD H[LJLGD H são perdidos no processamento culinário e nos hábitos
2 alimentares representam qual porcentagem em relação
destes, 2 têm bom preparo físico. ao total de alimentos que são perdidos no país?
 A 
A quantidade de candidatos selecionados pelo clube de B 
futebol foi C 
A 12. D 
B 14. E 
C 16.
QUESTÃO 178
D 
E 42. Um confeiteiro deseja fazer um bolo cuja receita
indica a utilização de açúcar e farinha de trigo em
QUESTÃO 176 quantidades fornecidas em gramas. Ele sabe que uma
Barras de cobre cilíndricas são utilizadas para fazer determinada xícara utilizada para medir os ingredientes
aterramentos elétricos. comporta 120 gramas de farinha de trigo e que três
dessas xícaras de açúcar correspondem, em gramas, a
Durante a instalação de um chuveiro, uma pessoa quatro de farinha de trigo.
utilizou uma barra de aterramento de densidade ȡ,
massa m, diâmetro D = 2R e altura h. Quantos gramas de açúcar cabem em uma dessas
xícaras?
Para fazer um novo aterramento, essa pessoa
utilizou uma barra com a mesma densidade, mas com A 
RGREURGDPDVVDHRGREURGRGLkPHWURHPUHODomRj B 40
usada no chuveiro.
m C 90
A densidade é dada por ȡ = e o volume da barra
V D 160
cilíndrica é V = Sͽ R 2 ͽ h.
E 
Qual a relação da altura da barra utilizada no novo
DWHUUDPHQWR FRPSDUDGD jTXHOD XWLOL]DGD QR DWHUUDPHQWR
do chuveiro?
A Quarta parte.
B Metade.
C Igual.
D Dobro.
E Quádruplo.

07žGLD_&DGHUQR&,1=$3iJLQD
2014 *CINZ25DOM31*
QUESTÃO 179 QUESTÃO 180
Uma fundição de alumínio utiliza, como matéria- O modelo matemático desenvolvido por Kirschner
prima, lingotes de alumínio para a fabricação de peças
e Webb descreve a dinâmica da interação das células
injetadas. Os lingotes são derretidos em um forno e o
alumínio, em estado líquido, é injetado em moldes para se não infectadas do sistema imunológico humano com os
VROLGL¿FDUQRIRUPDWRGHVHMDGR2JUi¿FRLQGLFDDVFXUYDV YtUXV +,9 2V JUi¿FRV PRVWUDP D HYROXomR QR WHPSR
de resfriamento do alumínio fundido no molde para três da quantidade de células não infectadas no sistema
GLIHUHQWHVÀXLGRVUHIULJHUDQWHV WLSR,WLSR,,HWLSR,,, TXH imunológico de cinco diferentes pacientes infectados
são utilizados para resfriar o molde, bem como a curva
de resfriamento quando não é utilizado nenhum tipo de SHOR YtUXV +,9 4XDQGR D SRSXODomR GDV FpOXODV QmR
ÀXLGRUHIULJHUDQWH$SHoDVySRGHVHUUHWLUDGDGRPROGH infectadas de um sistema imunológico é extinta, o
GHVPROGH  TXDQGR DWLQJH D WHPSHUDWXUD GH  ƒ& SDFLHQWH LQIHFWDGR ¿FD PDLV VXVFHWtYHO j PRUWH FDVR
Para atender a uma encomenda, a fundição não poderá contraia alguma outra doença.
gastar mais do que 8 segundos para o desmolde da peça
após a sua injeção. Gráfico A Gráfico B

700 3000 3000

Células/mL

Células/mL
600 2000 2000
Sem fluido refrigerante
500 1000 1000
(°C)

0 0
400 0 1000 2000 3000 4000 0 1000 2000 3000 4000
Temperatura

Tempo (dias) Tempo (dias)


300 I Gráfico C Gráfico D
200
3000 3000

II
Células/mL

Células/mL
100
2000 2000
III
1000 1000
0 2 4 6 8 10 12 14 16 18 20
Tempo de resfriamento (s) 0 0
0 1000 2000 3000 4000 0 1000 2000 3000 4000
Tempo (dias) Tempo (dias)

Com a exigência para o desmolde das peças injetadas, Gráfico E

TXDO LV ÀXLGR V UHIULJHUDQWH V SRGHUi mR VHUXWLOL]DGR V  3000


no resfriamento?
Células/mL

2000
A 4XDOTXHUXPGRVÀXLGRVGRWLSR,,,H,,,
B 6RPHQWHRVÀXLGRVGRWLSR,,H,,, 1000

C 6RPHQWHRÀXLGRGRWLSR,,, 0
0 1000 2000 3000 4000
D Não será necessário utilizar nenhum fluido Tempo (dias)

refrigerante. .,56&+1(5'(:(%%*)5HVLVWDQFH5HPLVVLRQDQG4XDOLWDWLYH'LIIHUHQFHVLQ+,9
Chemotherapy. Emerging Infectious DiseasesYQ
E 1HQKXP GRV ÀXLGRV UHIULJHUDQWHV LQGLFDGRV DWHQGH
jVH[LJrQFLDV A partir desses dados, o sistema imunológico do
SDFLHQWH LQIHFWDGR TXH ¿FRX PDLV UDSLGDPHQWH
VXVFHWtYHOjPRUWHHVWiUHSUHVHQWDGRSHORJUi¿FR
A A.
B B.
C C.
D D.
E E.

07žGLD_&DGHUQR&,1=$3iJLQD
*CINZ25DOM32* 2014

2014

Transcreva a sua Redação para a Folha de Redação.

10

HO
11

12

UN ÃO
C Ç
13

S A
14

A D
15

R RE
16

17

18

19

20

21
DA
22

23

24

25

26

27

28

29

30

/&žGLD_&DGHUQR&,1=$3iJLQD
EXAME NACIONAL DO ENSINO MÉDIO
PROVA DE CIÊNCIAS HUMANAS E SUAS TECNOLOGIAS
PROVA DE CIÊNCIAS DA NATUREZA E SUAS TECNOLOGIAS

1º DIA
CADERNO

2015 9
2ª APLICAÇÃO

ATENÇÃO: transcreva no espaço apropriado do seu CARTÃO-RESPOSTA,


com sua caligrafia usual, considerando as letras maiúsculas e minúsculas, a seguinte frase:

Tudo que não invento é falso.

LEIA ATENTAMENTE AS INSTRUÇÕES SEGUINTES: 3. Para cada uma das questões objetivas, são apresentadas
5 opções. Apenas uma responde corretamente à questão.
1. Este CADERNO DE QUESTÕES contém 90 questões
4. O tempo disponível para estas provas é de quatro horas e
numeradas de 1 a 90, dispostas da seguinte maneira:
trinta minutos.
a) as questões de número 1 a 45 são relativas à área de 5. Reserve os 30 minutos finais para marcar seu CARTÃO-RESPOSTA.
Ciências Humanas e suas Tecnologias; Os rascunhos e as marcações assinaladas no CADERNO DE
b) as questões de número 46 a 90 são relativas à área de QUESTÕES não serão considerados na avaliação.
Ciências da Natureza e suas Tecnologias. 6. Quando terminar as provas, acene para chamar o
2. Confira se o seu CADERNO DE QUESTÕES contém a aplicador e entregue este CADERNO DE QUESTÕES e
quantidade de questões e se essas questões estão na ordem o CARTÃO-RESPOSTA.
mencionada na instrução anterior. Caso o caderno esteja 7. Você poderá deixar o local de prova somente após
incompleto, tenha defeito ou apresente qualquer divergência, decorridas duas horas do início da aplicação e não poderá
comunique ao aplicador da sala para que ele tome as levar seu CADERNO DE QUESTÕES ao deixar em
providências cabíveis. definitivo a sala de prova.

Ministério
da Educação *BR0975SAB1*
*BR0975SAB2* 2015

CIÊNCIAS HUMANAS E SUAS TECNOLOGIAS QUESTÃO 03


A população negra teve que enfrentar sozinha o
Questões de 1 a 45 desafio da ascensão social, e frequentemente procurou
fazê-lo por rotas originais, como o esporte, a música e a
QUESTÃO 01 dança. Esporte, sobretudo o futebol, música, sobretudo o
samba, e dança, sobretudo o carnaval, foram os principais
Em 1881, a Câmara dos Deputados aprovou uma canais de ascensão social dos negros até recentemente.
A libertação dos escravos não trouxe consigo a igualdade
reforma na lei eleitoral brasileira, a fim de introduzir o efetiva. Essa igualdade era afirmada nas leis, mas negada
voto direto. A grande novidade, porém, ficou por conta da na prática. Ainda hoje, apesar das leis, aos privilégios e
exigência de que os eleitores soubessem ler e escrever. arrogâncias de poucos correspondem o desfavorecimento
As consequências logo se refletiram nas estatísticas. e a humilhação de muitos.
CARVALHO, J. M. Cidadania no Brasil: o longo caminho. Rio de Janeiro:
Em 1872, havia mais de 1 milhão de votantes, já em
Civilização Brasileira, 2006 (adaptado).
1886, pouco mais de 100 mil cidadãos participaram das
Em relação ao argumento de que no Brasil existe uma
eleições parlamentares. Houve um corte de quase 90 por democracia racial, o autor demonstra que
cento do eleitorado.
A essa ideologia equipara a nação a outros países
CARVALHO, J. M. Cidadania no Brasil: o longo caminho. Rio de Janeiro: modernos.
Civilização Brasileira, 2006 (adaptado).
B esse modelo de democracia foi possibilitado pela
Nas últimas décadas do século XIX, o Império do Brasil miscigenação.
passou por transformações como as descritas, que C essa peculiaridade nacional garantiu mobilidade
social aos negros.
representaram a
D esse mito camuflou formas de exclusão em relação
A ascensão dos “homens bons”. aos afrodescendentes.
E essa dinâmica política depende da participação ativa
B restrição dos direitos políticos. de todas as etnias.
C superação dos currais eleitorais.
QUESTÃO 04
D afirmação do eleitorado monarquista.
E ampliação da representação popular. TEXTO I
Os problemas ambientais são consequência direta
QUESTÃO 02
da intervenção humana nos diferentes ecossistemas
da Terra, causando desequilíbrios no meio ambiente e
Em 1943, Getúlio Vargas criou o Departamento de
comprometendo a qualidade de vida.
Propaganda e Difusão Cultural junto ao Ministério da
Disponível em: www.repository.utl.pt. Acesso em: 29 jul. 2012.
Justiça, esvaziando o Ministério da Educação não só da
TEXTO II
propaganda, mas também do rádio e do cinema. A decisão
tinha como objetivo colocar os meios de comunicação
de massa a serviço direto do Poder Executivo, iniciativa
que tinha inspiração direta no recém-criado Ministério da
Propaganda alemão.
CAPELATO, M. H. Propaganda política e controle dos meios de comunicação.
Rio de Janeiro: FGV, 1999.

No contexto citado, a transferência de funções entre


ministérios teve como finalidade o(a)
A desativação de um sistema tradicional de
comunicação voltado para a educação.
B controle do conteúdo da informação por meio de
uma orientação política e ideológica. Disponível em: www.netuno.eco.br. Acesso em: 29 jul. 2012.
C subordinação do Ministério da Educação ao As imagens representam as geleiras da Groenlândia,
Ministério da Justiça e ao Poder Executivo. que sofreram e sofrem impactos, resultantes do(a)
D ampliação do raio de atuação das emissoras de rádio A ilha de calor.
como forma de difusão da cultura popular. B chuva ácida.
E demonstração de força política do Executivo diante C erosão eólica.
D inversão térmica.
de ministérios herdados do governo anterior.
E aquecimento global.
CH - 1º dia | Caderno 9 - BRANCO - Página 2
2015 *BR0975SAB3*
QUESTÃO 05 QUESTÃO 07
A crescente conscientização sobre os efeitos do
modelo intensivo de produção, adotado de forma geral na
agricultura, tem gerado também uma série de reações.
De fato, a agricultura está cada vez mais pressionada
pelo conjunto de relações que mantém com a sociedade
em geral, sendo emergente o que comumente se
denomina “questão ambiental”. Essas relações, às
vezes de dependência, às vezes de conflito, são as
que determinam uma chamada ampla para mudanças
orientadas à sustentabilidade, não só da atividade
agrícola em si, senão que afete de maneira geral a todo o
entorno no qual a agricultura está inserida.
GOMES, J. C. C. Desenvolvimento rural, transição de formatos tecnológicos, elaboração
social da qualidade, interdisciplinaridade e participação. In: PORTO, V. H.; WIZNIEWSKY,
C. R. F. ; SIMICH, T. (Org.). Agricultor familiar: sujeito de um novo método
de pesquisa, o participativo. Pelotas: Embrapa, 2004.
Disponível em: www.rededemocratica.org. Acesso em: 28 set. 2012.
No texto, faz-se referência a um tipo de pressão
da sociedade contemporânea sobre a agricultura.
Essa pressão objetiva a seguinte transformação na Na imagem, encontram-se referências a um momento de
atividade agrícola: intensa agitação estudantil no país. Tal mobilização se
A Ampliação de políticas de financiamento voltadas explica pela
para a produção de transgênicos.
A divulgação de denúncias de corrupção envolvendo o
B Modernização do modo de produção focado na alta
produtividade da terra. presidente da República.
C Expansão do agronegócio relacionado ao mercado B criminalização dos movimentos sociais realizada pelo
consumidor externo.
Governo Federal.
D Promoção de práticas destinadas à conservação de
recursos naturais. C adoção do arrocho salarial implementada pelo
E Inserção de modelos orientados ao uso intensivo de Ministério da Fazenda.
agroquímicos.
D compra de apoio político promovida pelo Poder
QUESTÃO 06 Executivo.
Uma dimensão da flexibilização do tempo de E violência da repressão estatal atribuída às Forças
trabalho é a sutileza cada vez maior das fronteiras Armadas.
que separam o espaço de trabalho e o do lar, o tempo
de trabalho e o de não trabalho. Os mecanismos QUESTÃO 08
modernos de comunicação permitem que, no horário
de descanso, os trabalhadores permaneçam ligados O reconhecimento da união homoafetiva levou o
à empresa. Mesmo não exercendo diretamente suas debate à esfera pública, dividindo opiniões. Apesar da
atividades profissionais, o trabalhador fica à disposição grande repercussão gerada pela mídia, a população ainda
da empresa ou leva problemas para refletir em casa.
É muito comum o trabalhador estar de plantão, para o não se faz suficientemente esclarecida, confundindo
caso de a empresa ligar para o seu celular ou pager. o conceito de união estável com casamento. Apesar
A remuneração para esse estado de alerta é irrisória de ter sido legitimado pelo Supremo Tribunal Federal
ou inexistente. (STF), o reconhecimento da união homoafetiva é fruto do
KREIN, J. D. Mudanças e tendências recentes na regulação do trabalho. In: DEDECCA, protagonismo dos movimentos sociais como um todo.
C. S.; PRONI, M. W. (Org.). Políticas públicas e trabalho: textos para estudo dirigido.
Campinas: IE/Unicamp; Brasília: MTE, 2006 (adaptado). ARÊDES, N.; SOUZA, I.; FERREIRA, E. Disponível em: http://reporterpontocom.wordpress.com.
Acesso em: 1 mar. 2012 (adaptado).
A relação entre mudanças tecnológicas e tempo de
trabalho apresentada pelo texto implica o As decisões em favor das minorias, tomadas pelo Poder
A prolongamento da jornada de trabalho com a Judiciário, foram possíveis pela organização desses
intensificação da exploração. grupos. Ainda que não sejam assimiladas por toda a
B aumento da fragmentação da produção com a população, essas mudanças
racionalização do trabalho.
A contribuem para a manutenção da ordem social.
C privilégio de funcionários familiarizados com
equipamentos eletrônicos. B reconhecem a legitimidade desses pleitos.
D crescimento da contratação de mão de obra pouco C dependem da iniciativa do Poder Legislativo Federal.
qualificada.
D resultam na celebração de um consenso político.
E declínio dos salários pagos aos empregados mais
idosos. E excedem o princípio da isonomia jurídica.

CH - 1º dia | Caderno 9 - BRANCO - Página 3


*BR0975SAB4* 2015

QUESTÃO 09

Decreto-lei 3.509, de 12 de setembro de 1865


Art. 1º – O cidadão guarda-nacional que por si apresentar outra pessoa para o serviço do Exército por tempo de
nove anos, com a idoneidade regulada pelas leis militares, ficará isento não só do recrutamento, senão também do
serviço da Guarda Nacional. O substituído é responsável por o que o substituiu, no caso de deserção.
Arquivo Histórico do Exército. Ordem do dia do Exército, n. 455, 1865 (adaptado).

No artigo, tem-se um dos mecanismos de formação dos “Voluntários da Pátria”, encaminhados para lutar na Guerra do
Paraguai. Tal prática passou a ocorrer com muita frequência no Brasil nesse período e indica o(a)
A forma como o Exército brasileiro se tornou o mais bem equipado da América do Sul.
B incentivo dos grandes proprietários à participação dos seus filhos no conflito.
C solução adotada pelo país para aumentar o contingente de escravos no conflito.
D envio de escravos para os conflitos armados, visando sua qualificação para o trabalho.
E fato de que muitos escravos passaram a substituir seus proprietários em troca de liberdade.

QUESTÃO 10

SANTIAGO. O interior. In: LEMOS, R. (Org.). Uma história do Brasil através da caricatura: 1840-2001. Rio de Janeiro: Letras & Expressões, 2001 (adaptado).

O diálogo entre os personagens da charge evidencia, no Brasil, a(s)


A reinserção do país na economia globalizada.
B transformações políticas na vigência do Estado Novo.
C alterações em áreas estratégicas para o desenvolvimento do país.
D suspensão das eleições legislativas durante o período da Ditadura Militar.
E volta da democracia após um período sem eleições diretas para o Executivo Federal.

CH - 1º dia | Caderno 9 - BRANCO - Página 4


2015 *BR0975SAB5*
QUESTÃO 11 QUESTÃO 13

Estimativa do número de escravos africanos


desembarcados no Brasil entre os 138
140
anos de 1846 a 1852
120 111

Número de escravos africanos

Milhões de habitantes
Ano 100
desembarcados no Brasil 80,4
80
1846 64 262
60 52,1
1847 75 893 38,8 41,1 38,6
35,8
40 33,2 31,3 31,8
28,3
1848 76 338 18,8
12,9
20
1849 70 827
0
1850 37 672 1940 1950 1960 1970 1980 1991 2000

1851 7 058 Urbana Rural


IBGE. Tendências demográficas: uma análise da sinopse preliminar
1852 1 234 do censo demográfico 2000. Rio de Janeiro: IBGE, 2001.

Disponível em: www.slavevoyages.org. Acesso em: 24 fev. 2012 (adaptado). O processo indicado no gráfico demonstra um aumento
significativo da população urbana em relação à população
A mudança apresentada na tabela é reflexo da Lei
rural no Brasil. Esse fenômeno pode ser explicado pela
Eusébio de Queiróz que, em 1850,
A atração de mão de obra pelo setor produtivo
A aboliu a escravidão no território brasileiro. concentrado nas áreas urbanas.
B definiu o tráfico de escravos como pirataria. B manutenção da instabilidade climática nas áreas
C elevou as taxas para importação de escravos. rurais.
D libertou os escravos com mais de 60 anos. C concentração da oferta de ensino nas áreas urbanas.
E garantiu o direito de alforria aos escravos. D inclusão da população das áreas urbanas em
programas assistenciais.
QUESTÃO 12 E redução dos subsídios para os setores da economia
localizados nas áreas rurais.
O acúmulo gradual de sais nas camadas superiores
do solo, um processo chamado salinização, retarda o QUESTÃO 14
crescimento das safras, diminui a produção das culturas
e, consequentemente, mata as plantas e arruína o solo. Os nossos ancestrais dedicavam-se à caça, à
A salinização mais grave ocorre na Ásia, em especial na pesca e à coleta de frutas e vegetais, garantindo sua
China, na Índia e no Paquistão. subsistência, porque ainda não conheciam as práticas de
agricultura e pecuária. Uma vez esgotados os alimentos,
MILLER, G. Ciência ambiental. São Paulo: Thomson, 2007.
viam-se obrigados a transferir o acampamento para
O fenômeno descrito no texto representa um grande outro lugar.
impacto ambiental em áreas agrícolas e tem como causa HALL, P. P. Gestão ambiental. São Paulo: Pearson, 2011 (adaptado).

direta o(a) O texto refere-se ao movimento migratório denominado


A rotação de cultivos. A sedentarismo.
B associação de culturas. B transumância.
C plantio em curvas de nível. C êxodo rural.
D manipulação genética das plantas. D nomadismo.
E instalação de sistemas de irrigação. E pendularismo.

CH - 1º dia | Caderno 9 - BRANCO - Página 5


*BR0975SAB6* 2015

QUESTÃO 15

QUEIROZ FILHO, A. P.; BIASI, M. Técnicas de cartografia. In: VENTURI, L. A. B. (Org.). Geografia: práticas de campo, laboratório e sala de aula. São Paulo: Sarandi, 2011 (adaptado).

As figuras representam a distância real (D) entre duas residências e a distância proporcional (d) em uma representanção
cartográfica, as quais permitem estabelecer relações espaciais entre o mapa e o terreno. Para a ilustração apresentada,
a escala numérica correta é
A 1/50.
B 1/5 000.
C 1/50 000.
D 1/80 000.
E 1/80 000 000.

QUESTÃO 16

Dubai é uma cidade-estado planejada para estarrecer os visitantes. São tamanhos e formatos grandiosos, em
hotéis e centros comerciais reluzentes, numa colagem de estilos e atrações que parece testar diariamente os limites
da arquitetura voltada para o lazer. O maior shopping do tórrido Oriente Médio abriga uma pista de esqui, a orla do
Golfo Pérsico ganha milionárias ilhas artificiais, o centro financeiro anuncia para breve a torre mais alta do mundo
(a Burj Dubai) e tem ainda o projeto de um campo de golfe coberto! Coberto e refrigerado, para usar com sol e chuva,
inverno e verão.
Disponível em: http://viagem.uol.com.br. Acesso em: 30 jul. 2012 (adaptado).

No texto, são descritas algumas características da paisagem de uma cidade do Oriente Médio. Essas características
descritas são resultado do(a)
A criação de territórios políticos estratégicos.
B preocupação ambiental pautada em decisões governamentais.
C utilização de tecnologia para transformação do espaço.
D demanda advinda da extração local de combustíveis fósseis.
E emprego de recursos públicos na redução de desigualdades sociais.

CH - 1º dia | Caderno 9 - BRANCO - Página 6


2015 *BR0975SAB7*
QUESTÃO 17 QUESTÃO 19

Sabe-se o que era a mata do Nordeste, antes da Colonizar, afirmava, em 1912, um eminente jurista,
monocultura da cana: um arvoredo tanto e tamanho e tão “é relacionar-se com os países novos para tirar benefícios
basto e de tantas prumagens que não podia homem dar dos recursos de qualquer natureza desses países,
aproveitá-los no interesse nacional, e ao mesmo tempo
conta. O canavial desvirginou todo esse mato grosso do levar às populações primitivas as vantagens da cultura
modo mais cru: pela queimada. A fogo é que foram se intelectual, social, científica, moral, artística, literária,
abrindo no mato virgem os claros por onde se estendeu comercial e industrial, apanágio das raças superiores.
o canavial civilizador, mas ao mesmo tempo devastador. A colonização é, pois, um estabelecimento fundado em
FREYRE, G. Nordeste. São Paulo: Global, 2004 (adaptado).
país novo por uma raça de civilização avançada, para
realizar o duplo fim que acabamos de indicar”.
Analisando os desdobramentos da atividade canavieira MÉRIGNHAC. Précis de législation et d´économie coloniales. Apud LINHARES, M. Y.
sobre o meio físico, o autor salienta um paradoxo, A luta contra a Metrópole (Ásia e África). São Paulo: Brasiliense, 1981.

caracterizado pelo(a) A definição de colonização apresentada no texto tinha a


A demanda de trabalho, que favorecia a escravidão. função ideológica de
B modelo civilizatório, que acarretou danos ambientais. A dissimular a prática da exploração mediante a ideia
de civilização.
C rudimento das técnicas produtivas, que eram
B compensar o saque das riquezas mediante a
ineficientes. educação formal dos colonos.
D natureza da atividade econômica, que concentrou C formar uma identidade colonial mediante a
riqueza. recuperação de sua ancestralidade.
E predomínio da monocultura, que era voltada para D reparar o atraso da Colônia mediante a incorporação
exportação. dos hábitos da Metrópole.
E promover a elevação cultural da Colônia mediante a
QUESTÃO 18 incorporação de tradições metropolitanas.

Na sociedade democrática, as opiniões de cada QUESTÃO 20


um não são fortalezas ou castelos para que neles nos TEXTO I
encerremos como forma de autoafirmação pessoal. Não é possível passar das trevas da ignorância para a
Não só temos de ser capazes de exercer a razão luz da ciência a não ser lendo, com um amor sempre mais
vivo, as obras dos Antigos. Ladrem os cães, grunhem os
em nossas argumentações, como também devemos porcos! Nem por isso deixarei de ser um seguidor dos
desenvolver a capacidade de ser convencidos pelas Antigos. Para eles irão todos os meus cuidados e, todos
melhores razões. A partir dessa perspectiva, a verdade os dias, a aurora me encontrará entregue ao seu estudo.
buscada é sempre um resultado, não ponto de partida: BLOIS, P. Apud PEDRERO SÁNCHEZ, M. G. História da Idade Média:
texto e testemunhas. São Paulo: Unesp, 2000.
e essa busca inclui a conversação entre iguais, a
TEXTO II
polêmica, o debate, a controvérsia.
A nossa geração tem arraigado o defeito de recusar
SAVATER, F. As perguntas da vida. São Paulo: Martins Fontes, 2001 (adaptado). admitir tudo o que parece vir dos modernos. Por isso,
quando descubro uma ideia pessoal e quero torná-la
A ideia de democracia presente no texto, baseada na pública, atribuo-a a outrem e declaro: — Foi fulano de tal
concepção de Habermas acerca do discurso, defende que o disse, não sou eu. E para que acreditem totalmente
nas minhas opiniões, digo: — O inventor foi fulano de tal,
que a verdade é um(a) não sou eu.
A alvo objetivo alcançável por cada pessoa, como BATH, A. Apud PEDRERO SÁNCHEZ, M. G. História da Idade Média: texto e testemunhas.
São Paulo: Unesp, 2000.
agente racional autônomo.
Nos textos são apresentados pontos de vista distintos
B critério acima dos homens, de acordo com o qual sobre as mudanças culturais ocorridas no século XII no
podemos julgar quais opiniões são as melhores. Ocidente. Comparando os textos, os autores discutem
o(a)
C construção da atividade racional de comunicação
A produção do conhecimento face à manutenção dos
entre os indivíduos, cujo resultado é um consenso.
argumentos de autoridade da Igreja.
D produto da razão, que todo indivíduo traz latente B caráter dinâmico do pensamento laico frente à
desde o nascimento, mas que só se firma no processo estagnação dos estudos religiosos.
educativo. C surgimento do pensamento científico em oposição à
tradição teológica cristã.
E resultado que se encontra mais desenvolvido nos
D desenvolvimento do racionalismo crítico ao opor fé
espíritos elevados, a quem cabe a tarefa de convencer e razão.
os outros. E construção de um saber teológico científico.
CH - 1º dia | Caderno 9 - BRANCO - Página 7
*BR0975SAB8* 2015

QUESTÃO 21 QUESTÃO 23

Suponha homens numa morada subterrânea, em Em 1960, os 20% mais ricos da população mundial
forma de caverna, cuja entrada, aberta à luz, se estende dispunham de um capital trinta vezes mais elevado do
que o dos 20% mais pobres, o que já era escandaloso.
sobre todo o comprimento da fachada; eles estão lá Mas, ao invés de melhorar, a situação ainda se agravou.
desde a infância, as pernas e o pescoço presos por Hoje, o capital dos ricos em relação ao dos pobres é, não
correntes, de tal sorte que não podem trocar de lugar e mais trinta, mas oitenta e duas vezes mais elevado.
só podem olhar para frente, pois os grilhões os impedem RAMONET, I. Guerras do século XXI: novos temores e novas ameaças.
Petrópolis: Vozes, 2003 (adaptado).
de voltar a cabeça; a luz de uma fogueira acesa ao
longe, numa elevada do terreno, brilha por detrás deles; Que característica socioeconômica está expressa no
entre a fogueira e os prisioneiros, há um caminho texto?
ascendente; ao longo do caminho, imagine um pequeno A Expansão demográfica.
muro, semelhante aos tapumes que os manipuladores B Homogeneidade social.
de marionetes armam entre eles e o público e sobre os C Concentração de renda.
quais exibem seus prestígios. D Desemprego conjuntural.
PLATÃO. A República. Lisboa: Fundação Calouste Gulbenkian, 2007. E Desenvolvimento econômico.
Essa narrativa de Platão é uma importante manifestação QUESTÃO 24
cultural do pensamento grego antigo, cuja ideia central,
Mediante o Código de Posturas de 1932, o poder
do ponto de vista filosófico, evidencia o(a) público enumera e prevê, para os habitantes de Fortaleza,
A caráter antropológico, descrevendo as origens do uma série de proibições condicionadas pela hora: após
as 22 horas era vetada a emissão de sons em volume
homem primitivo. acentuado. O uso de buzinas, sirenes, vitrolas, motores
B sistema penal da época, criticando o sistema ou qualquer objeto que produzisse barulho seria punido
carcerário da sociedade ateniense. com multa. No início dos anos 1940 o último bonde partia
da Praça do Ferreira às 23 horas.
C vida cultural e artística, expressa por dramaturgos
SILVA FILHO, A. L. M. Fortaleza: imagens da cidade. Fortaleza:
trágicos e cômicos gregos. Museu do Ceará; Secult, 2001 (adaptado).

D sistema político elitista, provindo do surgimento da Como Fortaleza, muitas capitais brasileiras
pólis e da democracia ateniense. experimentaram, na primeira metade do século XX, um
novo tipo de vida urbana, marcado por condutas que
E teoria do conhecimento, expondo a passagem do evidenciam uma
mundo ilusório para o mundo das ideias.
A experiência temporal regida pelo tempo orgânico e
QUESTÃO 22 pessoal.
B experiência que flexibilizava a obediência ao tempo
Se os nossos adversários, que admitem a existência do relógio.
de uma natureza não criada por Deus, o Sumo Bem, C relação de códigos que estimulavam o trânsito de
quisessem admitir que essas considerações estão certas, pessoas na cidade.
deixariam de proferir tantas blasfêmias, como a de atribuir D normatização do tempo com vistas à disciplina dos
a Deus tanto a autoria dos bens quanto dos males. Pois corpos na cidade.
sendo Ele fonte suprema da Bondade, nunca poderia ter E cultura urbana capaz de conviver com diferentes
experiências temporais.
criado aquilo que é contrário à sua natureza.
AGOSTINHO. A natureza do Bem. Rio de Janeiro: Sétimo Selo, 2005 (adaptado).

Para Agostinho, não se deve atribuir a Deus a origem do


mal porque
A o surgimento do mal é anterior à existência de Deus.
B o mal, enquanto princípio ontológico, independe de
Deus.
C Deus apenas transforma a matéria, que é, por
natureza, má.
D por ser bom, Deus não pode criar o que lhe é oposto,
o mal.
E Deus se limita a administrar a dialética existente entre
o bem e o mal.

CH - 1º dia | Caderno 9 - BRANCO - Página 8


2015 *BR0975SAB9*
QUESTÃO 25 QUESTÃO 27
TEXTO I A conquista pelos ingleses de grandes áreas da Índia
A melhor banda de todos os tempos da última semana deu o impulso inicial à produção e venda organizada
de ópio. A Companhia das Índias Orientais obteve o
O melhor disco brasileiro de música americana monopólio da compra do ópio indiano e depois vendeu
O melhor disco dos últimos anos de sucessos do licenças para mercadores selecionados, conhecidos como
passado “mercadores nativos”. Depois de vender ópio na China,
O maior sucesso de todos os tempos entre os dez esses mercadores depositavam a prata que recebiam
maiores fracassos por ele com agentes da companhia em Cantão, em troca
Não importa contradição de cartas de crédito; a companhia, por sua vez, usava a
prata para comprar chá, porcelana e outros artigos que
O que importa é televisão seriam vendidos na Inglaterra.
Dizem que não há nada que você não se acostume
SPENCE, J. Em busca da China moderna. São Paulo: Cia. das Letras, 1996 (adaptado).
Cala a boca e aumenta o volume então.
MELLO, B.; BRITTO, S. A melhor banda de todos os tempos da última semana.
A análise das trocas comerciais citadas permite
São Paulo: Abril Music, 2001 (fragmento). interpretar as relações de poder que foram estabelecidas.
A partir desse pressuposto, o processo sócio-histórico
TEXTO II identificado no texto é
O fetichismo na música e a regressão da audição A a expansão político-econômica de países do Oriente,
iniciada nas últimas décadas do século XX.
Aldous Huxley levantou em um de seus ensaios a
seguinte pergunta: quem ainda se diverte realmente hoje B a consolidação do cenário político entreguerras, na
primeira metade do século XX.
num lugar de diversão? Com o mesmo direito poder-se-ia
perguntar: para quem a música de entretenimento serve C o colonialismo europeu, que marcou a expansão
europeia no século XV.
ainda como entretenimento? Ao invés de entreter, parece
que tal música contribui ainda mais para o emudecimento D o imperialismo, cujo ápice ocorreu na segunda
metade do século XIX.
dos homens, para a morte da linguagem como expressão,
para a incapacidade de comunicação. E as libertações nacionais, ocorridas na segunda
metade do século XX.
ADORNO, T. Textos escolhidos. São Paulo: Nova Cultural, 1999.

A aproximação entre a letra da canção e a crítica de


Adorno indica o(a)
A lado efêmero e restritivo da indústria cultural.
B baixa renovação da indústria de entretenimento.
C influência da música americana na cultura brasileira.
D fusão entre elementos da indústria cultural e da
cultura popular.
E declínio da forma musical em prol de outros meios de
entretenimento.

QUESTÃO 26
A pura lealdade na amizade, embora até o presente
não tenha existido nenhum amigo leal, é imposta a todo
homem, essencialmente, pelo fato de tal dever estar
implicado como dever em geral, anteriormente a toda
experiência, na ideia de uma razão que determina a
vontade segundo princípios a priori.
KANT, I. Fundamentação da metafísica dos costumes. São Paulo: Barcarolla, 2009.

A passagem citada expõe um pensamento


caracterizado pela
A eficácia prática da razão empírica.
B transvaloração dos valores judaico-cristãos.
C recusa em fundamentar a moral pela experiência.
D comparação da ética a uma ciência de rigor
matemático.
E importância dos valores democráticos nas relações
de amizade.
CH - 1º dia | Caderno 9 - BRANCO - Página 9
*BR0975SAB10* 2015

QUESTÃO 28 QUESTÃO 30

Após ter examinado cuidadosamente todas as


coisas, cumpre enfim concluir e ter por constante que
esta proposição, eu sou, eu existo, é necessariamente
verdadeira todas as vezes que a enuncio ou que a
concebo em meu espírito.
DESCARTES, R. Meditações. Pensadores. São Paulo: Abril Cultural, 1979.

A proposição “eu sou, eu existo” corresponde a um dos


momentos mais importantes na ruptura da filosofia do
século XVII com os padrões da reflexão medieval, por
A estabelecer o ceticismo como opção legítima.
B utilizar silogismos linguísticos como prova ontológica.
C inaugurar a posição teórica conhecida como
DUARTE, P. A. Fundamentos de cartografia. Florianópolis: UFSC, 2002.
empirismo.
D estabelecer um princípio indubitável para o
As diferentes representações cartográficas trazem
consigo as ideologias de uma época. A representação conhecimento.
destacada se insere no contexto das Cruzadas por E questionar a relação entre a filosofia e o tema da
existência de Deus.
A revelar aspectos da estrutura demográfica de um
povo.
QUESTÃO 31
B sinalizar a disseminação global de mitos e preceitos
políticos. O impulso para o ganho, a perseguição do lucro, do
C utilizar técnicas para demonstrar a centralidade de dinheiro, da maior quantidade possível de dinheiro não
algumas regiões.
tem, em si mesma, nada que ver com o capitalismo.
D mostrar o território para melhor administração dos
recursos naturais. Tal impulso existe e sempre existiu. Pode-se dizer que
E refletir a dinâmica sociocultural associada à visão de tem sido comum a toda sorte e condição humanas em
mundo eurocêntrica. todos os tempos e em todos os países, sempre que se
tenha apresentada a possibilidade objetiva para tanto.
QUESTÃO 29 O capitalismo, porém, identifica-se com a busca do
Não acho que seja possível identificar a globalização lucro, do lucro sempre renovado por meio da empresa
apenas com a criação de uma economia global, embora permanente, capitalista e racional. Pois assim deve
este seja seu ponto focal e sua característica mais óbvia. ser: numa ordem completamente capitalista da
Precisamos olhar além da economia. Antes de tudo, sociedade, uma empresa individual que não tirasse
a globalização depende da eliminação de obstáculos
vantagem das oportunidades de obter lucros estaria
técnicos, não de obstáculos econômicos. Isso tornou
possível organizar a produção, e não apenas o comércio, condenada à extinção.
em escala internacional. WEBER, M. A ética protestante e o espírito do capitalismo.
São Paulo: Martin Claret, 2001 (adaptado).
HOBSBAWM, E. O novo século: entrevista a Antonio Polito.
São Paulo: Cia. das Letras, 2000 (adaptado).
O capitalismo moderno, segundo Max Weber, apresenta
Um fator essencial para a organização da produção, na como característica fundamental a
conjuntura destacada no texto, é a
A competitividade decorrente da acumulação de capital.
A criação de uniões aduaneiras.
B implementação da flexibilidade produtiva e comercial.
B difusão de padrões culturais.
C ação calculada e planejada para obter rentabilidade.
C melhoria na infraestrutura de transportes.
D supressão das barreiras para comercialização. D socialização das condições de produção.
E organização de regras nas relações internacionais. E mercantilização da força de trabalho.

CH - 1º dia | Caderno 9 - BRANCO - Página 10


2015 *BR0975SAB11*
QUESTÃO 32

O filósofo Auguste Comte (1798-1857) preenche sua doutrina com uma imagem do progresso social na qual
se conjugam ciência e política: a ação política deve assumir o aspecto de uma ação científica e a política deve ser
estudada de maneira científica (a física social). Desde que a Revolução Francesa favoreceu a integração do povo
na vida social, o positivismo obstina-se no programa de uma comunidade pacífica. E o Estado, instituição do “reino
absoluto da lei”, é a garantia da ordem que impede o retorno potencial das revoluções e engendra o progresso.
RUBY, C. Introdução à filosofia política. São Paulo: Unesp, 1998 (adaptado).

A característica do Estado positivo que lhe permite garantir não só a ordem, como também o desejado progresso das
nações, é ser
A espaço coletivo, onde as carências e desejos da população se realizam por meio das leis.
B produto científico da física social, transcendendo e transformando as exigências da realidade.
C elemento unificador, organizando e reprimindo, se necessário, as ações dos membros da comunidade.
D programa necessário, tal como a Revolução Francesa, devendo portanto se manter aberto a novas insurreições.
E agente repressor, tendo um papel importante a cada revolução, por impor pelo menos um curto período de ordem.

QUESTÃO 33
A humanidade conhece, atualmente, um fenômeno espacial novo: pela primeira vez na história humana, a
população urbana ultrapassa a rural no mundo. Todavia, a urbanização é diferenciada entre os continentes.
DURAND, M. F. et al. Atlas da mundialização: compreender o espaço mundial contemporâneo. São Paulo: Saraiva, 2009.

No texto, faz-se referência a um processo espacial de escala mundial. Um indicador das diferenças continentais
desse processo espacial está presente em:
A Orientação política de governos locais.
B Composição religiosa de povos originais.
C Tamanho desigual dos espaços ocupados.
D Distribuição etária dos habitantes do território.
E Grau de modernização de atividades econômicas.

QUESTÃO 34

Consumo de energia elétrica


per capita 2007 (tep)
0,0 a 1,5
1,5 a 3,0
3,0 a 4,5
4,5 a 6,0
>6,0

BRASIL. Atlas da energia elétrica do Brasil. Brasília: Agência Nacional de Energia Elétrica, 2008 (adaptado).

A distribuição do consumo de energia elétrica per capita, verificada no cartograma, é resultado da


A extensão territorial dos Estados-nação.
B diversificação da matriz energética local.
C capacidade de integração política regional.
D proximidade com áreas de produção de petróleo.
E instalação de infraestrutura para atender à demanda.

CH - 1º dia | Caderno 9 - BRANCO - Página 11


*BR0975SAB12* 2015

QUESTÃO 35 QUESTÃO 37
O ícone dos conflitos que assolam a região da bacia
do Xingu na atualidade é o projeto da hidrelétrica de Belo
Monte. Prevista para ser implantada no Médio Xingu, tem
a capacidade de gerar, segundo os estudos da Eletronorte,
11 mil megawatts de energia, o que faria dela a segunda
maior hidrelétrica do Brasil. Entre adesivos que refletem o
teor polêmico do projeto — “Eu quero Belo Monte” e “Fora
Belo Monte” —, os moradores de Altamira, cidade polo da
região onde a usina deverá ser construída, se dividem.
MARTINHO, N. O coração do Brasil. Horizonte Geográfico, n. 129, jun. 2010 (adaptado).

Na polêmica apresentada, de acordo com a perspectiva


dos trabalhadores da região, um argumento favorável
e outro contrário à implementação do projeto estão,
respectivamente, na
A urbanização da periferia e valorização dos imóveis
rurais.
NANI. Disponível em: www.nanihumor.com. Acesso em: 7 ago. 2012.
B recuperação da autoestima e criação de empregos
qualificados. As novas tecnologias foram massificadas, alcançando
C expansão de lavouras e crescimento do e impactando de diferentes formas os lugares. A ironia
assalariamento agrícola. proposta pela charge indica que o acesso à tecnologia está
D captação de investimentos e expropriação dos A vinculado a mudanças na paisagem.
posseiros pobres. B garantido de forma equitativa aos cidadãos.
E adoção do preservacionismo e estabelecimento de C priorizado para resolver as desigualdades.
reservas permanentes. D relacionado a uma ação redentora na vida social.
QUESTÃO 36 E dissociado de revoluções na realidade socioespacial.

É simplesmente espantoso que esses núcleos tão


desiguais e tão diferentes se tenham mantido aglutinados
numa só nação. Durante o período colonial, cada um
deles teve relação direta com a metrópole. Ocorreu o
extraordinário, fizemos um povo-nação, englobando
todas aquelas províncias ecológicas numa só entidade
cívica e política.
RIBEIRO, D. O povo brasileiro: formação e sentido do Brasil. São Paulo: Cia. das Letras, 1988.

Após a conquista da autonomia, a questão primordial


do Brasil residia em como garantir sua unidade
político-territorial diante das características e práticas
herdadas da colonização. Relacionando o projeto
de independência à construção do Estado nacional
brasileiro, a sua particularidade decorreu da
A ordenação de um pacto que reconheceu os direitos
políticos aos homens, independentemente de cor,
sexo ou religião.
B estruturação de uma sociedade que adotou
os privilégios de nascimento como critério de
hierarquização social.
C realização de acordos entre as elites regionais, que
evitou confrontos armados contrários ao projeto
luso-brasileiro.
D concessão da autonomia política regional, que
atendeu aos interesses socioeconômicos dos
grandes proprietários.
E afirmação de um regime constitucional monárquico,
que garantiu a ordem associada à permanência da
escravidão.

CH - 1º dia | Caderno 9 - BRANCO - Página 12


2015 *BR0975SAB13*
QUESTÃO 38 QUESTÃO 40
Se vamos ter mais tempo de lazer no futuro
automatizado, o problema não é como as pessoas vão
consumir essas unidades adicionais de tempo de lazer,
mas que capacidade para a experiência terão as pessoas
com esse tempo livre. Mas se a notação útil do emprego
do tempo se torna menos compulsiva, as pessoas talvez
tenham de reaprender algumas das artes de viver que
foram perdidas na Revolução Industrial: como preencher
os interstícios de seu dia com relações sociais e pessoais;
como derrubar mais uma vez as barreiras entre o trabalho
e a vida.
THOMPSON, E. P. Costumes em comum: estudos sobre a cultura popular tradicional.
São Paulo: Cia. das Letras, 1998 (adaptado).

A partir da reflexão do historiador, um argumento contrário


à transformação promovida pela Revolução Industrial na
relação dos homens com o uso do tempo livre é o(a)
A intensificação da busca do lucro econômico.
B flexibilização dos períodos de férias trabalhistas.
C esquecimento das formas de sociabilidade
tradicionais.
D aumento das oportunidades de confraternização
familiar.
E multiplicação das possibilidades de entretenimento GILMAR. Disponível em: www.deficientefisico.com. Acesso em: 6 dez. 2012.
virtual.
O cartum evidencia um desafio que o tema da inclusão
QUESTÃO 39 social impõe às democracias contemporâneas. Esse
desafio exige a combinação entre
A razão principal que leva o capitalismo como sistema A participação política e formação profissional
a ser tão terrivelmente destrutivo da biosfera é que, na diferenciada.
maioria dos casos, os produtores que lucram com a B exercício da cidadania e políticas de transferência de
destruição não a registram como um custo de produção, renda.
mas sim, precisamente ao contrário, como uma redução
C modernização das leis e ampliação do mercado de
no custo. Por exemplo, se um produtor joga lixo em um rio, trabalho.
poluindo suas águas, esse produtor considera que está
economizando o custo de outros métodos mais seguros, D universalização de direitos e reconhecimento das
diferenças.
porém mais caros de dispor do lixo.
E crescimento econômico e flexibilização dos processos
WALLERSTEIN, I. Utopística ou as decisões históricas do século vinte e um. seletivos.
Petrópolis: Vozes, 2003.

A pressão dos movimentos socioambientais, na tentativa


de reverter a lógica descrita no texto, aponta para a
A emergência de um sistema econômico global que
secundariza os lucros.
B redução dos custos de tratamento de resíduos pela
isenção fiscal das empresas.
C flexibilização do trabalho como estratégia positiva de
corte de custos empresariais.
D incorporação de um sistema normativo ambiental no
processo de produção industrial.
E minimização do papel do Estado em detrimento das
organizações não governamentais.

CH - 1º dia | Caderno 9 - BRANCO - Página 13


*BR0975SAB14* 2015

QUESTÃO 41 QUESTÃO 43

Confidência do itabirano As autoridades de Kiribati, arquipélago do Oceano


De Itabira trouxe prendas diversas que ora te ofereço: Pacífico formado por 33 atóis e uma ilha de coral, estão
esta pedra de ferro, futuro aço do Brasil; conscientizando sua população para que aceitem que,
este São Benedito do velho santeiro Alfredo Durval; nas próximas décadas, terão de fugir do país. A estimativa
este couro de anta, estendido no sofá de visitas; é que, em um período de 50 anos, as ilhas podem
este orgulho, esta cabeça baixa. desaparecer. O governo convocou os líderes de todas
as ilhas para convencê-los da importância de mudar a
Tive ouro, tive gado, tive fazendas. mentalidade das pessoas, com pleno conhecimento que
Hoje sou funcionário público. é uma questão muito sensível porque ameaça a própria
Itabira é apenas uma fotografia na parede. identidade de um país. Kiribati já antecipou convênios
Mas como dói. com Austrália e Nova Zelândia para enviar seus cidadãos
ANDRADE, C. D. Sentimento do mundo. São Paulo: Cia. das Letras, 2012 (fragmento). aos países vizinhos, algo que muitos dos moradores do
O poeta pensa a região como lugar, pleno de afetos. arquipélago não aceitam.
A longa história da ocupação de Minas Gerais, iniciada Disponível em: http://noticias.terra.com.br. Acesso em: 28 jul. 2012.
com a mineração, deixou marcas que se atualizam em
Itabira, pequena cidade onde nasceu o poeta. Nesse No texto, faz-se referência a um problema que se tornou
sentido, a evocação poética indica o(a) um tema recorrente na agenda global. Nesse sentido,
A pujança da natureza resistindo à ação humana. a preocupação apresentada pela população de Kiribati
B sentido de continuidade do progresso. fundamenta-se na previsão de
C cidade como imagem positiva da identidade mineira. A submersão de terras habitadas, decorrente da
D percepção da cidade como paisagem da memória. elevação do nível do mar.
E valorização do processo de ocupação da região. B ocorrência de tsunamis, derivada de mudanças no
QUESTÃO 42 eixo de rotação do planeta.
C erupções vulcânicas frequentes, visto que estão
A utilidade do escravo é semelhante à do animal.
Ambos prestam serviços corporais para atender às assentados sobre o Círculo do Fogo.
necessidades da vida. A natureza faz o corpo do D terremotos com magnitude extrema, devido à
escravo e do homem livre de forma diferente. O escravo proximidade de bordas de placas tectônicas.
tem corpo forte, adaptado naturalmente ao trabalho
servil. Já o homem livre tem corpo ereto, inadequado ao E furacões de grande intensidade, em função de
trabalho braçal, porém apto à vida do cidadão. redução da temperatura média do Oceano Pacífico.
ARISTÓTELES. Política. Brasília: UnB, 1985.

O trabalho braçal é considerado, na filosofia


aristotélica, como
A indicador da imagem do homem no estado de
natureza.
B condição necessária para a realização da virtude
humana.
C atividade que exige força física e uso limitado da
racionalidade.
D referencial que o homem deve seguir para viver uma
vida ativa.
E mecanismo de aperfeiçoamento do trabalho por meio
da experiência.

CH - 1º dia | Caderno 9 - BRANCO - Página 14


2015 *BR0975SAB15*
QUESTÃO 44 QUESTÃO 45

Energia de Noronha virá da força das águas Falava-se, antes, de autonomia da produção para
significar que uma empresa, ao assegurar uma produção,
A energia de Fernando de Noronha virá do mar, do
ar, do sol e até do lixo produzido por seus moradores e buscava também manipular a opinião pela via da
visitantes. É o que promete o projeto de substituição da publicidade. Nesse caso, o fato gerador do consumo seria
matriz energética da ilha, que prevê a troca dos geradores a produção. Mas, atualmente, as empresas hegemônicas
atuais, que consomem 310 mil litros de diesel por mês. produzem o consumidor antes mesmo de produzirem
GUIBU, F. Folha de S. Paulo, 19 ago. 2012 (adaptado). os produtos. Um dado essencial do entendimento do
No texto, está apresentada a nova matriz energética do consumo é que a produção do consumidor, hoje, precede
Parque Nacional Marinho de Fernando de Noronha. a produção dos bens e dos serviços.
A escolha por essa nova matriz prioriza o(a) SANTOS, M. Por uma outra globalização: do pensamento único à consciência universal.
Rio de Janeiro: Record, 2000 (adaptado).
A expansão da oferta de energia, para aumento da
atividade turística. O tipo de relação entre produção e consumo discutido no
B uso de fontes limpas, para manutenção das condições texto pressupõe o(a)
ecológicas da região.
A aumento do poder aquisitivo.
C barateamento dos custos energéticos, para estímulo
da ocupação permanente. B estímulo à livre concorrência.
D desenvolvimento de unidades complementares, para C criação de novas necessidades.
solução da carência energética local.
D formação de grandes estoques.
E diminuição dos gastos operacionais de transporte,
E implantação de linhas de montagem.
para superação da distância do continente.

CH - 1º dia | Caderno 9 - BRANCO - Página 15


*BR0975SAB16* 2015

CIÊNCIAS DA NATUREZA E SUAS QUESTÃO 47


TECNOLOGIAS Durante a aula, um professor apresentou uma
pesquisa nacional que mostrava que o consumo de sódio
Questões de 46 a 90 pelos adolescentes brasileiros é superior ao determinado
pela Organização Mundial da Saúde. O professor, então,
QUESTÃO 46 destacou que esse hábito deve ser evitado.
Em uma flauta, as notas musicais possuem A doença associada a esse hábito é a
frequências e comprimentos de onda (λ) muito bem A obesidade.
definidos. As figuras mostram esquematicamente
um tubo de comprimento L, que representa de forma B osteoporose.
simplificada uma flauta, em que estão representados: C diabetes tipo II.
em A o primeiro harmônico de uma nota musical D hipertensão arterial.
(comprimento de onda λA), em B seu segundo harmônico E hipercolesterolemia.
(comprimento de onda λB) e em C o seu terceiro
harmônico (comprimento de onda λC), onde λA > λB > λC. QUESTÃO 48

O cladograma representa, de forma simplificada, o


processo evolutivo de diferentes grupos de vertebrados.
Nesses organismos, o desenvolvimento de ovos
protegidos por casca rígida (pergaminácea ou calcárea)
L possibilitou a conquista do ambiente terrestre.

A B C
Em função do comprimento do tubo, qual o comprimento
de onda da oscilação que forma o próximo harmônico?

L
A
4

L
B
5
O surgimento da característica mencionada está
L representado, no cladograma, pelo número
C
2 A 1.
B 2.
L C 3.
D
8 D 4.
E 5.
6L
E
8

CN - 1º dia | Caderno 9 - BRANCO - Página 16


2015 *BR0975SAB17*
QUESTÃO 49 QUESTÃO 50
Um eletricista projeta um circuito com três lâmpadas O urânio é um elemento cujos átomos contêm 92 prótons,
incandescentes idênticas, conectadas conforme a figura. 92 elétrons e entre 135 e 148 nêutrons. O isótopo de
Deseja-se que uma delas fique sempre acesa, por isso é urânio 235U é utilizado como combustível em usinas
ligada diretamente aos polos da bateria, entre os quais se nucleares, onde, ao ser bombardeado por nêutrons, sofre
mantém uma tensão constante. As outras duas lâmpadas
fissão de seu núcleo e libera uma grande quantidade
são conectadas em um fio separado, que contém uma
chave. Com a chave aberta (desligada), a bateria fornece de energia (2,35×1010 kJ/mol). O isótopo 235U ocorre
uma potência X. naturalmente em minérios de urânio, com concentração
de apenas 0,7%. Para ser utilizado na geração de
2 3 energia nuclear, o minério é submetido a um processo
de enriquecimento, visando aumentar a concentração do
isótopo 235U para, aproximadamente, 3% nas pastilhas.
Em décadas anteriores, houve um movimento mundial
para aumentar a geração de energia nuclear buscando
Chave 1 substituir, parcialmente, a geração de energia elétrica
a partir da queima do carvão, o que diminui a emissão
atmosférica de CO2 (gás com massa molar igual a 44 g/mol).
A queima do carvão é representada pela equação química:

C (s) + O2 (g) → CO2 (g) ∆H = −400 kJ/mol

Qual é a massa de CO2, em toneladas, que deixa de ser


Bateria liberada na atmosfera, para cada 100 g de pastilhas de
urânio enriquecido utilizadas em substituição ao carvão
Assumindo que as lâmpadas obedeçam à Lei de Ohm,
com a chave fechada, a potência fornecida pela bateria, como fonte de energia?
em função de X, é: A 2,10
B 7,70
2
A X. C 9,00
3
D 33,0
B X. E 300

3 QUESTÃO 51
C X.
2 O cobre presente nos fios elétricos e instrumentos
musicais é obtido a partir da ustulação do minério
D 2X. calcosita (Cu2S). Durante esse processo, ocorre o
aquecimento desse sulfeto na presença de oxigênio, de
E 3X. forma que o cobre fique “livre” e o enxofre se combine
com o O2 produzindo SO2, conforme a equação química:
Cu S (s) + O (g) ∆ 2 Cu (l) + SO (g)
2 2 2

As massas molares dos elementos Cu e S são,


respectivamente, iguais a 63,5 g/mol e 32 g/mol.
CANTO, E. L. Minerais, minérios, metais: de onde vêm?, para onde vão?
São Paulo: Moderna, 1996 (adaptado).

Considerando que se queira obter 16 mols do metal em


uma reação cujo rendimento é de 80%, a massa, em
gramas, do minério necessária para obtenção do cobre
é igual a
A 955.
B 1 018.
C 1 590.
D 2 035.
E 3 180.

CN - 1º dia | Caderno 9 - BRANCO - Página 17


*BR0975SAB18* 2015

QUESTÃO 52
Sais de amônio são sólidos iônicos com alto ponto de fusão, muito mais solúveis em água que as aminas
originais e ligeiramente solúveis em solventes orgânicos apolares, sendo compostos convenientes para serem
usados em xaropes e medicamentos injetáveis. Um exemplo é a efedrina, que funde a 79 °C, tem um odor
desagradável e oxida na presença do ar atmosférico formando produtos indesejáveis. O cloridrato de efedrina funde a
217 °C, não se oxida e é inodoro, sendo o ideal para compor os medicamentos.
OH OH

CH3 CH3
+ HCl

NHCH3 +
NH2CH3 Cl−

Efedrina Cloridrato de efedrina

SOUTO, C. R. O.; DUARTE, H. C. Química da vida: aminas. Natal: EDUFRN, 2006.

De acordo com o texto, que propriedade química das aminas possibilita a formação de sais de amônio estáveis,
facilitando a manipulação de princípios ativos?
A Acidez.
B Basicidade.
C Solubilidade.
D Volatilidade.
E Aromaticidade.

QUESTÃO 53
Além de ser uma prática ilegal, a adulteração de combustíveis é prejudicial ao meio ambiente, ao governo e,
especialmente, ao consumidor final. Em geral, essa adulteração é feita utilizando compostos com propriedades físicas
semelhantes às do combustível, mas de menor valor agregado.
Considerando um combustível com 20% de adulterante, a mistura em que a adulteração seria identificada visualmente é
A etanol e água.
B etanol e acetona.
C gasolina e água.
D gasolina e benzeno.
E gasolina e querosene.

QUESTÃO 54
Sabe-se que nas proximidades dos polos do planeta Terra é comum a formação dos icebergs, que são grandes
blocos de gelo, flutuando nas águas oceânicas. Estudos mostram que a parte de gelo que fica emersa durante a
flutuação corresponde a aproximadamente 10% do seu volume total. Um estudante resolveu simular essa situação
introduzindo um bloquinho de gelo no interior de um recipiente contendo água, observando a variação de seu nível
desde o instante de introdução até o completo derretimento do bloquinho.
Com base nessa simulação, verifica-se que o nível da água no recipiente
A subirá com a introdução do bloquinho de gelo e, após o derretimento total do gelo, esse nível subirá ainda mais.
B subirá com a introdução do bloquinho de gelo e, após o derretimento total do gelo, esse nível descerá, voltando
ao seu valor inicial.
C subirá com a introdução do bloquinho de gelo e, após o derretimento total do gelo, esse nível permanecerá
sem alteração.
D não sofrerá alteração com a introdução do bloquinho de gelo, porém, após seu derretimento, o nível subirá devido
a um aumento em torno de 10% no volume de água.
E subirá em torno de 90% do seu valor inicial com a introdução do bloquinho de gelo e, após seu derretimento, o
nível descerá apenas 10% do valor inicial.
CN - 1º dia | Caderno 9 - BRANCO - Página 18
2015 *BR0975SAB19*
QUESTÃO 55 QUESTÃO 58
A rede elétrica de uma residência tem tensão de O avanço tecnológico da medicina propicia o
110 V e o morador compra, por engano, uma lâmpada desenvolvimento de tratamento para diversas doenças,
incandescente com potência nominal de 100 W e tensão como as relacionadas à visão. As correções que utilizam
nominal de 220 V. laser para o tratamento da miopia são consideradas
Se essa lâmpada for ligada na rede de 110 V, o que seguras até 12 dioptrias, dependendo da espessura e
acontecerá? curvatura da córnea. Para valores de dioptria superiores a
A A lâmpada brilhará normalmente, mas como a tensão esse, o implante de lentes intraoculares é mais indicado.
é a metade da prevista, a corrente elétrica será o Essas lentes, conhecidas como lentes fácicas (LF), são
dobro da normal, pois a potência elétrica é o produto implantadas junto à córnea, antecedendo o cristalino (C),
de tensão pela corrente. sem que esse precise ser removido, formando a imagem
B A lâmpada não acenderá, pois ela é feita para trabalhar correta sobre a retina (R).
apenas com tensão de 220 V, e não funciona com O comportamento de um feixe de luz incidindo no olho
tensão abaixo desta.
que possui um implante de lentes fácicas para correção
C A lâmpada irá acender dissipando uma potência de do problema de visão apresentado é esquematizado por
50 W, pois como a tensão é metade da esperada, a
potência também será reduzida à metade.
D A lâmpada irá brilhar fracamente, pois com a metade
da tensão nominal, a corrente elétrica também será
menor e a potência dissipada será menos da metade
da nominal.
E A lâmpada queimará, pois como a tensão é menor do A
que a esperada, a corrente será maior, ultrapassando
a corrente para a qual o filamento foi projetado. LF C R
QUESTÃO 56
A obtenção de sistemas coloidais estáveis depende
das interações entre as partículas dispersas e o meio
onde se encontram. Em um sistema coloidal aquoso,
cujas partículas são hidrofílicas, a adição de um solvente B
orgânico miscível em água, como etanol, desestabiliza
o coloide, podendo ocorrer a agregação das partículas
preliminarmente dispersas. LF C R
A desestabilização provocada pelo etanol ocorre porque
A a polaridade da água no sistema coloidal é reduzida.
B as cargas superficiais das partículas coloidais são
diminuídas.
C as camadas de solvatação de água nas partículas
são diminuídas. C
D o processo de miscibilidade da água e do solvente
libera calor para o meio. LF C R
E a intensidade dos movimentos brownianos das
partículas coloidais é reduzida.

QUESTÃO 57
Dentre outras características, uma determinada
vegetação apresenta folhas durante três a quatro meses D
ao ano, com limbo reduzido, mecanismo rápido de
abertura e fechamento dos estômatos e caule suculento. LF C R
Essas são algumas características adaptativas das
plantas ao bioma onde se encontram.
Que fator ambiental é o responsável pela ocorrência
dessas características adaptativas?
A Escassez de nutrientes no solo.
B Estratificação da vegetação. E
C Elevada insolação.
D Baixo pH do solo. LF C R
E Escassez de água.

CN - 1º dia | Caderno 9 - BRANCO - Página 19


*BR0975SAB20* 2015

QUESTÃO 59

O papel tem na celulose sua matéria-prima, e uma das etapas de sua produção é o branqueamento, que visa
remover a lignina da celulose. Diferentes processos de branqueamento usam, por exemplo, cloro (Cl2), hipoclorito
de sódio (NaClO), oxigênio (O2), ozônio (O3) ou peróxido de hidrogênio (H2O2). Alguns processos de branqueamento
levam à formação de compostos organoclorados. São apresentadas as estruturas de um fragmento da lignina e do
tetracloroguaiacol, um dos organoclorados formados no processo de branqueamento.

CH2OH
CH
CHOH

CH2OH CH2OH
CH CH OCH3 CH2
CHOH CH O CH
O CH

CH3O OCH3 OCH3 CH2OH


O O CH CH3O OCH3
CH O

CH3O OCH3
O

Fragmento da Lignina

OH
CI OCH3

CI CI
CI
Tetracloroguaiacol

SANTOS, C. P. et al. Papel: como se fabrica? Química Nova na Escola, n. 14, 2001 (adaptado).

Os reagentes capazes de levar à formação de organoclorados no processo citado são

A O2 e O3.
B Cl2 e O2.
C H2O2 e Cl2.
D NaClO e O3.
E NaClO e Cl2.

CN - 1º dia | Caderno 9 - BRANCO - Página 20


2015 *BR0975SAB21*
QUESTÃO 60 QUESTÃO 62
A remoção de petróleo derramado em ecossistemas Uma enzima foi retirada de um dos órgãos do sistema
marinhos é complexa e muitas vezes envolve a adição de digestório de um cachorro e, após ser purificada, foi
mais sustâncias ao ambiente. Para facilitar o processo de diluída em solução fisiológica e distribuída em três tubos
recuperação dessas áreas, pesquisadores têm estudado de ensaio com os seguintes conteúdos:
a bioquímica de bactérias encontradas em locais sujeitos
• Tubo 1: carne
a esse tipo de impacto. Eles verificaram que algumas
dessas espécies utilizam as moléculas de hidrocarbonetos • Tubo 2: macarrão
como fonte energética, atuando como biorremediadores,
• Tubo 3: banha
removendo o óleo do ambiente.
KREPSKY, N.; SILVA SOBRINHO, F.; CRAPEZ, M. A. C. Em todos os tubos foi adicionado ácido clorídrico (HCl),
Ciência Hoje, n. 223, jan.-fev. 2006 (adaptado). e o pH da solução baixou para um valor próximo a 2. Além
Para serem eficientes no processo de biorremediação disso, os tubos foram mantidos por duas horas a uma
citado, as espécies escolhidas devem possuir temperatura de 37 °C. A digestão do alimento ocorreu
somente no tubo 1.
A células flageladas, que capturem as partículas de
óleo presentes na água. De qual órgão do cachorro a enzima foi retirada?
B altas taxas de mutação, para se adaptarem ao A Fígado.
ambiente impactado pelo óleo. B Pâncreas.
C enzimas, que catalisem reações de quebra das C Estômago.
moléculas constituintes do óleo. D Vesícula biliar.
D parede celular espessa, que impossibilite que as E Intestino delgado.
bactérias se contaminem com o óleo.
E capacidade de fotossíntese, que possibilite a liberação QUESTÃO 63
de oxigênio para a renovação do ambiente poluído.
Observações astronômicas indicam que no centro
QUESTÃO 61 de nossa galáxia, a Via Láctea, provavelmente exista um
buraco negro cuja massa é igual a milhares de vezes a
Os calcários são materiais compostos por carbonato massa do Sol. Uma técnica simples para estimar a massa
de cálcio, que podem atuar como sorventes do dióxido desse buraco negro consiste em observar algum objeto
de enxofre (SO2), um importante poluente atmosférico. que orbite ao seu redor e medir o período de uma rotação
As reações envolvidas no processo são a ativação do completa, T, bem como o raio médio, R, da órbita do objeto,
calcário, por meio de calcinação, e a fixação do SO2 com que supostamente se desloca, com boa aproximação, em
a formação de um sal de cálcio, como ilustrado pelas movimento circular uniforme. Nessa situação, considere
equações químicas simplificadas. que a força resultante, devido ao movimento circular, é
igual, em magnitude, à força gravitacional que o buraco
calor
CaCO3 CaO + CO2 negro exerce sobre o objeto.
A partir do conhecimento do período de rotação, da
1
CaO + SO2 + O2 Sal de cálcio distância média e da constante gravitacional, G, a massa
2
do buraco negro é
Considerando-se as reações envolvidas nesse processo
de dessulfurização, a fórmula química do sal de cálcio 4p2R 2
corresponde a A .
GT 2
A CaSO3.
B CaSO4.
p2R 3
C CaS2O8. B .
2GT 2
D CaSO2.
E CaS2O7.
2p2R 3
C .
GT 2

4p2R 3
D .
GT 2

p2R 5
E .
GT 2

CN - 1º dia | Caderno 9 - BRANCO - Página 21


*BR0975SAB22* 2015

QUESTÃO 64
As superbactérias respondem por um número crescente de infecções e mortes em todo o mundo. O termo
superbactérias é atribuído às bactérias que apresentam resistência a praticamente todos os antibióticos. Dessa forma,
no organismo de um paciente, a população de uma espécie bacteriana patogênica pode ser constituída principalmente
por bactérias sensíveis a antibióticos usuais e por um número reduzido de superbactérias que, por mutação ou
intercâmbio de material genético, tornaram-se resistentes aos antibióticos existentes.
FERREIRA, F. A.; CRUZ, R. S.; FIGUEIREDO, A. M. S. Superbactérias: o problema mundial da resistência a antibióticos. Ciência Hoje, n. 287, nov. 2011 (adaptado).

Qual figura representa o comportamento populacional das bactérias ao longo de uma semana de tratamento
com um antibiótico comum?

I
Densidade populacional

II I - Bactérias sensíveis ao antibiótico

II - Bactérias resistentes ao antibiótico


A

1 2 3 4 5 6 7
Dias de tratamento
Densidade populacional

II I - Bactérias sensíveis ao antibiótico

II - Bactérias resistentes ao antibiótico


B
I
1 2 3 4 5 6 7
Dias de tratamento
Densidade populacional

I I - Bactérias sensíveis ao antibiótico

II - Bactérias resistentes ao antibiótico


C
II

1 2 3 4 5 6 7
Dias de tratamento
Densidade populacional

I - Bactérias sensíveis ao antibiótico

II - Bactérias resistentes ao antibiótico


D
I

II
1 2 3 4 5 6 7
Dias de tratamento
Densidade populacional

II I - Bactérias sensíveis ao antibiótico

II - Bactérias resistentes ao antibiótico


E

1 2 3 4 5 6 7
Dias de tratamento

CN - 1º dia | Caderno 9 - BRANCO - Página 22


2015 *BR0975SAB23*
QUESTÃO 65 QUESTÃO 67

Em altos-fornos siderúrgicos, as temperaturas acima Num sistema de freio convencional, as rodas do


de 600 °C são mensuradas por meio de pirômetros óticos. carro travam e os pneus derrapam no solo, caso a força
Esses dispositivos apresentam a vantagem de medir a
exercida sobre o pedal seja muito intensa. O sistema
temperatura de um objeto aquecido sem necessidade
ABS evita o travamento das rodas, mantendo a força de
de contato. Dentro de um pirômetro ótico, um filamento
metálico é aquecido pela passagem de corrente elétrica atrito no seu valor estático máximo, sem derrapagem.
até que sua cor seja a mesma que a do objeto aquecido O coeficiente de atrito estático da borracha em contato
em observação. Nessa condição, a temperatura com o concreto vale µe = 1,0 e o coeficiente de atrito
conhecida do filamento é idêntica à do objeto aquecido cinético para o mesmo par de materiais é µc = 0,75. Dois
em observação. carros, com velocidades iniciais iguais a 108 km/h, iniciam
Disponível em: www.if.usp.br. Acesso em: 4 ago. 2012 (adaptado). a frenagem numa estrada perfeitamente horizontal de
A propriedade da radiação eletromagnética avaliada concreto no mesmo ponto. O carro 1 tem sistema ABS e
nesse processo é a utiliza a força de atrito estática máxima para a frenagem;
A amplitude. já o carro 2 trava as rodas, de maneira que a força de
B coerência. atrito efetiva é a cinética. Considere g = 10 m/s2.
C frequência. As distâncias, medidas a partir do ponto em que iniciam
D intensidade. a frenagem, que os carros 1 (d1) e 2 (d2) percorrem até
E velocidade. parar são, respectivamente,

QUESTÃO 66 A d1 = 45 m e d2 = 60 m.
B d1 = 60 m e d2 = 45 m.
A fenilcetonúria é uma doença hereditária
autossômica recessiva, associada à mutação do C d1 = 90 m e d2 = 120 m.
gene PAH, que limita a metabolização do aminoácido D d1 = 5,8×102 m e d2 = 7,8×102 m.
fenilalanina. Por isso, é obrigatório, por lei, que E d1 = 7,8×102 m e d2 = 5,8×102 m.
as embalagens de alimentos, como refrigerantes
dietéticos, informem a presença de fenilalanina em sua QUESTÃO 68
composição. Uma mulher portadora de mutação para
o gene PAH tem três filhos normais, com um homem Cinco indústrias de ramos diferentes foram instaladas
normal, cujo pai sofria de fenilcetonúria, devido à ao longo do curso de um rio. O descarte dos efluentes
mesma mutação no gene PAH encontrada em um dos dessas indústrias acarreta impacto na qualidade de suas
alelos da mulher. águas. O pH foi determinado em diferentes pontos desse
Qual a probabilidade de a quarta criança gerada por rio, a 25 °C, e os resultados são apresentados no quadro.
esses pais apresentar fenilcetonúria?
Pontos de coleta Valor do pH
A 0% Antes da primeira indústria 5,5
B 12,5% Entre a primeira e a segunda indústria 5,5
C 25% Entre a segunda e a terceira indústria 7,5
D 50% Entre a terceira e a quarta indústria 7,0
E 75%
Entre a quarta e a quinta indústria 7,0
Após a quinta indústria 6,5

A indústria que descarta um efluente com características


básicas é a
A primeira.
B segunda.
C terceira.
D quarta.
E quinta.

CN - 1º dia | Caderno 9 - BRANCO - Página 23


*BR0975SAB24* 2015

QUESTÃO 69
O poli(ácido lático) ou PLA é um material de interesse tecnológico por ser um polímero biodegradável e bioabsorvível.
O ácido lático, um metabólito comum no organismo humano, é a matéria-prima para produção do PLA, de acordo com
a equação química simplificada:
CH3 CH3 O CH3

OH polimerização O OH
HO HO O
n
O O CH3 O

Ácido d/l-lático 100 ≤ n ≤ 10 000

Que tipo de polímero de condensação é formado nessa reação?


A Poliéster.
B Polivinila.
C Poliamida.
D Poliuretana.
E Policarbonato.

QUESTÃO 70
O alumínio é um metal bastante versátil, pois, a partir dele, podem-se confeccionar materiais amplamente utilizados
pela sociedade. A obtenção do alumínio ocorre a partir da bauxita, que é purificada e dissolvida em criolita fundida
(Na3AlF6) e eletrolisada a cerca de 1 000 °C. Há liberação do gás dióxido de carbono (CO2), formado a partir da reação
de um dos produtos da eletrólise com o material presente nos eletrodos. O ânodo é formado por barras de grafita
submergidas na mistura fundida. O cátodo é uma caixa de ferro coberta de grafita. A reação global do processo é:
2 Al2O3 (l) + 3 C (s) → 4 Al (l) + 3 CO2 (g)
Na etapa de obtenção do alumínio líquido, as reações que ocorrem no cátodo e ânodo são:

cátodo: Al3+ + 3 e− → Al
A 2 O2− → O2 + 4 e−
ânodo
C + O2 → CO2

2 O2− → O2 + 4 e−
cátodo
B C + O2 → CO2
ânodo: Al3+ + 3 e− → Al

Al3+ + 3 e− → Al
cátodo
C 2 O2− → O2 + 4 e−
ânodo: C + O2 → CO2

Al3+ + 3 e− → Al
cátodo
D C + O2 → CO2
ânodo: 2 O2− → O2 + 4 e−

cátodo: 2 O2− → O2 + 4 e−
E Al3+ + 3 e− → Al
ânodo
C + O2 → CO2

CN - 1º dia | Caderno 9 - BRANCO - Página 24


2015 *BR0975SAB25*
QUESTÃO 71
O quadro apresenta a composição do petróleo.

Faixa de tamanho Faixa de ponto de


Fração Usos
das moléculas ebulição (°C)
Gás C1 a C5 −160 a 30 combustíveis gasosos
Gasolina C5 a C12 30 a 200 combustível de motor
Querosene C12 a C18 180 a 400 diesel e combustível de alto-forno
Lubrificantes maior que C16 maior que 350 lubrificantes
Parafinas maior que C20 sólidos de baixa fusão velas e fósforos
Asfalto maior que C30 resíduos pastosos pavimentação
BROWN, T. L. et al. Química: a ciência central. São Paulo: Person Prentice Hall, 2005.

Para a separação dos constituintes com o objetivo de produzir a gasolina, o método a ser utilizado é a
A filtração.
B destilação.
C decantação.
D precipitação.
E centrifugação.

QUESTÃO 72
Um gel vaginal poderá ser um recurso para as mulheres na prevenção contra a aids. Esse produto tem como
princípio ativo um composto que inibe a transcriptase reversa viral.
Essa ação inibidora é importante, pois a referida enzima
A corta a dupla hélice do DNA, produzindo um molde para o RNA viral.
B produz moléculas de DNA viral que vão infectar células sadias.
C polimeriza molécula de DNA, tendo como molde o RNA viral.
D promove a entrada do vírus da aids nos linfócitos T.
E sintetiza os nucleotídeos que compõem o DNA viral.

QUESTÃO 73
O Nylon® é um polímero (uma poliamida) obtido pela reação do ácido adípico com a hexametilenodiamina, como
indicado no esquema reacional.

O O O O

( ) ( ) ( ) ( )
HO 4 OH + H2N 4 NH2 4 N 4 N + n H2O

H H
n
ácido hexanodioico 1,6-diamino-hexano
Nylon 6,6
(ácido adípico) (hexametilenodiamina)
Na época da invenção desse composto, foi proposta uma nomenclatura comercial, baseada no número de átomos
de carbono do diácido carboxílico, seguido do número de carbonos da diamina.
De acordo com as informações do texto, o nome comercial de uma poliamida resultante da reação do ácido butanodioico
com o 1,2-diamino-etano é
A Nylon 4,3.
B Nylon 6,2.
C Nylon 3,4.
D Nylon 4,2.
E Nylon 2,6.

CN - 1º dia | Caderno 9 - BRANCO - Página 25


*BR0975SAB26* 2015

QUESTÃO 74 QUESTÃO 76
O vinagre vem sendo usado desde a Antiguidade A figura representa uma embalagem cartonada e
como conservante de alimentos, bem como agente de sua constituição em multicamadas. De acordo com as
limpeza e condimento. Um dos principais componentes orientações do fabricante, essas embalagens não devem
do vinagre é o ácido acético (massa molar 60 g/mol), cuja ser utilizadas em fornos micro-ondas.
faixa de concentração deve se situar entre 4% a 6% (m/v).
Em um teste de controle de qualidade foram analisadas
cinco marcas de diferentes vinagres, e as concentrações
de ácido acético, em mol/L, se encontram no quadro.
polietileno.....
alumínio......
Amostra Concentração de ácido acético (mol/L) polietileno...
1 0,007 papel...........
2 0,070 polietileno....
3 0,150
4 0,400
5 0,700
RIZZON, L. A. Sistema de produção de vinagre.
Disponível em: www.sistemasdeproducao.cnptia.embrapa.br. NASCIMENTO, R. M. M. et al. Embalagem cartonada longa vida: lixo ou luxo?
Acesso em: 14 ago. 2012 (adaptado). Química Nova na Escola, n. 25, maio 2007 (adaptado).

A amostra de vinagre que se encontra dentro do limite de A restrição citada deve-se ao fato de a
concentração tolerado é a
A embalagem aberta se expandir pela pressão do vapor
A 1. formado em seu interior.
B 2. B camada de polietileno se danificar, colocando o
C 3. alumínio em contato com o alimento.
D 4. C fina camada de alumínio blindar a radiação, não
permitindo que o alimento se aqueça.
E 5.
D absorção de radiação pelo papel, que se aquece e
QUESTÃO 75 pode levar à queima da camada de polietileno.
E geração de centelhas na camada de alumínio,
Os parasitoides são insetos diminutos, que têm
que pode levar à queima da camada de papel e de
hábitos bastante peculiares: suas larvas se desenvolvem
polietileno.
dentro do corpo de outros animais. Em geral, cada
parasitoide ataca hospedeiros de determinada espécie
e, por isso, esses organismos vêm sendo amplamente
usados para o controle biológico de pragas agrícolas.
SANTO, M. M. E. et al. Parasitoides: insetos benéficos e cruéis.
Ciência Hoje, n. 291, abr. 2012 (adaptado).

O uso desses insetos na agricultura traz benefícios


ambientais, pois diminui o(a)
A tempo de produção agrícola.
B diversidade de insetos-praga.
C aplicação de inseticidas tóxicos.
D emprego de fertilizantes agrícolas.
E necessidade de combate a ervas daninhas.

CN - 1º dia | Caderno 9 - BRANCO - Página 26


2015 *BR0975SAB27*
QUESTÃO 77 QUESTÃO 78

No manual de uma torneira elétrica são fornecidas De acordo com estatísticas do Ministério da
instruções básicas de instalação para que o produto
Saúde, cerca de 5% das pessoas com dengue
funcione corretamente:
hemorrágica morrem. A dengue hemorrágica tem como
• Se a torneira for conectada à caixa-d’água
domiciliar, a pressão da água na entrada da base fisiopatológica uma resposta imune anômala,
torneira deve ser no mínimo 18 kPa e no máximo causando aumento da permeabilidade de vasos
38 kPa. sanguíneos, queda da pressão arterial e manifestações
• Para pressões da água entre 38 kPa e 75 kPa hemorrágicas, podendo ocorrer manchas vermelhas
ou água proveniente diretamente da rede pública, na pele e sangramento pelo nariz, boca e gengivas.
é necessário utilizar o redutor de pressão que
acompanha o produto. O hemograma do paciente pode apresentar como
resultado leucopenia (diminuição do número de glóbulos
• Essa torneira elétrica pode ser instalada em um
brancos), linfocitose (aumento do número de linfócitos),
prédio ou em uma casa.
aumento do hematócrito e trombocitopenia (contagem de
Considere a massa específica da água 1 000 kg/m3 e a
aceleração da gravidade 10 m/s2. plaquetas abaixo de 100 000/mm3).
Disponível em: www.ciencianews.com.br. Acesso em: 28 fev. 2012 (adaptado).

Caixa-d’água
Relacionando os sintomas apresentados pelo paciente
com dengue hemorrágica e os possíveis achados do
hemograma, constata-se que

A as manifestações febris ocorrem em função da


diminuição dos glóbulos brancos, uma vez que estes
Altura h controlam a temperatura do corpo.
B a queda na pressão arterial é ocasionada pelo
aumento do número de linfócitos, que têm como
função principal a produção de anticorpos.
Torneira elétrica C o sangramento pelo nariz, pela boca e gengiva é
ocasionado pela quantidade reduzida de plaquetas,
Para que a torneira funcione corretamente, sem o uso que são responsáveis pelo transporte de oxigênio.
do redutor de pressão, quais deverão ser a mínima e a D as manifestações hemorrágicas estão associadas
máxima altura entre a torneira e a caixa-d’água?
à trombocitopenia, uma vez que as plaquetas estão
A 1,8 m e 3,8 m envolvidas na cascata de coagulação sanguínea.
B 1,8 m e 7,5 m E os sangramentos observados ocorrem em função
C 3,8 m e 7,5 m da linfocitose, uma vez que os linfócitos são
D 18 m e 38 m responsáveis pela manutenção da integridade dos
E 18 m e 75 m vasos sanguíneos.

CN - 1º dia | Caderno 9 - BRANCO - Página 27


*BR0975SAB28* 2015

QUESTÃO 79
O caramujo gigante africano, Achatina fulica, é uma espécie exótica que tem despertado o interesse das
autoridades brasileiras, uma vez que tem causado danos ambientais e prejuízos econômicos à agricultura.
A introdução da espécie no Brasil ocorreu clandestinamente, com o objetivo de ser utilizada na alimentação humana.
Porém, o molusco teve pouca aceitação no comércio de alimentos, o que resultou em abandono e liberação
intencional das criações por vários produtores. Por ser uma espécie herbívora generalista (alimenta-se de mais
de 500 espécies diferentes de vegetais), com grande capacidade reprodutiva, tornou-se uma praga agrícola de
difícil erradicação. Associada a isto, a ausência de predadores naturais fez com que ocorresse um crescimento
descontrolado da população.
O desequilíbrio da cadeia alimentar observado foi causado pelo aumento da densidade populacional de
A consumidores terciários, em função da elevada disponibilidade de consumidores secundários.
B consumidores primários, em função da ausência de consumidores secundários.
C consumidores secundários, em função da ausência de consumidores primários.
D consumidores terciários, em função da elevada disponibilidade de produtores.
E consumidores primários, em função do aumento de produtores.

QUESTÃO 80
O álcool utilizado como combustível automotivo (etanol hidratado) deve apresentar uma taxa máxima de água em
sua composição para não prejudicar o funcionamento do motor. Uma maneira simples e rápida de estimar a quantidade
de etanol em misturas com água é medir a densidade da mistura. O gráfico mostra a variação da densidade da mistura
(água e etanol) com a fração percentual da massa de etanol (fe), dada pela expressão

me
fe = 100 × ,
(me + ma)

em que me e ma são as massas de etanol e de água na mistura, respectivamente, a uma temperatura de 20 °C.

1
0,98
0,96
0,94
0,92
0,9
ρ (g/cm³)

0,88
0,86
0,84
0,82
0,8
0,78
0,76

0 10 20 30 40 50 60 70 80 90 100
fe
Disponível em: www.handymath.com. Acesso em: 8 ago. 2012.

Suponha que, em uma inspeção de rotina realizada em determinado posto, tenha-se verificado que 50,0 cm3 de álcool
combustível tenham massa igual a 45,0 g. Qual é a fração percentual de etanol nessa mistura?
A 7%
B 10%
C 55%
D 90%
E 93%

CN - 1º dia | Caderno 9 - BRANCO - Página 28


2015 *BR0975SAB29*
QUESTÃO 81 QUESTÃO 83

A toxina botulínica (produzida pelo bacilo A reprodução vegetativa de plantas por meio de
Clostridium botulinum) pode ser encontrada em estacas é um processo natural. O homem, observando
alimentos malconservados, causando até a morte de esse processo, desenvolveu uma técnica para propagar
consumidores. No entanto, essa toxina modificada plantas em escala comercial.
em laboratório está sendo usada cada vez mais A base genética dessa técnica é semelhante àquela
para melhorar a qualidade de vida das pessoas com presente no(a)
problemas físicos e/ou estéticos, atenuando problemas
como o blefaroespasmo, que provoca contrações A transgenia.
involuntárias das pálpebras. B clonagem.
BACHUR, T. P. R. et al. Toxina botulínica: de veneno a tratamento. Revista Eletrônica
C hibridização.
Pesquisa Médica, n. 1, jan.-mar. 2009 (adaptado).
D controle biológico.
O alívio dos sintomas do blefaroespasmo é consequência
da ação da toxina modificada sobre o tecido E melhoramento genético.

A glandular, uma vez que ela impede a produção de QUESTÃO 84


secreção de substâncias na pele.
O acúmulo de plásticos na natureza pode levar a
B muscular, uma vez que ela provoca a paralisia das impactos ambientais negativos, tanto em ambientes
fibras que formam esse tecido. terrestres quanto aquáticos. Uma das formas de
C epitelial, uma vez que ela leva ao aumento da camada minimizar esse problema é a reciclagem, para a qual é
de queratina que protege a pele. necessária a separação dos diferentes tipos de plásticos.
D conjuntivo, uma vez que ela aumenta a quantidade de Em um processo de separação foi proposto o seguinte
substância intercelular no tecido. procedimento:
E adiposo, uma vez que ela reduz a espessura da I.Coloque a mistura de plásticos picados em um
camada de células de gordura do tecido. tanque e acrescente água até a metade da sua
capacidade.
QUESTÃO 82 II. Mantenha essa mistura em repouso por cerca de
A cafeína é um alcaloide, identificado como 10 minutos.
1,3,7-trimetilxantina (massa molar igual a 194 g/mol), III. Retire os pedaços que flutuaram e transfira-os
cuja estrutura química contém uma unidade de purina, para outro tanque com uma solução de álcool.
conforme representado. Esse alcaloide é encontrado em IV. Coloque os pedaços sedimentados em outro
grande quantidade nas sementes de café e nas folhas de tanque com solução de sal e agite bem.
chá-verde. Uma xícara de café contém, em média, 80 mg Qual propriedade da matéria possibilita a utilização do
de cafeína. procedimento descrito?
A Massa.
O
CH3 B Volume.
C Densidade.
H3C
N N D Porosidade.
E Maleabilidade.
N
O N

CH3
MARIA, C. A. B.; MOREIRA, R. F. A. Cafeína: revisão sobre métodos de análise.
Química Nova, n. 1, 2007 (adaptado).

Considerando que a xícara descrita contém um volume


de 200 mL de café, a concentração, em mol/L, de cafeína
nessa xícara é mais próxima de:
A 0,0004.
B 0,002.
C 0,4.
D 2.
E 4.

CN - 1º dia | Caderno 9 - BRANCO - Página 29


*BR0975SAB30* 2015

QUESTÃO 85 QUESTÃO 86
Para irrigar sua plantação, um produtor rural construiu A fotografia feita sob luz polarizada é usada por
um reservatório a 20 metros de altura a partir da barragem dermatologistas para diagnósticos. Isso permite ver
de onde será bombeada a água. Para alimentar o motor detalhes da superfície da pele que não são visíveis com o
elétrico das bombas, ele instalou um painel fotovoltaico. reflexo da luz branca comum. Para se obter luz polarizada,
A potência do painel varia de acordo com a incidência pode-se utilizar a luz transmitida por um polaroide ou a
solar, chegando a um valor de pico de 80 W ao meio-dia. luz refletida por uma superfície na condição de Brewster,
Porém, entre as 11 horas e 30 minutos e as 12 horas e como mostra a figura. Nessa situação, o feixe da luz
30 minutos, disponibiliza uma potência média de 50 W. refratada forma um ângulo de 90° com o feixe da luz
Considere a aceleração da gravidade igual a 10 m/s2 e refletida, fenômeno conhecido como Lei de Brewster.
uma eficiência de transferência energética de 100%. Nesse caso, o ângulo de incidência θp , também chamado
Qual é o volume de água, em litros, bombeado para o de ângulo de polarização, e o ângulo de refração θr estão
reservatório no intervalo de tempo citado? em conformidade com a Lei de Snell.

A 150
B 250 Raio de luz incidente Raio de luz refletido
(não polarizado) (polarizado)
C 450
D 900
θp θp
E 1 440

Lâmina

θr

Raio de luz refratado


(parcialmente polarizado)
Dado:
1
sen 30° = cos 60° =
2
√3
sen 60° = cos 30° =
2

Considere um feixe de luz não polarizada proveniente


de um meio com índice de refração igual a 1, que incide
sobre uma lâmina e faz um ângulo de refração θr de 30°.
Nessa situação, qual deve ser o índice de refração da
lâmina para que o feixe refletido seja polarizado?

A √3

√3
B
3

C 2

1
D
2
√3
E
2

CN - 1º dia | Caderno 9 - BRANCO - Página 30


2015 *BR0975SAB31*
QUESTÃO 87 QUESTÃO 89

Na natureza a matéria é constantemente transformada Durante uma aula experimental de física, os


por meio dos ciclos biogeoquímicos. Além do ciclo
estudantes construíram um sistema ressonante com
da água, existem os ciclos do carbono, do enxofre, do
fósforo, do nitrogênio e do oxigênio. pêndulos simples. As características de cada pêndulo
O elemento que está presente em todos os ciclos são apresentadas no quadro. Inicialmente, os estudantes
nomeados é o colocaram apenas o pêndulo A para oscilar.
A fósforo. Comprimento
Pêndulo Massa
B enxofre. do barbante
C carbono. A M L
D oxigênio. 1 M L
E nitrogênio.
M
2 2L
QUESTÃO 88 2
Bioindicador ou indicador biológico é uma espécie L
ou grupo de espécies que reflete o estado biótico ou 3 2M
2
abiótico de um meio ambiente, o impacto produzido sobre
um hábitat, comunidade ou ecossistema, entre outras M L
4
funções. A posição trófica do organismo bioindicador é 2 2
uma das características mais relevantes quanto ao seu 5 2M L
grau de importância para essa função: quanto mais baixo
o nível trófico do organismo, maior é a sua utilidade, pois Quais pêndulos, além desse, passaram também a oscilar?
pressupõe-se que toda a cadeia trófica é contaminada a
partir dele. A 1, 2, 3, 4 e 5.
ANDRÉA, M. M. Bioindicadores ecotoxicológicos de agrotóxicos.
Disponível em: www.biologico.sp.gov.br. Acesso em: 11 mar. 2013 (adaptado).
B 1, 2 e 3.
C 1 e 4.
O grupo de organismos mais adequado para essa
condição, do ponto de vista da sua posição na cadeia D 1 e 5.
trófica, é constituído por
E 3 e 4.
A algas.
B peixes. QUESTÃO 90
C baleias.
CO2
D camarões.
E anêmonas.

1 2

C6H12O6

No esquema representado, o processo identificado pelo


número 2 é realizado por

A seres herbívoros.
B fungos fermentadores.
C bactérias heterótrofas.
D organismos produtores.
E microrganismos decompositores.

CN - 1º dia | Caderno 9 - BRANCO - Página 31


*BR0975SAB32* 2015

2015

CN - 1º dia | Caderno 9 - BRANCO - Página 32


EXAME NACIONAL DO ENSINO MÉDIO
PROVA DE REDAÇÃO E DE LINGUAGENS, CÓDIGOS E SUAS TECNOLOGIAS
PROVA DE MATEMÁTICA E SUAS TECNOLOGIAS

2º DIA
CADERNO

2015 15
2ª APLICAÇÃO

ATENÇÃO: transcreva no espaço apropriado do seu CARTÃO-RESPOSTA,


com sua caligrafia usual, considerando as letras maiúsculas e minúsculas, a seguinte frase:

A minha diferença é sempre menos.

LEIA ATENTAMENTE AS INSTRUÇÕES SEGUINTES: 3. Para cada uma das questões objetivas, são apresentadas
5 opções. Apenas uma responde corretamente à questão.
1. Este CADERNO DE QUESTÕES contém a Proposta de
Redação e 90 questões numeradas de 91 a 180, dispostas 4. O tempo disponível para estas provas é de cinco horas e
da seguinte maneira: trinta minutos.
a) as questões de número 91 a 135 são relativas à área de
5. Reserve os 30 minutos finais para marcar seu CARTÃO-RESPOSTA.
Linguagens, Códigos e suas Tecnologias;
Os rascunhos e as marcações assinaladas no CADERNO DE
b) as questões de número 136 a 180 são relativas à área de QUESTÕES não serão considerados na avaliação.
Matemática e suas Tecnologias.
ATENÇÃO: as questões de 91 a 95 são relativas à língua 6. Somente serão corrigidas as redações transcritas na FOLHA
estrangeira. Você deverá responder apenas às questões DE REDAÇÃO.
relativas à língua estrangeira (inglês ou espanhol) escolhida
7. Quando terminar as provas, acene para chamar o aplicador
no ato de sua inscrição.
e entregue este CADERNO DE QUESTÕES e o CARTÃO-
2. Confira se o seu CADERNO DE QUESTÕES contém a RESPOSTA/FOLHA DE REDAÇÃO.
quantidade de questões e se essas questões estão na ordem
mencionada na instrução anterior. Caso o caderno esteja 8. Você poderá deixar o local de prova somente após
incompleto, tenha defeito ou apresente qualquer divergência, decorridas duas horas do início da aplicação e não poderá
comunique ao aplicador da sala para que ele tome as levar seu CADERNO DE QUESTÕES ao deixar em
providências cabíveis. definitivo a sala de prova.

Ministério
da Educação *CZ1525dom1*
*CZ1525dom2* 2015

PROPOSTA DE REDAÇÃO
A partir da leitura dos textos motivadores seguintes e com base nos conhecimentos construídos ao longo
de sua formação, redija texto dissertativo-argumentativo em modalidade escrita formal da língua portuguesa
sobre o tema “O histórico desafio de se valorizar o professor”, apresentando proposta de intervenção que
respeite os direitos humanos. Selecione, organize e relacione, de forma coerente e coesa, argumentos e fatos
para defesa de seu ponto de vista.
TEXTO I TEXTO II
A escolha profissional passava necessariamente pela
ideia de frequentar um curso de qualidade, que dava uma
excelente cultura geral e preparo adequado para exercer
uma profissão que era reputada como digna e prestigiada,
fosse ela exercida por homens ou por mulheres. A figura
da mulher que lecionava era bem aceita e apontada
às moças como exemplo de honestidade e ideal a ser
seguido. O mesmo acontecia com o professor. A família
tinha a figura da professora e do professor em grande
consideração e estes detinham um prestígio social que
estava em claro desacordo com a remuneração salarial
percebida. Eles desfrutavam um prestígio advindo do
saber, e não do poder aquisitivo.
ALMEIDA, J. S. D. Mulher e educação: a paixão pelo possível. São Paulo: Unesp, 1998 (adaptado). Disponível em: http://www.sinpro-rs.org.br. Acesso em: 26 jun. 2015 (adaptado).

TEXTO III
O estatuto social e econômico é a chave para o estudo dos professores e da sua profissão. Num olhar rápido
temos a impressão que a imagem social e a condição econômica dos professores se encontram num estado de
grande degradação, sentimento que é confirmado por certos discursos das organizações sindicais e mesmo das
autoridades estatais. Mas, cada vez que a análise é mais fina, os resultados são menos concludentes e a profissão
docente continua a revelar facetas atrativas. É evidente que há uma perda de prestígio, associada à alteração do
papel tradicional dos professores no meio local: os professores do ensino primário já não são, ao lado dos párocos,
os únicos agentes culturais nas aldeias e vilas da província; os professores do ensino secundário já não pertencem à
elite social das cidades. NÓVOA, A. O passado e o presente dos professores. In NÓVOA, A. (Ed.). Profissão professor. Porto: Porto Editora, 1995 (adaptado).

TEXTO IV

Disponível em: http://www.sinprodf.org.br. Acesso em: 26 jun.2015.


INSTRUÇÕES:
• O rascunho da redação deve ser feito no espaço apropriado.
• O texto definitivo deve ser escrito à tinta, na folha própria, em até 30 linhas.
• A redação que apresentar cópia dos textos da Proposta de Redação ou do Caderno de Questões terá o número de linhas copiadas
desconsiderado para efeito de correção.
Receberá nota zero, em qualquer das situações expressas a seguir, a redação que:
• tiver até 7 (sete) linhas escritas, sendo considerada “texto insuficiente”.
• fugir ao tema ou que não atender ao tipo dissertativo-argumentativo.
• apresentar proposta de intervenção que desrespeite os direitos humanos.
• apresentar parte do texto deliberadamente desconectada do tema proposto.

LC - 2º dia | Caderno 15 - CINZA - Página 2


2015 *CZ1525dom3*
LINGUAGENS, CÓDIGOS E SUAS Nesse artigo de jornal, Buenos Aires é apresentada como
a capital argentina, que
TECNOLOGIAS
A foi objeto de novelas televisivas baseadas em sua
Questões de 91 a 135 vida noturna e artística.
Questões de 91 a 95 (opção inglês) B manteve sua elegância e espírito cosmopolita, apesar
das crises econômicas.
QUESTÃO 91 C teve sua energia e aspecto empreendedor ofuscados
pela incerteza da economia.
D foi marcada historicamente por uma vida financeira
estável, com repercussão na arte.
E parou de atrair apreciadores da gastronomia, devido
ao alto valor de sua moeda.

QUESTÃO 93

Horse or cow
Prior to taking retirement and selling off his land, a
farmer needed to get rid of all the animals he owned, so
he decided to call on every house in his village. At houses
where the man was the boss, he gave a horse; at houses
where the woman was the boss, he gave a dairy cow.
Approaching one cottage, he saw a couple gardening
and called out, ‘Who’s the boss around here?’
‘I am,’ said the man.
The farmer said: ‘I have a black horse and a brown
Disponível em: www.barhampc.kentparishes.gov.uk. Acesso em: 31 jul. 2012. horse. Which one would you like?’
The man thought for a minute and said, ‘The black one.’
Uma campanha pode ter por objetivo conscientizar ‘No, no, get the brown one,’ said his wife.
a população sobre determinada questão social. The farmer said, ‘Here’s your cow.’
Na campanha realizada no Reino Unido, a frase “A third TIBBALLS, G. The book of senior jokes. Great Britain: Michael O’Mara, 2009 (adaptado).
of the food we buy in the UK ends up being thrown away”
O texto relata o caso de um fazendeiro prestes a se
foi utilizada para enfatizar o(a) aposentar e vender sua fazenda. O aspecto cômico desse
A desigualdade social. texto provém da
B escassez de plantações. A constatação pelo fazendeiro da razão de sua
aposentadoria.
C reeducação alimentar. B opinião dos vizinhos referente à forma de se livrar dos
D desperdício de comida. animais.
E custo dos alimentos. C percepção do fazendeiro quanto à relação de poder
entre o casal.
QUESTÃO 92 D agressividade da esposa relacionada a um
questionamento inocente.
36 hours in Buenos Aires E indecisão dos cônjuges quanto à melhor escolha a
ser feita no momento.
Contemporary Argentine history is a roller coaster of
financial booms and cracks, set to gripping political soap
operas. But through all the highs and lows, one thing has
remained constant: Buenos Aires’s graceful elegance
and cosmopolitan cool. This attractive city continues to
draw food lovers, design buffs and party people with its
riotous night life, fashion-forward styling and a favorable
exchange rate. Even with the uncertain economy, the
creative energy and enterprising spirit of Porteños, as
residents are called, prevail — just look to the growing
ranks of art spaces, boutiques, restaurants and hotels.
SINGER, P. Disponível em: www.nytimes.com. Acesso em: 30 jul. 2012.

LC - 2º dia | Caderno 15 - CINZA - Página 3


*CZ1525dom4* 2015

QUESTÃO 94 LINGUAGENS, CÓDIGOS E SUAS


First Footing TECNOLOGIAS
One of the major Hogmanay customs was “first-
footing”. Shortly after “the bells” — the stroke of midnight Questões de 91 a 135
when public clocks would chime to signal the start of the Questões de 91 a 95 (opção espanhol)
new year —, neighbours would visit one another’s houses
to wish each other a good new year. This visiting was
known as “first-footing”, and the luckiest first-foot into any QUESTÃO 91
house was a tall, dark and handsome man — perhaps as Desde luego que para quienes continuamos
a reward to the woman who traditionally had spent the escribiendo en quechua, en aymara o en las lenguas
previous day scrubbing her house (another Hogmanay
amazónicas, o recreamos en castellano el subyugante
ritual). Women or red heads, however, were always
considered bad luck as first-foots. universo andino, el mayor obstáculo es, sin duda, el
First-foots brought symbolic gifts to “handsel” the lenguaje: cómo hacer verosímil — mediante la palabra —
house: coal for the fire, to ensure that the house would lo que de por sí es increíble en ese arcano territorio donde las
be warm and safe, and shortbread or black bun (a type of fronteras entre vida/muerte, mundo natural/sobrenatural,
fruit cake) to symbolise that the household would never go no existen y es común, más bien, toparse en un cruce
hungry that year. de caminos con un ángel andariego o recibir, tal vez, en
First-footing has faded in recent years, particularly with una siembra de papas, la visita inesperada de un familiar
the growth of the major street celebrations in Edinburgh muerto que viene — del más allá — a prevenirnos sobre
and Glasgow, although not the Scots love of a good party, el clima o porque simplemente tiene sed y desea un poco
of which there are plenty on the night!
Disponível em: www.visitscotland.com. Acesso em: 23 nov. 2011.
de chicha de maíz. No obstante a ello, la poesía quechua
contemporánea, la escrita por Alencastre por ejemplo,
A partir da leitura do texto sobre a comemoração do
Ano-novo na Escócia, observa-se que, com o tempo, tiene autor y códigos propios y ya no más ese carácter
aspectos da cultura de um povo podem ser colectivo, anónimo y oral de los inicios, cuando estaba
conformada por oraciones e himnos que, de acuerdo a su
A passados para outros povos.
B substituídos por outras práticas. naturaleza, eran wawakis (invocaciones para enterrar a
C reforçados pelas novas gerações. un infante muerto), hayllis (poesía épica), harawis (poesía
D valorizados pelas tradições locais. amorosa), qhaswas (cantos de regocijo), wankas, entre
E representados por festas populares. otros. Ni siquiera la luminosa personalidad de José María
Arguedas confinó al limbo al poeta Alencastre, de quien
QUESTÃO 95 dijo era el más grande poeta quechua del siglo XX.
GONZÁLEZ, O. Disponível em: www.lenguandina.org. Acesso em: 30 jul. 2012.
Annual Greenhouse Gas Emissions by Sector
Segundo Odi González, embora seja difícil dar
Industrial
16.8% verossimilhança ao universo cultural andino ao escrever
processes
Power stations em línguas indígenas ou em castelhano, nos dias de hoje,
21.3% a poesia quíchua
Transportation fuels A baseia-se na tradição oral.
14.0% Waste disposal B constitui-se de poemas cerimoniais.
and treatment
3.4%
C costuma ter um caráter anônimo.
D possui marcas autorais.
Agricultural
12.5% Land use and E busca uma temática própria.
by-products 10.0%
biomass burning
QUESTÃO 92
Fossil fuel retrieval, 10.3% Residential, commercial,
processing, and distribution 11.3% and other sources Siete crisantemos
Disponível em: www.globalwarming.org. Acesso em: 31 jul. 2012 (adaptado). A las buenas costumbres nunca me he acostumbrado,
del calor de la lumbre del hogar me aburrí.
A emissão de gases tóxicos na atmosfera traz diversas También en el infierno llueve sobre mojado,
consequências para nosso planeta. De acordo com o
lo sé porque he pasado más de una noche allí.
gráfico, retirado do texto Global warming is an international
issue, observa-se que SABINA, J. Esta boca es mía. Madri: Ariola, 1994 (fragmento).

A as queimadas poluem um pouco mais do que os Nessa estrofe da canção Siete crisantemos, do cantor
combustíveis usados nos meios de transporte. espanhol Joaquín Sabina, a expressão “llueve sobre
B as residências e comércios são os menores emissores mojado” faz referência ao(à)
de gases de efeito estufa na atmosfera.
C o processo de tratamento de água contribui para a A constância necessária para viver.
emissão de gases poluentes no planeta. B esperança de uma vida melhor.
D os combustíveis utilizados nos meios de transportes C desprezo pelos bons costumes.
poluem mais do que as indústrias.
D rotina entediante da vida.
E os maiores emissores de gases de efeito estufa na
atmosfera são as usinas elétricas. E rechaço a uma vida confortável.

LC - 2º dia | Caderno 15 - CINZA - Página 4


2015 *CZ1525dom5*
QUESTÃO 93 O texto é uma carta de leitor sobre a reportagem
“¡Desenchúfalo... y a jugar!”, publicada em uma revista.
Las lenguas existen para comunicarse y para Ao relatar sua experiência pessoal, a leitora retoma o
mantener la diversidad cultural de las sociedades. Perder tema da reportagem e confirma a necessidade de
una lengua es perder parte del patrimonio cultural de los
A cercar as crianças da tecnologia disponível e
pueblos, de ahí que un proyecto de colaboración on-line treiná-las a usá-la.
se haya puesto como meta la protección de la diversidad B desconectar as crianças dos aparelhos tecnológicos
lingüística mundial. e brincar com elas.
Según los expertos, en 2100 solo se hablará la mitad C oferecer às crianças uma variedade de brinquedos
de las lenguas que siguen vivas en la actualidad, de ahí la não tecnológicos.
importancia de esta iniciativa. D revezar o tempo que cada um dedica às brincadeiras
En el mundo existen más de 3 000 idiomas en peligro com os filhos.
de extinción, pero la tecnología puede impulsar su E controlar o tempo de que os filhos dispõem para usar
utilización y conservación. Gracias a la digitalización de os aparelhos tecnológicos.
documentos, grabación de vídeos y audio en alta calidad, QUESTÃO 95
y a la capacidad de compartirlos con el resto del mundo se
espera que muchas lenguas que solo hablan o escriben
unas miles de personas no caigan en el olvido y estén
avocadas a la desaparición.
Es el caso de la ya extinguida lengua Miami-Illinois,
que hablaban comunidades de indios americanos en el
actual Medio Oeste de Estados Unidos y cuyos últimos
parlantes murieron en la década de los 1960. Años
más tarde un ciudadano de la tribu Miami de Oklahoma
aprendió la lengua a través de manuscritos y ahora trata
de revitalizar el idioma a base de archivos de audio, Tapar bien los recipientes donde guardarmos el agua para nuestro consumo (1).
relatos. Se trata de solo un ejemplo, pero puede servir Lavar periódicamente las pilas y en caso de almacenar el agua
utilizar bolsa matalarva (2).
como muestra de otros muchos trabajos y del uso de la
Eliminar de nuestro hogar cualquier objeto inservible: botellas, latas o
tecnología y la red con fines lingüísticos. llantas donde se acumula agua (3).
Disponível em: www.muyinteresante.es. Acesso em: 22 jul. 2012 (adaptado). Cambiar el agua del bebedero de los animales diariamente (4).
Limpiar canaletas y evitar cualquier agua estancada (5).
Mais que uma forma de comunicação, o idioma de um Cambiar el agua de los floreros cada tres días (6).
povo é a marca de sua cultura. Nesse sentido, o texto
informa sobre o(a)
A uso da tecnologia como ferramenta para a
conservação de línguas em vias de extinção.
B importância da valorização da língua oral para a
conservação da cultura de um povo.
C forma como a língua Miami-Illinois sobreviveu à
ameaça de extinção.
D evolução natural das línguas, suas adaptações e seu
possível desaparecimento.
E tendência à substituição dos meios de comunicação
tradicionais por ferramentas digitais.
QUESTÃO 94
Disponível em: www.mspas.gob.sv. Acesso em: 14 dez. 2009.
Soy madre de un pequeño de 3 años y a partir del
artículo ”Desenchúfalo… ¡y a jugar!”, me puse a pensar Os programas de prevenção à dengue não estão restritos
a cidades brasileiras. No material elaborado sobre esse
en el tiempo que le dedico a mi hijo. Todos los días,
tema pelo Ministério da Saúde de El Salvador, país da
cuando llego a mi casa, mi prioridad es mi hijo y nos América Central, objetiva-se
turnamos con mi marido para ver quién cocina y quién se
tira en el piso a jugar con Santiago. Nuestro hijo tiene toda A apresentar a sequência de ações necessárias à
tecnología a su disposición, porque su papá es técnico prevenção da doença.
B instruir o leitor sobre como impedir a formação de
en sistemas, pero cuando llegamos a casa después de
criadouros do mosquito.
un agotador día laboral, nos desenchufamos los tres y C descrever como se dá a proliferação do Aedes aegypti
usamos cualquier cosa que tengamos a mano: una pelota em El Salvador.
o una sábana para divertirnos. Esa pequeña terapia de D convencer o leitor sobre a necessidade do tratamento
risa es altamente curativa contra los bajones anímicos, da doença.
contra el estrés, contra los pequeños enojos cotidianos, E relatar experiências sobre como lidar com a
contra todo. OVIEDO, P. Sophia, n. 130, ago. 2012 (adaptado). multiplicação do Aedes aegypti.

LC - 2º dia | Caderno 15 - CINZA - Página 5


*CZ1525dom6* 2015

A obra do artista plástico Leonilson (1953-1993) marca


Questões de 96 a 135 presença no panorama da arte brasileira e internacional.
Nessa obra, ele utilizou a habilidade técnica do bordado
QUESTÃO 96
manual para
Da timidez
A obtenção das linhas retas paralelas.
Ser um tímido notório é uma contradição. O tímido tem
horror a ser notado, quanto mais a ser notório. Se ficou B valorização do tracejado retilíneo.
notório por ser tímido, então tem que se explicar. Afinal, C exploração de diferentes texturas.
que retumbante timidez é essa, que atrai tanta atenção?
Se ficou notório apesar de ser tímido, talvez estivesse se D obtenção do equilíbrio assimétrico.
enganando junto com os outros e sua timidez seja apenas E inscrição homogênea das formas e palavras.
um estratagema para ser notado. Tão secreto que nem
ele sabe. É como no paradoxo psicanalítico, só alguém QUESTÃO 98
que se acha muito superior procura o analista para tratar E: Diva ... tem algumas ... alguma experiência pessoal
um complexo de inferioridade, porque só ele acha que se
sentir inferior é doença. que você passou e que você poderia me contar ... alguma
[...] coisa que marcou você? Uma experiência ... você poderia
O tímido tenta se convencer de que só tem problemas contar agora …
com multidões, mas isto não é vantagem. Para o tímido, I: É ... tem uma que eu vivi quando eu estudava o
duas pessoas são uma multidão. Quando não consegue terceiro ano científico lá no Atheneu ... né ... é:: eu gostava
escapar e se vê diante de uma plateia, o tímido não pensa
muito do laboratório de química ... eu ... eu ia ajudar os
nos membros da plateia como indivíduos. Multiplica-os
por quatro, pois cada indivíduo tem dois olhos e dois professores a limpar aquele material todo ... aqueles vidros
ouvidos. Quatro vias, portanto, para receber suas gafes. ... eu achava aquilo fantástico ... aquele monte de coisa ...
Não adianta pedir para a plateia fechar os olhos, ou tapar né ... então ... todos os dias eu ia ... quando terminavam
um olho e um ouvido para cortar o desconforto do tímido as aulas eu ajudava o professor a limpar o laboratório ...
pela metade. Nada adianta. O tímido, em suma, é uma nesse dia não houve aula e o professor me chamou pra
pessoa convencida de que é o centro do Universo, e que fazer uma limpeza geral no laboratório ... chegando lá
seu vexame ainda será lembrado quando as estrelas ... ele me fez uma experiência ... ele me mostrou uma
virarem pó.
VERISSIMO, L. F. Comédias para se ler na escola. Rio de Janeiro: Objetiva, 2001.
coisa bem interessante que ... pegou um béquer com
meio d’água e colocou um pouquinho de cloreto de sódio
Entre as estratégias de progressão textual presentes pastoso ... então foi aquele fogaréu desfilando ... aquele
nesse trecho, identifica-se o emprego de elementos fogaréu ... quando o professor saiu ... eu chamei umas
conectores. Os elementos que evidenciam noções
semelhantes estão destacados em: duas colegas minhas pra mostrar a experiência que eu
tinha achado fantástico ... só que ... eu achei o seguinte ...
A “Se ficou notório por ser tímido” e “[...] então tem que se o professor colocou um pouquinho ... foi aquele desfile
se explicar”. ... imagine se eu colocasse mais ... peguei o mesmo
B “[...] então tem que se explicar” e “[...] quando as béquer ... coloquei uma colher ... uma colher de cloreto
estrelas virarem pó”. de sódio ... foi um fogaréu tão grande ... foi uma explosão
C “[...] ficou notório apesar de ser tímido [...]” e “[...] mas ... quebrou todo o material que estava exposto em cima
isto não é vantagem [...]”. da mesa ... eu branca ... eu fiquei ... olha ... eu pensei que
D “[...] um estratagema para ser notado [...]” e “Tão eu fosse morrer sabe ... quando ... o colégio inteiro correu
secreto que nem ele sabe”. pro laboratório pra ver o que tinha sido ...
E “[...] como no paradoxo psicanalítico [...]” e “[...] CUNHA, M. A. F. (Org.) . Corpus discurso & gramática: a língua falada e
porque só ele acha [...]”. escrita na cidade de Natal. Natal: EdUFRN, 1998.

QUESTÃO 97 Na transcrição de fala, especialmente, no trecho “eu


branca ... eu fiquei ... olha ... eu pensei que eu fosse
morrer sabe...”, há uma estrutura sintática fragmentada,
embora facilmente interpretável. Sua presença na fala
revela
A distração e poucos anos de escolaridade.
B falta de coesão e coerência na apresentação das
ideias.
C afeto e amizade entre os participantes da conversação.
D desconhecimento das regras de sintaxe da norma
padrão.
E característica do planejamento e execução simultânea
LEONILSON. O recruta, o aranha e o penélope. Bordado sobre tecido, 1992. desse discurso.
Disponível em: www.projetoleonilson.com.br. Acesso em: 3 ago. 2012.

LC - 2º dia | Caderno 15 - CINZA - Página 6


2015 *CZ1525dom7*
QUESTÃO 99 que veem em certas ervas da floresta o enigma das
doenças mais temíveis, com as infusões de coloração
O mundo das grandes inovações tecnológicas, dos
sanguínea aconselhadas para aliviar trinta e seis dores do
avanços das pesquisas médicas e que já presenciou o
corpo humano. “E existem ervas que não curam nada”,
envio de homens ao espaço é o mesmo lugar onde 1 bilhão
revelava a lavadeira, “mas assanham a mente da gente.
de pessoas dormem e acordam com fome. A desnutrição
Basta tomar um gole do líquido fervendo para que o
ocupa o primeiro lugar no ranking dos 10 maiores riscos à
cristão sonhe uma única noite muitas vidas diferentes”.
saúde e mata mais do que a aids, a malária e a tuberculose
Esse relato poderia ser de duvidosa veracidade para
combinadas. O equivalente às populações da Europa e
outras pessoas, mas não para Emilie.
da América do Norte, juntas, está de barriga vazia. E um
HATOUM, M. São Paulo: Cia. das Letras, 2008.
futuro famélico aguarda a raça humana. Em 2050, apenas
por razões ligadas às mudanças climáticas, o número de As representações da Amazônia na literatura brasileira
pessoas sem comida no prato vai aumentar em até 20%. mantêm relação com o papel atribuído à região na
construção do imaginário nacional. Pertencentes a
Disponível em: www.correiobraziliense.com.br. Acesso em: 22 jan. 2012.
contextos históricos distintos, os fragmentos diferenciam-se
Considerando a natureza do tema, a forma como está ao propor uma representação da realidade amazônica em
apresentado e o meio pelo qual é veiculado o texto, que se evidenciam
percebe-se que seu principal objetivo é
A aspectos da produção econômica e da cura na
A divulgar dados estatísticos recentes sobre a fome no tradição popular.
mundo e sobre as inovações tecnológicas. B manifestações culturais autênticas e da resignação
B esclarecer questões científicas acerca dos danos familiar.
causados pela fome e pela aids nos indivíduos. C valores sociais autóctones e influência dos
C demonstrar que a fome, juntamente com as doenças estrangeiros.
endêmicas, também é um problema de saúde pública. D formas de resistência locais e do cultivo das
D convidar o leitor a engajar-se em alguma ação superstições.
positiva contra a fome, a partir da divulgação de E costumes domésticos e levantamento das tradições
dados alarmantes. indígenas.
E alertar sobre o problema da fome, apresentando-o
como um contraste no mundo de tantos recursos QUESTÃO 101
tecnológicos.

QUESTÃO 100

TEXTO I
Voluntário
Rosa tecia redes, e os produtos de sua pequena
indústria gozavam de boa fama nos arredores. A reputação
da tapuia crescera com a feitura de uma maqueira de
tucum ornamentada com a coroa brasileira, obra de
ingênuo gosto, que lhe valera a admiração de toda a
comarca e provocara a inveja da célebre Ana Raimunda,
de Óbidos, a qual chegara a formar uma fortunazinha com
aquela especialidade, quando a indústria norte-americana CABRAL, I. Disponível em: www.ivancabral.com. Acesso em: 30 jul. 2012.
reduzira à inatividade os teares rotineiros do Amazonas.
A palavra inglesa “involution” traduz-se como involução
SOUSA, I. Contos amazônicos. São Paulo: Martins Fontes, 2004.
ou regressão. A construção da imagem com base
TEXTO II na combinação do verbal com o não verbal revela a
Relato de um certo oriente intenção de

Emilie, ao contrário de meu pai, de Dorner e dos A denunciar o retrocesso da humanidade.


nossos vizinhos, não tinha vivido no interior do Amazonas. B criticar o consumo de bebida alcoólica pelos humanos.
Ela, como eu, jamais atravessara o rio. Manaus era o seu C satirizar a caracterização dos humanos como
mundo visível. O outro latejava na sua memória. Imantada primatas.
por uma voz melodiosa, quase encantada, Emilie D elogiar a teoria da evolução humana pela seleção
maravilha-se com a descrição da trepadeira que espanta natural.
a inveja, das folhas malhadas de um tajá que reproduz
E fazer um trocadilho com as palavras inovação e
a fortuna de um homem, das receitas de curandeiros
involução.

LC - 2º dia | Caderno 15 - CINZA - Página 7


*CZ1525dom8* 2015

QUESTÃO 102 O modo como o filho qualifica os presentes é


incompreendido pela mãe, e essas escolhas lexicais
O peru de Natal
revelam diferenças entre os interlocutores, que estão
O nosso primeiro Natal de família, depois da relacionadas
morte de meu pai acontecida cinco meses antes, foi A à linguagem infantilizada.
de consequências decisivas para a felicidade familiar. B ao grau de escolaridade.
Nós sempre fôramos familiarmente felizes, nesse C à dicotomia de gêneros.
sentido muito abstrato da felicidade: gente honesta, sem
D às especificidades de cada faixa etária.
crimes, lar sem brigas internas nem graves dificuldades
E à quebra de regras da hierarquia familiar.
econômicas. Mas, devido principalmente à natureza
cinzenta de meu pai, ser desprovido de qualquer lirismo, QUESTÃO 104
duma exemplaridade incapaz, acolchoado no medíocre, TEXTO I
sempre nos faltara aquele aproveitamento da vida,
Versos de amor
aquele gosto pelas felicidades materiais, um vinho bom,
uma estação de águas, aquisição de geladeira, coisas A um poeta erótico
assim. Meu pai fora de um bom errado, quase dramático,
Oposto ideal ao meu ideal conservas.
o puro-sangue dos desmancha-prazeres.
Diverso é, pois, o ponto outro de vista
ANDRADE, M. In: MORICONI, I. Os cem melhores contos brasileiros do século.
São Paulo: Objetiva, 2000 (fragmento).
Consoante o qual, observo o amor, do egoísta
Modo de ver, consoante o qual, o observas.
No fragmento do conto de Mário de Andrade, o tom
confessional do narrador em primeira pessoa revela uma Porque o amor, tal como eu o estou amando,
concepção das relações humanas marcada por É Espírito, é éter, é substância fluida,
É assim como o ar que a gente pega e cuida,
A distanciamento de estados de espírito acentuado pelo
papel das gerações. Cuida, entretanto, não o estar pegando!

B relevância dos festejos religiosos em família na É a transubstanciação de instintos rudes,


sociedade moderna. Imponderabilíssima, e impalpável,
C preocupação econômica em uma sociedade urbana Que anda acima da carne miserável
em crise. Como anda a garça acima dos açudes!
D consumo de bens materiais por parte de jovens, ANJOS, A. Obra completa. Rio de Janeiro: Nova Aguilar, 1996 (fragmento).

adultos e idosos. TEXTO II


E pesar e reação de luto diante da morte de um familiar Arte de amar
querido.
Se queres sentir a felicidade de amar, esquece a tua
QUESTÃO 103 alma.

— Não, mãe. Perde a graça. Este ano, a senhora vai A alma é que estraga o amor.
ver. Compro um barato. Só em Deus ela pode encontrar satisfação.

— Barato? Admito que você compre uma lembrancinha Não noutra alma.
barata, mas não diga isso a sua mãe. É fazer pouco-caso Só em Deus — ou fora do mundo.
de mim. As almas são incomunicáveis.
— Ih, mãe, a senhora está por fora mil anos. Não Deixa o teu corpo entender-se com outro corpo.
sabe que barato é o melhor que tem, é um barato!
Porque os corpos se entendem, mas as almas não.
— Deixe eu escolher, deixe... BANDEIRA, M. Estrela da vida inteira. Rio de Janeiro: Nova Fronteira, 1993.

— Mãe é ruim de escolha. Olha aquele blazer furado Os Textos I e II apresentam diferentes pontos de vista
sobre o tema amor. Apesar disso, ambos definem esse
que a senhora me deu no Natal!
sentimento a partir da oposição entre
— Seu porcaria, tem coragem de dizer que sua mãe A satisfação e insatisfação.
lhe deu um blazer furado? B egoísmo e generosidade.
— Viu? Não sabe nem o que é furado? Aquela cor já C felicidade e sofrimento.
era, mãe, já era! D corpo e espírito.
ANDRADE, C. D. Poesia e prosa. Rio de Janeiro: Nova Aguilar, 1998. E ideal e real.

LC - 2º dia | Caderno 15 - CINZA - Página 8


2015 *CZ1525dom9*
QUESTÃO 105 Considerando o texto em suas cinco partes, constata-se
Conecte-se que há o emprego de argumento de autoridade no trecho:
Estabeleça relações com A “Seja curioso, saboreie os momentos da vida e tome
as pessoas a sua volta. Os
relacionamentos são a base
consciência de como se sente. Refletir sobre suas
da vida diária e investir tempo neles experiências ajuda a descobrir o que realmente
enriquecerá seu dia e garantirá apoio importa”.
quando precisar. As pesquisas mostram
B “As pesquisas mostram que quem tem menos de três
que quem tem menos de três pessoas
em sua rede de contatos próxima — entre pessoas em sua rede de contatos próxima [...] tem
família e amigos — tem mais chance de mais chances de desenvolver uma doença mental.”
desenvolver uma doença mental. C “Caminhe ou corra, ande de bicicleta, pratique um
esporte, dance. Os exercícios fazem as pessoas se
Seja ativo sentirem bem”.
Caminhe ou corra, ande de D “Tente algo novo, matricule-se em um curso [...]
bicicleta, pratique um esporte,
Escolha um desafio que você vai gostar de perseguir.”
dance. Os exercícios fazem as
pessoas se sentirem bem — o importante é E “Fazer parte de uma comunidade traz benefícios —
cada pessoa achar a atividade que lhe dá entre eles relações sociais mais significativas.”
prazer e que é adequada a seus limites.
Estudos de longo prazo sugerem que a
prática de uma atividade física previne
QUESTÃO 106
o declínio das capacidades mentais e
protege contra a ansiedade e a depressão.
Não adianta isolar o fumante
Se quiser mesmo combater o fumo, o governo precisa
ir além das restrições. É preciso apoiar quem quer largar
Preste atenção o cigarro.
Seja curioso, saboreie os
momentos da vida e tome Ao apoiar uma medida provisória para combater o
consciência de como se sente.
Refletir sobre suas experiências ajuda
fumo em locais públicos nos 27 estados brasileiros, o
a descobrir o que realmente importa e Senado reafirmou um valor fundamental: a defesa da
garantir que você viva o presente. Uma saúde e da vida.
pesquisa mostrou que pessoas treinadas
a prestar atenção em seus sentimentos Em pelo menos um aspecto a MP 540/2011 é ainda
durante oito a 12 semanas apresentaram mais rigorosa que as medidas em vigor em São Paulo,
melhora no bem-estar por anos. no Rio de Janeiro e no Paraná, estados que até agora
adotaram as legislações mais duras contra o tabagismo.
Ela proíbe os fumódromos em 100% dos locais fechados,
Continue aprendendo incluindo até tabacarias, onde o fumo era autorizado sob
Tente algo novo, matricule-
se em um curso, faça uma determinadas condições.
nova tarefa no trabalho. Tente
consertar algo em casa. Aprenda a tocar Uma das principais medidas atinge o fumante no
um instrumento ou a cozinhar. Escolha um bolso. O governo fica autorizado a fixar um novo preço
desafio que você vai gostar de perseguir. para o maço de cigarros. O Imposto sobre Produtos
Os estudos sugerem que o bem-estar está Industrializados (IPI) será elevado em 300%. Somando
ligado a ter metas — desde que elas sejam uma coisa e outra, o sabor de fumar se tornará muito mais
estabelecidas pelos próprios indivíduos e
tenham a ver com seus valores pessoais. ácido. Deverá subir 20% em 2012 e 55% em 2013.
A visão fundamental da MP está correta. Sabe-se, há
muito, que o tabaco faz mal à saúde. É razoável, portanto,
Doe-se que o Estado aja em nome da saúde pública.
Agradeça a alguém, ajude um
amigo ou um estranho. Sorria, Época, 28 nov. 2011 (adaptado).
faça trabalho voluntário, junte-
se à associação do bairro. Olhe para fora, O autor do texto analisa a aprovação da MP 540/2011
além de olhar para dentro de si. Fazer parte pelo Senado, deixando clara a sua opinião sobre o tema.
de uma comunidade traz benefícios — entre O trecho que apresenta uma avaliação pessoal do autor
eles relações sociais mais significativas.
como uma estratégia de persuasão do leitor é:
As pesquisas mostram que as pessoas
que têm um interesse maior pelo outro A “Ela proíbe os fumódromos em 100% dos locais
tendem a se considerar mais felizes.
fechados”.
B “O governo fica autorizado a fixar um novo preço para
o maço de cigarros.’’
Disponível em: www.revistaepoca.globo.com. Acesso em: 27 fev. 2012.
C “O Imposto sobre Produtos Industrializados (IPI) será
Ao interagirmos socialmente, é comum deixarmos elevado em 300%.”
claro nosso posicionamento a respeito do assunto D “Somando uma coisa e outra, o sabor de fumar se
discutido. Para isso, muitas vezes, recorremos a tornará muito mais ácido.”
determinadas estratégias argumentativas, dentre as
quais se encontra o argumento de autoridade. E “Deverá subir 20% em 2012 e 55% em 2013.”

LC - 2º dia | Caderno 15 - CINZA - Página 9


*CZ1525dom10* 2015

QUESTÃO 107 aceitável, por exemplo, aquela das românticas figuras


indígenas? A Carioca oferecia um corpo simultaneamente
ideal e obsceno: o alto — uma beleza imaterial — e o
baixo — uma carnalidade excessiva. Sugeria uma mistura
de estilos que, sem romper com a regra do decoro
artístico, insinuava na tela algo inadequado ao repertório
simbólico oficial. A exótica morena, que não é índia — nem
mulata ou negra — poderia representar uma visualidade
feminina brasileira e desfrutar de um lugar de destaque no
imaginário da nossa “monarquia tropical”?
OLIVEIRA, C. Disponível em: http://anpuh.org.br. Acesso em: 20 maio 2015.

O texto revela que a aceitação da representação do belo


na obra de arte está condicionada à
A incorporação de grandes correntes teóricas de uma
época, conferindo legitimidade ao trabalho do artista.
B atemporalidade do tema abordado pelo artista,
garantindo perenidade ao objeto de arte então
elaborado.
C inserção da produção artística em um projeto estético
e ideológico determinado por fatores externos.
D apropriação que o pintor faz dos grandes temas
universais já recorrentes em uma vertente artística.
E assimilação de técnicas e recursos já utilizados por
movimentos anteriores que trataram da temática.

QUESTÃO 109
Como estamos na “Era Digital”, foi necessário rever os
velhos ditados existentes e adaptá-los à nova realidade.
Disponível em: http://newsgerais.blogspot.com.br. Acesso em: 1 ago. 2012. Veja abaixo...
Esse texto trata de uma campanha sobre o trânsito e visa 1. A pressa é inimiga da conexão.
a orientação dos motociclistas quanto ao(à) 2. Amigos, amigos, senhas à parte.
A intolerância com a morosidade do tráfego. 3. Para bom provedor uma senha basta.
B desconhecimento da legislação. 4. Não adianta chorar sobre arquivo deletado.
C crescente número de motocicletas.
5. Mais vale um arquivo no HD do que dois baixando.
D manutenção preventiva do veículo.
E cuidado com a própria segurança. 6. Quem clica seus males multiplica.
7. Quem semeia e-mails, colhe spams.
QUESTÃO 108
8. Os fins justificam os e-mails.
Em 1866, tendo encerrado seus estudos na Escola
Disponível em: www.abusar.org.br. Acesso em: 20 maio 2015 (adaptado).
de Belas Artes, em Paris, Pedro Américo ofereceu a tela
A Carioca ao imperador Pedro II, em reconhecimento ao No texto, há uma reinterpretação de ditados populares
seu mecenas. O nu feminino obedecia aos cânones da com o uso de termos da informática. Essa reinterpretação
grande arte e pretendia ser uma alegoria feminina da A torna o texto apropriado para profissionais da
nacionalidade. A tela, entretanto, foi recusada por imoral e informática.
licenciosa: mesmo não fugindo à regra oitocentista relativa
B atribui ao texto um caráter humorístico.
à nudez na obra de arte, A Carioca não pôde, portanto,
ser absorvida de imediato. A sensualidade tangível da C restringe o acesso ao texto por público não
figura feminina, próxima do orientalismo tão em voga especializado.
na Europa, confrontou-se não somente com os limites D deixa a terminologia original mais acessível ao
morais, mas também com a orientação estética e cultural público em geral.
do Império. O que chocara mais: a nudez frontal ou um nu E dificulta a compreensão do texto por quem não
tão descolado do que se desejava como nudez nacional domina a língua inglesa.

LC - 2º dia | Caderno 15 - CINZA - Página 10


2015 *CZ1525dom11*
QUESTÃO 110 Nesse sentido, Machado de Assis, que foi o primeiro
Famigerado presidente desde a sua inauguração até a data de sua
morte, em 1908, imaginava que a nossa Academia deveria
Com arranco, [o sertanejo] calou-se. Como arrependido ser uma academia de Letras, portanto, de literatos.
de ter começado assim, de evidente. Contra que aí estava BECHARA, E. Disponível em: www.academiagalega.org. Acesso em: 31 jul. 2012.
com o fígado em más margens; pensava, pensava.
Cabismeditado. Do que, se resolveu. Levantou as feições. No trecho da palestra proferida por Evanildo Bechara, na
Se é que se riu: aquela crueldade de dentes. Encarar, não Academia Galega da Língua Portuguesa, verifica-se o
me encarava, só se fito à meia esguelha. Latejava-lhe um uso de estruturas gramaticais típicas da norma padrão da
orgulho indeciso. Redigiu seu monologar. língua. Esse uso
O que frouxo falava: de outras, diversas pessoas A torna a fala inacessível aos não especialistas no
e coisas, da Serra, do São Ão, travados assuntos, assunto abordado.
insequentes, como dificultação. A conversa era para B contribui para a clareza e a organização da fala no
teias de aranha. Eu tinha de entender-lhe as mínimas nível de formalidade esperado para a situação.
entonações, seguir seus propósitos e silêncios. Assim no
fechar-se com o jogo, sonso, no me iludir, ele enigmava. C atribui à palestra características linguísticas restritas
E, pá: à modalidade escrita da língua portuguesa.
D dificulta a compreensão do auditório para preservar o
— Vosmecê agora me faça a boa obra de querer
caráter rebuscado da fala.
me ensinar o que é mesmo que é: fasmisgerado...
faz-me-gerado... falmisgeraldo... familhas-gerado...? E evidencia distanciamento entre o palestrante e o
ROSA, J. G. Primeiras estórias. Rio de Janeiro: Nova Fronteira, 1988.
auditório para atender os objetivos do gênero palestra.

A linguagem peculiar é um dos aspectos que conferem QUESTÃO 112


a Guimarães Rosa um lugar de destaque na literatura
brasileira. No fragmento lido, a tensão entre a personagem Mudança linguística
e o narrador se estabelece porque Ataliba de Castilho, professor de língua portuguesa
A o narrador se cala, pensa e monologa, tentando assim da USP, explica que o internetês é parte da metamorfose
evitar a perigosa pergunta de seu interlocutor. natural da língua.
B o sertanejo emprega um discurso cifrado, com — Com a internet, a linguagem segue o caminho
enigmas, como se vê em “a conversa era para teias dos fenômenos da mudança, como o que ocorreu com
de aranha”. “você”, que se tornou o pronome átono “cê”. Agora,
C entre os dois homens cria-se uma comunicação o interneteiro pode ajudar a reduzir os excessos da
impossível, decorrente de suas diferenças ortografia, e bem sabemos que são muitos. Por que o
socioculturais. acento gráfico é tão importante assim para a escrita?
D a fala do sertanejo é interrompida pelo gesto de Já tivemos no Brasil momentos até mais exacerbados por
impaciência do narrador, decidido a mudar o assunto acentos e dispensamos muitos deles. Como toda palavra
da conversa. é contextualizada pelo falante, podemos dispensar ainda
E a palavra desconhecida adquire o poder de gerar muitos outros. O interneteiro mostra um caminho, pois faz
conflito e separar as personagens em planos um casamento curioso entre oralidade e escrituralidade.
incomunicáveis. O internetês pode, no futuro, até tornar a comunicação
mais eficiente. Ou evoluir para um jargão complexo, que,
QUESTÃO 111 em vez de aproximar as pessoas em menor tempo, estimule
Em primeiro lugar gostaria de manifestar os meus o isolamento dos iniciados e a exclusão dos leigos.
agradecimentos pela honra de vir outra vez à Galiza e Para Castilho, no entanto, não será uma reforma
conversar não só com os antigos colegas, alguns dos ortográfica que fará a mudança de que precisamos na
quais fazem parte da mesa, mas também com novos língua. Será a internet. O jeito eh tc e esperar pra ver?
colegas, que pertencem à nova geração, em cujas mãos,
Disponível em: http://revistalingua.com.br. Acesso em: 3 jun. 2015 (adaptado).
com toda certeza, está também o destino do Galego na
Galiza, e principalmente o destino do Galego incorporado Na entrevista, o fragmento “O jeito eh tc e esperar pra
à grande família lusófona. ver?” tem por objetivo
E, portanto, é com muito prazer que teço algumas A ilustrar a linguagem de usuários da internet que
considerações sobre o tema apresentado. Escolhi como poderá promover alterações de grafias.
tema como os fundadores da Academia Brasileira de
B mostrar os perigos da linguagem da internet como
Letras viam a língua portuguesa no seu tempo. Como
sabem, a nossa Academia, fundada em 1897, está agora potencializadora de dificuldades de escrita.
completando 110 anos, foi organizada por uma reunião de C evidenciar uma forma de exclusão social para as
jornalistas, literatos, poetas que se reuniam na secretaria pessoas com baixa proficiência escrita.
da Revista Brasileira, dirigida por um crítico literário e por D explicar que se trata de um erro linguístico por destoar
um literato chamado José Veríssimo, natural do Pará, e do padrão formal apresentado ao longo do texto.
desse entusiasmo saiu a ideia de se criar a Academia
Brasileira, depois anexada ao seu título: Academia E exemplificar dificuldades de escrita dos interneteiros
Brasileira de Letras. que desconhecem as estruturas da norma padrão.

LC - 2º dia | Caderno 15 - CINZA - Página 11


*CZ1525dom12* 2015

QUESTÃO 113 Com o advento das novas tecnologias, a sociedade tem


vivenciado mudanças de paradigmas em vários setores.
Nesse sentido, o telecommuting traz novidades para o
mundo do trabalho porque proporciona prioritariamente o(a)
A aumento da produtividade do empregado.
B equilíbrio entre vida pessoal e profissional do
trabalhador.
C fortalecimento da relação entre empregador e
empregado.
D participação do profissional nas decisões da
organização.
E maleabilidade dos locais de atuação do profissional
da empresa.

QUESTÃO 115

Perder a tramontana

A expressão ideal para falar de desorientados e outras


palavras de perder a cabeça
É perder o norte, desorientar-se. Ao pé da letra,
“perder a tramontana” significa deixar de ver a estrela
polar, em italiano stella tramontana, situada do outro lado
dos montes, que guiava os marinheiros antigos em suas
Disponível em: http://fsindical-rs.org.br. Acesso em: 16 ago. 2012 (adaptado).
viagens desbravadoras.
Nesse texto, associam-se recursos verbais e não verbais
na busca de mudar o comportamento das pessoas Deixar de ver a tramontana era sinônimo de
quanto a uma questão de saúde pública. No cartaz, essa desorientação. Sim, porque, para eles, valia mais o céu
associação é ressaltada no(a) estrelado que a terra. O Sul era região desconhecida,
A destaque dado ao laço, símbolo do combate à aids, imprevista; já o Norte tinha como referência no firmamento
seguido da frase “Use camisinha”. um ponto luminoso conhecido como a estrela Polar, uma
B centralização da mensagem “Previna-se”. espécie de farol para os navegantes do Mediterrâneo,
C foco dado ao objeto camisinha em imagem e em sobretudo os genoveses e os venezianos. Na linguagem
palavra. deles, ela ficava transmontes, para além dos montes, os
D laço como elemento de ligação entre duas Alpes. Perdê-la de vista era perder a tramontana, perder
recomendações.
o Norte.
E sobreposição da imagem da camisinha e da boia,
relacionada à frase “Salve vidas”. No mundo de hoje, sujeito a tantas pressões, muita
gente não resiste a elas e entra em parafuso. Além de
QUESTÃO 114
perder as estribeiras, perde a tramontana...
Telecommuting redefine o tradicional entendimento
sobre o espaço de trabalho. Atualmente, as organizações COTRIM, M. Língua Portuguesa, n. 15, jan. 2007.

estão se focando em novos valores, tais como, inovações, Nesse texto, o autor remonta às origens da expressão
satisfação, responsabilidades, resultados e ambiente
de trabalho familiar. A alternativa do telecommuting “perder a tramontana”. Ao tratar do significado dessa
complementa esses princípios e oferece flexibilidade expressão, utilizando a função referencial da linguagem,
aos patrões e empregados. É um conceito novo que, a o autor busca
cada dia, ganha mais força ao redor do mundo. Grandes
empresas escolheram o trabalho de telecommuting pelas A apresentar seus indícios subjetivos.
facilidades que ele gera para o empregador. A implantação B convencer o leitor a utilizá-la.
do telecommuting determina regras para se trabalhar
em casa em dias específicos da semana e, nos demais C expor dados reais de seu emprego.
dias, trabalhar no escritório. O local de trabalho pode D explorar sua dimensão estética.
ser a casa ou, temporariamente, por motivo de viagem,
outros escritórios. E criticar sua origem conceitual.
FERREIRA JR., J.C. Disponível em: www.ccuec.unicamp.br. Acesso em: 1 ago. 2012 (adaptado).

LC - 2º dia | Caderno 15 - CINZA - Página 12


2015 *CZ1525dom13*
QUESTÃO 116 brinquedos e suas ferramentas de aprendizado. Daqui
a 50 anos, talvez concluamos que não houve nenhuma
Ai se sêsse
crise educacional no mundo — apenas ocorreu uma
Se um dia nois se gostasse incongruência crescente entre a maneira como as escolas
Se um dia nois se queresse do século XX ensinavam e a maneira como as crianças
Se nois dois se empareasse do fim do século XX aprendiam.
Se juntim nois dois vivesse DRUCKER, P. O melhor de Peter Drucker: obra completa. São Paulo: Nobel, 2002.

Se juntim nois dois morasse O artigo apresenta uma reflexão sobre a Revolução da
Se juntim nois dois drumisse Informação, que, assim como a Revolução Industrial,
Se juntim nois dois morresse provocou impactos significativos nas sociedades
Se pro céu nois assubisse contemporâneas. Ao tratar da Revolução da Informação,
o autor enfatiza que
Mas porém se acontecesse
De São Pedro não abrisse A o comércio eletrônico é um dos canais mais
A porta do céu e fosse importantes dessa revolução.
Te dizer qualquer tulice B o computador desenvolve na criança uma inteligência
E se eu me arriminasse maior que a dos pais.
E tu cum eu insistisse C o aumento no número de empregos via internet é uma
Pra que eu me arresolvesse realidade atualmente.
E a minha faca puxasse D o colapso educacional é fruto de uma incongruência
E o bucho do céu furasse no ensino do século XX.
Tarvês que nois dois ficasse E o advento da Revolução da Informação causará
impactos nos próximos 50 anos.
Tarvês que nois dois caísse
E o céu furado arriasse QUESTÃO 118
E as virgi toda fugisse
Quem não se recorda de Aurélia Camargo, que
ZÉ DA LUZ. Cordel do Fogo Encantado. Recife: Álbum de estúdio, 2001.
atravessou o firmamento da corte como brilhante meteoro,
O poema foi construído com formas do português não e apagou-se de repente no meio do deslumbramento que
padrão, tais como “juntim”, “nois”, “tarvês”. Essas formas produzira seu fulgor? Tinha ela dezoito anos quando
legitimam-se na construção do texto, pois apareceu a primeira vez na sociedade. Não a conheciam;
A revelam o bom humor do eu lírico do poema. e logo buscaram todos com avidez informações acerca
da grande novidade do dia. Dizia-se muita coisa que não
B estão presentes na língua e na identidade popular. repetirei agora, pois a seu tempo saberemos a verdade,
C revelam as escolhas de um poeta não escolarizado. sem os comentos malévolos de que usam vesti-la os
D tornam a leitura fácil de entender para a maioria dos noveleiros. Aurélia era órfã; tinha em sua companhia
brasileiros. uma velha parenta, viúva, D. Firmina Mascarenhas, que
E compõem um conjunto de estruturas linguísticas sempre a acompanhava na sociedade. Mas essa parenta
inovadoras. não passava de mãe de encomenda, para condescender
com os escrúpulos da sociedade brasileira, que naquele
QUESTÃO 117 tempo não tinha admitido ainda certa emancipação
feminina. Guardando com a viúva as deferências devidas
Além da Revolução da Informação à idade, a moça não declinava um instante do firme
O impacto da Revolução da Informação está propósito de governar sua casa e dirigir suas ações como
apenas começando. Mas a força motriz desse impacto entendesse. Constava também que Aurélia tinha um
não é a informática, a inteligência artificial, o efeito tutor; mas essa entidade era desconhecida, a julgar pelo
caráter da pupila, não devia exercer maior influência em
dos computadores sobre a tomada de decisões ou
sua vontade, do que a velha parenta.
a elaboração de políticas ou de estratégias. É algo
ALENCAR, J. Senhora. São Paulo: Ática, 2006.
que praticamente ninguém previu, nem mesmo se
falava há 10 ou 15 anos: o comércio eletrônico — o O romance Senhora, de José de Alencar, foi publicado em
aparecimento explosivo da internet como um canal 1875. No fragmento transcrito, a presença de D. Firmina
importante, talvez principal, de distribuição mundial Mascarenhas como “parenta” de Aurélia Camargo assimila
de produtos, serviços e, surpreendentemente, de práticas e convenções sociais inseridas no contexto do
empregos de nível gerencial. Essa nova realidade está Romantismo, pois
modificando profundamente economias, mercados e A o trabalho ficcional do narrador desvaloriza a mulher
estruturas setoriais, os produtos e serviços e seu fluxo, ao retratar a condição feminina na sociedade brasileira
a segmentação, os valores e o comportamento dos da época.
consumidores, o mercado de trabalho. B o trabalho ficcional do narrador mascara os hábitos
O impacto, porém, pode ser ainda maior nas sociais no enredo de seu romance.
sociedades e nas políticas empresariais e, acima de tudo, C as características da sociedade em que Aurélia
na maneira como encaramos o mundo e nós mesmos vivia são remodeladas na imaginação do narrador
dentro dele. O impacto psicológico da Revolução da romântico.
Informação, como o da Revolução Industrial, foi enorme. D o narrador evidencia o cerceamento sexista à
Talvez tenha sido mais forte na maneira como as crianças autoridade da mulher, financeiramente independente.
aprendem. Já aos 4 anos (e às vezes até antes), as E o narrador incorporou em sua ficção hábitos muito
crianças desenvolvem habilidades de computação, logo avançados para a sociedade daquele período
ultrapassando seus pais. Os computadores são seus histórico.
LC - 2º dia | Caderno 15 - CINZA - Página 13
*CZ1525dom14* 2015

QUESTÃO 119

Disponível em: www.istoe.com.br. Acesso em: 5 dez. 2012.

Esse infográfico resume as conclusões de diversas pesquisas científicas sobre a adolescência. Tais conclusões
A desconstroem os estereótipos a respeito dos adolescentes.
B estabelecem novos limites de duração para essa fase da vida.
C reiteram a ideia da adolescência como um período conturbado.
D confirmam a proximidade entre os universos adolescente e adulto.
E apontam a insegurança como uma característica típica dos adolescentes.
QUESTÃO 120
Síntese entre erudito e popular
Na região mineira, a separação entre cultura popular (as artes mecânicas) e erudita (as artes liberais) é marcada
pela elite colonial, que tem como exemplo os valores europeus, e o grupo popular, formado pela fusão de várias
culturas: portugueses aventureiros ou degredados, negros e índios. Aleijadinho, unindo as sofisticações da arte
erudita ao entendimento do artífice popular, consegue fazer essa síntese característica deste momento único na
história da arte brasileira: o barroco colonial. MAJORA, C. BrHistória, n. 3, mar. 2007 (adaptado).

No século XVIII, a arte brasileira, mais especificamente a de Minas Gerais, apresentava a valorização da técnica
e um estilo próprio, incluindo a escolha dos materiais. Artistas como Aleijadinho e Mestre Ataíde têm suas obras
caracterizadas por peculiaridades que são identificadas por meio
A do emprego de materiais oriundos da Europa e da interpretação realista dos objetos representados.
B do uso de recursos materiais disponíveis no local e da interpretação formal com características próprias.
C da utilização de recursos materiais vindos da Europa e da homogeneização e linearidade representacional.
D da observação e da cópia detalhada do objeto representado e do emprego de materiais disponíveis na região.
E da utilização de materiais disponíveis no Brasil e da interpretação idealizada e linear dos objetos representados.
LC - 2º dia | Caderno 15 - CINZA - Página 14
2015 *CZ1525dom15*
QUESTÃO 121 QUESTÃO 123
Minha mãe achava estudo a coisa mais fina do mundo.
Não é.
A coisa mais fina do mundo é o sentimento.
Aquele dia de noite, o pai fazendo serão,
ela falou comigo:
“Coitado, até essa hora no serviço pesado”.
Arrumou pão e café, deixou tacho no fogo com
água quente.
Não me falou em amor. DAHMER, A. Disponível em: www.malvados.com.br. Acesso em: 18 fev. 2013.
Essa palavra de luxo.
PRADO, A. Poesia reunida. São Paulo: Siciliano, 1991.
As redes sociais permitem que seus usuários facilmente
Um dos procedimentos consagrados pelo Modernismo foi compartilhem entre si ideias e opiniões. Na tirinha, há um
a percepção de um lirismo presente nas cenas e fatos do
cotidiano. No poema de Adélia Prado, o eu lírico resgata tom de crítica àqueles que
a poesia desses elementos a partir do(a)
A fazem uso inadequado das redes sociais para criticar
A reflexão irônica sobre a importância atribuída aos o mundo.
estudos por sua mãe.
B sentimentalismo, oposto à visão pragmática que B são usuários de redes sociais e têm seus desejos
reconhecia na mãe. atendidos.
C olhar comovido sobre seu pai, submetido ao trabalho
pesado. C se supõem críticos, porém não apresentam ação
D reconhecimento do amor num gesto de aparente efetiva.
banalidade.
D são usuários das redes sociais e não criticam o
E enfoque nas relações afetivas abafadas pela vida
conjugal. mundo.

QUESTÃO 122 E se esforçam para promover mudanças no mundo.

QUESTÃO 124
Organizados pelo Comitê Intertribal Indígena,
com apoio do Ministério dos Esportes, os Jogos dos
Povos Indígenas têm o seguinte mote: “O importante
não é competir, e sim, celebrar”. A proposta é recente,
já que a primeira edição dos jogos ocorreu em 1996, e
tem como objetivo a integração das diferentes tribos,
assim como o resgate e a celebração dessas culturas
tradicionais. A edição dos jogos de 2003, por exemplo,
teve a participação de sessenta etnias, dentre elas os
kaiowá, guarani, bororo, pataxó e yanomami. A última
edição ocorreu em 2009, e foi a décima vez que o torneio
Disponível em: http://portal.saude.gov.br. Acesso em: 31 jul. 2012. foi realizado. A periodicidade dos jogos é anual, com
Campanhas educativas têm o propósito de provocar exceção do intervalo ocorrido em 1997, 1998, 2006 e
uma reflexão em torno de questões sociais de grande 2008, quando não houve edições.
relevância, tais como as relacionadas à cidadania
RONDINELLI, P. Disponível em: www.brasilescola.com. Acesso em: 15 ago. 2013.
e também à saúde. Com a imagem de um relógio
despertador e o slogan “Sempre é hora de combater a Considerando o texto, os Jogos dos Povos Indígenas
dengue”, a Campanha Nacional de Combate à Dengue
objetiva convencer a população de que é preciso assemelham-se aos Jogos Olímpicos em relação à
A eliminar potenciais criadouros, quando aparecer a A quantificação de medalhas e vitórias.
doença.
B posicionar-se criticamente sobre as ações de combate B melhora de resultados e performance.
ao mosquito. C realização anual dos eventos e festejos.
C prevenir-se permanentemente contra a doença.
D repensar as ações de prevenção da doença. D renovação de técnicas e táticas esportivas.
E preparar os agentes de combate ao mosquito. E aproximação de diferentes sujeitos e culturas.

LC - 2º dia | Caderno 15 - CINZA - Página 15


*CZ1525dom16* 2015

QUESTÃO 125 Então passou Caiuanogue, a estrela da manhã.


Macunaíma já meio enjoado de tanto viver pediu pra ela
que o carregasse pro céu.
Caiuanogue foi se chegando porém o herói fedia muito.
— Vá tomar banho! — ela fez. E foi-se embora.
Assim nasceu a expressão “Vá tomar banho”
que os brasileiros empregam se referindo a certos
imigrantes europeus.
ANDRADE, M. Macunaíma: o herói sem nenhum caráter. Rio de Janeiro: Agir, 2008.

O fragmento de texto faz parte do capítulo VII, intitulado


“Vei, a Sol”, do livro Macunaíma, de Mário de Andrade,
pertencente à primeira fase do Modernismo brasileiro.
Considerando a linguagem empregada pelo narrador, é
possível identificar
Disponível em: www.casualciclo.com. Acesso: 2 ago. 2012.
A resquícios do discurso naturalista usado pelos
A charge retrata um comportamento recorrente nos escritores do século XIX.
dias atuais: a insatisfação das pessoas com o peso. B ausência de linearidade no tratamento do tempo,
No entanto, do ponto de vista orgânico, o peso corporal recurso comum ao texto narrativo da primeira fase
se torna um problema à saúde quando modernista.
A estimula a adesão à dieta. C referência à fauna como meio de denunciar o
B aumenta conforme a idade. primitivismo e o atraso de algumas regiões do país.
C expressa a inatividade da pessoa. D descrição preconceituosa dos tipos populares
brasileiros, representados por Macunaíma e
D provoca modificações na aparência. Caiuanogue.
E acomete o funcionamento metabólico. E uso da linguagem coloquial e de temáticas do
QUESTÃO 126 lendário brasileiro como meio de valorização da
cultura popular nacional.
O rap constitui-se em uma expressão artística por
meio da qual os MCs relatam poeticamente a condição QUESTÃO 128
social em que vivem e retratam suas experiências
cotidianas. Anfíbio com formato de cobra é descoberto
SOUZA, J.; FIALHO, V. M.; ARALDI, J. Hip hop: da rua para a escola. Porto Alegre: Sulina, 2008. no Rio Madeira (RO)
O “relato poético” é uma característica fundamental desse Animal raro foi encontrado por biólogos em canteiro
gênero musical, em que o de obras de usina. Exemplares estão no Museu Emilio
Goeldi, no Pará
A MC canta de forma melodiosa as letras, que retratam
a complexa realidade em que se encontra. O trabalho de um grupo de biólogos no canteiro
B rap se limita a usar sons eletrônicos nas músicas, de obras da Usina Hidrelétrica Santo Antônio, no Rio
que seriam responsáveis por retratar a realidade da Madeira, em Porto Velho, resultou na descoberta de um
periferia. anfíbio de formato parecido com uma cobra. Atretochoana
eiselti é o nome científico do animal raro descoberto em
C rap se caracteriza pela proximidade das notas na Rondônia. Até então, só havia registro do anfíbio no
melodia, em que a letra é mais recitada do que Museu de História Natural de Viena e na Universidade
cantada, como em uma poesia. de Brasília. Nenhum deles tem a descrição exata de
D MC canta enquanto outros músicos o acompanham localidade, apenas “América do Sul”. A descoberta
com instrumentos, tais como o contrabaixo elétrico e ocorreu em dezembro do ano passado, mas apenas
o teclado. agora foi divulgada.
E MC canta poemas amplamente conhecidos, XIMENES, M. Disponível em: http://g1.globo.com. Acesso em: 1 ago. 2012.

fundamentando sua atuação na memorização de A notícia é um gênero textual em que predomina a função
suas letras. referencial da linguagem. No texto, essa predominância
QUESTÃO 127 evidencia-se pelo(a)
A recorrência de verbos no presente para convencer o
Vei, a Sol
leitor.
Ora o pássaro careceu de fazer necessidade, fez B uso da impessoalidade para assegurar a objetividade
e o herói ficou escorrendo sujeira de urubu. Já era da informação.
de madrugadinha e o tempo estava inteiramente frio.
Macunaíma acordou tremendo, todo lambuzado. Assim C questionamento do código linguístico na construção
mesmo examinou bem a pedra mirim da ilhota para vê da notícia.
si não havia alguma cova com dinheiro enterrado. Não D utilização de expressões úteis que mantêm aberto o
havia não. Nem a correntinha encantada de prata que canal de comunicação com o leitor.
indica pro escolhido, tesouro de holandês. Havia só as E emprego dos sinais de pontuação para expressar as
formigas jaquitaguas ruivinhas. emoções do autor.

LC - 2º dia | Caderno 15 - CINZA - Página 16


2015 *CZ1525dom17*
QUESTÃO 129 Tendo em vista seus elementos constitutivos e o meio de
divulgação, esse texto identifica-se como
A verbete enciclopédico, pois contém a definição de um
item lexical.
B cartaz, pois instrui sobre a localização de um ambiente
que oferece atrações turísticas.
C cartão-postal, pois a imagem mostra ao destinatário o
local onde se encontra o remetente.
D anúncio publicitário, pois busca persuadir o público-alvo
VEIGA, D. Disponível em: http://dirceuveiga.com.br. Acesso em: 3 maio 2012. a visitar um determinado local.
E fotografia, pois retrata uma paisagem urbana de
Considerando que a internet influencia os modos de
grande impacto.
comunicação contemporânea, a charge faz uma crítica
ao uso vicioso dessa tecnologia, pois QUESTÃO 131
A gera diminuição no tempo de descanso, substituído TEXTO I
pelo contato com outras pessoas. Quem sabe, devido às atividades culinárias da
B propicia a continuação das atividades de trabalho, esposa, nesses idílios Vadinho dizia-lhe “Meu manuê
ainda que em ambiente doméstico. de milho verde, meu acarajé cheiroso, minha franguinha
C promove o distanciamento nos relacionamentos, gorda”, e tais comparações gastronômicas davam justa
mesmo entre pessoas próximas fisicamente. ideia de certo encanto sensual e caseiro de dona Flor a
D tem impacto negativo no tempo disponível para o esconder-se sob uma natureza tranquila e dócil. Vadinho
lazer do casal. conhecia-lhe as fraquezas e as expunha ao sol, aquela
ânsia controlada de tímida, aquele recatado desejo
E implica a adoção de atitudes agressivas entre os
fazendo-se violência e mesmo incontinência ao libertar-
membros de uma mesma família. se na cama.
QUESTÃO 130 AMADO, J. Dona Flor e seus dois maridos. São Paulo: Martins, 1966.

TEXTO II
As suas mãos trabalham na braguilha das calças
do falecido. Dulcineusa me confessou mais tarde: era
assim que o marido gostava de começar as intimidades.
Um fazer de conta que era outra coisa, a exemplo do gato
que distrai o olhar enquanto segura a presa nas patas.
Esse o acordo silencioso que tinham: ele chegava em
casa e se queixava que tinha um botão a cair. Calada,
Dulcineusa se armava dos apetrechos da costura e se
posicionava a jeito dos prazeres e dos afazeres.
COUTO, M. Um rio chamado tempo, uma casa chamada terra. São Paulo: Cia. das Letras, 2002.

Tema recorrente na obra de Jorge Amado, a figura feminina


aparece, no fragmento, retratada de forma semelhante à
que se vê no texto do moçambicano Mia Couto. Nesses
dois textos, com relação ao universo feminino em seu
contexto doméstico, observa-se que
A o desejo sexual é entendido como uma fraqueza
moral, incompatível com a mulher casada.
B a mulher tem um comportamento marcado por
convenções de papéis sexuais.
C à mulher cabe o poder da sedução, expresso pelos
gestos, olhares e silêncios que ensaiam.
D a mulher incorpora o sentimento de culpa e age com
apatia, como no mito bíblico da serpente.
E a dissimulação e a malícia fazem parte do repertório
Caras, n. 34, ago. 2011. feminino nos espaços público e íntimo.

LC - 2º dia | Caderno 15 - CINZA - Página 17


*CZ1525dom18* 2015

QUESTÃO 132 QUESTÃO 134

Um relacionamento de grupo saudável exige um Manter as contas sob controle e as finanças saudáveis
parece um objetivo inatingível para você? Tenha certeza
número de indivíduos trabalhando interdependentemente de que você não está sozinho. A bagunça na vida
para completar um projeto, com total participação financeira compromete os sonhos de muita gente no
individual e contribuição pessoal. Se uma pessoa domina, Brasil. É por isso que nós lançamos, pelo terceiro ano
os outros membros têm pouco crescimento ou prazer consecutivo, este especial com informações que ajudam
na atividade, não existe um verdadeiro relacionamento a encarar a situação de forma prática. Sem malabarismos
no grupo. O teatro é uma atividade artística que exige — mas com boa dose de disciplina! — é possível quitar
as dívidas, organizar os gastos, fazer planos de consumo
o talento e a energia de muitas pessoas — desde a que caibam em seus rendimentos mensais e estruturar os
primeira ideia de uma peça ou cena até o último eco investimentos para fazer o dinheiro que sobra render mais.
de aplauso. Sem esta interação não há lugar para o Ter dinheiro para viver melhor está diretamente
ator individualmente, pois sem o funcionamento do relacionado a sua capacidade de se organizar e de
grupo, para quem iria ele representar, que materiais eleger prioridades na hora de gastar. Aceite o desafio e
boa leitura!
usaria e que efeitos poderia produzir? O aluno-ator deve Você S/A, n. 16, 2011 (adaptado).

aprender que “como atuar”, assim como no jogo, está No trecho apresentado, são utilizados vários argumentos
intrinsecamente ligado a todas as outras pessoas na que demonstram que o objetivo principal do produtor do
complexidade da forma da arte. O teatro improvisacional texto, em relação ao público-alvo da revista, é
requer relacionamento de grupo muito intenso, pois é a A conscientizar o leitor de que ele é capaz de
partir do acordo e da atuação em grupo que emerge o economizar.
material para as cenas e peças. B levar o leitor a envolver-se com questões de ordem
SPOLIN, V. Improvisação para o teatro. São Paulo: Perspectiva, 2008. econômica.
C ajudar o leitor a quitar suas dívidas e organizar sua
Com base no texto, as diferenças e similaridades dos vida financeira.
atores são aceitas no teatro de improvisação quando D persuadir o leitor de que ele não é o único com
problemas financeiros.
A todos experimentam o teatro juntos e sem julgamentos.
E convencer o leitor da importância de ler essa edição
B uma parte do grupo comanda a outra, exercendo o especial da revista.
poder.
QUESTÃO 135
C a opinião de alguns tem valor e demonstra a sua
capacidade individual. O primeiro contato dos suruís com o homem branco foi
em 1969. A população indígena foi dizimada por doenças
D a individualidade se destaca e traz à tona o talento e matanças, mas, recentemente, voltou a crescer.
daquele que é o melhor. Soa contraditório, mas a mesma modernidade que
quase dizimou os suruís nos tempos do primeiro contato
E uma pessoa precisa dominar, comandando as ações promete salvar a cultura e preservar o território desse
do grupo, sem acordos. povo. Em 2007, o líder Almir Suruí, de 37 anos, fechou
uma parceria inédita e levou a tecnologia às tribos.
QUESTÃO 133 Os índios passaram a valorizar a história dos anciãos.
E a resguardar, em vídeos e fotos on-line, as tradições
A dança moderna propõe em primeiro lugar o da aldeia. Ainda se valeram de smartphones e GPS para
conhecimento de si e o autodomínio. Minha proposta é delimitar suas terras e identificar os desmatamentos
esta: através do conhecimento e do autodomínio chego à ilegais.
RIBEIRO, A. Não temos o direito de ficar isolados. Época, n. 718, 20 fev. 2012 (adaptado).
forma, à minha forma — e não o contrário. É uma inversão
que muda toda a estética, toda a razão do movimento. Considerando-se as características históricas da relação
A técnica na dança tem apenas uma finalidade: preparar entre índios e não índios, a suposta contradição observada
o corpo para responder à exigência do espírito artístico. na relação entre suruís e recursos da modernidade
justifica-se porque os índios
VIANNA, K.; CARVALHO, M. A. A dança. São Paulo: Siciliano, 1990.
A aderiram à tecnologia atual como forma de assimilar a
Na abordagem dos autores, a técnica, o autodomínio e o cultura do homem branco.
conhecimento do bailarino estão a serviço da B fizeram uso do GPS para identificar áreas propícias a
novas plantações.
A padronização do movimento da dança.
C usaram recursos tecnológicos para registrar a cultura
B subordinação do corpo a um padrão. do seu povo.
C concretização da criação pessoal. D fecharam parceria para denunciar as vidas perdidas
por doenças e matanças.
D ideia preconcebida de forma.
E resguardaram as tradições da aldeia à custa do
E busca pela igualdade entre os bailarinos. isolamento provocado pela tecnologia moderna.

LC - 2º dia | Caderno 15 - CINZA - Página 18


2015 *CZ1525dom19*
MATEMÁTICA E SUAS TECNOLOGIAS QUESTÃO 137
Um granjeiro detectou uma infecção bacteriológica
Questões de 136 a 180 em sua criação de 100 coelhos. A massa de cada
coelho era de, aproximadamente, 4 kg. Um veterinário
QUESTÃO 136 prescreveu a aplicação de um antibiótico, vendido em
frascos contendo 16 mL, 25 mL, 100 mL, 400 mL ou
Em uma confeitaria, um cliente comprou um cupcake 1 600 mL. A bula do antibiótico recomenda que, em aves
(pequeno bolo no formato de um tronco de cone regular e coelhos, seja administrada uma dose única de 0,25 mL
mais uma cobertura, geralmente composta por um para cada quilograma de massa do animal.
creme), semelhante ao apresentado na figura: Para que todos os coelhos recebessem a dosagem do
7 cm antibiótico recomendada pela bula, de tal maneira que
não sobrasse produto na embalagem, o criador deveria
comprar um único frasco com a quantidade, em mililitros,
igual a
A 16.
5 cm B 25.
C 100.
D 400.
E 1 600.
QUESTÃO 138
Alguns brasileiros têm o hábito de trocar de carro a
4 cm cada um ou dois anos, mas essa prática nem sempre é
um bom negócio, pois o veículo desvaloriza com o uso.
Esse fator é chamado de depreciação, sendo maior nos
primeiros anos de uso.
Uma pessoa realizou uma pesquisa sobre o valor de
4 cm mercado dos dois veículos (X e Y) que possui. Colocou
os resultados obtidos em um mesmo gráfico, pois os
veículos foram comprados juntos.
Como o bolinho não seria consumido no
estabelecimento, o vendedor verificou que as caixas Valor (R$)
disponíveis para embalar o doce eram todas em formato
55 000
de blocos retangulares, cujas medidas estão apresentadas
no quadro: 50 000

45 000
Dimensões
Embalagem 40 000
(comprimento × largura × altura)
35 000
I 8,5 cm × 12,2 cm × 9,0 cm 30 000
II 10 cm × 11 cm × 15 cm 25 000
III 7,2 cm × 8,2 cm × 16 cm 20 000
X
IV 7,5 cm × 7,8 cm × 9,5 cm 15 000

V 15 cm × 8 cm × 9 cm 10 000 Y
5 000
A embalagem mais apropriada para armazenar o doce,
de forma a não deformá-lo e com menor desperdício de 0 2 4 6 8 10 12 14
espaço na caixa, é Tempo de uso (ano)
A I.
Após a pesquisa, ela decidiu vender os veículos no
B II. momento em que completarem quatro anos de uso.
C III. Disponível em: www.carrosnaweb.com.br. Acesso em: 3 ago. 2012 (adaptado).

D IV. Considerando somente os valores de compra e de venda


E V. dos veículos por essa pessoa, qual a perda, em reais, que
ela terá?
A 10 000,00
B 15 000,00
C 25 000,00
D 35 000,00
E 45 000,00

MT - 2º dia | Caderno 15 - CINZA - Página 19


*CZ1525dom20* 2015

QUESTÃO 139 QUESTÃO 141

Uma pesquisa recente aponta que 8 em cada Doenças relacionadas ao saneamento ambiental
10 homens brasileiros dizem cuidar de sua beleza, não inadequado (DRSAI) podem estar associadas ao
apenas de sua higiene pessoal. abastecimento deficiente de água, tratamento inadequado
CAETANO, M.; SOEIRO, R.; DAVINO, R. Cosméticos. de esgoto sanitário, contaminação por resíduos sólidos
Superinteressante, n. 304, maio 2012 (adaptado).
ou condições precárias de moradia. O gráfico apresenta o
Outra maneira de representar esse resultado é número de casos de duas DRSAI de uma cidade:
exibindo o valor percentual dos homens brasileiros que
dizem cuidar de sua beleza. Nº de casos
Doença A
Doença B
Qual é o valor percentual que faz essa representação?
1 200
A 80%
1 100
B 8%
1 000
C 0,8%
900
D 0,08%
800
E 0,008%
700
QUESTÃO 140
600
Em uma pesquisa sobre prática de atividade física, 500
foi perguntado aos entrevistados sobre o hábito de andar
400
de bicicleta ao longo da semana e com que frequência
o faziam. Entre eles, 75% afirmaram ter esse hábito, e a 300
frequência semanal com que o faziam é a apresentada 200
no gráfico:
100
Com que frequência?
mar

ago
mai mês

nov

dez
abr

out
jan

fev

jun

set
jul
26%

Disponível em: http://dados.gov.br. Acesso em: 7 dez. 2012 (adaptado).


17%
15%
13% 12%
O mês em que se tem a maior diferença entre o número
10% de casos das doenças de tipo A e B é
7%
A janeiro.
B abril.
1 vez 2 vezes 3 vezes 4 vezes 5 vezes 6 vezes Todos os C julho.
dias
D setembro.
Que porcentagem do total de entrevistados representa E novembro.
aqueles que afirmaram andar de bicicleta pelo menos três
vezes por semana?
A 70,0%
B 52,5%
C 22,5%
D 19,5%
E 5,0%

MT - 2º dia | Caderno 15 - CINZA - Página 20


2015 *CZ1525dom21*
QUESTÃO 142 Quantos metros uma criança sentada no cavalo C1
percorrerá a mais do que uma criança no cavalo C2, em
No comércio é comumente utilizado o salário mensal
uma sessão? Use 3,0 como aproximação para p.
comissionado. Além de um valor fixo, o vendedor tem um
incentivo, geralmente um percentual sobre as vendas. A 55,5
Considere um vendedor que tenha salário comissionado, B 60,0
sendo sua comissão dada pelo percentual do total de C 175,5
vendas que realizar no período. O gráfico expressa o
D 235,5
valor total de seu salário, em reais, em função do total de
E 240,0
vendas realizadas, também em reais.
QUESTÃO 144
Salário (R$)
2 400
2 200 O padrão internacional ISO 216 define os tamanhos de
2 000 papel utilizados em quase todos os países. O formato-base
1 800 é uma folha retangular de papel chamada de A0, cujas
1 600
dimensões estão na razão 1 : √2 . A partir de então, dobra-se
1 400
a folha ao meio, sempre no lado maior, definindo os demais
1 200
1 000
formatos, conforme o número da dobradura. Por exemplo,
800 A1 é a folha A0 dobrada ao meio uma vez, A2 é a folha A0
600 dobrada ao meio duas vezes, e assim sucessivamente,
400
Total de conforme figura.
200 Vendas (R$)
0
0 10 000 20 000 30 000 40 000 50 000 60 000 A2
A0 A1

Qual o valor percentual da sua comissão?


A 2,0% Um tamanho de papel bastante comum em escritórios
B 5,0% brasileiros é o A4, cujas dimensões são 21,0 cm por 29,7 cm.
C 16,7% Quais são as dimensões, em centímetros, da folha A0?
D 27,7%
A 21,0 × 118,8
E 50,0%
B 84,0 × 29,7
QUESTÃO 143 C 84,0 × 118,8
D 168,0 × 237,6
A figura é uma representação simplificada do
carrossel de um parque de diversões, visto de cima. E 336,0 × 475,2
Nessa representação, os cavalos estão identificados QUESTÃO 145
pelos pontos escuros, e ocupam circunferências de raios
3 m e 4 m, respectivamente, ambas centradas no ponto O. Uma barraca de tiro ao alvo de um parque de diversões
Em cada sessão de funcionamento, o carrossel efetua dará um prêmio de R$ 20,00 ao participante, cada vez
10 voltas. que ele acertar o alvo. Por outro lado, cada vez que ele
C 1 errar o alvo, deverá pagar R$ 10,00. Não há cobrança
inicial para participar do jogo. Um participante deu 80 tiros
e, ao final, recebeu R$ 100,00.
Qual foi o número de vezes que esse participante acertou
o alvo?

O C2 A 30
B 36
C 50
D 60
E 64

MT - 2º dia | Caderno 15 - CINZA - Página 21


*CZ1525dom22* 2015

QUESTÃO 146 QUESTÃO 148

No próximo final de semana, um grupo de alunos Um promotor de eventos foi a um supermercado para
participará de uma aula de campo. Em dias chuvosos, comprar refrigerantes para uma festa de aniversário.
aulas de campo não podem ser realizadas. A ideia é que Ele verificou que os refrigerantes estavam em garrafas
essa aula seja no sábado, mas, se estiver chovendo no de diferentes tamanhos e preços. A quantidade de
sábado, a aula será adiada para o domingo. Segundo a refrigerante e o preço de cada garrafa, de um mesmo
meteorologia, a probabilidade de chover no sábado é de refrigerante, estão na tabela.
30% e a de chover no domingo é de 25%.
Quantidade de
A probabilidade de que a aula de campo ocorra no Preço
Garrafa refrigerante
domingo é de (R$)
(litro)
A 5,0% Tipo I 0,5 0,68
B 7,5% Tipo II 1,0 0,88
C 22,5% Tipo III 1,5 1,08
D 30,0% Tipo IV 2,0 1,68
E 75,0% Tipo V 3,0 2,58
QUESTÃO 147
Para economizar o máximo possível, o promotor de
Sabe-se que o valor cobrado na conta de energia eventos deverá comprar garrafas que tenham o menor
elétrica correspondente ao uso de cada eletrodoméstico preço por litro de refrigerante.
é diretamente proporcional à potência utilizada pelo O promotor de eventos deve comprar garrafas do tipo
aparelho, medida em watts (W), e também ao tempo
que esse aparelho permanece ligado durante o mês. A I.
Certo consumidor possui um chuveiro elétrico com B II.
potência máxima de 3 600 W e um televisor com potência C III.
máxima de 100 W. Em certo mês, a família do consumidor
D IV.
utilizou esse chuveiro elétrico durante um tempo total de
5 horas e esse televisor durante um tempo total de 60 E V.
horas, ambos em suas potências máximas. QUESTÃO 149
Qual a razão entre o valor cobrado pelo uso do chuveiro e
o valor cobrado pelo uso do televisor? Uma pessoa, ao fazer uma pesquisa com alguns
alunos de um curso, coletou as idades dos entrevistados
A 1 : 1 200 e organizou esses dados em um gráfico.
B 1 : 12
C 3:1
36 : 1
Frequência de ocorrência

D
E 432 : 1 21

15

12

9 12 18 Idade (ano)

Qual a moda das idades, em anos, dos entrevistados?


A 9
B 12
C 13
D 15
E 21

MT - 2º dia | Caderno 15 - CINZA - Página 22


2015 *CZ1525dom23*
QUESTÃO 150 O aumento percentual esperado do fluxo por esse vaso
Considere que os quarteirões de um bairro tenham está entre
sido desenhados no sistema cartesiano, sendo a origem A 7% e 8%
o cruzamento das duas ruas mais movimentadas
desse bairro. Nesse desenho, as ruas têm suas B 9% e 11%
larguras desconsideradas e todos os quarteirões são C 20% e 22%
quadrados de mesma área e a medida de seu lado é a D 39% e 41%
unidade do sistema.
A seguir há uma representação dessa situação, em E 46% e 47%
que os pontos A, B, C e D representam estabelecimentos
QUESTÃO 152
comerciais desse bairro.
y Um bairro residencial tem cinco mil moradores, dos
quais mil são classificados como vegetarianos. Entre os
vegetarianos, 40% são esportistas, enquanto que, entre
os não vegetarianos, essa porcentagem cai para 20%.
A Uma pessoa desse bairro, escolhida ao acaso, é
esportista.
C
A probabilidade de ela ser vegetariana é
B
x
2
A
25
D 1
B
5

1 quarteirão: 1
C
4
Suponha que uma rádio comunitária, de fraco sinal,
garante área de cobertura para todo estabelecimento que 1
se encontre num ponto cujas coordenadas satisfaçam à D
3
inequação: x² + y² – 2x – 4y – 31 ≤ 0.
A fim de avaliar a qualidade do sinal, e proporcionar 5
E
uma futura melhora, a assistência técnica da rádio realizou 6
uma inspeção para saber quais estabelecimentos estavam
dentro da área de cobertura, pois estes conseguem ouvir QUESTÃO 153
a rádio enquanto os outros não.
Os estabelecimentos que conseguem ouvir a rádio são Na construção de um conjunto habitacional de casas
apenas populares, todas serão feitas num mesmo modelo,
ocupando, cada uma delas, terrenos cujas dimensões
A A e C.
são iguais a 20 m de comprimento por 8 m de largura.
B B e C. Visando a comercialização dessas casas, antes do início
C B e D. das obras, a empresa resolveu apresentá-las por meio de
D A, B e C. maquetes construídas numa escala de 1 : 200.
E B, C e D. As medidas do comprimento e da largura dos terrenos,
QUESTÃO 151 respectivamente, em centímetros, na maquete construída,
foram de
O fisiologista francês Jean Poiseuille estabeleceu, na
primeira metade do século XIX, que o fluxo de sangue A 4 e 10.
por meio de um vaso sanguíneo em uma pessoa é B 5 e 2.
diretamente proporcional à quarta potência da medida
C 10 e 4.
do raio desse vaso. Suponha que um médico, efetuando
uma angioplastia, aumentou em 10% o raio de um vaso D 20 e 8.
sanguíneo de seu paciente. E 50 e 20.

MT - 2º dia | Caderno 15 - CINZA - Página 23


*CZ1525dom24* 2015

QUESTÃO 154
Os maias desenvolveram um sistema de numeração vigesimal que podia representar qualquer número inteiro,
não negativo, com apenas três símbolos. Uma concha representava o zero, um ponto representava o número 1 e uma
barrinha horizontal, o número 5. Até o número 19, os maias representavam os números como mostra a Figura 1:

0 1 2 3 4

5 6 7 8 9

10 11 12 13 14 Terceira
posição 8 x202

Segunda 6 x 201
posição
15 16 17 18 19
Primeira
posição 12 x 1

Figura 1 Figura 2
Números superiores a 19 são escritos na vertical, seguindo potências de 20 em notação posicional, como mostra
a Figura 2.
Ou seja, o número que se encontra na primeira posição é multiplicado por 200 = 1, o número que se encontra na
segunda posição é multiplicado por 201 = 20 e assim por diante. Os resultados obtidos em cada posição são somados
para obter o número no sistema decimal.
Um arqueólogo achou o hieroglifo da Figura 3 em um sítio arqueológico:

Terceira
posição

Segunda
posição

Primeira
posição

Figura 3
Disponível em: http://mdmat.mat.ufrgs.br. Acesso em: 13 ago. 2012 (adaptado).

O número, no sistema decimal, que o hieroglifo da Figura 3 representa é igual a


A 279.
B 539.
C 2 619.
D 5 219.
E 7 613.

MT - 2º dia | Caderno 15 - CINZA - Página 24


2015 *CZ1525dom25*
QUESTÃO 155 QUESTÃO 157

Durante um jogo de futebol foram anunciados os Uma fábrica vende pizzas congeladas de tamanhos
totais do público presente e do público pagante. Diante médio e grande, cujos diâmetros são respectivamente
da diferença entre os dois totais apresentados, um dos 30 cm e 40 cm. Fabricam-se apenas pizzas de sabor
comentaristas esportivos presentes afirmou que apenas muçarela. Sabe-se que o custo com os ingredientes para
75% das pessoas que assistiam àquele jogo no estádio
a preparação é diretamente proporcional ao quadrado
pagaram ingresso.
do diâmetro da pizza, e que na de tamanho médio esse
Considerando que a afirmativa do comentarista está custo é R$ 1,80. Além disso, todas possuem um custo
correta, a razão entre o público não pagante e o público fixo de R$ 3,00, referente às demais despesas da fábrica.
pagante naquele jogo foi Sabe-se ainda que a fábrica deseja lucrar R$ 2,50 em
1 cada pizza grande.
A Qual é o preço que a fábrica deve cobrar pela pizza
4
grande, a fim de obter o lucro desejado?
1
B A R$ 5,70
3
B R$ 6,20
3
C
4 C R$ 7,30
D R$ 7,90
4
D E R$ 8,70
3
QUESTÃO 158
3
E
1 No jogo mostrado na figura, uma bolinha desloca-se
somente de duas formas: ao longo de linhas retas ou por
QUESTÃO 156 arcos de circunferências centradas no ponto O e raios
variando de 1 a 8. Durante o jogo, a bolinha que estiver
Uma fábrica que trabalha com matéria-prima de
no ponto P deverá realizar a seguinte sequência de
fibra de vidro possui diversos modelos e tamanhos de
caixa-d’água. Um desses modelos é um prisma reto movimentos: 2 unidades no mesmo sentido utilizado para
com base quadrada. Com o objetivo de modificar a ir do ponto O até o ponto A e, no sentido anti-horário, um
capacidade de armazenamento de água, está sendo arco de circunferência cujo ângulo central é 120°.
construído um novo modelo, com as medidas das
arestas da base duplicadas, sem a alteração da altura,
mantendo a mesma forma. E

Em relação ao antigo modelo, o volume do novo modelo é


F
A oito vezes maior. D P

B quatro vezes maior. B A

H
C duas vezes maior. 1 2 3 4 5 6 7 8
O
D a metade.
C
E a quarta parte.

Após a sequência de movimentos descrita, a bolinha


estará no ponto
A B.
B D.
C E.
D F.
E G.

MT - 2º dia | Caderno 15 - CINZA - Página 25


*CZ1525dom26* 2015

QUESTÃO 159 A probabilidade de o primeiro país escolhido pertencer à


América do Norte e o segundo pertencer ao continente
O banheiro de uma escola pública, com paredes asiático é
e piso em formato retangular, medindo 5 metros de
1
largura, 4 metros de comprimento e 3 metros de altura, A
9
precisa de revestimento no piso e nas paredes internas,
excluindo a área da porta, que mede 1 metro de largura 1
B
por 2 metros de altura. Após uma tomada de preços 4
com cinco fornecedores, foram verificadas as seguintes
combinações de azulejos para as paredes e de lajotas 3
C
para o piso, com os preços dados em reais por metro 10
quadrado, conforme a tabela.
2
D
Fornecedor Azulejo (R$/m2) Lajota (R$/m2) 3
A 31,00 31,00
B 33,00 30,00 E 1
C 29,00 39,00
QUESTÃO 161
D 30,00 33,00
Ao se perfurar um poço no chão, na forma de um
E 40,00 29,00 cilindro circular reto, toda a terra retirada é amontoada
na forma de um cone circular reto, cujo raio da base é
Desejando-se efetuar a menor despesa total, deverá ser
o triplo do raio do poço e a altura é 2,4 metros. Sabe-se
escolhido o fornecedor que o volume desse cone de terra é 20% maior do que o
volume do poço cilíndrico, pois a terra fica mais fofa após
A A. ser escavada.
B B. Qual é a profundidade, em metros, desse poço?
C C. A 1,44
D D. B 6,00
C 7,20
E E.
D 8,64
QUESTÃO 160 E 36,00

Um protocolo tem como objetivo firmar acordos QUESTÃO 162


e discussões internacionais para conjuntamente Uma confecção possuía 36 funcionários, alcançando
uma produtividade de 5 400 camisetas por dia, com uma
estabelecer metas de redução de emissão de gases de
jornada de trabalho diária dos funcionários de 6 horas.
efeito estufa na atmosfera. O quadro mostra alguns dos Entretanto, com o lançamento da nova coleção e de uma
países que assinaram o protocolo, organizados de acordo nova campanha de marketing, o número de encomendas
cresceu de forma acentuada, aumentando a demanda
com o continente ao qual pertencem.
diária para 21 600 camisetas. Buscando atender essa
Países da América nova demanda, a empresa aumentou o quadro de
Países da Ásia funcionários para 96. Ainda assim, a carga horária de
do Norte
trabalho necessita ser ajustada.
Estados Unidos
China
da América Qual deve ser a nova jornada de trabalho diária dos
Canadá Índia funcionários para que a empresa consiga atender a
demanda?
México Japão
A 1 hora e 30 minutos.
Em um dos acordos firmados, ao final do ano, dois B 2 horas e 15 minutos.
dos países relacionados serão escolhidos aleatoriamente, C 9 horas.
um após o outro, para verificar se as metas de redução do D 16 horas.
protocolo estão sendo praticadas. E 24 horas.
MT - 2º dia | Caderno 15 - CINZA - Página 26
2015 *CZ1525dom27*
QUESTÃO 163 QUESTÃO 165

Na imagem, a personagem Mafalda mede a Um meio de transporte coletivo que vem ganhando
circunferência do globo que representa o planeta Terra. espaço no Brasil é a van, pois realiza, com relativo conforto
e preço acessível, quase todos os tipos de transportes:
escolar e urbano, intermunicipal e excursões em geral.
O dono de uma van, cuja capacidade máxima é de
15 passageiros, cobra para uma excursão até a capital de
seu estado R$ 60,00 de cada passageiro. Se não atingir
a capacidade máxima da van, cada passageiro pagará
mais R$ 2,00 por lugar vago.
Sendo x o número de lugares vagos, a expressão que
representa o valor arrecadado V(x), em reais, pelo dono
da van, para uma viagem até a capital é
A V(x) = 902x
B V(x) = 930x
C V(x) = 900 + 30x
D V(x) = 60x + 2x²
E V(x) = 900 - 30x - 2x²
Em uma aula de matemática, o professor considera
que a medida encontrada por Mafalda, referente à maior
circunferência do globo, foi de 80 cm. Além disso, informa
que a medida real da maior circunferência da Terra, a
linha do Equador, é de aproximadamente 40 000 km.
QUINO. Toda Mafalda. São Paulo: Martins Fontes, 2008 (adaptado).

A circunferência da linha do Equador é quantas vezes


maior do que a medida encontrada por Mafalda?
A 500
B 5 000
C 500 000
D 5 000 000
E 50 000 000

QUESTÃO 164

O prefeito de uma cidade deseja promover uma festa


popular no parque municipal para comemorar o aniversário
de fundação do município. Sabe-se que esse parque
possui formato retangular, com 120 m de comprimento
por 150 m de largura. Além disso, para segurança das
pessoas presentes no local, a polícia recomenda que
a densidade média, num evento dessa natureza, não
supere quatro pessoas por metro quadrado.
Seguindo as recomendações de segurança estabelecidas
pela polícia, qual é o número máximo de pessoas que
poderão estar presentes na festa?
A 1 000
B 4 500
C 18 000
D 72 000
E 120 000

MT - 2º dia | Caderno 15 - CINZA - Página 27


*CZ1525dom28* 2015

QUESTÃO 166 QUESTÃO 167


Uma empresa necessita colorir parte de suas O sindicato de trabalhadores de uma empresa
embalagens, com formato de caixas cúbicas, para que sugere que o piso salarial da classe seja de R$ 1 800,00,
possa colocar produtos diferentes em caixas distintas propondo um aumento percentual fixo por cada ano
pela cor, utilizando para isso um recipiente com tinta, dedicado ao trabalho. A expressão que corresponde à
conforme Figura 1. Nesse recipiente, mergulhou-se um proposta salarial (s), em função do tempo de serviço (t),
cubo branco, tal como se ilustra na Figura 2. Desta forma, em anos, é s(t) = 1 800 • (1,03)t.
a parte do cubo que ficou submersa adquiriu a cor da tinta.
De acordo com a proposta do sindicato, o salário de um
profissional dessa empresa com 2 anos de tempo de
serviço será, em reais,
A 7 416,00.
B 3 819,24.
C 3 709,62.
Figura 1 Figura 2
D 3 708,00.
Qual é a planificação desse cubo após submerso?
E 1 909,62.

QUESTÃO 168

O modelo predador-presa foi proposto de forma


A independente por Alfred J. Lotka, em 1925, e Vito
Volterra, em 1926. Esse modelo descreve a interação
entre duas espécies, sendo que uma delas dispõe de
alimentos para sobreviver (presa) e a outra se alimenta
da primeira (predador). Considere que o gráfico
representa uma interação predador-presa, relacionando
a população do predador com a população da sua presa
ao longo dos anos.

B Presa Predador

60

50

40
População

C 30

20

10

0
0 10 20 30 40 50 60 70 80
D Tempo (ano)
Disponível em: www.eventosufrpe.com.br. Acesso em: 22 mar. 2012 (adaptado)

De acordo com o gráfico, nos primeiros quarenta anos,


quantas vezes a população do predador se igualou à
da presa?
A 2
B 3
E
C 4
D 5
E 9

MT - 2º dia | Caderno 15 - CINZA - Página 28


2015 *CZ1525dom29*
QUESTÃO 169 QUESTÃO 171

A bandeira de um estado é formada por cinco faixas, Atendendo à encomenda de um mecânico, um


A, B, C, D e E, dispostas conforme a figura. soldador terá de juntar duas barras de metais diferentes.
A solda utilizada tem espessura de 18 milímetros,
B conforme ilustrado na figura.
A
C 18 mm

D
E 1m

Deseja-se pintar cada faixa com uma das cores verde,


1,5 m
azul ou amarelo, de tal forma que faixas adjacentes não
sejam pintadas com a mesma cor.
O cálculo do número de possibilidades distintas de se
Qual o comprimento, em metros, da peça resultante após
pintar essa bandeira, com a exigência acima, é
a soldagem?
A 1 × 2 × 1 × 1 × 2.
A 2,0230
B 3 × 2 × 1 × 1 × 2.
B 2,2300
C 3 × 2 × 1 × 1 × 3.
D 3 × 2 × 1 × 2 × 2. C 2,5018
E 3 × 2 × 2 × 2 × 2. D 2,5180
E 2,6800
QUESTÃO 170
QUESTÃO 172
Um artesão fabrica vários tipos de potes cilíndricos.
Mostrou a um cliente um pote de raio de base a e altura Uma fábrica brasileira de exportação de peixes vende
b. Esse cliente, por sua vez, quer comprar um pote com o para o exterior atum em conserva, em dois tipos de latas
dobro do volume do pote apresentado. O artesão diz que
cilíndricas: uma de altura igual a 4 cm e raio 6 cm, e outra
possui potes com as seguintes dimensões:
de altura desconhecida e raio de 3 cm, respectivamente,
• Pote I: raio a e altura 2b
conforme figura. Sabe-se que a medida do volume da
• Pote II: raio 2a e altura b lata que possui raio maior, V1, é 1,6 vezes a medida do
• Pote III: raio 2a e altura 2b volume da lata que possui raio menor, V 2 .
• Pote IV: raio 4a e altura b
3 cm
• Pote V: raio 4a e altura 2b
O pote que satisfaz a condição imposta pelo cliente é o
A I.
B II.
6 cm x
C III.
D IV.
4 cm

E V.

Disponível em: www.cbra.org.br. Acesso em: 3 mar. 2012.

A medida da altura desconhecida vale


A 8 cm.
B 10 cm.
C 16 cm.
D 20 cm.
E 40 cm.

MT - 2º dia | Caderno 15 - CINZA - Página 29


*CZ1525dom30* 2015

QUESTÃO 173 QUESTÃO 174

Uma empresa que embala seus produtos em caixas Um técnico precisa consertar o termostato do aparelho
de papelão, na forma de hexaedro regular, deseja que de ar-condicionado de um escritório, que está desregulado.
seu logotipo seja impresso nas faces opostas pintadas de A temperatura T, em graus Celsius, no escritório, varia de
cinza, conforme a figura:
acordo com a função T (h) = A + B sen (π
12
(h - 12) , )
sendo h o tempo, medido em horas, a partir da meia-noite
(0 < h < 24) e A e B os parâmetros que o técnico precisa
regular. Os funcionários do escritório pediram que a
temperatura máxima fosse 26°C, a mínima 18°C, e
que durante a tarde a temperatura fosse menor do que
durante a manhã.
A gráfica que fará as impressões dos logotipos
apresentou as seguintes sugestões planificadas: Quais devem ser os valores de A e de B para que o pedido
dos funcionários seja atendido?
A A = 18 e B = 8
B A = 22 e B = -4
I
C A = 22 e B = 4
D A = 26 e B = -8
E A = 26 e B = 8

QUESTÃO 175
II
Num campeonato de futebol de 2012, um time sagrou-se
campeão com um total de 77 pontos (P) em 38 jogos,
tendo 22 vitórias (V), 11 empates (E) e 5 derrotas (D).
No critério adotado para esse ano, somente as vitórias e
empates têm pontuações positivas e inteiras. As derrotas
têm valor zero e o valor de cada vitória é maior que o valor
III de cada empate.
Um torcedor, considerando a fórmula da soma de
pontos injusta, propôs aos organizadores do campeonato
que, para o ano de 2013, o time derrotado em cada partida
perca 2 pontos, privilegiando os times que perdem menos
ao longo do campeonato. Cada vitória e cada empate
IV continuariam com a mesma pontuação de 2012.
Qual a expressão que fornece a quantidade de pontos
(P), em função do número de vitórias (V), do número de
empates (E) e do número de derrotas (D), no sistema de
pontuação proposto pelo torcedor para o ano de 2013?
A P = 3V + E
V B P = 3V - 2D
C P = 3V + E - D
D P = 3V + E - 2D
Que opção sugerida pela gráfica atende ao desejo da E P = 3V + E + 2D
empresa?
A I
B II
C III
D IV
E V

MT - 2º dia | Caderno 15 - CINZA - Página 30


2015 *CZ1525dom31*
QUESTÃO 176 QUESTÃO 179
O gráfico mostra a variação percentual do valor do Cinco amigos marcaram uma viagem à praia em
Produto Interno Bruto (PIB) do Brasil, por trimestre, em dezembro. Para economizar, combinaram de ir num
relação ao trimestre anterior: único carro. Cada amigo anotou quantos quilômetros seu
2,57 carro fez, em média, por litro de gasolina, nos meses de
2,45 setembro, outubro e novembro. Ao final desse trimestre,
calcularam a média dos três valores obtidos para
escolherem o carro mais econômico, ou seja, o que teve
2,19
a maior média. Os dados estão representados na tabela:
1,87 1,59
1,1 Desempenho médio mensal (km/litro)
Carro
0,39 0,75 Setembro Outubro Novembro
-1,92 I 6,2 9,0 9,3
1º trimestre 2º trimestre 3º trimestre 4º trimestre 1º trimestre 2º trimestre 3º trimestre 4º trimestre 1º trimestre
2009 2010 2011 II 6,7 6,8 9,5
Disponível em: www.ibge.gov.br. Acesso em: 6 ago. 2012.
III 8,3 8,7 9,0
De acordo com o gráfico, no período considerado, o IV 8,5 7,5 8,5
trimestre em que o Brasil teve o maior valor do PIB foi o
V 8,0 8,0 8,0
A segundo trimestre de 2009.
B quarto trimestre de 2009. Qual carro os amigos deverão escolher para a viagem?
C terceiro trimestre de 2010. A I
D quarto trimestre de 2010. B II
E primeiro trimestre de 2011. C III
QUESTÃO 177 D IV
A Organização Mundial da Saúde (OMS) recomenda E V
que o consumo diário de sal de cozinha não exceda 5 g.
Sabe-se que o sal de cozinha é composto por 40% de QUESTÃO 180
sódio e 60% de cloro. Um paciente precisa ser submetido a um
Disponível em: http://portal.saude.gov.br. Acesso em: 29 fev. 2012 (adaptado).
tratamento, sob orientação médica, com determinado
Qual é a quantidade máxima de sódio proveniente do sal medicamento. Há cinco possibilidades de medicação,
de cozinha, recomendada pela OMS, que uma pessoa variando a dosagem e o intervalo de ingestão do
pode ingerir por dia? medicamento. As opções apresentadas são:
A 1 250 mg A: um comprimido de 400 mg, de 3 em 3 horas,
B 2 000 mg durante 1 semana;
C 3 000 mg
B: um comprimido de 400 mg, de 4 em 4 horas,
D 5 000 mg durante 10 dias;
E 12 500 mg
C: um comprimido de 400 mg, de 6 em 6 horas,
QUESTÃO 178 durante 2 semanas;
Um fornecedor vendia caixas de leite a um D: um comprimido de 500 mg, de 8 em 8 horas,
supermercado por R$ 1,50 a unidade. O supermercado durante 10 dias;
costumava comprar 3 000 caixas de leite por mês desse
E: um comprimido de 500 mg, de 12 em 12 horas,
fornecedor. Uma forte seca, ocorrida na região onde o
leite é produzido, forçou o fornecedor a encarecer o preço durante 2 semanas.
de venda em 40%. O supermercado decidiu então cortar Para evitar efeitos colaterais e intoxicação, a
em 20% a compra mensal dessas caixas de leite. Após recomendação é que a quantidade total de massa
essas mudanças, o fornecedor verificou que sua receita da medicação ingerida, em miligramas, seja a menor
nas vendas ao supermercado tinha aumentado. possível.
O aumento da receita nas vendas do fornecedor, em Seguindo a recomendação, deve ser escolhida a opção
reais, foi de
A 540. A A.
B 600. B B.
C 900. C C.
D 1 260. D D.
E 1 500. E E.
MT - 2º dia | Caderno 15 - CINZA - Página 31
*CZ1525dom32* 2015

2015

Transcreva a sua Redação para a Folha de Redação.

O
8

H
9

N O
10

U Ã
11

C Ç
12

S
13

A
14

A D
15

R RE
16

17

A
18

D
19

20

21

22

23

24

25

26

27

28

29

30

LC - 2º dia | Caderno 15 - CINZA - Página 32


EXAME NACIONAL DO ENSINO MÉDIO
PROVA DE CIÊNCIAS HUMANAS E SUAS TECNOLOGIAS
PROVA DE CIÊNCIAS DA NATUREZA E SUAS TECNOLOGIAS

1º DIA
CADERNO

2016 9
3ª APLICAÇÃO

ATENÇÃO: transcreva no espaço apropriado do seu CARTÃO-RESPOSTA,


FRPVXDFDOLJUD¿DXVXDOFRQVLGHUDQGRDVOHWUDVPDL~VFXODVHPLQ~VFXODVDVHJXLQWHIUDVH

E fui cuidar do que restava.

LEIA ATENTAMENTE AS INSTRUÇÕES SEGUINTES: 3. Para cada XPDGDVTXHVW}HVREMHWLYDVVmRDSUHVHQWDGDV


RSo}HV$SHQDVXPDUHVSRQGHFRUUHWDPHQWHjTXHVWmR
1. Este CADERNO DE QUESTÕES contém 90 questões
4. O tempo disponível para estas provas é de quatro horas e
numeradas de 1 a 90, dispostas da seguinte maneira:
trinta minutos.
D  DVTXHVW}HVGHQ~PHURDVmRUHODWLYDVjiUHDGH 5. 5HVHUYHRVPLQXWRV¿QDLVSDUDPDUFDUVHXCARTÃO-RESPOSTA.
Ciências Humanas e suas Tecnologias; Os rascunhos e as marcações assinaladas no CADERNO DE
E  DVTXHVW}HVGHQ~PHURDVmRUHODWLYDVjiUHDGH 48(67®(6QmRVHUmRFRQVLGHUDGRVQDDYDOLDomR
Ciências da Natureza e suas Tecnologias. 6. Quando terminar as provas, acene para chamar o
2. &RQ¿UD VH R VHX &$'(512 '( 48(67®(6 FRQWpP D aplicador e entregue este CADERNO DE QUESTÕES e
TXDQWLGDGHGHTXHVW}HVHVHHVVDVTXHVW}HVHVWmRQDRUGHP o CARTÃO-RESPOSTA.
PHQFLRQDGD QD LQVWUXomR DQWHULRU &DVR R FDGHUQR HVWHMD 7. 9RFrSRGHUiGHL[DURORFDOGHSURYDVRPHQWHDSyVGHFRUULGDV
LQFRPSOHWR WHQKD GHIHLWR RX DSUHVHQWH TXDOTXHU GLYHUJrQFLD GXDV KRUDV GR LQtFLR GD DSOLFDomR H SRGHUi OHYDU VHX
comunique ao aplicador da sala para que ele tome as &$'(512'(48(67®(6DRGHL[DUHPGH¿QLWLYRDVDODGH
providências cabíveis. prova nos 30 minutos que antecedem o término das provas.

Ministério
da Educação *BR0975SAB1*
*BR0975SAB2* 2016

CIÊNCIAS HUMANAS E SUAS TECNOLOGIAS


Questões de 01 a 45
QUESTÃO 01
2VJDUJDORVURGRYLiULRVGR%UDVLOHRFDyWLFRWUkQVLWRGDVVXDVPHWUySROHVIRUoDPRVJRYHUQRVHVWDGXDLVHIHGHUDO
DUHWRPDURVSODQRVGHLPSODQWDomRGRVWUHQVUHJLRQDLV'XUDQWHDV~OWLPDVTXDWURGpFDGDVDPDOKDIHUURYLiULDIRL
HVTXHFLGDHVXFDWHDGDWDQWRTXHKRMHHPWRGRRSDtVDSHQDVGXDVOLQKDVGHSDVVDJHLURVHVWmRHPIXQFLRQDPHQWR
7UDQVSRUWDPPLOKmRGHSHVVRDVHQWUH%HOR+RUL]RQWH 0* H9LWyULD (6 HHQWUH6mR/XtV 0$ H&DUDMiV 3$ ʊDV
GXDVRSHUDGDVSHODPLQHUDGRUD9DOH1RVDQRVPDLVGHPLOK}HVGHSDVVDJHLURVXWLOL]DYDPWUHQVLQWHUXUEDQRV
QRWHUULWyULRQDFLRQDO
'LVSRQtYHOHPZZZHVWDGDRFRPEU$FHVVRHPVHW

2VXFDWHDPHQWRGRPHLRGHWUDQVSRUWHGHVFULWRIRLSURYRFDGRSHOD
A redução da demanda populacional por trens interurbanos.
B LQDGHTXDomRGRVWUDMHWRVHPIXQomRGDH[WHQVmRGRSDtV
C SUHFDUL]DomRWHFQROyJLFDIUHQWHDRXWURVPHLRVGHGHVORFDPHQWR
D SULRUL]DomRGDPDOKDURGRYLiULDQRSHUtRGRGHPRGHUQL]DomRGRHVSDoR
E DPSOLDomRGRVSUREOHPDVDPELHQWDLVDVVRFLDGRVjFRQVHUYDomRGDVIHUURYLDV.
QUESTÃO 02

'$+0(5$'LVSRQtYHOHPKWWSPDOYDGRVZRUGSUHVVFRP$FHVVRHPGH]

$QDOLVDURSURFHVVRDWXDOGHFLUFXODomRHGHDUPD]HQDPHQWRGHGHWHUPLQDGRVEHQVFXOWXUDLVGLDQWHGDWUDQVIRUPDomR
GHFRUUHQWHGRLPSDFWRGHQRYDVWHFQRORJLDVLQGLFDTXHKRMH
A DVP~VLFDVHRVWH[WRVWrPSULYLOHJLDGRXPIRUPDWRGLJLWDOWRUQDQGRLQDGPLVVtYHOVXDDFXPXODomR
B DUHGHPXQGLDOGHFRPSXWDGRUHVDFDEDFRPRFKDPDGRGLUHLWRDXWRUDOTXHpLQDSOLFiYHOHPUHODo}HVYLUWXDLV
C DVHJXUDQoDHDLQFOXVmRGLJLWDOVmRSUREOHPDVH[SRQGRDLPSRVVLELOLGDGHGHUHDOL]DUXPFRPpUFLRIHLWRon-line.
D DVPtGLDVGLJLWDLVHDLQWHUQHWSHUPLWLUDPPDLRUÀX[RGHVVHVSURGXWRVSRLVVHXDF~PXORLQGHSHQGHGHJUDQGHV
bases materiais.
E DSLUDWDULDpRUHFXUVRXWLOL]DGRSHORVFRQVXPLGRUHVYLVWRTXHVmRLPSHGLGRVGHDGTXLULUOHJDOPHQWHDOJRGHVSURYLGR
GHVXSRUWHItsico.
&+žGLD_&DGHUQR%5$1&23iJLQD
2016 *BR0975SAB3*
QUESTÃO 03 QUESTÃO 05

$ FHQD GH WmR FRWLGLDQD Mi QmR FDXVD PDLV 'H DOFDQFH QDFLRQDO R 0RYLPHQWR GRV
HVWUDQKH]DD,VDEHO6ZDQ$RERWDUREDUFRQDViJXDVGD 7UDEDOKDGRUHV 5XUDLV 6HP 7HUUD 067  UHSUHVHQWD
%DtDGH*XDQDEDUDDYHOHMDGRUDSUHFLVDVHGHVYHQFLOKDU D LQFRUSRUDomR j YLGD SROtWLFD GH SDUFHOD LPSRUWDQWH
GH VDFRV SOiVWLFRV WDPSLQKDV GH UHIULJHUDQWHV ODWDV GD SRSXODomR WUDGLFLRQDOPHQWH H[FOXtGD SHOD IRUoD
GR ODWLI~QGLR 0LOKDUHV GH WUDEDOKDGRUHV UXUDLV VH
palitos de sorvete. Um dos cartões-postais cariocas
organizaram e pressionaram o governo em busca
recebe diariamente uma média de cem toneladas de GH WHUUD SDUD FXOWLYDU H GH ¿QDQFLDPHQWR GH VDIUDV
OL[R ÀXWXDQWH FDUUHJDGR SHORV ULRV TXH FRUWDP D UHJLmR 6HXVPpWRGRVʊDLQYDVmRGHWHUUDVS~EOLFDVRXQmR
metropolitana do Rio de Janeiro. FXOWLYDGDV ʊ WDQJHQFLDP D LOHJDOLGDGH PDV WHQGR
$/(1&$5(7RQHODGDVGHGHVFDVRO GloboDEU DGDSWDGR  HPYLVWDDRSUHVVmRVHFXODUGHTXHIRUDPYtWLPDVHD
H[WUHPDOHQWLGmRGRVJRYHUQRVHPUHVROYHURSUREOHPD
2SUREOHPDDPELHQWDOGHVFULWRWHPVXDFDXVDDVVRFLDGDj DJUiULRSRGHPVHUFRQVLGHUDGRVOHJtWLPRV
A LQH¿FLrQFLDGHHFREDUUHLUDV &$59$/+2-0Cidadania no Brasil: o longo caminho.
5LRGH-DQHLUR&LYLOL]DomR%UDVLOHLUD DGDSWDGR 
B GHVRUJDQL]DomRGRWXULVPRORFDO
Argumenta-se que as reivindicações apresentadas por
C LQDGHTXDomRGDFROHWDGRPLFLOLDU PRYLPHQWRVVRFLDLVFRPRRGHVFULWRQRWH[WRWrPFRPR
D PRYLPHQWDomRGDViUHDVSRUWXiULDV REMHWLYRFRQWULEXLUSDUDRSURFHVVRGH
E UDUHIDomRGDRFXSDomRSRSXODFLRQDO A LQRYDomRLQVWLWXFLRQDO
B RUJDQL]DomRSDUWLGiULD
QUESTÃO 04 C UHQRYDomRSDUODPHQWDU
D HVWDWL]DomRGDSURSULHGDGH
E GHPRFUDWL]DomRGRVLVWHPD

QUESTÃO 06
7HQGRVHOLYUDGRGRHQWXOKRGRPDTXLQiULRYROXPRVR
H GDV HQRUPHV HTXLSHV GH IiEULFD R FDSLWDO YLDMD OHYH
DSHQDV FRP D EDJDJHP GH PmR SDVWD FRPSXWDGRU
SRUWiWLOHWHOHIRQHFHOXODU2QRYRDWULEXWRGDYRODWLOLGDGH
IH]GHWRGRFRPSURPLVVRHVSHFLDOPHQWHGRFRPSURPLVVR
HVWiYHO DOJR DR PHVPR WHPSR UHGXQGDQWH H SRXFR
inteligente: seu estabelecimento paralisaria o movimento
HIXJLULDGDGHVHMDGDFRPSHWLWLYLGDGHUHGX]LQGRa priori as
opções que poderiam levar ao aumento da produtividade.
%$80$1=Modernidade líquida5LRGH-DQHLUR=DKDU
'LVSRQtYHOHPZZZFXOWXUDEDJRYEU$FHVVRHPMDQ
1RWH[WRID]VHUHIHUrQFLDDXPSURFHVVRGHWUDQVIRUPDomR
$LPDJHPUHWUDWDXPDSUiWLFDFXOWXUDOEUDVLOHLUDFXMDUDL] GRPXQGRSURGXWLYRFXMDFRQVHTXrQFLDpR D
KLVWyULFDHVWiDVVRFLDGDj A UHJXODPHQWDomRGHOHLVWUDEDOKLVWDVPDLVUtJLGDV
A liberdade religiosa. B IUDJLOL]DomRGDVUHODo}HVKLHUiUTXLFDVGHWUDEDOKR
B PLJUDomRIRUoDGD C GHFUpVFLPRGRQ~PHURGHIXQFLRQiULRVGDVHPSUHVDV
C GHYRomRHFXPrQLFD D incentivo ao investimento de longos planos de
carreiras.
D DWLYLGDGHPLVVLRQiULD
E GHVYDORUL]DomR GRV SRVWRV GH JHUHQFLDPHQWR
E PRELOL]DomRSROtWLFD corporativo.

&+žGLD_&DGHUQR%5$1&23iJLQD
*BR0975SAB4* 2016

QUESTÃO 07 QUESTÃO 09
2V HVFUDYRV WRUQDPVH SURSULHGDGH QRVVD VHMD HP $H[SHULrQFLDGRPRYLPHQWRRUJDQL]DGRGHPXOKHUHV
YLUWXGH GD OHL FLYLO VHMD GD OHL FRPXP GRV SRYRV HP QR %UDVLO RIHUHFH H[FHOHQWH H[HPSOR GH FRPR VH SRGH
virtude da lei civil, se qualquer pessoa de mais de vinte XWLOL]DU D OHL HP IDYRU GD PHOKRULD GR status MXUtGLFR GD
DQRV SHUPLWLU D YHQGD GH VL SUySULD FRP D ¿QDOLGDGH GH FRQGLomRVRFLDOGRDYDQoRQRVHQWLGRGHXPDSUHVHQoD
lucrar conservando uma parte do preço da compra; e em PDLVHIHWLYDQRSURFHVVRGHGHFLVmRSROtWLFD$RORQJRGH
YLUWXGH GD OHL FRPXP GRV SRYRV VmR QRVVRV HVFUDYRV quase todo o século XX, com mais intensidade em algumas
DTXHOHVTXHIRUDPFDSWXUDGRVQDJXHUUDHDTXHOHVTXH décadas do que em outras, as mulheres brasileiras
VmR¿OKRVGHQRVVDVHVFUDYDV FRQVHJXLUDP REWHU YLWyULDV H[SUHVVLYDV$OJXPDV YH]HV
CARDOSO, C. F. Trabalho compulsório na Antiguidade6mR3DXOR*UDDO
DEROLQGR GLVSRVLWLYRV OHJDLV GLVFULPLQDWyULRV RXWUDV
conseguindo aprovar novas leis.
A obra InstitutasGRMXULVWD$HOLXV0DUFLDQXV VpFXOR,,,G&  TABAK, F. A lei como instrumento de mudança social. In: TABAK, F.; VERUCCI, F.
LQVWUXL VREUH D HVFUDYLGmR QD 5RPD DQWLJD 1RGLUHLWR A difícil igualdade: os direitos da mulher como direitos humanos.
5LRGH-DQHLUR5HOXPH'XPDUi
e na sociedade romana desse período, os escravos
compunham uma $ DWXDomR GR PRYLPHQWR VRFLDO DERUGDGR QR WH[WR
A PmRGHREUDHVSHFLDOL]DGDSURWHJLGDSHODOHL resultou, na década de 1930, em
B IRUoDGHWUDEDOKRVHPDSUHVHQoDGHH[FLGDGmRV A direito de voto.
C categoria de trabalhadores oriundos dos mesmos B garantia de cotas.
povos. C acesso ao trabalho.
D FRQGLomR OHJDO LQGHSHQGHQWH GD RULJHP pWQLFD GR D RUJDQL]DomRSDUWLGiULD
indivíduo. E igualdade de oportunidades.
E comunidade criada a partir do estabelecimento das
leis escritas. QUESTÃO 10

QUESTÃO 08 1R DQLYHUViULR GR SULPHLUR GHFrQLR GD 0DUFKD VREUH


5RPD HP RXWXEUR GH  0XVVROLQL LUi LQDXJXUDU VXD
Estamos, pois, de acordo quando, ao ver algum Via dell Impero; a nova Via Sacra do Fascismo, ornada
REMHWRGL]HPRV³(VWHREMHWRTXHHVWRXYHQGRDJRUDWHP FRPHVWiWXDVGH&pVDU$XJXVWR7UDMDQRVHUYLUiDRFXOWR
tendência para assemelhar-se a um outro ser, mas, por GR DQWLJR H j JOyULD GR ,PSpULR 5RPDQR H GH HVSDoR
WHUGHIHLWRVQmRFRQVHJXHVHUWDOFRPRRVHUHPTXHVWmR FRPHPRUDWLYRGRXIDQLVPRLWDOLDQR¬VVRPEUDVGRSDVVDGR
H OKH p SHOR FRQWUiULR LQIHULRU´ $VVLP SDUD SRGHUPRV recriado ergue-se a nova Roma, que pode vangloriar-se e
ID]HUHVWDVUHÀH[}HVpQHFHVViULRTXHDQWHVWHQKDPRV FHOHEUDUVHXVLPSHUDGRUHVHKRPHQVIRUWHVVHXVJUDQGHV
WLGRRFDVLmRGHFRQKHFHUHVVHVHUGHTXHVHDSUR[LPDR SRHWDVHDSyORJRVFRPR+RUiFLRH9LUJtOLR
GLWRREMHWRDLQGDTXHLPSHUIHLWDPHQWH 6,/9$*História antiga e usos do passado: um estudo de apropriações da Antiguidade
VRERUHJLPHGH9LFK\6mR3DXOR$QQDEOXPH DGDSWDGR 
3/$7­2Fédon6mR3DXOR$EULO&XOWXUDO
$UHWRPDGDGD$QWLJXLGDGHFOiVVLFDSHODSHUVSHFWLYDGR
Na epistemologia platônica, conhecer um determinado SDWULP{QLRFXOWXUDOIRLUHDOL]DGDFRPRREMHWLYRGH
REMHWRLPSOLFD
A D¿UPDU R LGHiULR FULVWmR SDUD UHFRQTXLVWDU D
A estabelecer semelhanças entre o que é observado grandeza perdida.
em momentos distintos. B XWLOL]DU RV YHVWtJLRV UHVWDXUDGRV SDUD MXVWL¿FDU R
B FRPSDUDU R REMHWR REVHUYDGR FRP XPD GHVFULomR regime político.
detalhada dele. C GLIXQGLU RV VDEHUHV DQFHVWUDLV SDUD PRUDOL]DU RV
C GHVFUHYHU FRUUHWDPHQWH DV FDUDFWHUtVWLFDV GR REMHWR costumes sociais.
observado. D UHID]HU R XUEDQLVPR FOiVVLFR SDUD IDYRUHFHU D
D ID]HU FRUUHVSRQGrQFLD HQWUH R REMHWR REVHUYDGR H SDUWLFLSDomRSROtWLFD
seu ser. E UHFRPSRUDRUJDQL]DomRUHSXEOLFDQDSDUDIRUWDOHFHUD
E LGHQWL¿FDURXWURH[HPSODULGrQWLFRDRREVHUYDGR DGPLQLVWUDomRHVWDWDO

&+žGLD_&DGHUQR%5$1&23iJLQD
2016 *BR0975SAB5*
QUESTÃO 11 QUESTÃO 13

2V GLDV GR 1X FRPR XP GRV ~OWLPRV ULRV GH FXUVR Enquanto o pensamento de Santo Agostinho
OLYUH GD UHJLmR HVWmR WHUPLQDQGR 2 JRYHUQR FKLQrV UHSUHVHQWD R GHVHQYROYLPHQWR GH XPD ¿ORVR¿D FULVWm
surpreendeu ambientalistas este ano ao reavivar planos LQVSLUDGD HP 3ODWmR R SHQVDPHQWR GH 6mR 7RPiV
GH FRQVWUXLU XVLQDV KLGUHOpWULFDV HP iUHDV UHPRWDV GR UHDELOLWD D ¿ORVR¿D GH $ULVWyWHOHV ʊ DWp HQWmR YLVWD
VRE VXVSHLWD SHOD ,JUHMD ʊ PRVWUDQGR VHU SRVVtYHO
FXUVRVXSHULRUGR1XRFHQWURGHXP3DWULP{QLR0XQGLDO
GHVHQYROYHU XPD OHLWXUD GH$ULVWyWHOHV FRPSDWtYHO FRP
da Unesco na província de Yunnan, sudoeste da China,
D GRXWULQD FULVWm 2 DULVWRWHOLVPR GH 6mR 7RPiV DEULX
TXH VH FODVVL¿FD HQWUH RV OXJDUHV HFRORJLFDPHQWH PDLV caminho para o estudo da obra aristotélica e para a
GLYHUVL¿FDGRVHIUiJHLVGRPXQGR2VFUtWLFRVGL]HPTXHR OHJLWLPDomR GR LQWHUHVVH SHODV FLrQFLDV QDWXUDLV XP
SURMHWRREULJDUiDUHPDQHMDUGH]HQDVGHPLQRULDVpWQLFDV GRV SULQFLSDLV PRWLYRV GR LQWHUHVVH SRU $ULVWyWHOHV
QRVSODQDOWRVGH<XQQDQHGHVWUXLUiRVFDPSRVGHGHVRYD nesse período.
GHGH]HQDVGHHVSpFLHVGHSHL[HVDPHDoDGDV 0$5&21'(6'7H[WRVEiVLFRVGH¿ORVR¿D5LRGH-DQHLUR=DKDU

'LVSRQtYHOHPZZZIROKDXROFRPEU$FHVVRHPPDLR DGDSWDGR 
$,JUHMD&DWyOLFDSRUPXLWRWHPSRLPSHGLXDGLYXOJDomRGD
(VVH SURMHWR VLQDOL]D XPD LQWHUIHUrQFLD QR PHLR ItVLFR REUDGH$ULVWyWHOHVSHORIDWRGHDREUDDULVWRWpOLFD
PRWLYDGDSHOR D A YDORUL]DUDLQYHVWLJDomRFLHQWt¿FDFRQWUDULDQGRFHUWRV
A EXVFDGRVHWRUSULPiULRSRULQIUDHVWUXWXUD dogmas religiosos.
B GHFODUDUDLQH[LVWrQFLDGH'HXVFRORFDQGRHPG~YLGD
B GHPDQGDGDSRSXODomRSRUHQHUJLDVOLPSDV
toda a moral religiosa.
C LQWHUHVVH GR (VWDGR HP GLYHUVL¿FDU D PDWUL]
C FULWLFDU D ,JUHMD &DWyOLFD LQVWLJDQGR D FULDomR GH
energética. outras instituições religiosas.
D QHFHVVLGDGH GRV FHQWURV XUEDQRV HP REWHU iJXD D evocar pensamentos de religiões orientais, minando a
SRWiYHO H[SDQVmRGRFULVWLDQLVPR
E compromisso da iniciativa privada com o E contribuir para o desenvolvimento de sentimentos
GHVHQYROYLPHQWRVXVWHQWiYHO antirreligiosos, seguindo sua teoria política.

QUESTÃO 12 QUESTÃO 14

$ /HL GR 6LVWHPD 1DFLRQDO GH 8QLGDGHV GH $LPSRUWkQFLDGRDUJXPHQWRGH+REEHVHVWiHPSDUWH


&RQVHUYDomR VXUJH GH XP FRQÀLWR PXLWR VpULR GH QR IDWR GH TXH HOH VH DPSDUD HP VXSRVLo}HV EDVWDQWH
LQWHUHVVHVGHXPODGRDDWLYLGDGHLOLPLWDGDHH[SDQVLYDGH plausíveis sobre as condições normais da vida humana.
3DUD H[HPSOL¿FDU R DUJXPHQWR QmR VXS}H TXH WRGRV
H[SORUDomRGHUHFXUVRVQDWXUDLVGHRXWURDQHFHVVLGDGH
VHMDPGHIDWRPRYLGRVSRURUJXOKRHYDLGDGHSDUDEXVFDU
GH JDUDQWLU D PDQXWHQomR GDV EDVHV QDWXUDLV SDUD D
R GRPtQLR VREUH RV RXWURV HVVD VHULD XPD VXSRVLomR
H[LVWrQFLD GR KRPHP H SDUD D SUySULD FRQWLQXLGDGH GD GLVFXWtYHO TXH SRVVLELOLWDULD D FRQFOXVmR SUHWHQGLGD
DWLYLGDGHHFRQ{PLFDH[SDQVLYDTXHVHTXHUUHSUHVDU SRU +REEHV PDV GH PRGR IiFLO GHPDLV 2 TXH WRUQD R
52'5,*8(6-(56LVWHPD1DFLRQDOGH8QLGDGHVGH&RQVHUYDomR DUJXPHQWR DVVXVWDGRU H OKH DWULEXL LPSRUWkQFLD H IRUoD
Revista dos Tribunais GUDPiWLFD p TXH HOH DFUHGLWD TXH SHVVRDV QRUPDLV DWp
PHVPRDVPDLVDJUDGiYHLVSRGHPVHULQDGYHUWLGDPHQWH
$ GLYHUVLGDGH QD FODVVL¿FDomR GDV XQLGDGHV GH
ODQoDGDVQHVVHWLSRGHVLWXDomRTXHUHVYDODUiHQWmRHP
FRQVHUYDomRGH¿QLGDVSHODOHLUHYHODDH[LVWrQFLDGHXP um estado de guerra.
impasse, pois
5$:/6-&RQIHUrQFLDVVREUHDKLVWyULDGD¿ORVR¿DSROtWLFD.
A UHVWULQJHRXVRGDSRSXODomRORFDOjIXQomRWXUtVWLFD 6mR3DXOR:0) DGDSWDGR 

B amplia as possibilidades do termo desenvolvimento 2 WH[WR DSUHVHQWD XPD FRQFHSomR GH ¿ORVR¿D SROtWLFD
VXVWHQWiYHO conhecida como
C UHIRUoDDOyJLFDGDSUHVHUYDomRGRVUHFXUVRVQDWXUDLV A DOLHQDomRLGHROyJLFD
D devolve a gerência desses espaços para o poder B PLFURItVLFDGRSRGHU
S~EOLFR C estado de natureza.
E JDUDQWH D SULRULGDGH GD FULDomR GH QRYDV iUHDV QR D contrato social.
espaço rural. E vontade geral.

&+žGLD_&DGHUQR%5$1&23iJLQD
*BR0975SAB6* 2016

QUESTÃO 15

48,120DIDOGD'LVSRQtYHOHPZZZQRYDDFURSROHSW$FHVVRHPIHY

$¿JXUDGRLQTXLOLQRDRTXDODSHUVRQDJHPGDWLULQKDVHUHIHUHpR D
A FRQVWUDQJLPHQWRSRUROKDUHVGHUHSURYDomR
B FRVWXPHLPSRVWRDRV¿OKRVSRUFRDomR
C FRQVFLrQFLDGDREULJDomRPRUDO
D pessoa habitante da mesma casa.
E temor de possível castigo.

QUESTÃO 16

Flor da negritude
1DVFLGRQXPDFDVDDQWLJDSHTXHQDFRPJUDQGHTXLQWDODUERUL]DGRORFDOL]DGDQRVXE~UELRGH/LQVGH9DVFRQFHORV
R5HQDVFHQoD&OXEHIRLIXQGDGRSRUVyFLRVWRGRVQHJURV%XVFDYDVHLQVWDXUDUSRUPHLRGR5HQDVFHQoDXP
FDPSRGHUHODo}HVHPTXHRV¿OKRVGHIDPtOLDVQHJUDVEHPVXFHGLGDVSXGHVVHPHQFRQWUDUSHVVRDVFRQVLGHUDGDV
GRPHVPRQtYHOVRFLDOHFXOWXUDOSDUD¿QVGHDPL]DGHRXFDVDPHQWR2VKRPHQVXVDYDPWUDMHVREULJDWRULDPHQWH
IRUPDLVÀRUHVQDODSHODjVYH]HVGHsummerRXDWpGHIUDTXH$VPXOKHUHVVHYHVWLDPFRPPXLWDVVHGDVFHWLQVH
UHQGDVQmRHVTXHFHQGRDVOXYDVHRVFKDSpXV
*,$&20,1,60Revista de História da Biblioteca NacionalVHW DGDSWDGR 

1RLQtFLRGRVDQRVDIXQGDomRGR5HQDVFHQoD&OXEHFRPRHVSDoRGHFRQYLYrQFLDGHPRQVWUDR D
A LQH[SHULrQFLDDVVRFLDWLYDTXHOHYRXDHOLWHQHJUDDLPLWDURVFOXEHVGRVEUDQFRV
B isolamento da comunidade destacada que ignorava a democracia racial brasileira.
C LQWHUHVVHGHXPJUXSRGHQHJURVQDD¿UPDomRVRFLDOSDUDVHOLYUDUGRSUHFRQFHLWR
D H[LVWrQFLDGHXPDHOLWHQHJUDLPXQHDRSUHFRQFHLWRSHODSRVLomRVRFLDOTXHRFXSDYD
E FULDomRGHXPUDFLVPRLQYHUWLGRTXHLPSHGLDDSUHVHQoDGHSHVVRDVEUDQFDVQHVVHVFOXEHV

QUESTÃO 17
8PD IiEULFD QD TXDO RV RSHUiULRV IRVVHP HIHWLYD H LQWHJUDOPHQWH VLPSOHV SHoDV GH PiTXLQDV H[HFXWDQGR
FHJDPHQWH DV RUGHQV GD GLUHomR SDUDULD HP TXLQ]H PLQXWRV 2 FDSLWDOLVPR Vy SRGH IXQFLRQDU FRP D FRQWULEXLomR
FRQVWDQWHGDDWLYLGDGHSURSULDPHQWHKXPDQDGHVHXVVXEMXJDGRVTXHDRPHVPRWHPSRWHQWDUHGX]LUHGHVXPDQL]DU
o mais possível.
CASTORIADIS, C. A instituição imaginária da sociedade5LRGH-DQHLUR3D]H7HUUD

2WH[WRGHVWDFDDOpPGDGLQkPLFDPDWHULDOGRFDSLWDOLVPRDLPSRUWkQFLDGDGLPHQVmRVLPEyOLFDGDVRFLHGDGHTXH
consiste em
A HODERUDUVLJQL¿FDo}HVHYDORUHVQRPXQGRSDUDGRWiORGHXPVHQWLGRTXHWUDQVFHQGHDFRQFUHWXGHGDYLGD
B HVWDEHOHFHUUHODo}HVO~GLFDVHQWUHDYLGDHDUHDOLGDGHVHPDSUHWHQVmRGHWUDQVIRUPDURPXQGRGRVKRPHQV
C DWXDUVREUHDYLYrQFLDUHDOHPRGL¿FiODSDUDHVWDEHOHFHUUHODo}HVLQWHUSHVVRDLVEDVHDGDVQRLQWHUHVVHP~WXR
D FULDU GLVFXUVRV GHVWLQDGRV D H[HUFHU R FRQYHQFLPHQWR VREUH DXGLrQFLDV LQGHSHQGHQWHPHQWH GDV SRVLo}HV
GHIHQGLGDV
E GHIHQGHUDFDULGDGHFRPRUHDOL]DomRSHVVRDOSRUPHLRGHSUiWLFDVDVVLVWHQFLDLVQDGHIHVDGRVPHQRVIDYRUHFLGRV

&+žGLD_&DGHUQR%5$1&23iJLQD
2016 *BR0975SAB7*
QUESTÃO 18 QUESTÃO 20
4XDQGRUHÀHWLPRVVREUHDTXHVWmRGDMXVWLoDDOJXPDV
DVVRFLDo}HV VmR IHLWDV TXDVH LQWXLWLYDPHQWH WDLV FRPR
a de equilíbrio entre as partes, princípio de igualdade,
GLVWULEXLomR HTXLWDWLYD PDV ORJR DV GL¿FXOGDGHV VH
mostram. Isso porque a nossa sociedade, sendo bastante
GLYHUVL¿FDGD DSUHVHQWD XPD KHWHURJHQHLGDGH WDQWR HP
WHUPRVGDVGLYHUVDVFXOWXUDVTXHFRH[LVWHPHPXPPXQGR
LQWHUOLJDGR FRPR HP UHODomR DRV PRGRV GH YLGD H DRV
valores que surgem no interior de uma mesma sociedade.
&+(',$..$SOXUDOLGDGHFRPRLGHLDUHJXODGRUDDQRomRGHMXVWLoDDSDUWLUGD¿ORVR¿DGH
/\RWDUGTrans/Form/AçãoQ DGDSWDGR 

$ UHODomR HQWUH MXVWLoD H SOXUDOLGDGH DSUHVHQWDGD SHOD


DXWRUDHVWiLQGLFDGDHP
A $ FRPSOH[LGDGH GD VRFLHGDGH OLPLWD R H[HUFtFLR GD
MXVWLoD H D LPSHGH GH DWXDU D IDYRU GD GLYHUVLGDGH
cultural.
B $ GLYHUVLGDGH FXOWXUDO H GH YDORUHV WRUQD D MXVWLoD
PDLV FRPSOH[D H GLVWDQWH GH XP SDUkPHWUR JHUDO
orientador.
C 2 SDSHO GD MXVWLoD UHIHUHVH j PDQXWHQomR GH
SULQFtSLRV ¿[RV H LQFRQGLFLRQDLV HP IXQomR GD
diversidade cultural e de valores.
D 2 SUHVVXSRVWR GD MXVWLoD p IRPHQWDU R FULWpULR GH
LJXDOGDGHD¿PGHTXHHVVHYDORUWRUQHVHDEVROXWR OITICICA, H. Parangolé'LVSRQtYHOHPZZZPXKNDEH$FHVVRHPPDLR
em todas as sociedades.
,QVSLUDGDHPIDQWDVLDVGH&DUQDYDODDUWHDSUHVHQWDGDVH
E 2 DVSHFWR IXQGDPHQWDO GD MXVWLoD p R H[HUFtFLR GH RSXQKDjFRQFHSomRGHSDWULP{QLRYLJHQWHQDVGpFDGDV
GRPLQDomR H FRQWUROH HYLWDQGR D GHVLQWHJUDomR GH GHHQDPHGLGDHPTXH
XPDVRFLHGDGHGLYHUVL¿FDGD
A VHDSURSULDYDGDVH[SUHVV}HVGDFXOWXUDSRSXODUSDUD
QUESTÃO 19 SURGX]LUXPDDUWHHIrPHUDGHVWLQDGDDRSURWHVWR
B UHVJDWDYD VtPERORV DPHUtQGLRV H DIULFDQRV SDUD VH
$ UHQDWXUDOL]DomR GH ULRV H FyUUHJRV p Ki PXLWR DGDSWDUDH[SRVLo}HVHPHVSDoRVS~EOLFRV
WHPSRXPDUHDOLGDGHQD(XURSDQR-DSmRQD&RUHLDGR
C DEVRUYLDHOHPHQWRVJUi¿FRVGDSURSDJDQGDSDUDFULDU
Sul, nos Estados Unidos e em outros países. No Brasil
REMHWRVFRPHUFLDOL]iYHLVSHODVJDOHULDV
DLQGD VmR PXLWR WtPLGDV DV LQLFLDWLYDV PDV DOJXPDV
SRXFDVFLGDGHVHVWmRDGRWDQGRHVVDLPSRUWDQWHSUiWLFD D valorizava elementos da arte popular para construir
representações da identidade brasileira.
'LVSRQtYHOHPKWWSVRVULRVGREUDVLOEORJVSRWFRPEU$FHVVRHPGH] DGDSWDGR 
E incorporava elementos da cultura de massa para
$ OHJLVODomR EUDVLOHLUD DYDQoRX DR HVWDEHOHFHU FRPR DWHQGHUjVH[LJrQFLDVGRVPXVHXV
XQLGDGHWHUULWRULDOSDUDDJHVWmRGHVVHUHFXUVR
A os biomas.
B DVUHVHUYDVHFROyJLFDV
C as unidades do relevo.
D DVEDFLDVKLGURJUi¿FDV
E DViUHDVGHSUHVHUYDomRDPELHQWDO

&+žGLD_&DGHUQR%5$1&23iJLQD
*BR0975SAB8* 2016

QUESTÃO 21 QUESTÃO 23

2 0DU GH $UDO ORFDOL]DGR HQWUH R &D]DTXLVWmR H R 1RVVDVYLGDVVmRGRPLQDGDVQmRVySHODVLQXWLOLGDGHV


8]EHTXLVWmRHUDDWpRTXDUWRPDLRUODJRGRPXQGR GH QRVVRV FRQWHPSRUkQHRV FRPR WDPEpP SHODV GH
FREULQGRXPDiUHDGHPLOTXLO{PHWURVTXDGUDGRVFRP KRPHQV TXH Mi PRUUHUDP Ki YiULDV JHUDo}HV $OpP
XPYROXPHHVWLPDGRGHPDLVGHTXLO{PHWURVF~ELFRV disso, cada inutilidade ganha credibilidade e reverência
O Aral e toda a bacia do lago ganharam notoriedade FRPFDGDGpFDGDSDVVDGDGHVGHVXDSURPXOJDomR,VVR
mundial como uma das maiores degradações ambientais VLJQL¿FDTXHFDGDVLWXDomRVRFLDOHPTXHQRVHQFRQWUDPRV
GRVpFXOR;;FDXVDGDVSHORKRPHPeUHIHULGDFRPRD QmR Vy p GH¿QLGD SRU QRVVRV FRQWHPSRUkQHRV FRPR
³&KHUQRE\O&DODGD´XPDFDWiVWURIHVLOHQFLRVDTXHHYROXLX DLQGDSUHGH¿QLGDSRUQRVVRVSUHGHFHVVRUHV(VVHIDWRp
OHQWDPHQWHGHIRUPDTXDVHLPSHUFHSWtYHODRORQJRGDV H[SUHVVRQRDIRULVPRVHJXQGRRTXDORVPRUWRVVmRPDLV
~OWLPDVGpFDGDV2IXWXURGR$UDOpLQFHUWR$~QLFDFHUWH]D poderosos que os vivos.
pTXHRODJRpDJRUDFHQiULRGHXPDFDWiVWURIHDPELHQWDO %(5*(53Perspectivas sociológicasXPDYLVmRKXPDQtVWLFD
j PHGLGD TXH R QtYHO GH iJXD GHFOLQD H R HFRVVLVWHPD 3HWUySROLV9R]HV DGDSWDGR 

GHJUDGDVHSURYRFDQGRXPDPELHQWHGHGHWHULRUDomRH
6HJXQGR D SHUVSHFWLYD DSUHVHQWDGD QR WH[WR RV
FRQGLo}HV GH YLGD H GH VD~GH SUHFiULDV SDUD RV SRYRV
LQGLYtGXRV GH GLIHUHQWHV JHUDo}HV FRQYLYHP QXPD
TXHYLYHPjVPDUJHQVGRODJR
mesma sociedade, com tradições que
6$17,$*2('LVSRQtYHOHPZZZLQIRHVFRODFRP$FHVVRHPGH] DGDSWDGR 
A SHUPDQHFHP FRPR GHWHUPLQDo}HV GD RUJDQL]DomR
2VLPSDFWRVDPELHQWDLVQR0DUGH$UDOVmRGLUHWDPHQWH social.
resultantes da B promovem o esquecimento dos costumes.
A H[SORUDomR GH SHWUyOHR HP iJXDV SURIXQGDV GHVVH C FRQ¿JXUDPDVXSHUDomRGHYDORUHV
PDUSDUDDWHQGHUjGHPDQGDFHQWURDVLiWLFD D sobrevivem como heranças sociais.
B DSOLFDomRGHSHVWLFLGDVQDVODYRXUDVGHVHXHQWRUQR E atuam como aptidões instintivas.
para aumentar a produtividade.
C FRQVWUXomR GH HGL¿FDo}HV HP VXDV PDUJHQV SDUD
desenvolver a atividade turística.
D XWLOL]DomRGHVXDViJXDVSDUDDWHQGHUjVQHFHVVLGDGHV
GDLQG~VWULDSHVTXHLUD
E H[WUDomRGDViJXDVGHVHXVDÀXHQWHVSDUDDLUULJDomR
de lavouras.

QUESTÃO 22

Ô ô, com tanto pau no mato


(PED~ED pFRURQp
Com tanto pau no mato, ê ê
Com tanto pau no mato
(PED~EDpFRURQp

EmbaúbaiUYRUHFRPXPHLQ~WLOSRUVHUSRGUHSRU
dentro, segundo o historiador Stanley Stein.
STEIN, S. J. VassourasXPPXQLFtSLREUDVLOHLURGRFDIp
5LRGH-DQHLUR1RYD)URQWHLUD DGDSWDGR 

2V YHUVRV ID]HP SDUWH GH XP MRQJR JrQHUR SRpWLFR


musical cantado por escravos e seus descendentes no
%UDVLOQRVpFXOR;,;HSURFXUDPH[SUHVVDUD
A H[SORUDomRUXUDO
B bravura senhorial.
C resistência cultural.
D violência escravista.
E ideologia paternalista.

&+žGLD_&DGHUQR%5$1&23iJLQD
2016 *BR0975SAB9*
QUESTÃO 24
TEXTO I
Embora eles, artistas modernos, se deem como novos precursores duma arte a ir, nada é mais velho que a arte
DQRUPDO'HKiPXLWRVMiTXHDHVWXGDPRVSVLTXLDWUDVHPVHXVWUDWDGRVGRFXPHQWDQGRVHQRVLQ~PHURVGHVHQKRV
TXHRUQDPDVSDUHGHVLQWHUQDVGRVPDQLF{PLRV(VVDVFRQVLGHUDo}HVVmRSURYRFDGDVSHODH[SRVLomRGD6UD0DOIDWWL
6HMDPVLQFHURVIXWXULVPRFXELVPRLPSUHVVLRQLVPRHtutti quanti QmRSDVVDPGHRXWURVWDQWRVUDPRVGDDUWHFDULFDWXUDO
/2%$7203DUDQRLDRXPLVWL¿FDomRDSURSyVLWRGDH[SRVLomRGH$QLWD0DOIDWWLO Estado de São PauloGH] DGDSWDGR 

TEXTO II
$QLWD0DOIDWWLSRVVXLGRUDGHXPDDOWDFRQVFLrQFLDGRTXHID]DYLEUDQWHDUWLVWDQmRWHPHXOHYDQWDUFRPRVVHXV
cinquenta trabalhos as mais irritadas opiniões e as mais contrariantes hostilidades. As suas telas chocam o preconceito
IRWRJUi¿FRTXHJHUDOPHQWHVHOHYDQRHVStULWRSDUDDVQRVVDVH[SRVLo}HVGHSLQWXUD1DDUWHDUHDOLGDGHQDLOXVmRp
RTXHWRGRVSURFXUDP(RVQDWXUDOLVWDVPDLVSHUIHLWRVVmRRVTXHPHOKRUFRQVHJXHPLOXGLU
$1'5$'(2$H[SRVLomR$QLWD0DOIDWWLJornal do CommercioMDQ DGDSWDGR 

TEXTO III

0$/)$77,$O homem amareloÏOHRVREUHWHOD[FP'LVSRQtYHOHPZZZHVWDGDRFRPEU$FHVVRHPIHY

$DQiOLVHGRVGRFXPHQWRVDSUHVHQWDGRVGHPRQVWUDTXHRFHQiULRDUWtVWLFREUDVLOHLURQRSULPHLURTXDUWHOGRVpFXOR;;
HUDFDUDFWHUL]DGRSHOR D
A GRPtQLRGRDFDGHPLFLVPRTXHGL¿FXOWDYDDUHFHSomRGDYHUWHQWHUHDOLVWDQDREUDGH$QLWD0DOIDWWL
B GLVVRQkQFLDHQWUHDVYHUWHQWHVDUWtVWLFDVTXHGLYHUJLDPVREUHDYDOLGDGHGRPRGHORHVWpWLFRHXURSHX
C H[DOWDomRGDEHOH]DHGDULJLGH]GDIRUPDTXHMXVWL¿FDYDPDDGDSWDomRGDHVWpWLFDHXURSHLDjUHDOLGDGHEUDVLOHLUD
D LPSDFWRGHQRYDVOLQJXDJHQVHVWpWLFDVTXHDOWHUDYDPRFRQFHLWRGHDUWHHDEDVWHFLDPDEXVFDSRUXPDSURGXomR
artística nacional.
E LQÀXrQFLDGRVPRYLPHQWRVDUWtVWLFRVHXURSHXVGHYDQJXDUGDTXHOHYDYDRVPRGHUQLVWDVDFRSLDUHPVXDVWpFQLFDV
HWHPiWLFDV

&+žGLD_&DGHUQR%5$1&23iJLQD
*BR0975SAB10* 2016

QUESTÃO 25 QUESTÃO 26
A atividade atualmente chamada de ciência tem
VH PRVWUDGR IDWRU LPSRUWDQWH QR GHVHQYROYLPHQWR
GD FLYLOL]DomR OLEHUDO VHUYLX SDUD HOLPLQDU FUHQoDV H
SUiWLFDVVXSHUVWLFLRVDVSDUDDIDVWDUWHPRUHVEURWDGRV
GD LJQRUkQFLD H SDUD IRUQHFHU EDVH LQWHOHFWXDO GH
DYDOLDomR GH FRVWXPHV KHUGDGRV H GH QRUPDV
tradicionais de conduta.
1$*(/(HWDO CiênciaQDWXUH]DHREMHWLYR6mR3DXOR&XOWUL[ DGDSWDGR 

Quais características permitem conceber a ciência com


os aspectos críticos mencionados?
A $SUHVHQWDU H[SOLFDo}HV HP XPD OLQJXDJHP
determinada e isenta de erros.
B 3RVVXLU SURSRVLo}HV TXH VmR UHFRQKHFLGDV FRPR
LQTXHVWLRQiYHLVHQHFHVViULDV
C 6HU IXQGDPHQWDGD HP XP FRUSR GH FRQKHFLPHQWR
autoevidente e verdadeiro.
D Estabelecer rigorosa correspondência entre princípios
H[SOLFDWLYRVHIDWRVREVHUYDGRV
E Constituir-se como saber organizado ao permitir
FODVVL¿FDo}HVGHGX]LGDVGDUHDOLGDGH

QUESTÃO 27

A eugenia, tal como originalmente concebida, era


D DSOLFDomR GH ³ERDV SUiWLFDV GH PHOKRUDPHQWR´ DR
DSULPRUDPHQWRGDHVSpFLHKXPDQD)UDQFLV*DOWRQIRLR
SULPHLUR D VXJHULU FRP GHVWDTXH R YDORU GD UHSURGXomR
+(1),/'LUHWDV-i,Q/(0265 2UJ  Uma história do Brasil através da humana controlada, considerando-a produtora do
caricatura (1840-2001)5LRGH-DQHLUR/HWUDV ([SUHVV}HV DSHUIHLoRDPHQWRGDHVSpFLH
526(0O espectro de Darwin5LRGH-DQHLUR=DKDU DGDSWDGR 
$ LPDJHP ID] UHIHUrQFLD D XPD LQWHQVD PRELOL]DomR
popular e pode ser traduzida como 8P UHVXOWDGR GD DSOLFDomR GHVVD WHRULD GLVVHPLQDGD D
A D FDPSDQKD SRSXODU TXH FRQIURQWDYD D OHJLWLPLGDGH SDUWLUGDVHJXQGDPHWDGHGRVpFXOR;,;IRLR D
das eleições indiretas no país. A DSURYDomRGHPHGLGDVGHLQFOXVmRVRFLDO
B D PDQLIHVWDomR GH PLOKDUHV GH SHVVRDV HP SURO GD B DGRomR GH FULDQoDV FRP GLIHUHQWHV FDUDFWHUtVWLFDV
UHDOL]DomRGHHOHLo}HVSDUDR6HQDGR ItVLFDV
C DV SDVVHDWDV UHDOL]DGDV HP SURO GR ¿P GD 'LWDGXUD C HVWDEHOHFLPHQWR GH OHJLVODomR TXH FRPEDWLD DV
0LOLWDUQR%UDVLOHQD$UJHQWLQD divisões sociais.
D RVFRPtFLRVHPDQLIHVWDo}HVSRSXODUHVSHODDEHUWXUD D SULVmRHHVWHULOL]DomRGHSHVVRDVFRPFDUDFWHUtVWLFDV
SROtWLFDGHIRUPDOHQWDHVHJXUD FRQVLGHUDGDVLQIHULRUHV
E RPRYLPHQWRTXHH[LJLDRGLUHLWRjLJXDOGDGHGHYRWR E GHVHQYROYLPHQWR GH SUyWHVHV TXH SRVVLELOLWDYDP D
para homens e mulheres. UHDELOLWDomRGHSHVVRDVGH¿FLHQWHV

&+žGLD_&DGHUQR%5$1&23iJLQD
2016 *BR0975SAB11*
QUESTÃO 28 QUESTÃO 29
2V ULFRV DGTXLULUDP XPD REULJDomR UHODWLYDPHQWH j As camadas dirigentes paulistas na segunda
FRLVDS~EOLFDXPDYH]TXHGHYHPVXDH[LVWrQFLDDRDWR PHWDGH GR VpFXOR ;,; UHFRUULDP j KLVWyULD H j ¿JXUD
GH VXEPLVVmR j VXD SURWHomR H ]HOR R TXH QHFHVVLWDP dos bandeirantes. Para os paulistas, desde o início da
SDUD YLYHU R (VWDGR HQWmR IXQGDPHQWD R VHX GLUHLWR GH FRORQL]DomRRVKDELWDQWHVGH3LUDWLQLQJD DQWLJRQRPHGH
FRQWULEXLomRGRTXHpGHOHVQHVVDREULJDomRYLVDQGRD
6mR3DXOR WLQKDPVLGRUHVSRQViYHLVSHODDPSOLDomRGR
PDQXWHQomRGHVHXVFRQFLGDGmRV,VVRSRGHVHUUHDOL]DGR
WHUULWyULRQDFLRQDOHQULTXHFHQGRDPHWUySROHSRUWXJXHVD
SHODLPSRVLomRGHXPLPSRVWRVREUHDSURSULHGDGHRXD
DWLYLGDGHFRPHUFLDOGRVFLGDGmRVRXSHORHVWDEHOHFLPHQWR FRP R RXUR H H[SDQGLQGR VXDV SRVVHVV}HV *UDoDV j
GHIXQGRVHGHXVRGRVMXURVREWLGRVDSDUWLUGHOHVQmR LQWHJUDomR WHUULWRULDO TXH SURPRYHUDP RV EDQGHLUDQWHV
SDUDVXSULUDVQHFHVVLGDGHVGR(VWDGR XPDYH]TXHHVWH HUDPWLGRVDLQGDFRPRIXQGDGRUHVGDXQLGDGHQDFLRQDO
é rico), mas para suprir as necessidades do povo. 5HSUHVHQWDYDP D OHDOGDGH j SURYtQFLD GH 6mR 3DXOR H
KANT, I. A metafísica dos costumes%DXUX(GLSUR ao Brasil.
$%8'.03DXOLVWDVXQLYRVRevista de História da Biblioteca Nacional,
6HJXQGRHVVHWH[WRGH.DQWR(VWDGR QMXO DGDSWDGR 

A deve sustentar todas as pessoas que vivem sob seu


1R SHUtRGR GD KLVWyULD QDFLRQDO DQDOLVDGR D HVWUDWpJLD
SRGHUD¿PGHTXHDGLVWULEXLomRVHMDSDULWiULD
GHVFULWDWLQKDFRPRREMHWLYR
B HVWiDXWRUL]DGRDFREUDULPSRVWRVGRVFLGDGmRVULFRV
SDUDVXSULUDVQHFHVVLGDGHVGRVFLGDGmRVSREUHV A SURPRYHU R SLRQHLULVPR LQGXVWULDO SHOD VXEVWLWXLomR
C dispõe de poucos recursos e, por esse motivo, de importações.
é obrigado a cobrar impostos idênticos dos seus B TXHVWLRQDU R JRYHUQR UHJHQFLDO DSyV D
membros. GHVFHQWUDOL]DomRDGPLQLVWUDWLYD
D GHOHJD DRV FLGDGmRV R GHYHU GH VXSULU DV C UHFXSHUDUDKHJHPRQLDSHUGLGDFRPR¿PGDSROtWLFD
necessidades do Estado, por causa do seu elevado GRFDIpFRPOHLWH
FXVWRGHPDQXWHQomR
D DXPHQWDU D SDUWLFLSDomR SROtWLFD HP IXQomR GD
E tem a incumbência de proteger os ricos das
LPSRVLo}HV SHFXQLiULDV GRV SREUHV SRLV RV ULFRV H[SDQVmRFDIHHLUD
pagam mais tributos. E legitimar o movimento abolicionista durante a crise
do escravismo.

QUESTÃO 30

2EMHWRV WULYLDOL]DGRV SRU VHX ODUJR XVR RV UHOyJLRV


VmRPDLVTXHLQVWUXPHQWRVLQGLVSHQViYHLVjURWLQDGLiULD
apontam para um modo historicamente construído de
lidar com o tempo. O emprego mais rigoroso e cotidiano
de instrumentos que registram a passagem do tempo
SRGH VHU FRQVWDWDGR SHOD SURGXomR PDVVL¿FDGD GH
UHOyJLRVHPHVSDoRVS~EOLFRVQRDPELHQWHGRPpVWLFRH
QRV LQFRQWiYHLV PRYLPHQWRV GR KRPHP XUEDQR RXWURUD
na algibeira, atualmente no pulso. Em seus ponteiros, a
VXFHVVmRGRVLQVWDQWHVpSDGURQL]DGDHPXQLGDGHV¿[DV
horas, minutos, segundos.
6,/9$),/+2$/0FortalezaLPDJHQVGDFLGDGH)RUWDOH]D0XVHXGR&HDUi
6HFXOW&( DGDSWDGR 

'XUDQWHRVpFXOR;;HVVDIRUPDGHFRQFHEHURWHPSR
H[SHULPHQWDGDVREUHWXGRQRHVSDoRXUEDQRWUD]LQGtFLRV
de uma cultura marcada pela
A RUJDQL]DomRGRWHPSRGHPRGRRUJkQLFRHSHVVRDO
B UHFXVDDRFRQWUROHGRWHPSRH[HUFLGRSHORVUHOyJLRV
C GHPRFUDWL]DomR QRV XVRV H DSURSULDo}HV GR WHPSR
cotidiano.
D QHFHVVLGDGHGHXPDPDLRUPDWHPDWL]DomRGRWHPSR
cotidiano.
E XWLOL]DomR GR UHOyJLR FRPR H[SHULrQFLD QDWXUDO GH
HODERUDomRGRWHPSR

&+žGLD_&DGHUQR%5$1&23iJLQD
*BR0975SAB12* 2016

QUESTÃO 31

$LPDJHPDSUHVHQWDXPH[HPSORGHFURTXLGHVtQWHVHVREUHRWXULVPRQD)UDQoD

Turismo na França
BENELUX
ROYAUME-UNI
ALLEMAGNE

NORMANDIE

BRETAGNE VOSGES

Châteaux
de la Loire
JURA SUISSE

VENDÉE
MASSIF
CENTRAL
ITALIE

ALPES
Provence
romaine
PAYS
BASQUE
CÔTE D’ AZUR
LANGUEDOC
PYRENÉES

Eixos e Fluxos Regiões Turísticas


Fluxo de Turistas Estrangeiros Principais Regiões de Turismo Balneário

Outras Regiões de Turismo Balneário


Eixos Principais
Turismo de Montanha
Eixos Secundários
Turismo Verde
Planejamento
Turismo Cultural Muito Importante
Litorais Planejados Turismo Cultural Importante

Parques Nacionais Estações

321786&+.$113$*$1(//,7/&$&(7(1+3DUDHQVLQDUHDSUHQGHUJHRJUD¿D6mR3DXOR&RUWH] DGDSWDGR 

2VFURTXLVVmRHVTXHPDVJUi¿FRVTXH
A WrPDVPHGLGDVUHSUHVHQWDGDVHPHVFDODXQLIRUPH
B UHVVDOWDPDGLVWULEXLomRHVSDFLDOGRVIHQ{PHQRVHRVIDWRUHVGHORFDOL]DomR
C WrPDUHSUHVHQWDomRJUi¿FDGHGLVWkQFLDVGRWHUUHQRIHLWDVREUHXPDOLQKDUHWDJUDGXDGD
D LQGLFDP D UHODomR HQWUH D GLPHQVmR GR HVSDoR UHDO H D GR HVSDoR UHSUHVHQWDGR SRU PHLR GH XPD SURSRUomR
numérica.
E SURSRUFLRQDPDREWHQomRGHLQIRUPDo}HVDFHUFDGHXPREMHWRiUHDRXIHQ{PHQRORFDOL]DGRQD7HUUDVHPTXH
KDMDFRQWDWRItVLFR

&+žGLD_&DGHUQR%5$1&23iJLQD
2016 *BR0975SAB13*
QUESTÃO 32

Carta de princípios
$VDOWHUQDWLYDVSURSRVWDVQR)yUXP6RFLDO0XQGLDOFRQWUDS}HPVHDXPSURFHVVRGHJOREDOL]DomRFRPDQGDGR
pelas grandes corporações multinacionais e pelos governos e instituições internacionais a serviço de seus interesses,
com a cumplicidade de governos nacionais.
'LVSRQtYHOHPKWWSIVPSRDFRPEU$FHVVRHPDJR

2IyUXPPHQFLRQDGRWHPFRPRREMHWLYRGLVFXWLUSURSRVWDVTXHFRQFUHWL]HPD
A SURSRVLomRGDLJXDOGDGHFXOWXUDOHQWUHRVSRYRV
B DPSOLDomRGRÀX[RSRSXODFLRQDOHQWUHRV(VWDGRV
C FRQVWUXomRGHXPDUHODomRVROLGiULDHQWUHRVSDtVHV
D LQWHQVL¿FDomRGDVUHODo}HVHFRQ{PLFDVHQWUHDVQDo}HV
E KRPRJHQHL]DomRGRFRQVXPRHQWUHDSRSXODomRPXQGLDO

QUESTÃO 33

São Luís

CENTRO DE VIGILÂNCIA

UNIDADE DE VIGILÂNCIA

UNIDADE DE TELECOMUNICAÇÕES
UNIDADE DE VIGILÂNCIA
E TELECOMUNICAÇÕES
PELOTÃO DE FRONTEIRA

PELOTÃO DE FRONTEIRA
EM IMPLANTAÇÃO
'LVSRQtYHOHPZZZJHRJUD¿DSDUDWRGRVFRPEU$FHVVRHPDJR

$DQiOLVHGDLPDJHPUHPHWHDXPDHVWUDWpJLDTXHSUHVVXS}HR D
A SUHRFXSDomRJRYHUQDPHQWDOFRPDHQWUDGDGHLPLJUDQWHVQRSDtV
B GHWHUPLQDomRGRJRYHUQRHPLPSHGLUDH[SDQVmRGHSDtVHVYL]LQKRV
C XWLOL]DomRGHWHFQRORJLDVQRSURFHVVRGHWHUULWRULDOL]DomRGRHVSDoREUDVLOHLUR
D GHFLVmRGRJRYHUQRHPSURWHJHUDViUHDVGHFRQVWUXomRGHKLGUHOpWULFDVQR%UDVLO
E GLUHFLRQDPHQWRGRVLQYHVWLPHQWRVPLOLWDUHVSDUDDSURWHomRGHUHFXUVRVELRJHQpWLFRV

&+žGLD_&DGHUQR%5$1&23iJLQD
*BR0975SAB14* 2016

QUESTÃO 34 QUESTÃO 35
$V LQIRUPDo}HV VXJHULGDV SRU$QW{QLR 0DQXHO HVWmR (QIHUPRDGHQRYHPEURQDVHJXQGDIHLUDRYHOKR
LPHUVDV HP XP MRUQDO GLYLGLGR HQWUH R ³UHDO´ H R TXH /LPD YROWRX DR WUDEDOKR LJQRUDQGR TXH QR HQWUHWHPSR
SRGHPRVFKDPDUGH³VLWXDFLRQDO´2DUWLVWDWUDQVIRUPDWRGR caíra o regime. Sentou-se e viu que tinham tirado da
RFOLPDGHUHSUHVVmRQDSUySULDPDWpULDGHVHXWUDEDOKR SDUHGH D YHOKD OLWRJUD¿D UHSUHVHQWDQGR ' 3HGUR GH
XWLOL]DQGRRVPHLRVGHFRPXQLFDomRFRPRDUPD LU{QLFD  $OFkQWDUD &RPR QD RFDVLmR SDVVDVVH XP FRQWtQXR
FRQWUDDHVWUXWXUDGHSRGHUGHXP(VWDGRDXWRULWiULR perguntou-lhe:
SCOVINO, F. Com as armas do inimigo. Revista de História da Biblioteca Nacional, — Por que tiraram da parede o retrato de Sua
QVHW DGDSWDGR 
0DMHVWDGH"
1R FRQWH[WR KLVWyULFR GHVFULWR D HVWUDWpJLD DGRWDGD SRU
O contínuo respondeu, num tom lentamente
DOJXQVVHJPHQWRVGDLPSUHQVDSDUDDFRQVWUXomRGHXPD
desdenhoso:
FUtWLFDVRFLRSROtWLFDIRLDGH
² 2UD FLGDGmR TXH ID]LD DOL D ¿JXUD GR 3HGUR
A EXUODU D FHQVXUD FRQWULEXLQGR SDUD D DQiOLVH GD
Banana?
vida social.
B MXVWL¿FDU R UHJLPH YLJHQWH DSUHVHQWDQGR YHUV}HV ²3HGUR%DQDQD²UHSHWLXUDLYRVRRYHOKR/LPD
diversas da realidade. E, sentando-se, pensou com tristeza:
C HVWLPXODU D OLYUH LQWHUSUHWDomR GRV IDWRV DWHQGHQGR
aos interesses dominantes. ² 1mR GRX WUrV DQRV SDUD TXH LVVR VHMD XPD
5HS~EOLFD
D DSULPRUDU R DOFDQFH GDV LQIRUPDo}HV DSUHVHQWDQGR
$=(9('2$Vidas alheias3RUWR$OHJUHVH DGDSWDGR 
as notícias em tempo real.
E PDQLSXODUDYLVmRFROHWLYDSURPRYHQGRLQWHUSUHWDo}HV A crônica de Artur Azevedo, retratando os dias imediatos
GLVWRUFLGDVGDVQRWtFLDVR¿FLDLV jLQVWDXUDomRGD5HS~EOLFDQR%UDVLOUHIHUHVHDR j
A DXVrQFLD GH SDUWLFLSDomR SRSXODU QR SURFHVVR GH
TXHGDGD0RQDUTXLD
B WHQVmR VRFLDO HQYROYLGD QR SURFHVVR GH LQVWDXUDomR
do novo regime.
C PRELOL]DomR GH VHWRUHV VRFLDLV QD UHVWDXUDomR GR
antigo regime.
D WHPRUGRVVHWRUHVEXURFUiWLFRVFRPRQRYRUHJLPH
E GHPRUDQDFRQVROLGDomRGRQRYRUHJLPH

&+žGLD_&DGHUQR%5$1&23iJLQD
2016 *BR0975SAB15*
QUESTÃO 36 QUESTÃO 37
$PHDoDUHDOjVHJXUDQoDGHPDLVGHSHVVRDVGH $UW ž ± 6mR GLUHLWRV GRV WUDEDOKDGRUHV XUEDQRV
FDVDVGH3ODQDOWLQDGH*RLiVDYRoRURFDTXHOHYRX e rurais, além de outros que visem a melhoria de sua
j GHFUHWDomR GH VLWXDomR GH HPHUJrQFLD QR PXQLFtSLR FRQGLomRVRFLDO
SHOR 0LQLVWpULR GD ,QWHJUDomR 1DFLRQDO IRL YLVWRULDGD
SHOR SURFXUDGRUJHUDO GH -XVWLoD GH *RLiV H SRU YiULDV ;;9±DVVLVWrQFLDJUDWXLWDDRV¿OKRVHGHSHQGHQWHV
DXWRULGDGHV GDV WUrV HVIHUDV GH JRYHUQR 'XUDQWH D GHVGH R QDVFLPHQWR DWp  FLQFR  DQRV GH LGDGH HP
YLVWRULDGDHURVmRTXHMiPHGHTXDVHTXLO{PHWURVGH FUHFKHV H SUpHVFRODV 5HGDomR GDGD SHOD (PHQGD
H[WHQVmRIRLFRQ¿UPDGDDOLEHUDomRGHUHFXUVRVYLVDQGR &RQVWLWXFLRQDOQGH 
paralisar o processo degradante. 'LVSRQtYHOHPZZZMXVEUDVLOFRPEU$FHVVRHPIHY DGDSWDGR 
'LVSRQtYHOHPKWWSPSJRMXVEUDVLOFRPEU$FHVVRHPDJR DGDSWDGR 
$ LQFOXVmR GR GLUHLWR j FUHFKH H j SUpHVFROD QD
&RQVWLWXLomRGD5HS~EOLFD)HGHUDWLYDGR%UDVLOSRGHVHU
H[SOLFDGDSHOD
A UHGXomRGDWD[DGHIHFXQGLGDGHQRSDtV
B SUHFDUL]DomR GDV UHGHV GH HVFRODV S~EOLFDV
brasileiras.
C PRELOL]DomR GDV PXOKHUHV LQVHULGDV QR PHUFDGR GH
trabalho.
D DWXDomRGDLQLFLDWLYDSULYDGDFRQVRDQWHjVGHPDQGDV
VRFLDLVGDSRSXODomR
E FRQVWDWDomR GRV HOHYDGRV tQGLFHV GH PDXVWUDWRV
VRIULGRVSHODVFULDQoDVQR%UDVLO

QUESTÃO 38
'LVSRQtYHOHPKWWSDOJROHJEU$FHVVRHPDJR DGDSWDGR 
$ 6HJXQGD 5HYROXomR ,QGXVWULDO QR ILQDO GR
2 IHQ{PHQR QRWLFLDGR VREUH D iUHD XUEDQD GH
século XIX e início do século XX, nos EUA, período em
3ODQDOWLQD *2 WHPVXDRULJHPH[SOLFDGDSHOD
TXH D HOHWULFLGDGH SDVVRX JUDGDWLYDPHQWH D ID]HU SDUWH
A IUDFD FREHUWXUD YHJHWDO H FRPSRVLomR GR VROR do cotidiano das cidades e a alimentar os motores das
UHVXOWDGRGDDomRHURVLYDQDWXUDOGDVFKXYDV IiEULFDVFDUDFWHUL]RXVHSHODDGPLQLVWUDomRFLHQWt¿FDGR
B UHODomRHQWUHRGHFOLYHGRWHUUHQRHDIRUoDHURVLYDGD WUDEDOKRHSHODSURGXomRHPVpULH
iJXDUHVXOWDGRGDHYROXomRGRUHOHYR 0(5/2$5&/$3,61/$VD~GHHRVSURFHVVRVGHWUDEDOKRQRFDSLWDOLVPR
UHÀH[}HVQDLQWHUIDFHGDSVLFRGLQkPLFDGRWUDEDOKRHGDVRFLRORJLDGRWUDEDOKR
C declividade do terreno e intensidade das chuvas, Psicologia e SociedadeQDEU
UHVXOWDGR GR HVFRDPHQWR VXSHU¿FLDO GDV iJXDV
pluviais. 'HDFRUGRFRPRWH[WRQDSULPHLUDPHWDGHGRVpFXOR;;
o capitalismo produziu um novo espaço geoeconômico e
D GHJUDGDomR DPELHQWDO H GH¿FLrQFLD QD GUHQDJHP
GH iJXDV SOXYLDLV UHVXOWDGR GD RFXSDomR H XVR XPDUHYROXomRTXHHVWiUHODFLRQDGDFRPD
inadequado do solo. A SUROLIHUDomRGHSHTXHQDVHPpGLDVHPSUHVDVTXHVH
E GHFRPSRVLomR H WUDQVSRUWH GH VHGLPHQWRV SRU equiparam com as novas tecnologias e aumentaram
HVFRDPHQWR VXSHU¿FLDO UHVXOWDGR GH SURFHVVRV DSURGXomRFRPDSRUWHGRJUDQGHFDSLWDO
HURVLYRVQDWXUDLVjVHQFRVWDV B WpFQLFDGHSURGXomRIRUGLVWDTXHLQVWLWXLXDGLYLVmRHD
KLHUDUTXL]DomRGRWUDEDOKRHPTXHFDGDWUDEDOKDGRU
realizava apenas uma etapa do processo produtivo.
C SDVVDJHP GR VLVWHPD GH SURGXomR DUWHVDQDO SDUD
R VLVWHPD GH SURGXomR IDEULO FRQFHQWUDQGRVH
SULQFLSDOPHQWH QD SURGXomR Wr[WLO GHVWLQDGD DR
mercado interno.
D independência política das nações colonizadas, que
permitiu igualdade nas relações econômicas entre
os países produtores de matérias-primas e os países
industrializados.
E FRQVWLWXLomR GH XPD FODVVH GH DVVDODULDGRV TXH
SRVVXtDPFRPRIRQWHGHVXEVLVWrQFLDDYHQGDGHVXD
IRUoD GH WUDEDOKR H TXH OXWDYDP SHOD PHOKRULD GDV
FRQGLo}HVGHWUDEDOKRQDVIiEULFDV

&+žGLD_&DGHUQR%5$1&23iJLQD
*BR0975SAB16* 2016

QUESTÃO 39 QUESTÃO 41
2 DSDUHFLPHQWR GD SyOLV VLWXDGR HQWUH RV VpFXORV e KRMH D QRVVD IHVWD QDFLRQDO 2 %UDVLO LQWHLUR GD
9,,, H 9,, D& FRQVWLWXL QD KLVWyULD GR SHQVDPHQWR FDSLWDOGR,PSpULRDPDLVUHPRWDHLQVLJQL¿FDQWHGHVXDV
grego, um acontecimento decisivo. Certamente, no plano DOGHRODVFRQJUHJDVHXQkQLPHSDUDFRPHPRUDURGLDTXH
intelectual como no domínio das instituições, a vida social RWLURXGHQWUHDVQDo}HVGHSHQGHQWHVSDUDFRORFiORHQWUH
HDVUHODo}HVHQWUHRVKRPHQVWRPDPXPDIRUPDQRYD as nações soberanas, e entregou-lhe os seus destinos,
FXMD RULJLQDOLGDGH IRL SOHQDPHQWH VHQWLGD SHORV JUHJRV TXHDWpHQWmRKDYLDP¿FDGRDFDUJRGHXPSRYRHVWUDQKR
PDQLIHVWDQGRVHQRVXUJLPHQWRGD¿ORVR¿D Gazeta de NotíciasVHW
VERNANT, J.-P. As origens do pensamento grego5LRGH-DQHLUR'LIHO DGDSWDGR 
$V IHVWLYLGDGHV HP WRUQR GD ,QGHSHQGrQFLD GR %UDVLO
6HJXQGR 9HUQDQW D ¿ORVR¿D QD DQWLJD *UpFLD IRL PDUFDPRQRVVRFDOHQGiULRGHVGHRVDQRVLPHGLDWDPHQWH
UHVXOWDGRGR D SRVWHULRUHVDRGHVHWHPEURGH(VVDFRPHPRUDomR
HVWiGLUHWDPHQWHUHODFLRQDGDFRP
A FRQVWLWXLomRGRUHJLPHGHPRFUiWLFR
B contato dos gregos com outros povos. A D FRQVWUXomR H PDQXWHQomR GH VtPERORV SDUD D
IRUPDomRGHXPDLGHQWLGDGHQDFLRQDO
C desenvolvimento no campo das navegações.
B o domínio da elite brasileira sobre os principais cargos
D aparecimento de novas instituições religiosas.
SROtWLFRVTXHVHHIHWLYRXORJRDSyV
E VXUJLPHQWRGDFLGDGHFRPRRUJDQL]DomRVRFLDO
C RV LQWHUHVVHV GH VHQKRUHV GH WHUUDV TXH DSyV D
,QGHSHQGrQFLDH[LJLUDPDDEROLomRGDHVFUDYLGmR
QUESTÃO 40
D R DSRLR SRSXODU jV PHGLGDV WRPDGDV SHOR JRYHUQR
7RGDVDVFRLVDVVmRGLIHUHQFLDo}HVGHXPDPHVPD LPSHULDOSDUDDH[SXOVmRGHHVWUDQJHLURVGRSDtV
FRLVD H VmR D PHVPD FRLVD ( LVWR p HYLGHQWH 3RUTXH E D FRQVFLrQFLD GD SRSXODomR VREUH RV VHXV GLUHLWRV
VHDVFRLVDVTXHVmRDJRUDQHVWHPXQGRʊWHUUDiJXD DGTXLULGRV SRVWHULRUPHQWH j WUDQVIHUrQFLD GD &RUWH
DU H IRJR H DV RXWUDV FRLVDV TXH VH PDQLIHVWDP QHVWH para o Rio de Janeiro.
PXQGR ʊ VH DOJXPD GHVWDV FRLVDV IRVVH GLIHUHQWH GH
TXDOTXHU RXWUD GLIHUHQWH HP VXD QDWXUH]D SUySULD H
VH QmR SHUPDQHFHVVH D PHVPD FRLVD HP VXDV PXLWDV
PXGDQoDV H GLIHUHQFLDo}HV HQWmR QmR SRGHULDP DV
FRLVDV GH QHQKXPD PDQHLUD PLVWXUDUVH XPDV jV
RXWUDV QHP ID]HU EHP RX PDO XPDV jV RXWUDV QHP
a planta poderia brotar da terra, nem um animal ou
TXDOTXHURXWUDFRLVDYLUjH[LVWrQFLDVHWRGDVDVFRLVDV
QmR IRVVHP FRPSRVWDV GH PRGR D VHUHP DV PHVPDV
7RGDVDVFRLVDVQDVFHPDWUDYpVGHGLIHUHQFLDo}HVGH
XPD PHVPD FRLVD RUD HP XPD IRUPD RUD HP RXWUD
retomando sempre a mesma coisa.
',Ï*(1(6,Q%251+(,0*$2V¿OyVRIRVSUpVRFUiWLFRV6mR3DXOR&XOWUL[

2WH[WRGHVFUHYHDUJXPHQWRVGRVSULPHLURVSHQVDGRUHV
GHQRPLQDGRV SUpVRFUiWLFRV 3DUD HOHV D SULQFLSDO
SUHRFXSDomR¿ORVy¿FDHUDGHRUGHP
A FRVPROyJLFD SURSRQGR XPD H[SOLFDomR UDFLRQDO GR
PXQGRIXQGDPHQWDGDQRVHOHPHQWRVGDQDWXUH]D
B SROtWLFDGLVFXWLQGRDVIRUPDVGHRUJDQL]DomRGDSyOLV
ao estabelecer as regras da democracia.
C pWLFD GHVHQYROYHQGR XPD ¿ORVR¿D GRV YDORUHV
YLUWXRVRVTXHWHPDIHOLFLGDGHFRPRREHPPDLRU
D estética, procurando investigar a aparência dos entes
sensíveis.
E KHUPHQrXWLFD FRQVWUXLQGR XPD H[SOLFDomR XQtYRFD
da realidade.

&+žGLD_&DGHUQR%5$1&23iJLQD
2016 *BR0975SAB17*
QUESTÃO 42 QUESTÃO 44
$ *XHUUD )ULD IRL DFLPD GH WXGR XP SURGXWR GD TEXTO I
KHWHURJHQHLGDGHQRVLVWHPDLQWHUQDFLRQDOʊSDUDUHSHWLU (QWUH RV DQRV  H  R FUHVFLPHQWR GD
GDKHWHURJHQHLGDGHGDRUJDQL]DomRLQWHUQDHGDSUiWLFD LPLJUDomR MXGDLFD SDUD D 3DOHVWLQD IRL H[SRQHQFLDO
LQWHUQDFLRQDO ʊ H VRPHQWH SRGHULD VHU HQFHUUDGD SHOD SDVVDQGR GH   LPLJUDQWHVDQR HP  SDUD PDLV
REWHQomRGHXPDQRYDKRPRJHQHLGDGH2UHVXOWDGRGLVWR GHHP(PYLQWHDQRVDSRSXODomRMXGDLFD
IRLTXHenquanto os dois sistemas distintos existiram, o havia passado de menos de 10% para mais de 30% da
FRQÀLWR GD *XHUUD )ULD HVWDYD GHVWLQDGR D FRQWLQXDU D SRSXODomRORFDO
*XHUUD)ULDQmRSRGHULDWHUPLQDUFRPRFRPSURPLVVRRX *$77$=$A Guerra da Palestina6mR3DXOR8VLQDGR/LYUR
a convergência, mas somente com a prevalência de um
destes sistemas sobre o outro. TEXTO II
+$//,'$<)Repensando as relações internacionais3RUWR$OHJUH(G8)5*6 8PHVWDGRVHPLLQGHSHQGHQWHVREFRQWUROHEULWkQLFR
IRLDIyUPXODTXHD*Um%UHWDQKDXVRXSDUDDDGPLQLVWUDomR
$ FDUDFWHUL]DomR GD *XHUUD )ULD DSUHVHQWDGD SHOR WH[WR GDViUHDVTXHWRPDUDGRLPSpULRWXUFR$H[FHomRIRLD
LPSOLFDLQWHUSUHWiODFRPRXP D Palestina, que eles administraram diretamente, tentando
A HVIRUoRGHKRPRJHQHL]DomRGRVLVWHPDLQWHUQDFLRQDO HP YmR FRQFLOLDU SURPHVVDV IHLWDV DRV MXGHXV VLRQLVWDV
QHJRFLDGRHQWUH(VWDGRV8QLGRVH8QLmR6RYLpWLFD HPWURFDGHDSRLRFRQWUDD$OHPDQKDHDRViUDEHVHP
troca de apoio contra os turcos.
B guerra, visando o estabelecimento de um renovado
HOBSBAWN, E. Era dos extremos6mR3DXOR&LDGDV/HWUDV
sistema social, híbrido de socialismo e capitalismo.
C FRQÀLWR LQWHUVLVWrPLFR HP TXH SDtVHV FDSLWDOLVWDV 1RVWUHFKRVVmRWHPDWL]DGRVRGHVWLQRGHXPWHUULWyULR
H VRFLDOLVWDV FRPSHWLULDP DWp R ¿P SHOR SRGHU GH QRSHUtRGRHQWUHDVGXDV*UDQGHV*XHUUDV0XQGLDLV
LQÀXrQFLDHPHVFDODPXQGLDO $ RULHQWDomR GD SROtWLFD EULWkQLFD UHODWLYD D HVVD UHJLmR
D FRPSURPLVVR FDSLWDOLVWD GH WUDQVIRUPDU DV HVWiLQGLFDGDQD
sociedades homogêneas dos países socialistas em A FULDomRGHXP(VWDGRDOLDGR
democracias liberais. B RFXSDomRGHiUHDVVDJUDGDV
E HQIUHQWDPHQWR EpOLFR HQWUH FDSLWDOLVPR H VRFLDOLVPR C UHDomRDRPRYLPHQWRVRFLDOLVWD
SHOD KRPRJHQHL]DomR VRFLDO GH VXDV UHVSHFWLYDV
D SURPRomRGRFRPpUFLRUHJLRQDO
iUHDVGHLQÀXrQFLDSROtWLFD
E H[SORUDomRGHMD]LGDVSHWUROtIHUDV
QUESTÃO 43
QUESTÃO 45
A teoria da democracia participativa é construída em
WRUQR GD D¿UPDomR FHQWUDO GH TXH RV LQGLYtGXRV H VXDV
LQVWLWXLo}HV QmR SRGHP VHU FRQVLGHUDGRV LVRODGDPHQWH
$ H[LVWrQFLD GH LQVWLWXLo}HV UHSUHVHQWDWLYDV HP QtYHO
QDFLRQDO QmR EDVWD SDUD D GHPRFUDFLD SRLV R Pi[LPR
GH SDUWLFLSDomR GH WRGDV DV SHVVRDV D VRFLDOL]DomR
RX ³WUHLQDPHQWR VRFLDO´ SUHFLVD RFRUUHU HP RXWUDV
HVIHUDV GH PRGR TXH DV DWLWXGHV H DV TXDOLGDGHV
SVLFROyJLFDV QHFHVViULDV SRVVDP VH GHVHQYROYHU (VVH 'LVSRQtYHOHPZZZPDOYDGRVFRPEU$FHVVRHPGH]
GHVHQYROYLPHQWRRFRUUHSRUPHLRGRSUySULRSURFHVVRGH
$ WLULQKD FRPSDUD GRLV YHtFXORV GH FRPXQLFDomR
SDUWLFLSDomR$SULQFLSDOIXQomRGDSDUWLFLSDomRQDWHRULD
DWULEXLQGRGHVWDTXHj
GHPRFUiWLFDSDUWLFLSDWLYDpSRUWDQWRHGXFDWLYD
3$7(0$1&Participação e teoria democrática5LRGH-DQHLUR3D]H7HUUD A UHVLVWrQFLDGRFDPSRYLUWXDOjDGXOWHUDomRGHGDGRV
B interatividade dos programas de entretenimento
1HVVDWHRULDDDVVRFLDomRHQWUHSDUWLFLSDomRHHGXFDomR
abertos.
WHPFRPRIXQGDPHQWRD
C FRQ¿DQoDGRWHOHVSHFWDGRUQDVQRWtFLDVYHLFXODGDV
A DVFHQVmRGDVFDPDGDVSRSXODUHV D FUHGLELOLGDGHGDVIRQWHVQDHVIHUDFRPSXWDFLRQDO
B RUJDQL]DomRGRVLVWHPDSDUWLGiULR E DXWRQRPLDGRLQWHUQDXWDQDEXVFDGHLQIRUPDo}HV
C H¿FLrQFLDGDJHVWmRS~EOLFD
D DPSOLDomRGDFLGDGDQLDDWLYD
E legitimidade do processo legislativo.

&+žGLD_&DGHUQR%5$1&23iJLQD
*BR0975SAB18* 2016

CIÊNCIAS DA NATUREZA
E SUAS TECNOLOGIAS
Questões de 46 a 90

QUESTÃO 46
8P SHVTXLVDGRU SUHSDURX XP IUDJPHQWR GR FDXOH GH XPD ÀRU GH PDUJDULGD SDUD TXH SXGHVVH VHU REVHUYDGR
HPPLFURVFRSLDySWLFD7DPEpPSUHSDURXXPIUDJPHQWRGHSHOHGHUDWRFRPDPHVPD¿QDOLGDGH,QIHOL]PHQWHDSyV
DOJXPGHVFXLGRDVDPRVWUDVIRUDPPLVWXUDGDV
4XHHVWUXWXUDVFHOXODUHVSHUPLWLULDPDVHSDUDomRGDVDPRVWUDVVHUHFRQKHFLGDV"
A Ribossomos e mitocôndrias, ausentes nas células animais.
B Centríolos e lisossomos, organelas muito numerosas nas plantas.
C (QYROWyULRQXFOHDUHQXFOpRORFDUDFWHUtVWLFRVGDVFpOXODVHXFDULyWLFDV
D /LVRVVRPRVHSHUR[LVVRPRVRUJDQHODVH[FOXVLYDVGHFpOXODVYHJHWDLV
E Parede celular e cloroplastos, estruturas características de células vegetais.

QUESTÃO 47
2 TXDGUR DSUHVHQWD DOJXQV H[HPSORV GH FRPEXVWtYHLV HPSUHJDGRV HP UHVLGrQFLDV LQG~VWULDV H PHLRV
de transporte.

Combustível Temperatura de fusão (°C) Temperatura de ebulição (°C)


Butano  
Etanol  
0HWDQR  
0HWDQRO  
Octano  

6mRFRPEXVWtYHLVOtTXLGRVjWHPSHUDWXUDDPELHQWHGHƒ&
A Butano, etanol e metano.
B Etanol, metanol e octano.
C 0HWDQRPHWDQROHRFWDQR
D 0HWDQROHPHWDQR
E Octano e butano.

&1žGLD_&DGHUQR%5$1&23iJLQD
2016 *BR0975SAB19*
QUESTÃO 48 QUESTÃO 50
Um navio petroleiro é capaz de transportar milhares (PXPDÀRUHVWDH[LVWLDPGXDVSRSXODo}HVKHUEtYRUDV
de toneladas de carga. Neste caso, uma grande TXH KDELWDYDP R PHVPR DPELHQWH $ SRSXODomR GD
TXDQWLGDGHGHPDVVDFRQVHJXHÀXWXDU HVSpFLH ; PRVWUDYD XP JUDQGH Q~PHUR GH LQGLYtGXRV
HQTXDQWR D SRSXODomR = HUD SHTXHQD $PEDV WLQKDP
1HVWDVLWXDomRRHPSX[Rp
KiELWRV HFROyJLFRV VHPHOKDQWHV &RP D LQWHUYHQomR
A PDLRUTXHDIRUoDSHVRGRSHWUROHLUR KXPDQD RFRUUHX IUDJPHQWDomR GD ÀRUHVWD HP GXDV
B LJXDOjIRUoDSHVRGRSHWUROHLUR SRUo}HVRTXHVHSDURXDVSRSXODo}HV;H=$SyVDOJXP
WHPSR REVHUYRXVH TXH D SRSXODomR ; PDQWHYH VXD
C PDLRUTXHDIRUoDSHVRGDiJXDGHVORFDGD
WD[D SRSXODFLRQDO HQTXDQWR D SRSXODomR = DXPHQWRX D
D LJXDOjIRUoDSHVRGRYROXPHVXEPHUVRGRQDYLR VXDDWpTXHDPEDVSDVVDUDPDWHUDSUR[LPDGDPHQWHD
E LJXDOjPDVVDGDiJXDGHVORFDGD mesma quantidade de indivíduos.

QUESTÃO 49 $UHODomRHFROyJLFDHQWUHDVHVSpFLHV;H=TXDQGRQR
mesmo ambiente, é de:
%HQMDPLQ )UDQNOLQ   SRU YROWD GH A 3UHGDomR
 SHUFHEHX TXH GRLV EDUFRV TXH FRPSXQKDP D
IURWD FRP D TXDO YLDMDYD SDUD /RQGUHV SHUPDQHFLDP B Parasitismo.
HVWiYHLVHQTXDQWRRVRXWURVHUDPMRJDGRVSHORYHQWR C &RPSHWLomR
$RTXHVWLRQDURSRUTXrGDTXHOHIHQ{PHQRIRLLQIRUPDGR D Comensalismo.
SHOR FDSLWmR TXH SURYDYHOPHQWH RV FR]LQKHLURV KDYLDP E 3URWRFRRSHUDomR
DUUHPHVVDGR yOHR SHORV ODGRV GRV EDUFRV ,QTXLULQGR
mais a respeito, soube que habitantes das ilhas do QUESTÃO 51
3DFt¿FRMRJDYDPyOHRQDiJXDSDUDLPSHGLUTXHRYHQWR
a agitasse e atrapalhasse a pesca. $GLFLRQDUTXDQWLGDGHVGHiOFRROjJDVROLQDGLIHUHQWHV
(P  )UDQNOLQ UHVROYHX WHVWDU R IHQ{PHQR GDTXHODVGHWHUPLQDGDVSHODOHJLVODomRpXPDGDVIRUPDV
MRJDQGRXPDFROKHUGHFKi P/ GHyOHRGHROLYDHPXP GHDGXOWHUiOD8PWHVWHVLPSOHVSDUDDIHULUDTXDQWLGDGH
ODJRRQGHSHTXHQDVRQGDVHUDPIRUPDGDV0DLVFXULRVR GHiOFRROSUHVHQWHQDPLVWXUDFRQVLVWHHPDGLFLRQDUXPD
TXHRHIHLWRGHDFDOPDUDVRQGDVIRLRIDWRGHTXHRyOHR VROXomRVDOLQDDTXRVDjDPRVWUDGHJDVROLQDVREDQiOLVH
KDYLDVHHVSDOKDGRFRPSOHWDPHQWHSHORODJRQXPDiUHD Essa metodRORJLDGHDQiOLVHSRGHVHUXVDGDSRUTXHR D
GHDSUR[LPDGDPHQWHPIRUPDQGRXP¿OPH¿QR
A iJXD GD VROXomR VDOLQD LQWHUDJH FRP D JDVROLQD
(PERUDQmRWHQKDVLGRDLQWHQomRRULJLQDOGH)UDQNOLQ GD PLVWXUD IRUPDQGR GXDV IDVHV XPD GHODV GH
HVVH H[SHULPHQWR SHUPLWH XPD HVWLPDWLYD GD RUGHP GH iOFRROSXUR
grandeza do tamanho das moléculas. Para isso, basta
VXSRUTXHRyOHRVHHVSDOKDDWpIRUPDUXPDFDPDGDFRP B iOFRROFRQWLGRQDJDVROLQDLQWHUDJHFRPDVROXomR
XPD~QLFDPROpFXODGHHVSHVVXUD VDOLQD IRUPDQGR GXDV IDVHV XPD GHODV GH
gasolina pura.
5$026&+,+LVWyULDCBME InformaçãoQMDQ DGDSWDGR 
C JDVROLQD GD PLVWXUD VRE DQiOLVH LQWHUDJH FRP D
1DVFRQGLo}HVGRH[SHULPHQWRUHDOL]DGRSRU)UDQNOLQDV VROXomR VDOLQD IRUPDQGR GXDV IDVHV XPD GHODV GH
PROpFXODVGRyOHRDSUHVHQWDPXPWDPDQKRGDRUGHPGH iOFRROSXUR
A 10 m.3 D iJXD GD VROXomR VDOLQD LQWHUDJH FRP R iOFRRO GD
B 10 m. PLVWXUDIRUPDQGRGXDVIDVHVXPDGHODVGHJDVROLQD
com o sal.
C 10 m.
E iOFRRO FRQWLGR QD JDVROLQD LQWHUDJH FRP R VDO GD
D 109 m. VROXomR VDOLQD IRUPDQGR GXDV IDVHV XPD GHODV GH
E 1011 m. JDVROLQDPDLViJXD

&1žGLD_&DGHUQR%5$1&23iJLQD
*BR0975SAB20* 2016

QUESTÃO 52

$ FUHVFHQWH SURGXomR LQGXVWULDO ODQoD DR DU GLYHUVDV VXEVWkQFLDV Wy[LFDV TXH SRGHP VHU UHPRYLGDV SHOD
SDVVDJHP GR DU FRQWDPLQDGR HP WDQTXHV SDUD ¿OWUDomR SRU PDWHULDLV SRURVRV RX SDUD GLVVROXomR HP iJXD RX
VROYHQWHVRUJkQLFRVGHEDL[DSRODULGDGHRXSDUDQHXWUDOL]DomRHPVROXo}HViFLGDVRXEiVLFDV8PGRVSROXHQWHV
PDLVWy[LFRVOLEHUDGRVQDDWPRVIHUDSHODDWLYLGDGHLQGXVWULDOpDWHWUDFORURGLR[LQD

CI O CI

CI O CI
Esse poluente pode ser removido do ar pela passagem através de tanques contendo
A KH[DQR
B metanol.
C iJXDGHVWLODGD
D iFLGRFORUtGULFRDTXRVR
E KLGUy[LGRGHDP{QLRDTXRVR

QUESTÃO 53

8PEDI{PHWURVLPSOHVFRQVLVWHHPXPWXERFRQWHQGRXPDPLVWXUDVyOLGDGHGLFURPDWRGHSRWiVVLRHPVtOLFD
XPHGHFLGDFRPiFLGRVXOI~ULFR1HVVHWHVWHDGHWHFomRGDHPEULDJXH]SRUFRQVXPRGHiOFRROVHGiYLVXDOPHQWH
SRLVDUHDomRTXHRFRUUHpDR[LGDomRGRiOFRRODDOGHtGRHDUHGXomRGRGLFURPDWR DODUDQMDGR DFURPR ,,,  YHUGH 
RXFURPR ,,  D]XO 
Sopro

Álcool Solução ácida de


dicromato de potássio

$HTXDomREDODQFHDGDGDUHDomRTXtPLFDTXHUHSUHVHQWDHVVHWHVWHp

A CrO DT + DT  3 CH3±&+±2+ J ĺ&U DT +2 O  3 CH3±&22+ J


B CrO DT + DT  3 CH3±&+±2+ J ĺ&U3 DT +2 O  3 CH3±&+2 J
C CrO DT + DT  3 CH3±&+±2+ J ĺ&U3 DT +2 O  3 CH3±&+2 J
D CrO DT + DT  3 CH3±&+2 J ĺ&U3 DT +2 O  3 CH3±&22+ J
E CrO DT + DT  3 CH3±&+2 J ĺ&U DT  H2 O  3 CH3±&22+ J

&1žGLD_&DGHUQR%5$1&23iJLQD
2016 *BR0975SAB21*
QUESTÃO 54 QUESTÃO 55

&OLPDWpULR p R QRPH GH XP HVWiJLR QR SURFHVVR GH 8PD IDPtOLD DGTXLULX XP WHOHYLVRU H QR PDQXDO GR
DPDGXUHFLPHQWR GH GHWHUPLQDGRV IUXWRV FDUDFWHUL]DGR XVXiULRFRQVWDYDPDVHVSHFLILFDo}HVWpFQLFDVFRPR
SHOR DXPHQWR GR QtYHO GD UHVSLUDomR FHOXODU H GR JiV apresentado no quadro. Esse televisor permaneceu
HWLOHQR &H &RPRFRQVHTXrQFLDKiRHVFXUHFLPHQWR GLDVHPUHSRXVR stand-by &RQVLGHUHTXHDH¿FLrQFLD
GRIUXWRRTXHUHSUHVHQWDDSHUGDGHPXLWDVWRQHODGDVGH HQWUHDJHUDomRHDWUDQVPLVVmRGHHOHWULFLGDGHQDXVLQD
alimentos a cada ano. é de 30%.

eSRVVtYHOSURORQJDUDYLGDGHXPIUXWRFOLPDWpULFR AC 100-240 V
Tensão de entrada
SHOD HOLPLQDomR GR HWLOHQR SURGX]LGR 1D LQG~VWULD 50/60Hz
XWLOL]DVH R SHUPDQJDQDWR GH SRWiVVLR .0Q2) para Consumo de potência :
R[LGDURHWLOHQRDHWLOHQRJOLFRO +2&+CHOH), sendo o Potência em repouso 1W
SURFHVVRUHSUHVHQWDGRGHIRUPDVLPSOL¿FDGDQDHTXDomR
4XHTXDQWLGDGHGHHQHUJLDHPMRXOHVIRLSURGX]LGDQD
.0Q2 + 3 CH+2ĺ0Q2 + 3 HOCHCH2+.2+
usina para manter o televisor em stand-by?
O processo de amadurecimento começa quando a
A 0-
FRQFHQWUDomRGHHWLOHQRQRDUHVWiHPFHUFDGHPJ
de CH por kg de ar. B 0-
C 0-
$VPDVVDVPRODUHVGRVHOHPHQWRV+& 2 . H 0Q
VmRUHVSHFWLYDPHQWHLJXDLVDJPROJPROJPRO D 0-
JPROHJPRO E 0-

$ ¿P GH GLPLQXLU HVVDV SHUGDV VHP GHVSHUGtFLR GH QUESTÃO 56


UHDJHQWHVDPDVVDPtQLPDGH.0Q2 por kg de ar é mais
1DSUHSDUDomRGDPDVVDGRSmRSUHVHQWHQDPHVD
SUy[LPDGH GR FDIp GD PDLRULD GRV EUDVLOHLURV XWLOL]DVH R IXQJR
A PJ Saccharomyces cerevisiaeYLYRFRQWLGRQRIHUPHQWR6XD
¿QDOLGDGHpID]HUFRPTXHDPDVVDFUHVoDSRUPHLRGD
B 1,0 mg. SURGXomRGHJiVFDUE{QLFR
C PJ (VVH SURFHVVR TXtPLFR GH OLEHUDomR GH JiV p
D PJ causado pela
E PJ A JOLFRJrQHVHOiWLFD
B IHUPHQWDomRDOFRyOLFD
C SURGXomRGHiFLGROiWLFR
D SURGXomRGHODFWREDFLORV
E IRUPDomRGRiFLGRSLU~YLFR

&1žGLD_&DGHUQR%5$1&23iJLQD
*BR0975SAB22* 2016

QUESTÃO 57 QUESTÃO 59

$WXDOPHQWHVROGDGRVHPFDPSRVHMDHPWUHLQDPHQWR 2FODGRJUDPDUHSUHVHQWDUHODo}HV¿ORJHQpWLFDVHQWUH
RXHPFRPEDWHSRGHPDTXHFHUVXDVUHIHLo}HVSURQWDV os vertebrados.
HHPEDODGDVHPEROVDVSOiVWLFDVXWLOL]DQGRDTXHFHGRUHV
TXtPLFRVVHPSUHFLVDUID]HUIRJR'HQWURGHVVDVEROVDV
H[LVWH PDJQpVLR PHWiOLFR HP Sy H TXDQGR R VROGDGR PEIXES ANFÍBIOS RÉPTEIS AVES MAMÍFEROS
TXHUDTXHFHUDFRPLGDHOHFRORFDiJXDGHQWURGDEROVD V
SURPRYHQGRDUHDomRGHVFULWDSHODHTXDomRTXtPLFD IV

0J V +2 O ĺ0J 2+  V + J N-


O aquecimento dentro da bolsa ocorre por causa da
A UHGXomR VRIULGD SHOR R[LJrQLR TXH p XPD UHDomR III
H[RWpUPLFD
B R[LGDomR VRIULGD SHOR PDJQpVLR TXH p XPD UHDomR II
H[RWpUPLFD
I
C UHGXomR VRIULGD SHOR PDJQpVLR TXH p XPD UHDomR
endotérmica.
D R[LGDomRVRIULGDSHORKLGURJrQLRTXHpXPDUHDomR A correspondência correta entre as indicações numéricas
H[RWpUPLFD no cladograma e a característica evolutiva é:
E UHGXomR VRIULGD SHOR KLGURJrQLR TXH p XPD UHDomR
A ,±HQGRWHUPLD
endotérmica.
B ,,±FROXQDYHUWHEUDO
QUESTÃO 58 C ,,,±RYRDPQLyWLFR
$ XWLOL]DomR GH SODFDV GH DTXHFLPHQWR VRODU FRPR D ,9±UHVSLUDomRSXOPRQDU
alternativa ao uso de energia elétrica representa um E 9±PHPEURVORFRPRWRUHV
importante mecanismo de economia de recursos naturais.
Um sistema de aquecimento solar com capacidade de
JHUDomRGHHQHUJLDGH0-GLDSRUPHWURTXDGUDGRGH
SODFDIRLLQVWDODGRSDUDDTXHFHUDiJXDGHXPFKXYHLUR
HOpWULFRGHSRWrQFLDGHN:XWLOL]DGRGXUDQWHPHLDKRUD
por dia.
$iUHDPtQLPDGDSODFDVRODUGHYHVHUGH
A 1,0 m.
B P.
C P.
D P.
E P.

&1žGLD_&DGHUQR%5$1&23iJLQD
2016 *BR0975SAB23*
QUESTÃO 60 QUESTÃO 62
Nem sempre é seguro colocar vírus inteiros numa 3DUDFRPSDUDUDH¿FLrQFLDGHGLIHUHQWHVFRPEXVWtYHLV
YDFLQD $OJXQV VmR WmR SHULJRVRV TXH RV FLHQWLVWDV costuma-se determinar a quantidade de calor liberada na
SUHIHUHPXVDUVyXPGHVHXVJHQHV±DTXHOHTXHIDEULFD FRPEXVWmRSRUPRORXJUDPDGHFRPEXVWtYHO2TXDGUR
o antígeno, proteína que é reconhecida pelas células PRVWUDRYDORUGHHQHUJLDOLEHUDGDQDFRPEXVWmRFRPSOHWD
GHGHIHVD8PDGHVVDVYDFLQDVGHDOWDWHFQRORJLDpD de alguns combustíveis.
anti-hepatite B. Um gene do vírus é emendado ao DNA
GHXPIXQJRLQRIHQVLYRTXHSDVVDHQWmRDSURGX]LUXPD Combustível ǻ+C° a 25 °C (kJ/mol)
VXEVWkQFLDTXHpLQMHWDGDQRFRUSRKXPDQR
+LGURJrQLR +) 
Vírus: guerra silenciosa. SuperinteressanteQDJR DGDSWDGR 

$ IXQomR GHVVD VXEVWkQFLD SURGX]LGD SHOR IXQJR QR (WDQRO &HOH) 
organismo humano é 0HWDQR &+) 
A neutralizar proteínas virais.
0HWDQRO &+3OH) 
B LQWHUURPSHUDDomRGDVWR[LQDV
C OLJDUVHDRSDWyJHQRMiLQVWDODGR 2FWDQR &H) 
D UHFRQKHFHUVXEVWkQFLDVHVWUDQKDV $V PDVVDV PRODUHV GRV HOHPHQWRV + & H 2 VmR
E GHVHQFDGHDUDSURGXomRGHDQWLFRUSRV LJXDLVDJPROJPROHJPROUHVSHFWLYDPHQWH
ATKINS, P. Princípios de química3RUWR$OHJUH%RRNPDQ DGDSWDGR 
QUESTÃO 61
4XDO FRPEXVWtYHO DSUHVHQWD PDLRU OLEHUDomR GH HQHUJLD
3DUDUHFLFODUXPPRWRUGHSRWrQFLDHOpWULFDLJXDOD: por grama?
XP HVWXGDQWH FRQVWUXLX XP HOHYDGRU H YHUL¿FRX TXH HOH
IRLFDSD]GHHUJXHUXPDPDVVDGHNJDXPDDOWXUDGH A Hidrogênio.
 PHWURV GXUDQWH  PLQXWR &RQVLGHUH D DFHOHUDomR GDB Etanol.
JUDYLGDGHPV. C 0HWDQR
4XDO D H¿FLrQFLD DSUR[LPDGD GR VLVWHPD SDUD UHDOL]DU D 0HWDQRO
WDOWDUHID" E Octano.
A 10%
QUESTÃO 63
B 
C  A Caatinga é um ecossistema que se encontra nos
D  lados equatoriais dos desertos quentes, com índices
SOXYLRPpWULFRV PXLWR EDL[RV &KRYH SRXFR QR LQYHUQR
E 100% H DV FKXYDV TXDQGR RFRUUHP DFRQWHFHP QR YHUmR
$SUHVHQWD SODQWDV VHPHOKDQWHV jV GDV UHJL}HV GH
GHVHUWR TXHQWH GR WLSR [HUy¿WDV FRPR DV FDFWiFHDV
FRPDGDSWDo}HVjVFRQGLo}HVGHHVFDVVH]GHiJXD
SADAVA, D. et al. VidaDFLrQFLDGDELRORJLD3RUWR$OHJUH$UWPHG DGDSWDGR 

Uma característica que permite a sobrevivência dessas


SODQWDVQDFRQGLomRGDHVFDVVH]FLWDGDpDSUHVHQoDGH
A FDXOHVXEWHUUkQHR
B VLVWHPDUDGLFXODUIDVFLFXODGR
C IROKDVPRGL¿FDGDVHPHVSLQKRV
D SDUrQTXLPDDPLOtIHURGHVHQYROYLGR
E OLPERIROLDUGHVSURYLGRGHHVW{PDWRV

&1žGLD_&DGHUQR%5$1&23iJLQD
*BR0975SAB24* 2016

QUESTÃO 64
$ iJXD FRQVXPLGD QD PDLRULD GDV FLGDGHV EUDVLOHLUDV p REWLGD SHOR WUDWDPHQWR GD iJXD GH PDQDQFLDLV
$SDUWHLQLFLDOGRWUDWDPHQWRFRQVLVWHQRSHQHLUDPHQWRHVHGLPHQWDomRGHSDUWtFXODVPDLRUHV1DHWDSDVHJXLQWH
GLVVROYHPVHQDiJXDFDUERQDWRGHVyGLRHHPVHJXLGDVXOIDWRGHDOXPtQLR2UHVXOWDGRpDSUHFLSLWDomRGHKLGUy[LGR
GHDOXPtQLRTXHpSRXFRVRO~YHOHPiJXDRTXDOOHYDFRQVLJRDVSDUWtFXODVSROXHQWHVPHQRUHV3RVWHULRUPHQWHD
iJXDSDVVDSRUXPSURFHVVRGHGHVLQIHFomRH¿QDOPHQWHpGLVSRQLELOL]DGDSDUDRFRQVXPR
1RSURFHVVRGHVFULWRDSUHFLSLWDomRGHKLGUy[LGRGHDOXPtQLRpYLDELOL]DGDSRUTXH
A DGLVVROXomRGRDOXPtQLRUHVIULDDVROXomR
B RH[FHVVRGHVyGLRLPSRVVLELOLWDVXDVROXELOL]DomR
C DR[LGDomRSURYRFDGDSHORVXOIDWRSURGX]KLGUR[LODV
D DVSDUWtFXODVFRQWDPLQDQWHVPHQRUHVDWUDHPHVVDVXEVWkQFLD
E RHTXLOtEULRTXtPLFRGRFDUERQDWRHPiJXDWRUQDRPHLRDOFDOLQR

QUESTÃO 65

Almanaque do Biotônico'LVSRQtYHOHPZZZPLQLZHEFRPEU$FHVVRHPDEU DGDSWDGR 

2UyWXORGRSURGXWRGHVFUHYHFDUDFWHUtVWLFDVGHXPDGRHQoDTXHSRGHVHUSUHYHQLGDFRPR D
A uso de calçados.
B DSOLFDomRGHLQVHWLFLGD
C XWLOL]DomRGHPRVTXLWHLURV
D HOLPLQDomRGHiJXDSDUDGD
E VXEVWLWXLomRGHFDVDVGHEDUURSRUGHDOYHQDULD

&1žGLD_&DGHUQR%5$1&23iJLQD
2016 *BR0975SAB25*
QUESTÃO 66 QUESTÃO 69
A corrida dos 100 m rasos é uma das principais Todo ano, cresce a demanda mundial de energia
SURYDVGRDWOHWLVPRHTXDOL¿FDRKRPHPPDLVUiSLGRGR com o aumento das populações e do consumo. É cada
PXQGR8PFRUUHGRUGHHOLWHIRLFDSD]GHSHUFRUUHUHVVD YH] PDLV QHFHVViULR EXVFDU IRQWHV DOWHUQDWLYDV TXH QmR
GLVWkQFLDHPVFRPSDVVDGDV(OHLQLFLRXDFRUULGD degradem os recursos do planeta nem comprometam
com o pé direito. D VREUHYLYrQFLD GDV HVSpFLHV$LQGD Ki PXLWR R TXH VH
2 SHUtRGR GH RVFLODomR GR Sp GLUHLWR GHVVH FRUUHGRU IRL GHVFREULU VREUH R XVR H¿FLHQWH GH UHFXUVRV HQHUJpWLFRV
PDLVSUy[LPRGH SURYHQLHQWHVGHIRQWHVUHQRYiYHLVPDVHODVHVWmRPDLV
SUy[LPDVGRTXHSDUHFHGDDGRomRHPODUJDHVFDOD
A V %$5%26$0$VXVWHQWDELOLGDGHGDHQHUJLDUHQRYiYHOSuperinteressanteQ
B V
2VUHFXUVRVHQHUJpWLFRVGRWLSRFLWDGRVmRSURYHQLHQWHVGH
C V
D V A pilhas e baterias.
E V B usinas nucleares e hidrelétricas.
C FpOXODVVRODUHVHJHUDGRUHVHyOLFRV
QUESTÃO 67 D centrais geotérmicas e termoelétricas.
$SyV D JHUPLQDomR QRUPDOPHQWH RV WRPDWHV E XVLQDVPDUHPRWUL]HVHFRPEXVWtYHLVIyVVHLV
SURGX]HP XPD SURWHtQD TXH RV ID] DPROHFHU GHSRLV GH
colhidos. Os cientistas introduziram, em um tomateiro, QUESTÃO 70
XPJHQHDQWLVVHQWLGR LPDJHPHVSHOKRGRJHQHQDWXUDO 
jTXHOHTXHFRGL¿FDDHQ]LPD³DPROHFHGRUD´2QRYRJHQH $ WHOHIRQLD PyYHO QR %UDVLO RSHUD FRP FHOXODUHV
antissentido bloqueou a síntese da proteína amolecedora. FXMDSRWrQFLDPpGLDGHUDGLDomRpFHUFDGH:3RU
UHFRPHQGDomR GR$16,,((( IRUDP HVWLSXODGRV OLPLWHV
6,=(5):+,71(<( NutriçãoFRQFHLWRVHFRQWURYpUVLDV%DUXHUL0DQROH DGDSWDGR 
SDUD H[SRVLomR KXPDQD j UDGLDomR HPLWLGD SRU HVVHV
8PEHQHItFLRDRVHREWHURWRPDWHWUDQVJrQLFRIRLRIDWR DSDUHOKRV 3DUD R DWHQGLPHQWR GHVVD UHFRPHQGDomR
GHRSURFHVVRELRWHFQROyJLFRWHU valem os conselhos: segurar o aparelho a uma pequena
GLVWkQFLD GR RXYLGR XVDU IRQHV GH RXYLGR SDUD DV
A DXPHQWDGR D FROHomR GH SURWHtQDV TXH R SURWHJHP chamadas de voz e utilizar o aparelho no modo viva voz
GR DSRGUHFLPHQWR SHOD SURGXomR GD SURWHtQD ou com dispositivos bluetooth. Essas medidas baseiam-se
antissentido. QR IDWR GH TXH D LQWHQVLGDGH GD UDGLDomR HPLWLGD GHFDL
B diminuído a necessidade do controle das pragas, pela UDSLGDPHQWH FRQIRUPH D GLVWkQFLD DXPHQWD SRU LVVR
PDLRUUHVLVWrQFLDFRQIHULGDSHODQRYDSURWHtQD DIDVWDURDSDUHOKRUHGX]ULVFRV
C IDFLOLWDGR D JHUPLQDomR GDV VHPHQWHV SHOD IDOWD GD COSTA, E. A. F. Efeitos na saúde humana da exposição aos campos de radiofrequência.
proteína que o leva a amolecer. 'LVSRQtYHOHPZZZFHGXIVFEU$FHVVRHPQRY DGDSWDGR 

D substituído a proteína amolecedora por uma invertida, 3DUDUHGX]LUDH[SRVLomRjUDGLDomRGRFHOXODUGHIRUPD


que endurece o tomate. PDLVH¿FLHQWHRXVXiULRGHYHXWOL]DU
E SURORQJDGRRWHPSRGHYLGDGRWRPDWHSHODIDOWDGD
A IRQHVGHRXYLGRFRPRDSDUHOKRQDPmR
proteína que o amolece.
B IRQHVGHRXYLGRFRPRDSDUHOKRQREROVRGDFDOoD
QUESTÃO 68 C IRQHVbluetooth, com o aparelho no bolso da camisa.
(P PtGLDV ySWLFDV FRPR &'V '9'V H blue-rays, a D R DSDUHOKR PDQWLGR D  FP GR RXYLGR VHJXUDGR
LQIRUPDomRpUHSUHVHQWDGDQDIRUPDGHbits ]HURVHXQV  SHODPmR
H p ¿VLFDPHQWH JUDYDGD H OLGD SRU IHL[HV GH OX] laser. E o sistema viva voz, com o aparelho apoiado numa
3DUDJUDYDUXPYDORU³]HUR´Rlaser brilha intensamente, mesa de trabalho.
GH PRGR D ³TXHLPDU´ WRUQDU RSDFD  XPD SHTXHQD iUHD
GRGLVFRGHWDPDQKRFRPSDUiYHODVHXFRPSULPHQWRGH
onda. Ao longo dos anos, as empresas de tecnologia vêm
conseguindo aumentar a capacidade de armazenamento
GH GDGRV HP FDGD GLVFR HP RXWUDV SDODYUDV D iUHD
usada para se representar um bit vem se tornando cada
vez mais reduzida.
4XDO DOWHUDomR GD RQGD HOHWURPDJQpWLFD TXH FRQVWLWXL R
laser SHUPLWHRDYDQoRWHFQROyJLFRFLWDGRQRWH[WR"
A $GLPLQXLomRGHVXDHQHUJLD
B 2DXPHQWRGHVXDIUHTXrQFLD
C $GLPLQXLomRGHVXDDPSOLWXGH
D O aumento de sua intensidade.
E $GLPLQXLomRGHVXDYHORFLGDGH

&1žGLD_&DGHUQR%5$1&23iJLQD
*BR0975SAB26* 2016

QUESTÃO 71
$EXVFDSRUVXEVWkQFLDVFDSD]HVGHPLQLPL]DUDDomRGRLQVHWRTXHDWDFDDVSODQWDo}HVGHWRPDWHQR%UDVLOOHYRX
jVtQWHVHHDRHPSUHJRGHXPIHURP{QLRVH[XDOFRPDVHJXLQWHIyUPXODHVWUXWXUDO
OCOCH3

8PDLQG~VWULDDJURTXtPLFDQHFHVVLWDVLQWHWL]DUXPGHULYDGRFRPPDLRUH¿FiFLD3DUDWDQWRRSRWHQFLDOVXEVWLWXWR
GHYHUi SUHVHUYDU DV VHJXLQWHV SURSULHGDGHV HVWUXWXUDLV GR IHURP{QLR VH[XDO IXQomR RUJkQLFD FDGHLD QRUPDO H D
isomeria geométrica original.
$IyUPXODHVWUXWXUDOGRVXEVWLWXWRDGHTXDGRDRIHURP{QLRVH[XDOREWLGRLQGXVWULDOPHQWHp

COCH3
A

B OCOCH3

C OCOCH3

O COCH3
D

OCOCH3
E

QUESTÃO 72

'LVSRQtYHOHPKWWSDQJHOREUDQFREORJVSRWFRPEU$FHVVRHPPDLR

1DWLULQKDRSURFHVVRPHQFLRQDGRSHORPROXVFRHVWiUHODFLRQDGRDXPPHFDQLVPRGH
A GHIHVD
B H[FUHomR
C FLUFXODomR
D ORFRPRomR
E UHSURGXomR

&1žGLD_&DGHUQR%5$1&23iJLQD
2016 *BR0975SAB27*
QUESTÃO 73
2VUDLRV;XWLOL]DGRVSDUDGLDJQyVWLFRVPpGLFRVVmRXPDUDGLDomRLRQL]DQWH2HIHLWRGDVUDGLDo}HVLRQL]DQWHVHP
XPLQGLYtGXRGHSHQGHEDVLFDPHQWHGDGRVHDEVRUYLGDGRWHPSRGHH[SRVLomRHGDIRUPDGDH[SRVLomRFRQIRUPH
relacionados no quadro.

Efeitos de uma radioexposição aguda em adulto

Forma Dose absorvida Sintomatologia

,QIUDFOtQLFD 0HQRUTXH-NJ Ausência de sintomas

Reações gerais $VWHQLDQiXVHDVHY{PLWRGHKDKDSyVD


GHD-NJ
leves H[SRVLomR

'/ GHD-NJ 0RUWHGHGRVLQGLYtGXRVLUUDGLDGRV

,QVX¿FLrQFLDUHVSLUDWyULDDJXGD
Pulmonar GHD-NJ
FRPDHPRUWHGHKDK

Cerebral 0DLRUTXH-NJ 0RUWHHPSRXFDVKRUDV


'LVSRQtYHOHPZZZFQHQJRYEU$FHVVRHPVHW DGDSWDGR 

3DUDXPWpFQLFRUDGLRORJLVWDGHNJTXH¿FRXH[SRVWRSRUGHVFXLGRGXUDQWHKRUDVDXPDIRQWHGHUDLRV;FXMD
SRWrQFLDpGHP-VDIRUPDGRVLQWRPDDSUHVHQWDGRFRQVLGHUDQGRTXHWRGDUDGLDomRLQFLGHQWHIRLDEVRUYLGDp
A '/.
B cerebral.
C pulmonar.
D LQIUDFOtQLFD
E reações gerais leves.

QUESTÃO 74
$ViJXDVGRVRFHDQRVDSUHVHQWDPXPDDOWDFRQFHQWUDomRGHtRQVHS+HQWUHH'HQWUHHVVHVtRQVHVWmR
HPHTXLOtEULRDVHVSpFLHVFDUERQDWR &23í HELFDUERQDWR +&23í UHSUHVHQWDGRSHODHTXDomRTXtPLFD

HCO3 DT  CO3 DT  H DT


$V iJXDV GRV ULRV DR FRQWUiULR DSUHVHQWDP FRQFHQWUDo}HV PXLWR EDL[DV GH tRQV H VXEVWkQFLDV EiVLFDV FRP
XPS+HPWRUQRGH$DOWHUDomRVLJQL¿FDWLYDGRS+GDViJXDVGRVULRVHRFHDQRVSRGHPXGDUVXDVFRPSRVLo}HV
TXtPLFDVSRUSUHFLSLWDomRGHHVSpFLHVGLVVROYLGDVRXUHGLVVROXomRGHHVSpFLHVSUHVHQWHVQRVVyOLGRVVXVSHQVRVRX
nos sedimentos.
$FRPSRVLomRGRVRFHDQRVpPHQRVDIHWDGDSHORODQoDPHQWRGHHÀXHQWHViFLGRVSRLVRVRFHDQRV
A FRQWrPJUDQGHTXDQWLGDGHGHFORUHWRGHVyGLR
B FRQWrPXPYROXPHGHiJXDSXUDPHQRUTXHRGRVULRV
C SRVVXHPS+iFLGRQmRVHQGRDIHWDGRVSHODDGLomRGHRXWURViFLGRV
D WrPDIRUPDomRGRVtRQVFDUERQDWRIDYRUHFLGDSHODDGLomRGHiFLGR
E DSUHVHQWDPXPHTXLOtEULRHQWUHRVtRQVFDUERQDWRHELFDUERQDWRTXHDWXDPFRPRVLVWHPDWDPSmR

&1žGLD_&DGHUQR%5$1&23iJLQD
*BR0975SAB28* 2016

QUESTÃO 75 QUESTÃO 77

Nos ambientes tropicais, os modelos convencionais $ ¿JXUD DSUHVHQWD GLIHUHQWHV IDVHV GR FLFOR GH XPD
GH SURGXomR DJUtFROD WrP JHUDGR GHJUDGDomR GRV FpOXODVRPiWLFDFXOWLYDGDHIRWRJUDIDGDHPPLFURVFySLR
UHFXUVRV QDWXUDLV H XP PDQHMR FDGD YH] PDLV FDUR FRQIRFDO GH YDUUHGXUD D laser. As partes mais claras
H WUDEDOKRVR 3HOD OHJLVODomR EUDVLOHLUD RV VLVWHPDV evidenciam o DNA.
DJURÀRUHVWDLV 6$)V VmRVLVWHPDVGHXVRHRFXSDomRGR
VRORHPTXHSODQWDVOHQKRVDVSHUHQHVVmRPDQHMDGDVHP
DVVRFLDomRFRPSODQWDVKHUEiFHDVDUEXVWLYDVDUEyUHDV
FXOWXUDVDJUtFRODVHIRUUDJHLUDVHPXPDPHVPDXQLGDGH
GH PDQHMR GH DFRUGR FRP DUUDQMR HVSDFLDO H WHPSRUDO
com alta diversidade de espécies e interações entre
esses componentes.
'LVSRQtYHOHPZZZDPELHQWHGXUDQHQJEU$FHVVRHPDJR DGDSWDGR 

2V 6$)V VmR DWXDOPHQWH PXLWR DGRWDGRV FRPR


HVWUDWpJLDGHPDQHMRDPELHQWDOQR%UDVLOSRUTXH
A JDUDQWHPDSURGXomRGHSODQWDVH[yWLFDV
B SRVVLELOLWDPDPDQXWHQomRGHPRQRFXOWXUDWtSLFD
C DXPHQWDPDSURGXomRFRPFXOWXUDVWUDQVJrQLFDV
D SHUPLWHPDXWLOL]DomRGRVRORFRPFXOWXUDVGLYHUVDV
E IDYRUHFHP D DGDSWDomR GH SODQWDV OHQKRVDV -8148(,5$/&&$51(,52-Histologia básica. Rio de Janeiro:
*XDQDEDUD.RRJDQ DGDSWDGR 
madeireiras.
1D IDVH UHSUHVHQWDGD HP ' REVHUYDVH TXH RV
QUESTÃO 76 cromossomos encontram-se em
Combustíveis automotivos têm sido adulterados A PLJUDomR
SHOD DGLomR GH VXEVWkQFLDV RX PDWHULDLV GH EDL[R YDORU B GXSOLFDomR
FRPHUFLDO (VVH WLSR GH FRQWUDYHQomR SRGH GDQL¿FDU RV C FRQGHQVDomR
PRWRUHVDXPHQWDURFRQVXPRGHFRPEXVWtYHOHSUHMXGLFDU
D UHFRPELQDomR
R PHLR DPELHQWH 9iULRV WHVWHV ODERUDWRULDLV SRGHP VHU
XWLOL]DGRV SDUD LGHQWL¿FDU VH XP FRPEXVWtYHO HVWi RX E UHHVWUXWXUDomR
QmR DGXOWHUDGR $ OHJLVODomR EUDVLOHLUD HVWDEHOHFH TXH QUESTÃO 78
R GLHVHO REWLGR GR SHWUyOHR FRQWHQKD FHUWD TXDQWLGDGH
de biodiesel. O quadro apresenta valores de quatro (PGHGH]HPEURGHXPtsunami devastador,
SURSULHGDGHV GR GLHVHO GR ELRGLHVHO H GR yOHR YHJHWDO originado a partir de um terremoto na costa da Indonésia,
um material comumente utilizado como adulterante. atingiu diversos países da Ásia, matando quase 300 mil
SHVVRDV2JUDXGHGHYDVWDomRGHYHXVHHPERDSDUWH
Óleo DRIDWRGHDVRQGDVGHXPtsunamiVHUHPH[WUHPDPHQWH
Propriedade Diesel Biodiesel
vegetal ORQJDVFRPFRPSULPHQWRGHRQGDGHFHUFDGHNP
'HQVLGDGH JFPñ    ,VWRpPXLWRPDLRUTXHDHVSHVVXUDGDOkPLQDGHOtTXLGRd,
WtSLFDGR2FHDQRËQGLFRTXHpGHFHUFDGHNP1HVVDV
3RGHUFDORUt¿FR 0-/  33,3  FRQGLo}HV FRP ERD DSUR[LPDomR D VXD YHORFLGDGH GH
9LVFRVLGDGH PPðV 3,9   SURSDJDomR WRUQDVH GHSHQGHQWH GH d REHGHFHQGR j
UHODomRv = ¥gd 1HVVDH[SUHVVmRg pDDFHOHUDomRGD
7HRUGHHQ[RIUH  1,3 < 0,001 < 0,001 JUDYLGDGHTXHSRGHVHUWRPDGDFRPRPV.
6,/9(,5$)/9$55,$/(0&3URSDJDomRGDVRQGDVPDUtWLPDVHGRVtsunami.
&RPEDVHQDVLQIRUPDo}HVDSUHVHQWDGDVQRTXDGURTXDLV Caderno Brasileiro de Ensino de FísicaQ DGDSWDGR 
VmR DV GXDV SURSULHGDGHV TXH SRGHP VHU HPSUHJDGDV
Sabendo-se que o tsunami consiste em uma série de
WHFQLFDPHQWH SDUD YHUL¿FDU VH XPD DPRVWUD GH GLHVHO
RQGDVVXFHVVLYDVTXDOpRYDORUPDLVSUy[LPRGRLQWHUYDOR
FRPHUFLDOHVWiRXQmRDGXOWHUDGDFRPyOHRYHJHWDO"
de tempo entre duas ondas consecutivas?
A Densidade e viscosidade. A 1 min
B 7HRUGHHQ[RIUHHGHQVLGDGH B PLQ
C 9LVFRVLGDGHHWHRUGHHQ[RIUH C PLQ
D 9LVFRVLGDGHHSRGHUFDORUt¿FR D PLQ
E 3RGHUFDORUt¿FRHWHRUGHHQ[RIUH E PLQ

&1žGLD_&DGHUQR%5$1&23iJLQD
2016 *BR0975SAB29*
QUESTÃO 79 QUESTÃO 81

'XUDQWH D IRUPDomR GH XPD WHPSHVWDGH VmR 2 SURFHVVR GH GHVVXOIXUL]DomR p XPD GDV HWDSDV
REVHUYDGDV YiULDV GHVFDUJDV HOpWULFDV RV UDLRV TXH XWLOL]DGDVQDSURGXomRGRGLHVHO(VVHSURFHVVRFRQVLVWH
SRGHP RFRUUHU GDV QXYHQV SDUD R VROR GHVFDUJD QD R[LGDomR GR HQ[RIUH SUHVHQWH QD IRUPD GH VXOIHWR
GHVFHQGHQWH  GR VROR SDUD DV QXYHQV GHVFDUJD GH KLGURJrQLR +6  D HQ[RIUH HOHPHQWDU VyOLGR  TXH p
ascendente) ou entre uma nuvem e outra. As descargas SRVWHULRUPHQWHUHPRYLGR8PPpWRGRSDUDHVVDH[WUDomR
ascendentes e descendentes podem ocorrer por causa química é o processo Claus, no qual parte do HS é
GR DF~PXOR GH FDUJDV HOpWULFDV SRVLWLYDV RX QHJDWLYDV R[LGDGDDGLy[LGRGHHQ[RIUH 62 HHQWmRHVVHJiVp
TXHLQGX]XPDSRODUL]DomRRSRVWDQRVROR XVDGR SDUD R[LGDU R UHVWDQWH GR +S. Os compostos de
Essas descargas elétricas ocorrem devido ao aumento da HQ[RIUHUHPDQHVFHQWHVHDVGHPDLVPROpFXODVSUHVHQWHV
LQWHQVLGDGHGR D QRGLHVHOVRIUHUmRFRPEXVWmRQRPRWRU
0$548(6),/+2-Estudo da fase térmica do processo Claus utilizando
A campo magnético da Terra. ÀXLGRGLQkPLFDFRPSXWDFLRQDO6mR3DXOR863 DGDSWDGR 

B corrente elétrica gerada dentro das nuvens.


2EHQHItFLRGRSURFHVVR&ODXVpTXHQDFRPEXVWmRGR
C resistividade elétrica do ar entre as nuvens e o solo. GLHVHOpPLQLPL]DGDDHPLVVmRGHJDVHV
D FDPSR HOpWULFR HQWUH DV QXYHQV H D VXSHUItFLH
A IRUPDGRUHVGHKLGURFDUERQHWRV
da Terra.
E IRUoD HOHWURPRWUL] LQGX]LGD QDV FDUJDV DFXPXODGDV B SURGXWRUHVGHy[LGRVGHQLWURJrQLR
no solo. C HPLVVRUHVGHPRQy[LGRGHFDUERQR
D SURPRWRUHVGDDFLGL¿FDomRGDFKXYD
QUESTÃO 80
E GHWHUPLQDQWHVSDUDRDXPHQWRGRHIHLWRHVWXID
2V LQGLYtGXRV GH XPD SRSXODomR GH XPD SHTXHQD
FLGDGH IXQGDGD SRU XPD IDPtOLD GH HXURSHXV VmR QUESTÃO 82
IUHTXHQWHPHQWH IUXWRV GH FDVDPHQWRV FRQVDQJXtQHRV
*UDQGH SDUWH GRV JUXSRV IDPLOLDUHV GHVVD ORFDOLGDGH 2 FKRTXH HOpWULFR p XPD VHQVDomR SURYRFDGD
apresenta membros acometidos por uma doença rara, pela passagem de corrente elétrica pelo corpo.
LGHQWL¿FDGDSRUIUDTXH]DPXVFXODUSURJUHVVLYDFRPLQtFLR $VFRQVHTXrQFLDVGHXPFKRTXHYmRGHVGHXPVLPSOHV
DRVDQRVGHLGDGH(PIDPtOLDVFRPSUHVHQoDGHVVD VXVWR DWp D PRUWH $ FLUFXODomR GDV FDUJDV HOpWULFDV
GRHQoDTXDQGRRVSDLVVmRVDXGiYHLVVRPHQWHRV¿OKRV depende da resistência do material. Para o corpo humano,
GRVH[RPDVFXOLQRSRGHPVHUDIHWDGRV0DVHPIDPtOLDV HVVD UHVLVWrQFLD YDULD GH   ȍ TXDQGR D SHOH HVWi
FXMR SDL p DFRPHWLGR SHOD GRHQoD H D PmH p SRUWDGRUD PROKDGDDWpȍTXDQGRDSHOHHVWiVHFD8PD
GR JHQH  GD GHVFHQGrQFLD LQGHSHQGHQWHPHQWH GR SHVVRD GHVFDOoD ODYDQGR VXD FDVD FRP iJXD PROKRX
VH[RpDIHWDGD RVSpVHDFLGHQWDOPHQWHSLVRXHPXP¿RGHVHQFDSDGR
&RQVLGHUDQGR DV FDUDFWHUtVWLFDV SRSXODFLRQDLV R VH[R VRIUHQGRXPDGHVFDUJDHOpWULFDHPXPDWHQVmRGH9
H D SURSRUomR GRV LQGLYtGXRV DIHWDGRV TXDO p R WLSR GH 4XDO D LQWHQVLGDGH Pi[LPD GH FRUUHQWH HOpWULFD TXH
KHUDQoDGDGRHQoDGHVFULWDQRWH[WR" passou pelo corpo da pessoa?
A Recessiva, ligada ao cromossomo X. A P$
B Dominante, ligada ao cromossomo X.
B P$
C Recessiva, ligada ao cromossomo Y.
C $
D Recessiva autossômica.
D $
E Dominante autossômica.
E N$

&1žGLD_&DGHUQR%5$1&23iJLQD
*BR0975SAB30* 2016

QUESTÃO 83 QUESTÃO 84
(PXPODERUDWyULRVmRDSUHVHQWDGRVDRVDOXQRVXPD $V HPLVV}HV GH GLy[LGR GH FDUERQR &2) por
OkPSDGDFRPHVSHFLILFDo}HVWpFQLFDVGH9H: YHtFXORV VmR GHSHQGHQWHV GD FRQVWLWXLomR GH FDGD WLSR
HXPFRQMXQWRGHSLOKDVGH9FDGD de combustível. Sabe-se que é possível determinar a
4XDO DVVRFLDomR GH JHUDGRUHV ID] FRP TXH D OkPSDGD quantidade emitida de CO, a partir das massas molares
produza maior brilho? GRFDUERQRHGRR[LJrQLRLJXDLVDJPROHJPRO
UHVSHFWLYDPHQWH(PXPDYLDJHPGHIpULDVXPLQGLYtGXR
SHUFRUUHXNPHPXPYHtFXORTXHFRQVRPHXPOLWURGH
JDVROLQDDFDGDNPGHSHUFXUVR
&RQVLGHUDQGR TXH R FRQWH~GR GH FDUERQR HP XP OLWUR
A GHVVDJDVROLQDpLJXDODNJDPDVVDGH&2 emitida
SHOR YHtFXOR QR DPELHQWH GXUDQWH D YLDJHP GH IpULDV
descrita, é igual a
A NJ
B 33 kg.
C NJ
D NJ
E NJ
B
QUESTÃO 85

3DUD YHUL¿FDU D H¿FiFLD GR WHVWH GH '1$ QD


GHWHUPLQDomR GH SDWHUQLGDGH FLQFR YROXQWiULRV GHQWUH
HOHV R SDL ELROyJLFR GH XP JDURWR FHGHUDP DPRVWUDV
ELROyJLFDVSDUDDUHDOL]DomRGHVVHWHVWH$¿JXUDPRVWUD
RUHVXOWDGRREWLGRDSyVDLGHQWL¿FDomRGRVIUDJPHQWRVGH
DNA de cada um deles.
C Garoto Mãe 1º 2º 3º 4º 5º

2/,9(,5$)%6,/9(,5$5092WHVWHGH'1$QDVDODGHDXODpSRVVtYHOHQVLQDU
biologia a partir de temas atuais. Revista Genética na EscolaDEU

$SyVDDQiOLVHGDVEDQGDVGH'1$SRGHVHFRQFOXLUTXH
RSDLELROyJLFRGRJDURWRpR
E
A ƒYROXQWiULR
B ƒYROXQWiULR
C ƒYROXQWiULR
D ƒYROXQWiULR
E ƒYROXQWiULR

&1žGLD_&DGHUQR%5$1&23iJLQD
2016 *BR0975SAB31*
QUESTÃO 86 QUESTÃO 88
Para o consumidor, é praticamente impossível 2 GHVFDUWH GR yOHR GH FR]LQKD QD UHGH GH HVJRWRV
LGHQWL¿FDU D GLIHUHQoD HQWUH D VDFROD ELRGHJUDGiYHO H D gera diversos problemas ambientais. Pode-se destacar a
FRPXPIHLWDGHSROLHWLOHQR±GHULYDGRGRSHWUyOHR$OJXQV FRQWDPLQDomRGRVFXUVRVG¶iJXDTXHWHPFRPRXPDGDV
JRYHUQRV PXQLFLSDLV Mi H[LJHP TXH RV VXSHUPHUFDGRV FRQVHTXrQFLDV D IRUPDomR GH XPD SHOtFXOD GH yOHR QD
RIHUHoDP VDFRODV ELRGHJUDGiYHLV HP VXEVWLWXLomR jV VXSHUItFLHFDXVDQGRGDQRVjIDXQDDTXiWLFDSRUGL¿FXOWDU
sacolas comuns. DV WURFDV JDVRVDV DOpP GH GLPLQXLU D SHQHWUDomR GRV
'LVSRQtYHOHPKWWSHSRFDQHJRFLRVJORERFRP$FHVVRHPDJR raios solares no curso hídrico.
'LVSRQtYHOHPKWWSUHYLVWDJDOLOHXJORERFRP$FHVVRHPDJR DGDSWDGR 
$DWLWXGHWRPDGDSHORVJRYHUQRVPXQLFLSDLVGHYHVHDR j
A PDLRU UHVLVWrQFLD TXH RV PDWHULDLV ELRGHJUDGiYHLV 4XDO GDV SURSULHGDGHV GRV yOHRV YHJHWDLV HVWi
DSUHVHQWDPHPUHODomRDRVFRPXQV relacionada aos problemas ambientais citados?
B HVFDVVH]GDVPDWpULDVSULPDVGHULYDGDVGRSHWUyOHR A $OWDPLVFLELOLGDGHHPiJXD
SDUDSURGXomRGDVVDFRODVFRPXQV B $OWDUHDWLYLGDGHFRPDiJXD
C custo consideravelmente menor das sacolas C %DL[DGHQVLGDGHHPUHODomRjiJXD
ELRGHJUDGiYHLVHPUHODomRDRGDVVDFRODVFRPXQV
D %DL[DYLVFRVLGDGHHPUHODomRjiJXD
D PDLRU FDSDFLGDGH GH SURGXomR GDV VDFRODV
E $OWRSRQWRGHHEXOLomRHPUHODomRjiJXD
ELRGHJUDGiYHLV Mi TXH DV IRQWHV SRGHP VHU
UHQRYiYHLV
QUESTÃO 89
E UiSLGD GHFRPSRVLomR GDV VDFRODV ELRGHJUDGiYHLV
SHOD DomR GH EDFWpULDV HP FRPSDUDomR jV VDFRODV Um produtor rural registrou queda de produtividade
comuns. QXPDGDViUHDVGHSODQWLRGHDUUR]GHVXDSURSULHGDGH
$QiOLVHV TXtPLFDV UHYHODUDP FRQFHQWUDo}HV HOHYDGDV
QUESTÃO 87 GR tRQ DP{QLR 1+  H EDL[DV GRV tRQV QLWULWR 12) e
QLWUDWR 123 QRVROR(VVHVFRPSRVWRVQLWURJHQDGRVVmR
Algumas crianças, ao brincarem de esconde- QHFHVViULRVSDUDRFUHVFLPHQWRGRVYHJHWDLVHSDUWLFLSDP
HVFRQGHWDSDPRVROKRVFRPDVPmRVDFUHGLWDQGRTXH do ciclo biogeoquímico do nitrogênio.
DRDGRWDUHPWDOSURFHGLPHQWRQmRSRGHUmRVHUYLVWDV
(P TXDO HWDSD GHVVH FLFOR ELRJHRTXtPLFR VmR
(VVD SHUFHSomR GD FULDQoD FRQWUDULD R FRQKHFLPHQWR IRUPDGRV RV FRPSRVWRV TXH HVWmR HP EDL[D
FLHQWt¿FRSRUTXHSDUDVHUHPYLVWRVRVREMHWRV FRQFHQWUDomRQHVVHVROR"
A UHIOHWHP SDUWtFXODV GH OX] IyWRQV  TXH DWLQJHP A 1LWUL¿FDomR
os olhos.
B $VVLPLODomR
B JHUDP SDUWtFXODV GH OX] IyWRQV  FRQYHUWLGDV SHOD
C $PRQL]DomR
IRQWHH[WHUQD
D 'HVQLWUL¿FDomR
C VmRDWLQJLGRVSRUSDUWtFXODVGHOX] IyWRQV HPLWLGDV
pelos olhos. E )L[DomRGHQLWURJrQLR
D UHÀHWHP SDUWtFXODV GH OX] IyWRQV  TXH VH FKRFDP
FRPRVIyWRQVHPLWLGRVSHORVROKRV QUESTÃO 90
E VmRDWLQJLGRVSHODVSDUWtFXODVGHOX] IyWRQV HPLWLGDV $REWHQomRGHHQHUJLDSRUPHLRGD¿VVmRQXFOHDUGR
SHODIRQWHH[WHUQDHSHORVROKRV 
8pPXLWRVXSHULRUTXDQGRFRPSDUDGDjFRPEXVWmRGD
JDVROLQD2FDORUOLEHUDGRQD¿VVmRGR8pu1010-J
HQDFRPEXVWmRGDJDVROLQDpu10-J
$ PDVVD GH JDVROLQD QHFHVViULD SDUD REWHU D PHVPD
HQHUJLDQD¿VVmRGHNJGHU é da ordem de
A 103 g.
B 10 g.
C 10 g.
D 10 g.
E 109 g.

&1žGLD_&DGHUQR%5$1&23iJLQD
*BR0975SAB32*

2016

&1žGLD_&DGHUQR%5$1&23iJLQD
EXAME NACIONAL DO ENSINO MÉDIO
PROVA DE REDAÇÃO E DE LINGUAGENS, CÓDIGOS E SUAS TECNOLOGIAS
PROVA DE MATEMÁTICA E SUAS TECNOLOGIAS

2º DIA
CADERNO

2016 16
3ª APLICAÇÃO

ATENÇÃO: transcreva no espaço apropriado do seu CARTÃO-RESPOSTA,


FRPVXDFDOLJUD¿DXVXDOFRQVLGHUDQGRDVOHWUDVPDL~VFXODVHPLQ~VFXODVDVHJXLQWHIUDVH

,QWHU¿UDPQRPHXYLYHUREMHWLYR

LEIA ATENTAMENTE AS INSTRUÇÕES SEGUINTES:  3DUDFDGDXPDGDVTXHVW}HVREMHWLYDVVmRDSUHVHQWDGDV


RSo}HV$SHQDVXPDUHVSRQGHFRUUHWDPHQWHjTXHVWmR
 Este CADERNO DE QUESTÕES contém a Proposta de
Redação e 90 questões numeradas de 91 a 180, dispostas  2 WHPSR GLVSRQtYHO SDUD HVWDV SURYDV p GH FLQFR KRUDV H
da seguinte maneira: WULQWDPLQXWRV
D  DV TXHVW}HV GH Q~PHUR  D  VmR UHODWLYDV j iUHD GH
Linguagens, Códigos e suas Tecnologias;  5HVHUYHRVPLQXWRV¿QDLVSDUDPDUFDUVHXCARTÃO-RESPOSTA
2VUDVFXQKRVHDVPDUFDo}HVDVVLQDODGDVQR&$'(512'(
E  DVTXHVW}HVGHQ~PHURDVmRUHODWLYDVjiUHDGH
0DWHPiWLFDHVXDV7HFQRORJLDV 48(67®(6QmRVHUmRFRQVLGHUDGRVQDDYDOLDomR

ATENÇÃO DV TXHVW}HV GH  D  VmR UHODWLYDV j OtQJXD  Somente serão corrigidas as redações transcritas na FOLHA
HVWUDQJHLUD 9RFr GHYHUi UHVSRQGHU DSHQDV jV TXHVW}HV '(5('$d­2
UHODWLYDVjOtQJXDHVWUDQJHLUD LQJOrVRXHVSDQKRO HVFROKLGD
QRDWRGHVXDLQVFULomR  4XDQGRWHUPLQDUDVSURYDVDFHQHSDUDFKDPDURDSOLFDGRU
e entregue este CADERNO DE QUESTÕES e o CARTÃO-
 &RQ¿UD Ve o seu CADERNO DE QUESTÕES contém a
5(63267$)2/+$'(5('$d­2
quantidade de questões e se essas questões estão na ordem
PHQFLRQDGD QD LQVWUXomR DQWHULRU &DVR R FDGHUQR HVWHMD  9RFrSRGHUiGHL[DURORFDOGHSURYDVRPHQWHDSyVGHFRUULGDV
LQFRPSOHWR WHQKD GHIHLWR RX DSUHVHQWH TXDOTXHU GLYHUJrQFLD GXDV KRUDV GR LQtFLR GD DSOLFDomR H SRGHUi OHYDU VHX
comunique ao aplicador da sala para que ele tome as &$'(512'(48(67®(6DRGHL[DUHPGH¿QLWLYRDVDODGH
provLGrQFLDVFDEtYHLV SURYDQRVPLQXWRVTXHDQWHFHGHPRWpUPLQRGDVSURYDV

0LQLVWpULR
GD(GXFDomR *CZ1625dom1*
*CZ1625DOM2* 2016

LINGUAGENS, CÓDIGOS E SUAS 48(67­2


TECNOLOGIAS 2QWKH0HDQLQJRI%HLQJ&KLQHVH
(WKQLFDOO\VSHDNLQJ,IHHO,DPFRPSOLFDWHGWRFODVVLI\
4XHVW}HVGHD EXW ZKR LVQ¶W ULJKW" 7R PH EHLQJ &KLQHVH%UD]LOLDQ
LQ $PHULFD PHDQV D KLVWRU\ RI OLYLQJ LQ WKUHH RSSRVLWH
4XHVW}HVGHD RSomRLQJOrV FXOWXUHV DQG VRPHWLPHV IHHOLQJ WKDW , GLG QRW EHORQJ LQ
QHLWKHUDFRQVWDQWVWUXJJOHWKDWLPPLJUDQWVDQGQDWLRQDO
48(67­2 FLWL]HQVIDFHZKHQWKHLUDSSHDUDQFHLVIRUHLJQWRQDWLYHV
LQWKHFRXQWU\-RNLQJO\,VD\WKDW,DP$VLDQLQ$PHULFD
+XQJHU*DPHV5HYLHZ)DPLO\)LOP*XLGH %UD]LOLDQLQ&KLQDDQGD³JULQJD´LQ%UD]LO1HYHUWKHOHVV
,EHOLHYHWKDWGHDOLQJZLWKWKHVHKDUGWRUHFRQFLOHH[WUHPHV
3DUHQW &RQFHUQV 7KHUH LV GH¿QLWHO\ YLROHQFH LQ WKLV KDYH VRPHKRZ KHOSHG PH WR EHFRPH PRUH FRPIRUWDEOH
¿OP7KHFHQWUDOHunger GamesPD\QRWEHDVEORRG\DQG ZLWKP\LGHQWLW\
EUXWDODVDXWKRU6X]DQQH&ROOLQVGHVFULEHVLQWKHQRYHOEXW
%(/(=$/,'LVSRQtYHOHPZZZDLLVIRUJ$FHVVRHPPDU
WKHUH¶VDYLVFHUDOUHDFWLRQWRVHHLQJWKHNLGRQNLGYLROHQFH
UDWKHUWKDQFRQMXULQJLWLQ\RXURZQLPDJLQDWLRQ7KHWULEXWHV 1HVVH IUDJPHQWR %HOH]D /L UHVXPH VXD H[SHULrQFLD GH
NLOOHDFKRWKHULQDKRVWRIZD\VIURPVSHDUNQLIHDQGDUURZ YLGDDRGHVFUHYHUDFRPSOH[LGDGHHP
ZRXQGVWRKDQGWRKDQGEDWWOHVWKDWOHDYHWHHQVZLWKWKHLU
KHDGVVPDVKHGLQRUQHFNVVQDSSHG7KHHGLWLQJLVTXLFN A YLYHUFRPRLPLJUDQWHHPXPSDtVDVLiWLFR
DQGWKHVKRWVQHYHUOLQJHURQDQ\WKLQJRYHUO\JUDSKLFEXW B GH¿QLUTXHPHODpQRTXHFRQFHUQHjHWQLD
WKHUHLVEORRGDQGWZHQW\WZRDGROHVFHQWVDJHGGLH C FRPSUHHQGHUDVFXOWXUDVTXHDFRQVWLWXHP
LQWKHDQQXDOEORRGVSRUWSDJHDQW,PPDWXUHWHHQVHYHQLI D OLGDUFRPEULQFDGHLUDVVREUHVXDDSDUrQFLD
WKH\¶YHUHDGWKHERRNVPD\QRWEHUHDG\WRKDQGOHWRWKH E OXWDUFRQWUDDGLVFULPLQDomRQRV(VWDGRV8QLGRV
¿OPMXVW\HW$JRRGUXOHRIWKXPELIWKH\¶UHQRWROGHQRXJK 48(67­2
WREHUHDSHGLQWRWKHHunger GamesWKH\¶UHSUREDEO\QRW
PDWXUHHQRXJKWRVHHLW
ANGUL2&+(16'LVSRQtYHOHPKWWSQHZVPRYLHIRQHFRP$FHVVRHPMXQ

Produções OLWHUiULDV H FLQHPDWRJUi¿FDV HVWmR PXLWDV


YH]HV DUWLFXODGDV 1R FDVR GR ¿OPH Hunger Games, a
DXWRUD GD UHVHQKD FKDPD D DWHQomR SDUD D TXHVWmR GD
YLROrQFLDTXHpPDLV
A GHWDOKDGDGRTXHDDXWRUDGROLYURJRVWDULDTXHIRVVH
B EUXWDOGRTXHRVSDLVSHUPLWLULDPSDUDVHXV¿OKRV
C DPHQDGRTXHRVDGROHVFHQWHVLPDJLQDYDP
D VXSHU¿FLDOGRTXHRS~EOLFRSRGHULDHVSHUDU
E impactanteGRTXHDUHSUHVHQWDGDQROLYUR

48(67­2

*HWWLQJ(YHU\&KLOGWR6FKRRO
5LJKWQRZPLOOLRQFKLOGUHQDUHPLVVLQJRXWRQWKHLU
ULJKW WR DQ HGXFDWLRQ 7KH\ FDQ¶W JR WR VFKRRO EHFDXVH
WKH\ KDYH WR ZRUN WR VXUYLYH EHFDXVH WKH\ DUH JLUOV RU
HYHQEHFDXVHWKHUHDUHQRVFKRROVZKHUHWKH\OLYH
:KHUH ZLOO WKHVH FKLOGUHQ EH ZKHQ WKH\ JURZ XS
ZLWKRXWWKHFKDQFHWROHDUQ"
:H¶UHZRUNLQJWRPDNHVXUHHYHU\FKLOGJRHVWRVFKRRO
:H¶UH KHOSLQJ EXLOG VFKRROV WUDLQ WHDFKHUV DGYRFDWLQJ
JLUO¶VHGXFDWLRQDQGUHDFKLQJFKLOGUHQZKRKDYHWRZRUNRU
DUHFDXJKWXSLQHPHUJHQFLHVZLWKOHDUQLQJ
<RX FDQ KHOS XSKROG HYHU\ FKLOG¶V ULJKW WR DQ
HGXFDWLRQ0DNHDGRQDWLRQWRGD\WRQRWRQO\JLYHFKLOGUHQ
WKHFKDQFHWRJRWRVFKRROEXWDOVRVDYHWKHLUOLYHVDQG
SURWHFWWKHLUFKLOGKRRGV
'LVSRQtYHOHPZZZVXSSRUWXQLFHIRUJ $FHVVRHPPDLR DGDSWDGR 
$%22/60'LVSRQtYHOHPKWWSEORJHFOHFWLFPHPHVFRP$FHVVRHPMDQ
(VVD FDPSDQKD SUHWHQGH FRQWULEXLU SDUD GLPLQXLU D
$VLJOD³$9´XVDGDQRWH[WRUHSUHVHQWDRDOYRGDFUtWLFD
GHVLJXDOGDGHVRFLDOXPDYH]TXH VREUHDHOHYDomRGRFXVWRGHXP D
A GHQXQFLDRWUDEDOKRGHPHQRUHV A HQTXHWHVREUHSUR¿VV}HVHPEDL[D
B DSRQWDPRWLYRVSDUDDHYDVmRHVFRODU B UHIHUHQGRUHDOL]DGRSHORJRYHUQR
C GLYXOJDRQ~PHURGHFULDQoDVIRUDGDHVFROD C WUDWDPHQWRPpGLFRLQRYDGRU
D GHIHQGHDUHIRUPDGHSROtWLFDVHGXFDFLRQDLV D softwareGH~OWLPDJHUDomR
E SHGHDMXGDSDUDJDUDQWLUjVFULDQoDVRGLUHLWRjHGXFDomR E QRYRVLVWHPDHOHLWRUDO

/&žGLD_&DGHUQR&,1=$3iJLQD
2016 *CZ1625DOM3*
48(67­2 LINGUAGENS, CÓDIGOS E SUAS
$UH7ZLWWHUDQG)DFHERRN$IIHFWLQJ+RZ:H7KLQN"
TECNOLOGIAS
4XHVW}HVGHD
Is constant use of electronic gadgets reshaping our
brains and making our thinking shallower? 4XHVW}HVGHD RSomRHVSDQKRO
%\1HLO7ZHHGLH
48(67­2
+RZ PDQ\ WLPHV GR \RX FOLFN RQ \RXU HPDLO LFRQ LQ
D GD\" 2U ORRN DW )DFHERRN RU7ZLWWHU"$QG KRZ PDQ\ 'HMDGDODJHQWHFRUUHU
WLPHVZKHQUHDGLQJRQWKHLQWHUQHWGR\RXFOLFNRQDOLQN
QDYLJDWLQJDZD\IURPWKHWH[WWKDWZDVWKHRULJLQDOREMHFW 1RKDEUiPDUDWyQHQORVSUy[LPRVDxRVHQODTXHORV
RI\RXUHQTXLU\"7KHZHELWVHHPVLVOLNHDQHOHFWURQLF FRUUHGRUHV QR VLHQWDQ OD PH]FOD GH WHPRU \ GH UHVSHWR
VZHHWVKRSIRUHYHUWHPSWLQJXVLQGLIIHUHQWGLUHFWLRQV%XW SRU ODV YtFWLPDV TXH VH GHVSUHQGH LQHYLWDEOHPHQWH GHO
GRHVWKLVPHQWDOSURPLVFXLW\WKLVWHQGHQF\WRÀLWDURXQG DWHQWDGRWHUURULVWDSHUSHWUDGRHQ%RVWRQHOGHDEULOGH
RQOLQHPDNHXVZHOOWKLFNHU"
(OORHVXQDFWRFDVLUHÀHMRGHLQTXLHWXGGHSpUGLGD
1LFKRODV&DUUWKH$PHULFDQVFLHQFHZULWHUKDVPLQHG de cierta inocencia en un evento convocado para unir a
WKLVWKHPHIRUKLVQHZERRN³7KH6KDOORZV´LQZKLFKKH SHUVRQDVGHSURFHGHQFLDVPX\GLVWLQWDVVLQLPSRUWDUPiV
DUJXHV WKDW QHZ PHGLD DUH QRW MXVW FKDQJLQJ RXU KDELWV FLUFXQVWDQFLDVLGHRORJtDVRFUHGRV
EXW RXU EUDLQV ,W WXUQV RXW WKDW WKH PDWXUH KXPDQ EUDLQ
LVQRWDQLPPXWDEOHVHDWRISHUVRQDOLW\DQGLQWHOOHFWEXWD $QWHV GH OD 3ULPHUD 0DUDWyQ GH &LVMRUGDQLD ORV
FKDQJHDEOH WKLQJ VXEMHFW WR ³QHXURSODVWLFLW\´ :KHQ RXU RUJDQL]DGRUHV \ SDUWLFLSDQWHV GH HVWD VH UHXQLHURQ HQ
DFWLYLWLHVDOWHUVRGRHVWKHDUFKLWHFWXUHRIRXUEUDLQ³,¶P %HOpQHQXQDYLJLOLDHQODTXHFRQYHODVKRPHQDMHDURQD
QRWWKLQNLQJWKHZD\,XVHGWRWKLQN´ZULWHV&DUU³,IHHOLW ODVYtFWLPDVGHODPDVDFUHRUTXHVWDGDSRUORVKHUPDQRV
PRVWVWURQJO\ZKHQ,¶PUHDGLQJ´
7VDUQDHY³7RGDODJHQWHWLHQHHOGHUHFKRDFRUUHU´VHOHtD
'LVSRQtYHOHPZZZWHOHJUDSKFRXN$FHVVRHPIHY
HQVXVSDQFDUWDV
1HLO 7ZHHGLH OHYDQWD YiULRV TXHVWLRQDPHQWRV VREUH D
XWLOL]DomRGHGLIHUHQWHVUHFXUVRVWHFQROyJLFRVGLVSRQtYHLV /D 3ULPHUD 0DUDWyQ GH &LVMRUGDQLD RUJDQL]DGD SRU
KRMH HP GLD $ SDUWLU GHVVHV TXHVWLRQDPHQWRV H GRV HO JUXSR LQGHSHQGLHQWH 'HUHFKR DO 0RYLPLHQWR OXFtD
DUJXPHQWRVGRHVFULWRUQRUWHDPHULFDQR1LFKRODV&DUUR como lema una breve cita de la Declaración Universal
WH[WRVXJHUHTXH de los Derechos Humanos³7RGDSHUVRQDWLHQHGHUHFKR
A R DWR GH FOLFDU HP tFRQHV H PDQXVHDU DSDUHOKRV D FLUFXODU OLEUHPHQWH´ /RV DJHQWHV GH SROLFtD SDOHVWLQRV
SUHMXGLFDRFRPSRUWDPHQWR KDEtDQ UHGREODGR OD VHJXULGDG HQ XQD PHGLGD PiV GH
B RPXQGRYLUWXDOSRGHVHUQRFLYRDRVMRYHQVSRUVHU SXURDFWRUHÀHMRTXHRWUDFRVD
PXLWRSURPtVFXR 0XFKDV VRQ ODV FDUJDV GHO SXHEOR SDOHVWLQR D QLYHO
C a internet contribui para o amadurecimento intelectual GHJREHUQDQ]DLQWHUQD\SRULPSRVLFLRQHVHQ,VUDHOSHUR
GRVXVXiULRV un ataque terrorista a los corredores no era realmente una
D R XVR LQWHQVR GH UHFXUVRV WHFQROyJLFRV SRGH DIHWDU SRVLELOLGDG)LQDOPHQWHSDUWLFLSDURQFRQWRWDOQRUPDOLGDG
QRVVRFpUHEUR  FRUUHGRUHV GH  SDtVHV (O  HUDQ SDOHVWLQRV
E DV UHGHV VRFLDLV YLUWXDLV DMXGDP D PHOKRUDU QRVVD Necesariamente, la maratón discurrió en varios tramos
IRUPDGHSHQVDU IUHQWHDOPXURHULJLGRSRU,VUDHO\DWUDYHVyGRVFDPSRV
GHUHIXJLDGRV
$/$1'(7(''LVSRQtYHOHPKWWSEORJVHOSDLVFRP$FHVVRHPDEU DGDSWDGR 

1R WH[WR VmR DERUGDGDV DV FLUFXQVWkQFLDV HP


TXH DFRQWHFHX D SULPHLUD PDUDWRQD UHDOL]DGD QD
&LVMRUGkQLD 3DOHVWLQD 2VHQYROYLGRVQHVVDPDUDWRQD
SURSXVHUDPXPOHPDHFRQIHFFLRQDUDPIDL[DVQDVTXDLV
reivindicavam a
A garantia de segurança em provas de atletismo e no
FRWLGLDQR
B PHOKRULDGDVYLDVGHDFHVVRHGDVLQVWDODo}HVHVSRUWLYDV
C SUHVHQoDGRVSDOHVWLQRVHPFRPSHWLo}HVLQWHUQDFLRQDLV
D SXQLomRGRVFXOSDGRVSRUDWRVGHWHUURULVPR
E OLEHUGDGHGHLUHYLUHGHSUDWLFDUHVSRUWHV

/&žGLD_&DGHUQR&,1=$3iJLQD
*CZ1625DOM4* 2016

48(67­2 48(67­2

0HGLRPLOOyQGHSHUVRQDVHQ/LPDKDEOD
XQDOHQJXDLQGtJHQD
4XHFKXD DLPDUD DVKDQLQND FDXTXL MDTDUX
PDWVLJHQND \ VKLSLERNRQLER VRQ OHQJXDV RULJLQDULDV
TXH WLHQHQ DOJR HQ FRP~Q WRGDV FRQYLYHQ HQ /LPD \
KR\FRPRWRGRGHPD\RVRQUHFRUGDGDVFRPRSDUWH
GHO 'tDGHO ,GLRPD1DWLYR (QOD FDSLWDO H[LVWH DO PHQRV
PHGLR PLOOyQ GH KDELWDQWHV TXH VH FRPXQLFDQ D WUDYpV
GHVLHWHGHODVOHQJXDVLQGtJHQDVTXHH[LVWHQHQWRGR
HO3HU~6RORHQHOFDVRGHTXHFKXDKDEODQWHV HQ/LPD
SRGHPRVHQFRQWUDUDOPHQRVPLOPiVGHPLOFX\D
OHQJXD RULJLQDULD HV HO DLPDUD   DVKDQLQND  
VKLSLERNRQLER\MDTDUX$JXVWtQ3DQL]ROLQJLVWDGHO
0LQLVWHULRGH&XOWXUDGHVWDFyTXHVLELHQHQORV~OWLPRV 'LVSRQtYHOHPZZZHIDURLQIR$FHVVRHPQRY DGDSWDGR 
DxRVVHKDDYDQ]DQGRHQHOUHFRQRFLPLHQWRGHOGHUHFKR
GH TXH FDGD FLXGDGDQR KDEOH VX LGLRPD QDWLYR WRGDYtD $ FKDUJH DSUHVHQWD XPD LQWHUSUHWDomR GRV HIHLWRV GD
KDFHIDOWDPiVGLIXVLyQVREUHODLPSRUWDQFLDGHUHVSHWDUODV FULVHHFRQ{PLFDHVSDQKRODHTXHVWLRQDR D
\SUHVHUYDUODV6HJ~QGDWRVGHO0LQLVWHULRGH&XOWXUDHQ
HO3HU~H[LVWHQOHQJXDVLQGtJHQDVKDEODGDVSRUPiV A GHFLVmRSROtWLFDGHVDOYDUDPRHGD~QLFDHXURSHLD
GHFXDWURPLOORQHVGHKDELWDQWHV1RREVWDQWHVHFDOFXOD B FRQJHODPHQWRGRVVDOiULRVGRVIXQFLRQiULRV
TXHDOPHQRVOHQJXDVQDWLYDVVHKDQH[WLQJXLGR\TXH
GHODVVREUHYLYLHQWHVHVWiQHQSHOLJURGHGHVDSDUHFHU C DSDWLDGDSRSXODomRHPUHODomRjSROtWLFD
'LVSRQtYHOHPKWWSHOFRPHUFLRSH$FHVVRHPMXO D FRQ¿DQoDGRVFLGDGmRVQRVLVWHPDEDQFiULR
$ GLYHUVLGDGH OLQJXtVWLFD p DQXDOPHQWH WUDWDGD QR E SODQRGRJRYHUQRSDUDVDOYDULQVWLWXLo}HV¿QDQFHLUDV
'tD GHO ,GLRPD 1DWLYR HP /LPD 1R WH[WR R GHVD¿R
apontado em relação a essa questão é 48(67­2
A GHOLQHDURTXDQWLWDWLYRGHOtQJXDVQDWLYDVUHPDQHVFHQWHV
'HWDOSDORWDODVWLOOD
B GHVSHUWDUSDUDDQHFHVVLGDGHGHSURWHJHUDVOtQJXDV
LQGtJHQDV &XDQGR 0LFKDHO $FXxD LQJUHVy HQ OD $FDGHP\ RI
C LQFHQWLYDU D FRPHPRUDomR GD VREUHYLYrQFLD GDV &XLVLQH HQ HO HVWDGR GH 0DU\ODQG \D KDFtD PXFKRV
OtQJXDVQDWLYDV VROHVTXHHUDXQH[FHOHQWHFRFLQHUR(VTXHVXVSDGUHV
D ID]HU R OHYDQWDPHQWR HVWDWtVWLFR GRV IDODQWHV GDV 0DQXHO\$OELWDIXHURQSURSLHWDULRVGH(O0HVyQ7LFRHQ
OtQJXDVQDWLYDV 0DGULG\GHVGHQLxR0LFKDHOQRVDOtDGHODFRFLQD
E PDQWHU D VRFLHGDGH DWXDOL]DGD VREUH D UHDOLGDGH
OLQJXtVWLFDSHUXDQD <D JUDGXDGR WUDEDMy HQ :DVKLQJWRQ HQ )LORPHQD¶V
)RXU 6HDVRQV HQWUH RWURV SUHVWLJLRVRV OXJDUHV &XDQGR
48(67­2 VXIDPLOLDUHJUHVyDVXSDtV&RVWD5LFD\DEULy/DV7DSDV
GH0DQXHODOHVWHGH6DQ-RVp±ODFDSLWDO±SURQWRVH
OHVUHXQLy
(O p[LWR QR VH KL]R HVSHUDU LQPHGLDWD DPSOLDFLyQ
SULPHUR \ OXHJR XQ VHJXQGR UHVWDXUDQWH HVWD YH] DO
RHVWHGHODFLXGDGFRQWDEODGRÀDPHQFR\XQDOHJUHEDU
0iV GH YHLQWLFLQFR WDSDV FOLHQWHV ¿HOHV TXH OOHJDQ
XQD \ RWUD YH] \ XQD FDOLGDG FRQVWDQWH WHVWLPRQLDQ OD
UD]yQGHVXp[LWR
52660$PHULFDQ$LUOLQHV1H[RVQPDU

2 WtWXOR GR WH[WR WUD] XPD H[SUHVVmR LGLRPiWLFD (VVD


H[SUHVVmR YLQFXODGD jV LQIRUPDo}HV GR WH[WR UHIRUoD
/Ï3(=$3HVFDGR'LVSRQtYHOHPKWWSEORJVSXEOLFRHV$FHVVRHPDJR TXHRVXFHVVRDOFDQoDGRSRU0LFKDHO$FXxDGHYHVHDR
(VVDFKDUJHWHPDIXQomRGHGHQXQFLDULURQLFDPHQWHR D IDWRGHHOHWHU
A UHEHOGLDGRV¿OKRVHPUHODomRjDOLPHQWDomR A HVWXGDGRHPXPDLQVWLWXLomRUHQRPDGD
B FRQWDPLQDomRGRVDOLPHQWRVLQJHULGRVSHODVRFLHGDGH B WUDEDOKDGRHPUHVWDXUDQWHVLQWHUQDFLRQDLV
C LQDGHTXDomRGRVKiELWRVDOLPHQWDUHVGDVRFLHGDGHDWXDO C DEHUWRVHXSULPHLURHPSUHHQGLPHQWRLQGLYLGXDO
D DXWRULWDULVPR GDV PmHV QD HVFROKD GD DOLPHQWDomR D YROWDGRjVUDt]HVJDVWURQ{PLFDVGHVHXSDtVGHRULJHP
GRV¿OKRV
E IDOWD GH KDELOLGDGH GD PXOKHU PRGHUQD QR SUHSDUR E FRQYLYLGR GHVGH D LQIkQFLD QR XQLYHUVR FXOLQiULR
GDVUHIHLo}HV GDIDPtOLD

/&žGLD_&DGHUQR&,1=$3iJLQD
2016 *CZ1625DOM5*
4XHVW}HVGHD135 2WH[WRaborda a linguagem como um campo de disputas
HSRGHU$VLQWHUURJDo}HVGDDXWRUDVmRHVWUDWpJLDVTXH
48(67­2 FRQGX]HPDRFRQYHQFLPHQWRGROHLWRUGHTXH

$FDUUHLUDQDVDOWXUDV A RSRUWXJXrVGR%UDVLOpPXLWRGLIHUHQWHGRSRUWXJXrV
GH3RUWXJDO
$iJXDHVWiQRMRHOKRGRVSUR¿VVLRQDLVGRPHUFDGR
$VIUDJLOLGDGHVQDIRUPDomRHP/tQJXD3RUWXJXHVDWrP B DV SUHVFULo}HV GRV JUDPiWLFRV HVWmR D VHUYLoR GDV
DOLPHQWDGRXPFDPSRGHUHFLFODJHPHP3RUWXJXrVQDV FODVVHVGRPLQDQWHV
escolas de idiomas e nos cursos de graduação para
SHVVRDVRULXQGDVGRPXQGRGRVQHJyFLRV2TXHDQWHV C D QRUPD OLQJXtVWLFD GD HOLWH EUDVLOHLUD p D ~QLFD
HUDUHVWULWRDSUR¿VVLRQDLVGHHGXFDomRHFRPXQLFDomR UHFRQKHFLGDFRPRWDO
DJRUD Mi ID] SDUWH GD URWLQD GH SUR¿VVLRQDLV GH YiULDV D R SRUWXJXrV GR %UDVLO Ki PXLWR GLVWDQFLRXVH GDV
iUHDV 3DUD HOHV D /tQJXD 3RUWXJXHVD FRPHoD D VHU
DVVLPLODGD FRPR XPD IHUUDPHQWD SDUD R GHVHPSHQKR SUHVFULo}HVGRVJUDPiWLFRV
HVWiYHO 6HP HOD R FRQKHFLPHQWR WpFQLFR ¿FD UHVWULWR j E D GHVYDORUL]DomR GDV YDULHGDGHV OLQJXtVWLFDV
SUySULDSHVVRDTXHQmRVDEHFRPXQLFiOR SRSXODUHVWHPPRWLYDomRVRFLDO
(PERUDDOJXPDVDWXDo}HVH[LMDPXPDSURGXomRRUDO
RX HVFULWD PDLV IUHTXHQWH FRPR GRFrQFLD H DGYRFDFLD 48(67­2
PXLWRV SUR¿VVLRQDLV SUHFLVDP HVFUHYHU UHODWyULR FDUWD
FRPXQLFDGR FLUFXODU 1D OLQJXDJHP RUDO WRGRV WrP GH &KHJRX GH 0RQWHV &ODURV XPD LUPm GD QRUD GH WLD
H[SUHVVDUVH GH IRUPD FRQYLQFHQWH QDV UHXQL}HV SDUD &ODULQKD H IRL YLVLWDU WLD$JRVWLQKD QR -RJR GD %ROD (OD
JDQKDU UHVSHLWR H FUHGLELOLGDGH ,VVR YDOH SDUD WRGRV RV p ERQLWD VLPSiWLFD H YHVWHVH PXLWR EHP >@ )LFDUDP
FDUJRVGDKLHUDUTXLDSUR¿VVLRQDO±H[SOLFDXPDSURIHVVRUD
GH/tQJXD3RUWXJXHVDGD)DFXOGDGHGH)LORVR¿D/HWUDVH WRGDV DV WLDV DGPLUDGDV GD EHOH]D GD PRoD H GH VHXV
&LrQFLDV+XPDQDVGD863 PRGRV SROtWLFRV GH FRQYHUVDU )DODYD H[SOLFDGR H WXGR
1$7$/,$5HYLVWD/tQJXDQMDQ DGDSWDGR  PXLWRFRUUHWR'L]LD³YRFr´HPYH]GH³RFr´3DODYUDTXH
HX QXQFD WLQKD YLVWR QLQJXpP IDODU WmR EHP WXGR FRPR
1RV XVRV FRWLGLDQRV GD OtQJXD DOJXPDV H[SUHVV}HV
SRGHPDVVXPLUGLIHUHQWHVVHQWLGRV1RWH[WRDH[SUHVVmR VH HVFUHYH VHP HQJROLU XP V QHP XP U 7LD $JRVWLQKD
³DiJXDHVWiQRMRHOKR´UHPHWHj PDQGRXYLUXPDEDQGHMDGHXYDVHOKHSHUJXQWRXVHHOD
A H[LJrQFLD GH DSURIXQGDPHQWR HP FRQKHFLPHQWRV JRVWDYDGHXYDV(ODUHVSRQGHX³$SUHFLRVREUHPDQHLUD
WpFQLFRV XPFDFKRGHXYDV'RQD$JRVWLQKD´(VWDVSDODYUDVQRV
B GHPDQGD SRU IRUPDomR SUR¿VVLRQDO GH SURIHVVRUHV ¿]HUDP¿FDUGHTXHL[RFDtGR'HSRLVHODIRLSDVVHDUFRP
HDGYRJDGRV RXWUDVH,DLiDSURYHLWRXSDUDOKHID]HUHORJLRVHFRPSDUDU
C SURFXUD SRU HVFRODV GH LGLRPDV SDUD R DSUHQGL]DGR FRQRVFR (OD GL]LD ³9RFrV QmR WLYHUDP LQYHMD GH YHU
GHOtQJXDV XPD PRoD >@ IDODU WmR ERQLWR FRPR HOD" 9RFrV GHYHP
D PHOKRULD GR GHVHPSHQKR SUR¿VVLRQDO QDV YiULDV DSURYHLWDU D FRPSDQKLD GHOD SDUD DSUHQGHUHP´ >@ 1D
iUHDVGRFRQKHFLPHQWR KRUDGRMDQWDUHXHDVSULPDVFRPHoDPRVDGL]HUSDUD
E QHFHVVLGDGH LPHGLDWD GH DSHUIHLoRDPHQWR GDV HQIH]DU ,DLi ³$SUHFLR VREUHPDQHLUD DV EDWDWDV IULWDV´
KDELOLGDGHVFRPXQLFDWLYDV
³$SUHFLRVREUHPDQHLUDXPDFR[DGHJDOLQKD´
025/(<+0LQKDYLGDGHPHQLQDFDGHUQRVGHXPDPHQLQDSURYLQFLDQDQRV¿QVGR
48(67­2 VpFXOR;,;5LRGH-DQHLUR-RVp2O\PSLR

$LQGDRVHTXtYRFRVQRFRPEDWH 1HVVH WH[WR QR TXH GL] UHVSHLWR DR YRFDEXOiULR


DRVHVWUDQJHLULVPRV empregado pela moça de Montes Claros, a narradora
3RUTXHQmRVHUHFRQKHFHDH[LVWrQFLDGHQRUPDQDV H[S}HXPDYLVmRLQGLFDWLYDGH
YDULHGDGHVSRSXODUHV"3DUDGHVTXDOL¿FiODV"3RUTXHVy A GHVFDVR XPD YH] TXH GHVDSURYD R XVR IRUPDO GD
XPDQRUPDpUHFRQKHFLGDFRPRQRUPDHQmRSRUDFDVR
DGDHOLWH" OtQJXDHPSUHJDGRSHODPRoD

3RU WDQWRV HTXtYRFRV Vy QRV UHVWD ODPHQWDU TXH B LURQLDXPDYH]TXHLQFRUSRUDRYRFDEXOiULRIRUPDOGD


DOJXPDV SHVVRDV LPEXtGDV GD FUHQoD GH TXH HVWmR PRoDQDVLWXDomRIDPLOLDU
GHIHQGHQGRDOtQJXDDLGHQWLGDGHHDSiWULDQDYHUGDGH C DGPLUDomRSHORIDWRGHGHOHLWDUVHFRPRYRFDEXOiULR
HVWHMDPUHIRUoDQGRYHOKRVSUHFRQFHLWRVHLPSRVLo}HV2
HPSUHJDGRSHODPRoD
SRUWXJXrVGR%UDVLOKiPXLWRGLVWDQFLRXVHGRSRUWXJXrV
GH 3RUWXJDO H GDV SUHVFULo}HV GRV JUDPiWLFRV FXMR D DQWLSDWLD SHOR IDWR GH FRELoDU RV HORJLRV GH ,DLi
VHUYLoRjVFODVVHVGRPLQDQWHVpGH¿QLUDOtQJXDGRSRGHU VREUHDPRoD
HPIDFHGHDPHDoDV±LQWHUQDVHH[WHUQDV
E LQGLJQDomR XPD YH] TXH FRQWHVWD DV DWLWXGHV
=,//(6$06,Q)$5$&2&$ 2UJ (VWUDQJHLULVPRVJXHUUDVHPWRUQRGDOtQJXD
6mR3DXOR3DUiEROD DGDSWDGR  GDPRoD

/&žGLD_&DGHUQR&,1=$3iJLQD
*CZ1625DOM6* 2016

48(67­2
%DLmRpXPULWPRSRSXODUGD5HJLmR1RUGHVWHGR%UDVLOGHULYDGRGHXPWLSRGHOXQGXGHQRPLQDGR³EDLDQR´FXMR
QRPHpFRUUXSWHOD1DVFHXVREDLQÀXrQFLDGRFDQWRFKmRFDQWROLW~UJLFRGD,JUHMD&DWyOLFDSUDWLFDGRSHORVPLVVLRQiULRV
HWRUQRXVHH[SUHVVLYDIRUPDPRGL¿FDGDSHODLQFRQVFLHQWHLQÀXrQFLDGHPDQLIHVWDo}HVORFDLV8PGRVJUDQGHVVXFHVVRV
YHLRFRPDP~VLFDKRP{QLPDBaião  GH/XL]*RQ]DJDH+XPEHUWR7HL[HLUD
&$6&8'2&'LFLRQiULRGRIROFORUHEUDVLOHLUR5LRGH-DQHLUR(GLRXUR DGDSWDGR 

2VHOHPHQWRVUHJLRQDLVTXHLQÀXHQFLDUDPFXOWXUDOPHQWHREDLmRDSDUHFHPHPRXWUDVIRUPDVDUWtVWLFDVHSRGHPVHU
YHUL¿FDGRVQDREUD

A D

Samba em terreiro+HLWRUGRV3UD]HUHV Lampião a cavalo0HVWUH9LWDOLQR

B E

9LROHLUR-RVp)HUUD]$OPHLGD-U
$PRODGRUGHIDFDV$GDOWRQ/RSHV

Folia de Reis5RVD*DXGLWDQR
QUESTÃO 100
RR%UDVLOQRPHXSRQWRGHYLVWDHQWHQGHX"RSDtVVyFUHVFHDWUDYpVGDHGXFDomRHQWHQGHX"(XSHQVRDVVLP
HQWmRTXHUGL]HUYRFrGDQGRXPDSULRULGDGHSUDSUDHGXFDomRDWHQGrQFLDpPHOKRUDUPDLVHQWHQGHX"HDV
SHVVRDVFRPRHXSRVVRH[SOLFDUDVVLP"DVSHVVRDVLUHPWRPDQGRFRQKHFLPHQWRPDLVGDVFRLVDVQp"SRUTXHHX
DFKRTXHDSLRUFRLVDTXHWHPpDSHVVRDDOLHQDGDQp"DSHVVRDTXHQmRWHPQRomRGHQDGDHQWHQGHX"
7UHFKRGDIDODGH-/VH[RPDVFXOLQRDQRV,Q9275(62/,9(,5$05 &RRUG $OtQJXDIDODGDHHVFULWDQDFLGDGHGR5LRGH-DQHLUR
'LVSRQtYHOHPZZZGLVFXUVRHJUDPDWLFDOHWUDVXIUMEU$FHVVRHPGH]

$OtQJXDIDODGDFDUDFWHUL]DVHSRUKHVLWDo}HVSDXVDVHRXWUDVSHFXOLDULGDGHV$VRFRUUrQFLDVGH³HQWHQGHX´H³Qp´, na
IDODGH-/LQGLFDPTXH
A a modalidade oraODSUHVHQWDSRXFRVUHFXUVRVFRPXQLFDWLYRVVHFRPSDUDGDjPRGDOLGDGHHVFULWD
B DOtQJXDIDODGDpPDUFDGDSRUSDODYUDVGLVSHQViYHLVHLUUHOHYDQWHVSDUDRHVWDEHOHFLPHQWRGDLQWHUDomR
C RHQXQFLDGRUSURFXUDLQWHUSHODURVHXLQWHUORFXWRUSDUDPDQWHURÀX[RFRPXQLFDWLYR
D RWHPDWUDWDGRQRWH[WRWHPDOWRJUDXGHFRPSOH[LGDGHHpGHVFRQKHFLGRGRHQWUHYLVWDGRU
E RIDODQWHPDQLIHVWDLQVHJXUDQoDDRDERUGDURDVVXQWRGHYLGRDRJrQHURVHUXPDHQWUHYLVWD
/&žGLD_&DGHUQR&,1=$3iJLQD
2016 *CZ1625DOM7*
QUESTÃO 101 QUESTÃO 103

0DULD'LDPED TEXTO I
3DUDQmRDSDQKDUPDLV QRYRVYHtFXORVSRUGLDQRHVWDGR
IDORXTXHVDELDID]HUERORV Frota, que chega a quase 1,4 milhão, deve
YLURXFR]LQKD dobrar em 13 anos
)RLRXWUDVFRLVDVSDUDTXHWLQKDMHLWR $ FDGD GLD XPD PpGLD GH  QRYRV YHtFXORV
1mRIDORXPDLV FKHJD jV UXDV GR (VStULWR 6DQWR VHJXQGR GDGRV
GR 'HSDUWDPHQWR (VWDGXDO GH 7UkQVLWR 'HWUDQ(6 
9LUDPTXHVDELDID]HUWXGR
1R ¿QDO GR PrV SDVVDGR D IURWD Mi HUD GH   
DWpPROHFDVSDUDD&DVD*UDQGH XQLGDGHV  PLO D PDLV GR TXH QR PHVPR PrV GH
'HSRLVIDORXVy  2V Q~PHURV LQFOXHP DXWRPyYHLV PRWRFLFOHWDV
só diante da ventania FDPLQK}HV H {QLEXV HQWUH RXWURV WLSRV 'H GH]HPEUR
SDUD Fi R FUHVFLPHQWR IRL GH PDLV GH  PLO YHtFXORV
que ainda vem do Sudão;
( VH HVVH ULWPR FRQWLQXDU D IURWD GR (VStULWR 6DQWR
IDORXTXHTXHULDIXJLU YDL GREUDU DWp  2 GLUHWRUJHUDO GR 'HWUDQ(6
GRVVHQKRUHVHGDVMXGLDULDVGHVWHPXQGR UHODFLRQD R FUHVFLPHQWR GHVVHV Q~PHURV j IDFLOLGDGH
SDUDRVXPLGRXUR HQFRQWUDGD SDUD VH FRPSUDU XP YHtFXOR ³+i WRGD XPD
TXHVWmR HFRQ{PLFD GD IDFLOLGDGH GH FUpGLWR &RPR
/,0$-3RHPDVQHJURV. 5LRGH-DQHLUR5HFRUG
RIHUHFHPRV XP WUDQVSRUWH FROHWLYR TXH DLQGD SUHFLVD
2SRHPDGH-RUJHGH/LPDVLQWHWL]DRSHUFXUVRGHYLGD VHU PHOKRUDGR LQHYLWDYHOPHQWH R FLGDGmR TXH SRGH
de Maria Diamba e sua reação ao sistema opressivo DGTXLUHVHXSUySULRYHtFXOR´
GD HVFUDYLGmR $ UHVLVWrQFLD GHVVD ¿JXUD IHPLQLQD p 'LVSRQtYHOHPKWWSJD]HWDRQOLQHJORERFRP$FHVVRHPDJR DGDSWDGR 
DVVLQDODGDQRWH[WRSHODUHODomRTXHVHID]HQWUH
TEXTO II
A RXVRGDIDODHRGHVHMRGHGHFLGLURSUySULRGHVWLQR
B DH[SORUDomRVH[XDOHDJHUDomRGHQRYDVHVFUDYDV
C D SUiWLFD QD FR]LQKD H D LQWHQomR GH DVFHQGHU
VRFLDOPHQWH
D RSUD]HUGHVHQWLURVYHQWRVHDHVSHUDQoDGHYROWDU
jÈIULFD
E RPHGRGDPRUWHHDYRQWDGHGHIXJLUGDYLROrQFLD
GRVEUDQFRV

QUESTÃO 102

6H R GDQoDULQR Mi SUHSDURX WRGD D VHQVDomR DQWHV


HOHQmRHVWiQRYD]LRMiHVWiDFDEDGR1HVVHPRPHQWR
YD]LR pRVHXFRUSRTXHHVWiGL]HQGRDOJRQmRpYRFr
4XDQGRRDWRUHVWiQHVVHPRPHQWRGHGHVLVWLUpQHVVH /,0$$'LVSRQtYHOHPKWWSDPDULOGRFKDUJHZRUGSUHVVFRP
momento que ele deve continuar; é nesse momento que $FHVVRHPDJR DGDSWDGR 

FKHJDDOJRSDUDTXHPHVWiDVVLVWLQGR1mRLPSRUWDWDQWR 2VWH[WRV,H,,WUDWDPGRPHVPRWHPDHPERUDVHMDP
DFRUHRJUD¿DHWRGRHVVHWUDEDOKR2PDLVLPSRUWDQWHp GH JrQHURV GLIHUHQWHV (VWDEHOHFHQGRVH DV UHODo}HV
LVVRRYD]LRHFRPRYRFrFRQWLQXDFRPLVVR HQWUH RV GRLV WH[WRV HQWHQGHVH TXH R 7H[WR ,, WHP D
&2//$$&&DPLQKDQWHQmRKiFDPLQKRVVyUDVWURV6mR3DXOR3HUVSHFWLYD IXQomRGH

2 WH[WR FRQVLGHUD TXH XP FRUSR YD]LR GH VRP A UHSURYDU DV PHGLGDV GR JRYHUQR GH LQFHQWLYR j
VHQWLPHQWR H SHQVDPHQWR  SRGH ID]HU TXDOTXHU FRLVD DTXLVLomRGRFDUURSUySULR
Nessa concepção, a atuação do dançarino alcança o B DSRQWDU XPD SRVVtYHO DOWHUQDWLYD SDUD UHVROYHU D
iSLFHGH TXHVWmRGRH[FHVVRGHYHtFXORV
A LQpUFLDHPFHQD C PRVWUDU D GL¿FXOGDGH GH VROXomR LPHGLDWD SDUD
UHVROYHURSUREOHPDGRFUHVFLPHQWRGDIURWD
B WUDQVFHQGrQFLDGHVL
D FULWLFDU SRU PHLR GD ViWLUD DV FRQVHTXrQFLDV GR
C VLJQL¿FDomRGRSUHSDUR DXPHQWRGDIURWDGHYHtFXORV
D DXVrQFLDGHFRPXQLFDomR E UHVSRQVDELOL]DU D Pi TXDOLGDGH GR VHUYLoR GH
E FRQVFLrQFLDGRPRYLPHQWR WUDQVSRUWHSHORFUHVFLPHQWRGRQ~PHrRGHYHtFXORV

/&žGLD_&DGHUQR&,1=$3iJLQD
*CZ1625DOM8* 2016

QUESTÃO 104 A reação da PmHSURIHVVRUD IUHQWH jV KDELOLGDGHV GD


³JHUDomRGLJLWDO´FRQWHPSRUkQHDUHÀHWHRGHVD¿RTXHVH
/LVERDDYHQWXUDV WHPHQIUHQWDGRGH
WRPHLXPH[SUHVVR A DSOLFDUDVPHVPDVIRUPDVGHOHUWH[WRVLPSUHVVRVD
FKHJXHLGHIRJXHWH WH[WRVGLJLWDLV
subi num bonde
desci de um elétrico B LQWHUSUHWDUDVYiULDVLQIRUPDo}HVQDOHLWXUDGHWH[WRV
SHGLXPFDIH]LQKR HPPXOWLPtGLD
serviram-me uma bica C OLGDUFRPDVQRYDVSUiWLFDVGHOHLWXUDTXHHPHUJHP
quis comprar meias FRPDWHFQRORJLD
VyYHQGLDPSH~JDV D VXSHUDU DV GL¿FXOGDGHV GH OHLWXUD JHUDGDV SHORV
IXLGDUDGHVFDUJD MRJRVGHFRPSXWDGRUHV
disparei um autoclisma
E WUDEDOKDU D GL¿FXOGDGH GH OHLWXUD XVDQGR DV
JULWHL³yFDUD´
UHVSRQGHUDPPH©ySiª WHFQRORJLDVFRPRIHUUDPHQWDV
positivamente QUESTÃO 106
DVDYHVTXHDTXLJRUMHLDPQmRJRUMHLDP
>FRPROi
3$(6-3$SRHVLDHVWiPRUWDPDVMXURTXHQmRIXLHX6mR3DXOR'XDV&LGDGHV

1R WH[WR D GLYHUVLGDGH OLQJXtVWLFD p DSUHVHQWDGD SHOD


ótica de um observador que entra em contato com
XPD FRPXQLGDGH OLQJXtVWLFD GLIHUHQWH GD VXD (VVH
observador é um
A IDODQWHGRSRUWXJXrVEUDVLOHLURUHODWDQGRRVHXFRQWDWR
QD(XURSDFRPRSRUWXJXrVOXVLWDQR
B LPLJUDQWHHP/LVERDFRPGRPtQLRGRVUHJLVWURVIRUPDO
HLQIRUPDOGRSRUWXJXrVHXURSHX
C WXULVWDHXURSHXFRPGRPtQLRGHGXDVYDULHGDGHVGR
SRUWXJXrVHPYLVLWDD/LVERD
D SRUWXJXrV FRP GRPtQLR GD YDULHGDGH FRORTXLDO GD
OtQJXDIDODGDQR%UDVLO
E SRHWD EUDVLOHLUR GHIHQVRU GR XVR SDGUmR GD OtQJXD
IDODGDHP3RUWXJDO

QUESTÃO 105
$PXOKHUHQWUDQRTXDUWRGR¿OKRGHFLGLGDDWHUXPD
FRQYHUVDVpULD'HQRYRDVUHVSRVWDVGHOHjLQWHUSUHWDomR
GR WH[WR QD SURYD VXJHUHP XPD JUDQGH GL¿FXOGDGH GH
)$1*&0LRSLD'LVSRQtYHOHPKWWSQHZVSVXHGX$FHVVRHPDEU
OHU 'LVSHUVmR SRGH VHU XPD UHVSRVWD SDUD SDUWH GR
SUREOHPD$H[WHQVmRGRWH[WRSRGHVHURXWUDPDVQHVWD O cartum Miopia GH &KHQ )DQJ IRL DSUHVHQWDGR HP
HODQmRYDLWRFDUSRUTXHWDPEpPpSURIHVVRUDHQmRYDL 2011 na quarta mostra Ecocartoon, que teve como tema
OKHGDUGHVFXOSDVSDUDLUPDOQDHVFROD3UHJXLoDGHOHU DHGXFDomRDPELHQWDO6HXWtWXORHRVHOHPHQWRVYLVXDLV
SDUHFH RXWUD IRUPD GH OLGDU FRP D H[WHQVmR GR WH[WR ID]HP UHIHUrQFLD DR H[DPH RIWDOPROyJLFR H D XP WLSR
(OH HVWi GH QRYR QR FRPSXWDGRU MRJDQGR /HYDQWD RV HVSHFt¿FR GH GL¿FXOGDGH YLVXDO &RP R XVR PHWDIyULFR
ROKRVFRPDTXHOHDUGHTXHPSRGHMRJDUHFRQYHUVDUDR GDPLRSLDHDH[SORUDomRGHFDUDFWHUtVWLFDVGDLPDJHP
PHVPRWHPSR$PmHOKHSHGHTXHLQWHUURPSDRMRJRH o cartum
HOHSHGHjPmH³VyXPLQVWDQWHSDUDVDOYDU´&XULRVDHOD
ROKD SDUD D WHOD H HVSDQWDVH FRP R MRJR HP MDSRQrV A HYLGHQFLD R SDSHO VHFXQGiULR TXH DQLPDLV H SODQWDV
3HUJXQWDOKHFRPRFRQVHJXHHQWHQGHURWH[WRSDUDMRJDU GHVHPSHQKDPQRSURFHVVRGHSURGXomRGHULTXH]DV
(OH OKH IDOD GH DOJXPD FRLVD SDUHFLGD FRP XPD ³OyJLFD B H[S}HRDOWRFXVWRSDUDDPDQXWHQomRGDYLGDWDQWR
GHMRJR´HVREUHDOJXPDVWHQWDWLYDVFRPRVtFRQHV'L] GRVVHUHVKXPDQRVFRPRGHDQLPDLVHSODQWDV
DLQGD TXH FRQKHFH D EDVH GD KLVWyULD H TXH DVVLP C GHQXQFLDDKLHUDUTXLDGHYDORUHVTXHVXSHUYDORUL]DR
PHVPR HP MDSRQrV WXGR ID] VHQWLGR $TXHOD FRQYHUVD GLQKHLURHPGHWULPHQWRGRVVHUHVYLYRV
DFDERX VHQGR DGLDGD$ PmHSURIHVVRUD FDSWXUDGD SRU
outros sentidos de leitura, não se sentia pronta naquele D UHYHODRGLVWDQFLDPHQWRHQWUHRKRPHPHDQDWXUH]D
PRPHQWR&RQVFLHQWHVXVSHQGHDDomR UHVXOWDQWHGDVDWLYLGDGHVHFRQ{PLFDV
%$55(725*)RUPDomRGHSURIHVVRUHVWHFQRORJLDVHOLQJXDJHQVPDSHDQGRYHOKRV
E questiona o antagonisPRHQWUHKRPHQVHPXOKHUHV
e novos GHV HQFRQWURV6mR3DXOR/R\ROD DGDSWDGR  PRWLYDGRSRUTXHVW}HVHFRQ{PLFDV

/&žGLD_&DGHUQR&,1=$3iJLQD
2016 *CZ1625DOM9*
QUESTÃO 107

1R%UDVLOPLOKDUHVGHFULDQoDVHDGROHVFHQWHVWUDEDOKDPHPFDVDVGHIDPtOLD,VVRQmRpOHJDO
2WUDEDOKRLQIDQWLOGRPpVWLFRHQFXUWDDLQIkQFLDSUHMXGLFDDDXWRHVWLPDHSURYRFDJUDQGHGHIDVDJHPHVFRODU
'HVHQYROYHPRVGLYHUVRVSURJUDPDVVRFLDLVTXHSURWHJHPHGmRGLJQLGDGHDFULDQoDVHMRYHQV
FRPRR3(7,352-29(085%$12352-29(0$'2/(6&(17((352-29(075$%$/+$'25HQWUHRXWURV
DisSRQtYHOHPKWWSVHUYLFRVSUWPSWPSEU$FHVVRHPMXO DGDSWDGR 

$ SHoD SXEOLFLWiULD HP SDXWD EXVFD SURPRYHU XPD FRQVFLHQWL]DomR VRFLDO 3HOD DQiOLVH GRV SURFHGLPHQWRV
DUJXPHQWDWLYRVXWLOL]DGRVSHORDXWRURWH[WR
A RS}HDIUDJLOLGDGHGDFULDQoDDRVGHVPDQGRVGRVDGXOWRV
B HOHQFDDVFDXVDVGDH[LVWrQFLDGRWUDEDOKRLQIDQWLOQR%UDVLO
C GHWDOKDDVLQLFLDWLYDVJRYHUQDPHQWDLVGHVROXomRGRSUREOHPDDERUGDGR
D GLYXOJDDo}HVLQVWLWXFLRQDLVORFDLVSDUDRHQIUHQWDPHQWRGHXPSUREOHPDQDFLRQDO
E UHVVDOWDDUHVSRQVDELOLGDGHGDVIDPtOLDVQDSURWHomRGDVFULDQoDVHGRVDGROHVFHQWHV

QUESTÃO 108
8PFDFKRUURFRUGHFDUYmRGRUPHQRD]XOHWpUHRGHXPDUHGHGHSHVFDHQURODGDVREUHDJUDPDGD3UDoD9LQWH
H8PGH$EULO2VROEDWHQDIUHQWHQRVGHJUDXVFLQ]HQWRVGDHVFDGDULDTXHVREHDHQFRVWDGRPRUURDWpD,JUHMDGD
0DWUL]$ODGHLUDGHSDUDOHOHStSHGRVFXUWDHtQJUHPHDRODGRGDLJUHMDSDVVDSRUXPJDOSmRGHEDUFRVHSRUXPDFDVD
GHPDGHLUDSUpPROGDGD$FHQDSDUDDYHOKLQKDPDUURPTXHWRPDVROQDYDUDQGDVHQWDGDQXPDFDGHLUDGHSUDLD
FRORULGD2YHQWRQRUGHVWHVDOJDGRWXPXOWXDDViUYRUHVHDVRQGDV1XYHQVHVSDUUDPDGDVDYDQoDPHPIRUPDomR
GRPDUSDUDRFRQWLQHQWHFRPRXPH[pUFLWRHPWUDQVH$ODGHLUDID]XPDFXUYDjHVTXHUGDSDVVDQGRHPIUHQWHDXP
SUHGLQKRGRVpFXORGH]RLWRFRPSDUHGHVEUDQFDVGHVFDVFDGDVHMDQHODVUHFpPSLQWDGDVGHD]XOFREDOWR
*$/(5$'%DUEDHQVRSDGDGHVDQJXH 6mR3DXOR&LDGDV/HWUDV

$GHVFULomRVXEMHWLYDRXREMHWLYDSHUPLWHDROHLWRUYLVXDOL]DURFHQiULRRQGHXPDDomRVHGHVHQYROYHHRVSHUVRQDJHQV
TXHGHODSDUWLFLSDP2IUDJPHQWRGRURPDQFHFDUDFWHUL]DVHFRPRXPDGHVFULomRVXEMHWLYDSRUTXH
A FRQVWUyLVHTXrQFLDVWHPSRUDLVSHORHPSUHJRGHH[SUHVV}HVDGYHUELDLV
B DSUHVHQWDIUDVHVFXUWDVGHRUGHPGLUHWDFRPHOHPHQWRVHQXPHUDWLYRV
C UHFRUUHDVXEVWDQWLYRVFRQFUHWRVSDUDUHSUHVHQWDUXPDPELHQWHHVWiWLFR
D FULDXPDDPELrQFLDSUySULDSRUPHLRGHQRPHVHYHUERVPHWDIRUL]DGRV
E SULRUL]DFRQVWUXo}HVRUDFLRQDLVGHYDORUVHPkQWLFRGHRSRVLomR
/&žGLD_&DGHUQR&,1=$3iJLQD
*CZ1625DOM10* 2016

48(67­2 QUESTÃO 111


Naquele tempo eu morava no Calango-Frito e não 2SDVVDGRQDWHODGRFRPSXWDGRU
DFUHGLWDYDHPIHLWLFHLURV
(RFRQWUDVVHQVRPDLVDYXOWDYDSRUTXHMiHQWmR௅H 8P GRV GHVD¿RV GR QRYR 0XVHX GD ,PLJUDomR
H[FOXtGDTXDQWDFRLVDHVRXVDGHQyVWRGRVOiHRXWUDV p VH FRQWUDSRU j LPDJHP GHL[DGD SHOD H[LELomR
FLVPDV FRUULTXHLUDV WDLV VDO GHUUDPDGR SDGUH YLDMDQGR do acervo permanente na época do Memorial do
FRPDJHQWHQRWUHPQmRIDODUHPUDLRTXDQGRPXLWRH ,PLJUDQWH PXLWR FULWLFDGD SRU GDU rQIDVH GHPDVLDGD
VHRWHPSRHVWiERP³IDtVFD´QHPGL]HUOHSUDVyR³PDO´ aos imigrantes estrangeiros e pouca atenção aos
passo de entrada com o pé esquerdo; ave do pescoço EUDVLOHLURV(UDXPDUHSUHVHQWDomRGHVSURSRUFLRQDOHP
SHODGR ULVDGD UHQJD GH VXLQGDUD FDFKRUUR ERGH H UHODomRDRVQ~PHURVGRVPLOK}HVGHSHVVRDVTXH
JDOR SUHWRV >@ ௅ SRUTXH Mi HQWmR FRPR LD GL]HQGR
HX SRGHULD FRQIHVVDU QXP UHFHQVHLR DSUR[LPDGR GR]H SDVVDUDPSHODKRVSHGDULDGHLPLJUDQWHVGH6mR3DXOR
WDEXV GH QmR XVR SUySULR RLWR UHJULQKDV RUWRGR[DV DSUR[LPDGDPHQWH  PLOKmR HUDP HVWUDQJHLUDV GH 
SUHYHQWLYDVYLQWHSpVVLPRVSUHVViJLRVGH]HVVHLVFDVRV QDFLRQDOLGDGHV H HWQLDV  H  PLOKmR HUDP EUDVLOHLUDV
GH EDWLGD REULJDWyULD QD PDGHLUD GH] RXWURV H[LJLQGR D RULXQGDVSULQFLSDOPHQWHGRVHVWDGRVQRUGHVWLQRV
¿JDGLJLWDOQDSROLWDQDPDVGDOHJtWLPDRFXOWDQGREHPD +(%0h//(533UREOHPDVEUDVLOHLURVQQRYGH] DGDSWDGR 
cabeça do polegar; e cinco ou seis indicações de ritual
PDLVFRPSOLFDGRWRWDOVHWHQWDHGRLV௅QRYHVIRUDQDGD 2DXWRUGRWH[WRVREUHDGLJLWDOL]DomRGRDFHUYRGRQRYR
0XVHX GD ,PLJUDomR DSUHVHQWD D rQIDVH QR LPLJUDQWH
526$-*6mR0DUFRV6DJDUDQD5LRGH-DQHLUR-RVp2O\PSLR DGDSWDGR 
estrangeiro como um problema de representação
-RmR*XLPDUmHV5RVDQHVVHIUDJPHQWRGHFRQWRUHVJDWD HTXLYRFDGD GD LPLJUDomR HP 6mR 3DXOR 3DUD WDQWR
a cultura popular ao registrar
IXQGDPHQWDVHXSRQWRGHYLVWDQR D
A WUHFKRVGHFDQWLJDV
B ULWXDLVGHPDQGLQJDV A panorama apresentado como a atual realidade do
C FLWDo}HVGHSUHFHLWRV LPLJUDQWHHP6mR3DXOR
D FHULP{QLDVUHOLJLRVDV B uso da tecnologia para aprimorar a imagem do
E H[HPSORVGHVXSHUVWLo}HV LPLJUDQWHHP6mR3DXOR
QUESTÃO 110 C GLIHUHQoDHQWUHR0HPRULDOGR,PLJUDQWHHRVGHPDLV
(VFUHYHU PXVHXVH[LVWHQWHVHP6mR3DXOR
A estudante perguntou como era essa coisa de D diversidade de nacionalidades e etnias como
HVFUHYHU(X¿]RJrQHURIRIR0ROH]DGLVVH SDUkPHWURGDLPLJUDomRHP6mR3DXOR
E GHVHTXLOtEULR QDV UHSUHVHQWDo}HV XVXDLV GRV
Primeiro evite esses coloquialismRV GH ³IRIR´ H LPLJUDQWHVHP6mR3DXOR
³PROH]D´SDVVHORQJHGDVJtULDVDLQGDQmRGLFLRQDUL]DGDV
HGHWXGRPDLVTXHVRHPDLVIDODGRGRTXHHVFULWR,VWR QUESTÃO 112
DTXL QmR p UiGLR )0 'H YH] HP TXDQGR DSOLTXH XPD
JtULD FRPR VH IRVVH XP SLSDURWH GH OHYH QR FDQJRWH GR $UJXPHQWR
WH[WRPDVHPJHUDOHYLWH)XMDGHVVDVULPDVERELQKDV 7iOHJDO
GHVVHVPRWHVVRQRURV2OHLWRUSRGHVHDFKDUGLDQWHGH
um rapperIUXVWUDGRHGDUFDPEDOKRWDV0DVDWHQomRVH Eu aceito o argumento
VRDUPXLWRHVWUDQKRUHHVFUHYD Mas não me altere o samba tanto assim
2OKDTXHDUDSD]LDGDHVWiVHQWLQGRDIDOWD
4XDQGR TXLVHU DSOLFDU XP ³PDV´ WRPH I{OHJR
OLJXH SDUD R  GR ,QVWLWXWR )HUQDQGR 3HVVRD SHoD De um cavaco, de um pandeiro e de um tamborim
DXWRUL]DomRDRViELRGHSODQWmRHSRUIDYRUYROWHDWUiV
eXPFDFRHWHIDFLOLWDGRU'HOHGHYHWHUYLQGRDH[SUHVVmR Sem preconceito
³FKHLRGHPDVPDV´RXVHMDXPDSHVVRDFKHLDGH³QmR Ou mania de passado
pEHPDVVLP´XPDFKDWDTXHXVDRWUXTXHSDUDD¿UPDUH 6HPTXHUHU¿FDUGRODGR
GHSRLVFRPRVHIRVVHHVWLORREWHPSHUDU De quem não quer navegar
6$1726-)2*ORERMDQ DGDSWDGR  )DoDFRPRRYHOKRPDULQKHLUR
Que durante o nevoeiro
$OtQJXDYDULDHPIXQomRGHGLIHUHQWHVIDWRUHV8PGHOHV /HYDREDUFRGHYDJDU
pDVLWXDomRHPTXHVHGiDFRPXQLFDomR1DFU{QLFD
ao ser interrogado sobre a arte de escrever, o autor 3$8/,1+2'$9,2/$'LVSRQtYHOHPZZZSDXOLQKRGDYLRODFRPEU$FHVVRHPGH]
XWLOL]DHPPHLRjOLQJXDJHPHVFULWDSDGUmRFRQGL]HQWH
FRPRFRQWH[WR 1D OHWUD GD FDQomR SHUFHEHVH XPD LQWHUORFXomR
A posição do emissor é conciliatória entre as tradições
A GH¿QLo}HVWHyULFDVSDUDSHUPLWLUTXHVHXVFRQVHOKRV
GR VDPED H RV PRYLPHQWRV LQRYDGRUHV GHVVH ULWPR
VHMDP~WHLVDRVIXWXURVMRUQDOLVWDV
$HVWUDWpJLDDUJXPHQWDWLYDGHFRQFHVVmRQHVVHFHQiULR
B JtULDVQmRGLFLRQDUL]DGDVSDUDLPLWDUDOLQJXDJHPGH pPDUFDGDQRWUHFKR
MRYHQVGHEDL[DHVFRODULGDGH
C palavras de uso coloquial, para estabelecer uma A ³0DVQmRPHDOWHUHRVDPEDWDQWRDVVLP´
LQWHUDomRVDWLVIDWyULDFRPDLQWHUORFXWRUD B ³2OKDTXHDUDSD]LDGDHVWiVHQWLQGRDIDOWD´
D WHUPRV GD OLQJXDJHP MRUQDOtVWLFD SDUD FDXVDU ERD C ³6HPSUHFRQFHLWR2XPDQLDGHSDVVDGR´
LPSUHVVmRQDMRYHPHQWUHYLVWDGRUD
E YRFDEXOiULR WpFQLFR SDUD DPSOLDU R UHSHUWyULR D ³6HPTXHUHU¿FDUGRODGR'HTXHPQmRTXHUQDYHJDU´
OLQJXtVWLFRGRVMRYHQVOHLWRUHVGRMRUQDO E ³/HYDREDUFRGHYDJDU´

/&žGLD_&DGHUQR&,1=$3iJLQD
2016 *CZ1625DOM11*
QUESTÃO 113 QUESTÃO 115
8PPHQLQRDSUHQGHDOHU
0LQKDPmHVHQWDYDVHDFRVHUHUHWLQKDPHGHOLYUR
QD PmR DR ODGR GHOD DR Sp GD PiTXLQD GH FRVWXUD
2OLYURWLQKDQXPDSiJLQDD¿JXUDGHXPELFKRFDUFXQGD
DR ODGR GD TXDO HP OHWUDV JUD~GDV GHVWDFDYDVH HVWD
SDODYUD (67Ð0$*2 'HSRLV GH VROHWUDU ³HVWRPDJR´
SURQXQFLHL ³HVWRPiJR´ (X KDYLD SURQXQFLDGR EHP DV
GXDV SULPHLUDV SDODYUDV TXH OL FDPHOR H GURPHGiULR
0DVHVW{PDJRSURQXQFLHLHVWRPiJR0LQKDPmHERQLWD
FRPRVySRGHVHUPmHMRYHPSDUD¿OKRSHTXHQRRURVWR
DOYtVVLPR RV FDEHORV HQURODGRV QR SHVFRoR SDURX D
FRVWXUDHPH¿WRXGHID]HUPHGR³*LOEHUWR´(VWUHPHFL
³(VWRPiJR" /HLD GH QRYR VROHWUH´ 6ROHWUHL UHSHWL
³(VWRPiJR´)RLRGLDER
$1'5$'(5'LVSRQtYHOHPZZZMRUQDOFLGDGHFRPEU$FHVVRHPRXW DGDSWDGR 
-DPDLVWLQKDRXYLGRDRTXHPHOHPEUDVVHHQWmRD
$ FKDUJH DERUGD XPD VLWXDomR GR FRWLGLDQR GH SDODYUDHVW{PDJR$FR]LQKHLUDRHVWULEHLURRVFULDGRV
DOJXPDV IDPtOLDV 1HVVH VHQWLGR HOD WHP R REMHWLYR %HUQDUGDGL]LDP³HVWDPER´³(VWRXFRPXPDGRUQDERFD
comunicativo de GRHVWDPER´³0HXHVWDPERHVWiWLQLQGR´0HXVSDLV
A GHQXQFLDURVSUHMXt]RVGDIDOWDGHGLiORJRHQWUHSDLV WHULDP SURQXQFLDGR GLUHLWR QD PLQKD SUHVHQoD PDV HX
H¿OKRV QmRPHOHPEUDYD(FULDQoDFRPRRSRYRVHPSUHTXH
SRGHUHSHOHSURSDUR[tWRQR
B PRVWUDU DV GLIHUHQoDV HQWUH DV SUHIHUrQFLDV GH
$0$'2*+LVWyULDGDPLQKDLQIkQFLD5LRGH-DQHLUR-RVp2O\PSLR
HQWUHWHQLPHQWRHQWUHSDLVH¿OKRV
C HYLGHQFLDURVH[FHVVRVGHXWLOL]DomRGDVUHGHVVRFLDLV 1R WUHFKR HP TXH R QDUUDGRU UHOHPEUD XP HSLVyGLR
HPPRPHQWRVGHFRQYLYrQFLDIDPLOLDU GH VXD LQIkQFLD UHYHODVH D SRVVLELOLGDGH GH D OtQJXD
VH UHDOL]DU GH IRUPDV GLIHUHQWHV &RP EDVH QR WH[WR D
D demonstrar que as mudanças culturais ocorridas passagem em que se constata uma marca de variedade
QD VRFLHGDGH LPS}HP QRYRV FRPSRUWDPHQWRV jV OLQJXtVWLFDSRXFRSUHVWLJLDGDp
IDPtOLDV
A ³2 OLYUR WLQKD QXPD SiJLQD D ¿JXUD GH XP ELFKR
E HQIDWL]DU TXH D VRFLDOL]DomR GH LQIRUPDo}HV VREUH
FDUFXQGD DR ODGR GD TXDO HP OHWUDV JUD~GDV
RV ¿OKRV p XPD IRUPD GH GHPRQVWUDU RUJXOKR GH
GHVWDFDYDVHHVWDSDODYUD(67Ð0$*2´
IDPLOLDUHV
B ³µ*LOEHUWR¶ (VWUHPHFL µ(VWRPiJR" /HLD GH QRYR
QUESTÃO 114 VROHWUH¶6ROHWUHLUHSHWLµ(VWRPiJR¶´
$ SDODYUD H D LPDJHP WrP R SRGHU GH FULDU C ³(X KDYLD SURQXQFLDGR EHP DV GXDV SULPHLUDV
H GHVWUXLU GH SURPHWHU H QHJDU $ SXEOLFLGDGH VH SDODYUDVTXHOLFDPHORHGURPHGiULR´
YDOH GHVWH UHFXUVR OLQJXtVWLFRLPDJpWLFR FRPR VHX D ³-DPDLVWLQKDRXYLGRDRTXHPHOHPEUDVVHHQWmRD
SULQFLSDOLQVWUXPHQWR9HQGHD¿FomRFRPRRUHDO SDODYUDHVW{PDJR´
 RQRUPDOFRPRDOJRIDQWiVWLFRWUDQVIRUPDXPFDUUR E ³$ FR]LQKHLUD R HVWULEHLUR RV FULDGRV %HUQDUGD
HPXPVtPERORGHSUHVWtJLRVRFLDOXPDFHUYHMDHP GL]LDPµHVWDPER¶´
uma loira bonita, e um cidadão comum num astro
RXHVWUHODEDVWDQGRWmRVRPHQWHXWLOL]DURSURGXWR
RXVHUYLoRGLYXOJDGRAssimID]HUREDQDOWRUQDUVH
10 RLGHDOpWDUHIDRUGLQiULDGDOLQJXDJHPSXEOLFLWiULD
$/0(,'$:0$OLQJXDJHPSXEOLFLWiULDHRHVWUDQJHLULVPR
/tQJXD3RUWXJXHVDQMDQ

$OJXQVHOHPHQWRVOLQJXtVWLFRVHVWDEHOHFHPUHODo}HVHQWUH
DV GLIHUHQWHV SDUWHV GR WH[WR 1HVVH WH[WR R YRFiEXOR
³Assim” Ɛ WHPDIXQomRGH
A FRQWUDULDURVDUJXPHQWRVDQWHULRUHV
B VLQWHWL]DUDVLQIRUPDo}HVDQWHULRUHV
C DFUHVFHQWDUXPQRYRDUJXPHQWR
D LQWURGX]LUXPDH[SOLFDomR
E DSUHVHQWDUXPDDQDORJLD

/&žGLD_&DGHUQR&,1=$3iJLQD
*CZ1625DOM12* 2016

QUESTÃO 116 QUESTÃO 118

eXPDSDUWLGDGHIXWHERO $WpFQLFDGHMRJRVWHDWUDLVSURS}HXPDDSUHQGL]DJHP
QmR YHUEDO HP TXH R DOXQR UH~QH RV VHXV SUySULRV
$EDQGHLUDQRHVWiGLRpXPHVWDQGDUWH GDGRVDSDUWLUGHXPDH[SHULPHQWDomRGLUHWD3RUPHLR
$ÀkPXODSHQGXUDGDQDSDUHGHGRTXDUWR do processo de solução de problemas, ele conquista o
2GLVWLQWLYRQDFDPLVDGRXQLIRUPH FRQKHFLPHQWRGDPDWpULD
.28'(/$,'-RJRVWHDWUDLV6mR3DXOR3HUVSHFWLYD DGDSWDGR 
4XHFRLVDOLQGDpXPDSDUWLGDGHIXWHERO
6RE RULHQWDomR GR SURIHVVRU RV MRJRV WHDWUDLV VmR
Posso morrer pelo meu time UHDOL]DGRVQDHVFRODGHIRUPDTXHRHVWXGDQWH
Se ele perder, que dor, imenso crime A VHMDXPERPUHSHWLGRUGHPRYLPHQWRVHDo}HVSRLV
3RVVRFKRUDUVHHOHQmRJDQKDU a cópia e a memória colaboram com seu processo de
GHVHQYROYLPHQWR
0DVVHHOHJDQKDQmRDGLDQWD
B obedeça a regras sem se posicionar criticamente
1mRKiJDUJDQWDTXHQmRSDUHGHEHUUDU H VHP GHVHQYROYHU PDWHULDO FULDWLYR IRUWDOHFHQGR
5(,61526$66DPEDSRFRQp6mR3DXOR6RQ\ IUDJPHQWR  DGLVFLSOLQD
1R%UDVLODOpPGHXPHVSRUWHGHFRPSHWLomRRIXWHERO C WHQKD XP PRPHQWR GH UHFUHDomR SRU PHLR GD
pXPPHLRGHLQWHUDomRVRFLDOTXHGHVSHUWDSDL[mRQDV FRQYLYrQFLD FRP RV FROHJDV PHOKRUDQGR VHX
SHVVRDV 1R WUHFKR GD OHWUD GD FDQomR HVVH HVSRUWH p UHQGLPHQWRHVFRODU
DSUHVHQWDGRFRPRXP D D desenvolva qualidades de ordem cognitiva e
A PRGDOLGDGHHVSRUWLYDWpFQLFD VHQVRULDO IDYRUHFHQGR VXD DXWRQRPLD H VHX
B IRUPDGHFRQWUROHGDYLROrQFLD DXWRFRQKHFLPHQWR
C HVSRUWHRUJDQL]DGRFRPUHJUDV E UHFRQKHoD R SURIHVVRU FRPR SULQFLSDO UHVSRQViYHO
D HOHPHQWRGDLGHQWLGDGHQDFLRQDO SHODV HVFROKDV D VHUHP IHLWDV HP DXOD GXUDQWH
DWLYLGDGHVGHWHDWUR
E IDWRUGHDOLHQDomRVRFLDOGRSRYR
48(67­2
QUESTÃO 117
+i PXLWR VH VDEH TXH D %DFLD %DXUX ± GHSyVLWR GH Estas palavras ecoavam docemente pelos atentos
URFKDV IRUPDGDV SRU VHGLPHQWRV ORFDOL]DGR HQWUH RV RXYLGRV GH *XDUDFLDED H OKH UHVVRDYDP Q¶DOPD FRPR
HVWDGRVGH6mR3DXOR0LQDV*HUDLV*RLiV3DUDQi0DWR XP KLQR FHOHVWLDO (OD VHQWLDVH DR PHVPR WHPSR
*URVVRH0DWR*URVVRGR6XO±IRLKDELWDGDKiPLOK}HV HQWHUQHFLGD H XIDQD SRU RXYLU DTXHOH DOWLYR H LQG{PLWR
GH DQRV SRU XPD DEXQGDQWH IDXQD GH FURFRGLOLIRUPHV guerreiro pronunciar a seus pés palavras do mais
um grupo de répteis em que estão inclusos os crocodilos, VXEPLVVR H PDYLRVR DPRU H UHVSRQGHXOKH FKHLD GH
MDFDUpV H VHXV SDUHQWHV SUpKLVWyULFRV H[WLQWRV (QWUH DV HPRomR ² ,WDMLED WXDV IDODV VmR PDLV GRFHV SDUD
IDPtOLDVTXHSRUOiYLYHUDPHVWiDBaurusuchidae, que, na PLQKDDOPDTXHRVIDYRVGDMDWDtRXRVXFRGHOLFLRVR
UHJLmRHQJOREDYDRXWUDVVHLVHVSpFLHVGHFURFRGLOLIRUPHV GR DEDFD[L (ODV ID]HPPH SDOSLWDU R FRUDomR FRPR D
H[FOXVLYDPHQWH WHUUHVWUHV H FRP JUDQGH FDSDFLGDGH GH ÀRU TXH HVWUHPHFH DR EDIHMR SHUIXPDGR GDV EULVDV GD
GHVORFDPHQWRFUkQLRDOWRHFRPSULPLGRODWHUDOPHQWHHFRP
PDQKm7XPHDPDVEHPRVHLHRDPRUTXHWHFRQVDJUR
ORQJRVGHQWHVVHUULOKDGRV$JRUDHPXPDUWLJRSXEOLFDGR
na versão on-line da revista Cretaceous Research, um WDPEpP QmR p SDUD WL QHQKXP VHJUHGR HPERUD PHXV
JUXSR GH SHVTXLVDGRUHV GDV XQLYHUVLGDGHV IHGHUDLV GR OiELRVQmRRWHQKDPUHYHODGR$ÀRUPHVPRQDVWUHYDV
5LRGH-DQHLURHGR7ULkQJXOR0LQHLURHP0LQDV*HUDLV VH WUDL SHOR VHX SHUIXPH D IRQWH GR GHVHUWR HVFRQGLGD
LGHQWL¿FDUDPPDLVXPPHPEURGHVVDDQWLJDIDPtOLD HQWUH RV URFKHGRV VH UHYHOD SRU VHX PXUP~ULR DR
'LVSRQtYHOHPKWWSUHYLVWDSHVTXLVDIDSHVSEU$FHVVRHPQRY FDPLQKDQWH VHTXLRVR 'HVGH RV SULPHLURV PRPHQWRV WX
YLVWHPHXFRUDomRDEULUVHSDUDWLFRPRDÀRUGRPDQDFi
$ FLUFXODomR GR FRQKHFLPHQWR FLHQWt¿FR RFRUUH GH DRVSULPHLURVUDLRVGRVRO
GLIHUHQWHV PDQHLUDV 3RU PHLR GD OHLWXUD GR WUHFKR
*8,0$5­(6%2HUPLWmRGH0XTXpP'LVSRQtYHOHPZZZGRPLQLRSXEOLFRJRYEU
LGHQWL¿FDVH TXH R WH[WR p XP DUWLJR GH GLYXOJDomR $FHVVRHPRXW
FLHQWt¿FDSRLVHQWUHRXWUDVFDUDFWHUtVWLFDV
A H[LJHGROHLWRUFRQKHFLPHQWRVHVSHFt¿FRVDFHUFDGR 2 WH[WR GH %HUQDUGR *XLPDUmHV p UHSUHVHQWDWLYR GD
WHPDH[SORUDGR HVWpWLFD URPkQWLFD (QWUH DV PDUFDV WH[WXDLV TXH
HYLGHQFLDP D ¿OLDomR D HVVH PRYLPHQWR OLWHUiULR HVWi
B GHVWLQDVH D OHLWRUHV YLQFXODGRV D GLIHUHQWHV
FRPXQLGDGHVFLHQWt¿FDV em destaque a
C ID] UHIHUrQFLD D DUWLJRV SXEOLFDGRV HP UHYLVWDV A UHIHUrQFLDDHOHPHQWRVGDQDWXUH]DORFDO
FLHQWt¿FDVLQWHUQDFLRQDLV B H[DOWDomRGH,WDMLEDFRPRQREUHJXHUUHLUR
D WUDWD GH GHVFREHUWDV GD FLrQFLD FRP OLQJXDJHP C FXPSOLFLGDGHHQWUHRQDUUDGRUHDSDLVDJHP
DFHVVtYHODRS~EOLFRHPJHUDO
E DERUGDWHPDVTXHUHFHEHUDPGHVWDTXHHPMRUQDLVH D UHSUHVHQWDomRLGHDOL]DGDGRFHQiULRGHVFULWR
UHYLVWDVQmRHVSHFLDOL]DGRV E H[SUHVVmRGDGHVLOXVmRDPRURVDGH*XDUDFLDED

/&žGLD_&DGHUQR&,1=$3iJLQD
2016 *CZ1625DOM13*
QUESTÃO 120 GDVOtQJXDVPDLVDPHDoDGDVGRPXQGR´GLVVHROLQJXLVWD
.'DYLG+DUULVRQ3DUDRVDWLYLVWDVGHFDGDFRPXQLGDGH
FRPLGLRPDDPHDoDGRHVVHGLFLRQiULRpXPDIHUUDPHQWD
TXH SRGH VHU XVDGD SDUD GLVVHPLQDU R FRQKHFLPHQWR GD
OtQJXDHQWUHRVPDLVMRYHQV3DUDWRGDVDVRXWUDVSHVVRDV
LQWHUHVVDGDVpDRSRUWXQLGDGHGHHQFRQWUDUVRQVHIRUPDV
GH GLVFXUVRV KXPDQRV GHVFRQKHFLGRV SDUD JUDQGH
SDUWHGDSRSXODomRGRJOREReDGLYHUVLGDGHOLQJXtVWLFD
HVFRQGLGDHTXHDJRUDSRGHVHUUHYHODGD
'LVSRQtYHOHPKWWSUHYLVWDOLQJXDXROFRPEU$FHVVRHPMXO DGDSWDGR 

&RQVLGHUDQGR R SURMHWR GRV ³GLFLRQiULRV IDODQWHV´


FRPSUHHQGHVHTXHRWUDEDOKRGRVOLQJXLVWDVDSUHVHQWDGR
QRWH[WRREMHWLYD
A GHVWDFDUDLPSRUWkQFLDGHVVHWLSRGHLQLFLDWLYDSDUDD
UHFRQVWLWXLomRGHOtQJXDVH[WLQWDV
B UDWL¿FDU D WUDGLomR RUDO FRPR LQVWUXPHQWR GH
SUHVHUYDomRGDVOtQJXDVQRPXQGR
C GHPRQVWUDUDH[LVWrQFLDGHUHJLVWURVOLQJXtVWLFRVVRE
ULVFRGHGHVDSDUHFHU
D preservar a memória cultural de um povo por meio de
UHJLVWURVHVFULWRV
E HVWLPXODU SURMHWRV YROWDGRV SDUD D HVFULWD GH
JUDPiWLFDVHGLFLRQiULRV
0$,7(1$'LVSRQtYHOHPZZZPDLWHQDFRPDU$FHVVRHPVHW
QUESTÃO 122
(VVD KLVWyULD HP TXDGULQKRV DERUGD D SDGURQL]DomR GD
LPDJHPFRUSRUDOQDFRQWHPSRUDQHLGDGH2IDWRUTXHSRGH %UDVLORSDtVGRVPLOK}HVGHUDLRV
VHU LGHQWL¿FDGR FRPR LQÀXHQFLDGRU GR FRPSRUWDPHQWR
'RV  ELOK}HV GH UDLRV TXH JROSHLDP D 7HUUD H
REVHVVLYRUHWUDWDGRQRVTXDGULQKRVpR
VHXVKDELWDQWHVGXUDQWHXPDQRPLOK}HVGHOHVYrP
A HQWHQGLPHQWR GD DSDUrQFLD FRUSRUDO UHODFLRQDGD GHVDEDU HP WHUUDV EUDVLOHLUDV 2 Q~PHUR GLYXOJDGR QR
jVD~GH DQR SDVVDGR SRU XPD HTXLSH GH FLHQWLVWDV GR ,QVWLWXWR
B FRQWUROH IHPLQLQR VREUH R LGHDO VRFLDO GH HVWpWLFD 1DFLRQDOGH3HVTXLVDV(VSDFLDLV ,QSH HP6mR-RVpGRV
FRUSRUDO &DPSRV 6mR 3DXOR QmR p VXSHUDGR SRU QHQKXP RXWUR
C GHVHMR SHOR PRGHOR GH FRUSR LGHDO FRQVWUXtGR SDtV(¿FRXEHPDFLPDGDVHVWLPDWLYDVTXHGDYDPFRQWD
VRFLDOPHQWH GHPLOK}HVDRDQR$JRUDVDEHPRVFRPVHJXUDQoD
D TXHVWLRQDPHQWR FUtWLFR GRV YDORUHV OLJDGRV DR HPTXDQWLGDGHGHUHOkPSDJRVQLQJXpPVHJXUDHVWHSDtV
VXFHVVRVRFLDO )21$&=$1&+(77$0,'LVSRQtYHOHPKWWSVXSHUDEULOFRPEU$FHVVRHPMDQ

E SRVLFLRQDPHQWR UHÀH[LYR GD PXOKHU IUHQWH jV 'LYHUVRV H[SHGLHQWHV DUJXPHQWDWLYRV VmR HPSUHJDGRV
LPSRVLo}HVHVWpWLFDV QRVWH[WRVSDUDVXVWHQWDUDVLGHLDVDSUHVHQWDGDV1HVVH
WH[WR D FLWDomR GH XP LQVWLWXWR HVSHFLDOL]DGR p XPD
QUESTÃO 121
estratégia para
e YLDMDQGR SHOR PXQGR TXH RV GRLV SUR¿VVLRQDLV A atestar a necessidade de ações de prevenção de
GR /LYLQJ 7RQJXHV ,QVWLWXWH IRU (QGDQJHUHG /DQJXDJHV
GDQRVFDXVDGRVSRUUDLRV
UH~QHP VXEVtGLRV SDUD IRUPDU RV FKDPDGRV ³GLFLRQiULRV
IDODQWHV´ GH LGLRPDV HP IDVH GH H[WLQomR FRP SRXFRV B DSUHVHQWDUDVHVWLPDWLYDVGHLQFLGrQFLDGHUDLRVHP
IDODQWHV QR SODQHWD ³$ PDLRULD GDV OtQJXDV GR PXQGR p WHUUDVEUDVLOHLUDV
RUDORTXHVLJQL¿FDTXHQmRHVWmRHVFULWDVRXVHXVIDODQWHV C SURPRYHU GLVFXVVmR VREUH DV FRQVHTXrQFLDV GDV
QmR FRVWXPDP HVFUHYrODV ( DSHVDU GH RV SURMHWRV GHVFDUJDVGHUDLRV
WUDGLFLRQDLVGRVOLQJXLVWDVVHUHPRVGHHVFUHYHUJUDPiWLFDV
H GLFLRQiULRV JRVWDPRV GH SHQVDU HP OtQJXDV YLYDV H D FRQIHULU FUHGLELOLGDGH DRV UHVXOWDGRV GH XPD
VDEHUTXHDVSHVVRDVDVHVWmRIDODQGR(QWmRVHYRFrYDL LQYHVWLJDomRVREUHUDLRV
DXPGLFLRQiULRGHYHVHUFDSD]GHRXYLOR)RLFRPLVVR E FRPSDUDURQ~PHURGHUDLRVLQFLGHQWHVQR%UDVLOH
em mente que criamRVRVGLFLRQiULRVSDUDRLWRGHDOJXPDV no muQGR

/&žGLD_&DGHUQR&,1=$3iJLQD
*CZ1625DOM14* 2016

QUESTÃO 123 QUESTÃO 124

Apuram o passo, por entre campinas ricas, onde 3DUHVWHVLDQmRIRUPLJDPHQWR


SDVWDP RX UXPLQDP RXWURV PLO H PDLV ERLV 0DV RV 7ULQWDHWUrVUHJUDVTXHPXGDPDUHGDomR
vaqueiros não esmorecem nos eias e cantigas, porque de bulas no Brasil
a boiada ainda tem passagens inquietantes: alarga-se e &RPR3URMHWR%XODVGHYROWDGRSDUDDWUDGXomR
recomprime-se, sem motivo, e mesmo dentro da multidão GRMDUJmRIDUPDFrXWLFRSDUDDOtQJXDSRUWXJXHVD±DTXHOD
PRYHGLoDKiJLURVHVWUDQKRVTXHQmRRVGHVORFDPHQWRV IDODGD HP WRGR R %UDVLO ± H D UHJXODPHQWDomR GR XVR
QRUPDLV GR JDGR HP PDUFKD ௅ TXDQGR VHPSUH DOJXQV GH PHGLFDPHQWRV QR SDtV FLQFR DQRV GHSRLV R %UDVLO
disputam a colocação na vanguarda, outros procuram FRPHoRXDVDLUGDVWUHYDV
R FHQWUR H PXLWRV VH GHL[DP OHYDU HPSXUUDGRV 2JUXSRFRPDQGDGRSRUXPDGRXWRUDHP/LQJXtVWLFDGD
VREUHQDGDQGR TXDVH FRP RV PDLV IUDFRV URODQGR SDUD 8)5- VXJHULX j$QYLVD PXGDU WXGR (ODERURX WDPEpP
RVODGRVHRVPDLVSHVDGRVWDUGDQGRSDUDWUiVQRFRLFH ³$ UHGDomR GH EXODV SDUD R SDFLHQWH XP JXLD FRP RV
GDSURFLVVmR SULQFtSLRV GH UHGDomR FODUD FRQFLVD H DFHVVtYHO SDUD R
OHLWRU GH EXODV´ GLVSRQtYHO HP YHUVmR DGDSWDGD QR site
²(KERLOi(KrrHKERL7RX7RX7RX GD$QYLVD 'LIHUHQWHPHQWH GR TXH DFRQWHFH FRP RXWURV
As ancas balançam, e as vagas de dorsos, das JrQHURVQDEXODQmRKiHVSDoRSDUDLQRYDo}HVGHHVWLOR
³2 XVR GH IyUPXODV UHSHWLWLYDV p EHPYLQGR Gi IRUoD
vacas e touros, batendo com as caudas, mugindo no LQVWLWXFLRQDODRWH[WR´H[SOLFDDGRXWRUD³$EXODQmRSRGH
meio, na massa embolada, com atritos de couros, DEULUSRVVLELOLGDGHVGHLQWHUSUHWDo}HVDRVHXOHLWRU´
estralos e guampas, estrondos e baques, e o berro
TXHL[RVR GR JDGR MXQTXHLUD GH FKLIUHV LPHQVRV FRP 6HREHGHFLGDVDVUHJUDVGRJXLDVmRGHVHUYHQWLD
JHQpULFD±TXHPOLGDFRPTXDOTXHUWLSRGHHVFULWDSRGH
PXLWD WULVWH]D VDXGDGH GRV FDPSRV TXHUrQFLD GRV VH EHQH¿FLDU GH VHXV HQVLQDPHQWRV $ UHJUD  SRU
SDVWRVGHOiGRVHUWmR H[HPSOR PDQGD DEROLU D OLQJXDJHP WpFQLFD IRQWH GH
³8PERLSUHWRXPERLSLQWDGR SRVVtYHOFRQVWUDQJLPHQWRSDUDTXHPQmRDFRPSUHHQGH
HUHFRPHQGD³1mRLUULWHROHLWRU´$UHJUDSUHJDXPWRP
FDGDXPWHPVXDFRU FRUGLDOHGXFDGRHVREUHWXGRFRQFLVR³1mRIDoDROHLWRU
SHUGHUWHPSR´
&DGDFRUDomRXPMHLWR 'LVSRQtYHOHPUHYLVWDSLDXLHVWDGDRFRPEU$FHVVRHPMXO DGDSWDGR 

GHPRVWUDUVHXDPRU´ $V EXODV GH UHPpGLR WrP FDUiWHU LQVWUXFLRQDO H


FRPSOHPHQWDPDVRULHQWDo}HVPpGLFDV1RFRQWH[WRGH
%RLEHPEUDYREDWHEDL[RERWDEDEDERLEHUUDQGR PXGDQoDV DSUHVHQWDGR D SULQFLSDO FDUDFWHUtVWLFD TXH
'DQoDGRLGRGiGHGXURGiGHGHQWURGiGLUHLWR PDUFDVXDQRYDOLQJXDJHPpR D
9DL YHP YROWD YHP QD YDUD YDL QmR YROWD YDL A SRVVLELOLGDGHGHLQFOXVmRGHQHRORJLVPR
YDUDQGR B UH¿QDPHQWRGDOLQJXDJHPIDUPDFrXWLFD
526$-*2EXUULQKRSHGUrV6DJDUDQD5LRGH-DQHLUR-RVp2O\PSLR C DGHTXDomRDROHLWRUQmRHVSHFLDOL]DGR
3Uy[LPR GR KRPHP H GR VHUWmR PLQHLURV *XLPDUmHV D GHWDOKDPHQWRGHLQIRUPDo}HV
5RVD FULRX XP HVWLOR TXH UHVVLJQL¿FD HVVHV HOHPHQWRV E LQIRUPDOLGDGHGRUHJLVWUR
2 IUDJPHQWR H[SUHVVD D SHFXOLDULGDGH GHVVH HVWLOR
QUESTÃO 125
narrativo, pois
$RDFRPSDQKDUPRVDKLVWyULDGRWHOHIRQHYHUL¿FDPRV
A demonstra a preocupação do narrador com a TXH HVVH PHLR HVWi VH PRVWUDQGR FDSD] GH UHXQLU HP
YHURVVLPLOKDQoD VHXFRQWH~GRXPDTXDQWLGDGHFDGDYH]PDLRUGHRXWURV
PHLRV௅HQYLRGHe-mailsUHFHELPHQWRGHQRWtFLDVP~VLFD
B UHYHOD DVSHFWRV GH FRQÀXrQFLD HQWUH DV YR]HV H RV DWUDYpVGHUiGLRHPHQVDJHQVGHWH[WR(VWD~OWLPDIXQomR
VRQVGDQDWXUH]D YHP VHUYLQGR FRPR VXSRUWH SDUD XPD QRYD IRUPD GH
VRFLDELOLGDGH R IHQ{PHQR GR ÀDVK PRE ௅ PRELOL]Do}HV
C UHFRUUHjSHUVRQL¿FDomRGRVDQLPDLVFRPRSULQFLSDO UHOkPSDJRTXHWrPFRPRFDUDFWHUtVWLFDSULQFLSDOUHDOL]DU
UHFXUVRHVWLOtVWLFR XPDHQFHQDomRHPDOJXPSRQWRGDFLGDGH
D SURGX]XPHIHLWRGHOHJLWLPLGDGHDWUHODGDjUHSURGXomR 3$03$1(//,*$$HYROXomRGRWHOHIRQHHXPDQRYDIRUPDGHVRFLDELOLGDGH: o ÀDVKPRE
'LVSRQtYHOHPZZZUD]RQ\SDODEUDRUJP[$FHVVRHPMXQ DGDSWDGR 
GDOLQJXDJHPUHJLRQDO
E H[SUHVVDRÀXLUGRUHEDQKRHGRVSH}HVSRUPHLRGH 'H DFRUGR FRP R WH[WR D HYROXomR GDV WHFQRORJLDV GH
UHFXUVRVVRQRURVHOH[LFDLV comunicação repercute na vida social, revelando que
A RDF~PXORGHLQIRUPDo}HVSURPRYHDVRFLDELOLGDGH
B as mudanças sociais demandam avanços
WHFQROyJLFRV
C RFUHVFLPHQWRWHFQROyJLFRDFDUUHWDPRELOL]Do}HVGDV
JUDQGHVPDVVDV
D a articulação entre meios tecnológicos pressupõe
GHVHQYROYLPHQWRVRFLDO
E a apropriação das tecnologias pela sociedade
possibilita ações inovadoUDV
/&žGLD_&DGHUQR&,1=$3iJLQD
2016 *CZ1625DOM15*
QUESTÃO 126 QUESTÃO 128

ÈUYRUHpFRUWDGDSDUDGDUOXJDUjSURSDJDQGDVREUH 4XLQ]HGH1RYHPEUR
SUHVHUYDomRDPELHQWDO Deodoro todo nos trinques
³8PD FULDQoD DEUDoD XPD iUYRUH FRP R VRUULVR %DWHQDSRUWDGH'mR3HGUR6HJXQGR
QR URVWR 1R IXQGR YHUGH XPD PHQVDJHP H[DOWD D ²6HXLPSHUDG{GrRIRUD
LPSRUWkQFLDGDSUHVHUYDomRGDQDWXUH]DHOHPEUDR'LD
TXHQyVTXHUHPRVWRPDUFRQWDGHVWDEXJLJDQJD
GD ÈUYRUH´ 2 TXH VHULD XP URWHLUR SDGUmR SDUD XPD
SHoDSXEOLFLWiULDYLURXPRWLYRGHUHYROWDHLQGLJQDomRHP 0DQGHYLURVP~VLFRV
XPD FLGDGH GR LQWHULRU GH 6mR 3DXOR ,VVR SRUTXH XPD 2LPSHUDGRUERFHMDQGRUHVSRQGH
empresa de outdoor GHUUXERXXPDiUYRUHFHQWHQiULDHP ²3RLVQmRPHXV¿OKRVQmRVHYH[HP
XPWHUUHQRSDUDDLQVWDODomRGHVXDVSODFDV PHGHL[HPFDOoDUDVFKLQHODV
$HPSUHVD WHULDLQIRUPDGR TXH WLQKD DXWRUL]DomR GD SRGHPHQWUDUjYRQWDGH
SUHIHLWXUD H GD SROtFLD DPELHQWDO SDUD FRUWDU D iUYRUH só peço que não me bulam nas obras completas de
6REUHDSURSDJDQGDDHPSUHVDGLVVHTXHIRL³XPDLQIHOL] >9LFWRU+XJR
FRLQFLGrQFLD´MiTXHQmRVDELDRTXHLULDVHUDQXQFLDGR
0(1'(603RHVLDFRPSOHWDHSURVD5LRGH-DQHLUR1RYD$JXLODU
(PQRWDD&RPSDQKLDGH7HFQRORJLDGH6DQHDPHQWR
$PELHQWDO &HWHVE OLJDGDj6HFUHWDULDGR0HLR$PELHQWH A poesia GH0XULOR0HQGHVGLDORJDFRPRLGHiULRSRpWLFR
GRV SULPHLURV PRGHUQLVWDV 1R SRHPD HVVD DWLWXGH
GR JRYHUQR SDXOLVWD LQIRUPRX TXH QmR Ki QHQKXPD PDQLIHVWDVHQD
DXWRUL]DomRHPQRPHGDHPSUHVDSDUDRFRUWHGDiUYRUH
A UHOHLWXUDLU{QLFDGHXPIDWRKLVWyULFR
$ PXOWD VHJXQGR D SROtFLD DPELHQWDO YDULD HQWUH
5H5SRUiUYRUHRXSODQWDFRUWDGD B YLVmRXIDQLVWDGHXPHSLVyGLRQDFLRQDO
C GHQ~QFLDLPSOtFLWDGHDWLWXGHVDXWRULWiULDV
'LVSRQtYHOHPKWWSRJORERJORERFRP$FHVVRHPVHW DGDSWDGR 
D LVHQomRLGHROyJLFDGRGLVFXUVRGRHXOtULFR
2WH[WRDSUHVHQWDXPDFUtWLFDDRXVRVRFLDOGHXPoutdoor E representação saudosista doUHJLPHPRQiUTXLFR
(VVDFUtWLFDHVWiDVVRFLDGDDRIDWRGH
A uma multa de R$ 100 a R$ 1 000 ser aplicada por 48(67­2
FRUWHGHiUYRUHRXGHSODQWD
2DGROHVFHQWH
B a Secretaria do Meio Ambiente ter negado a
DXWRUL]DomRGRFRUWHGDiUYRUH $YLGDpWmREHODTXHFKHJDDGDUPHGR
C DHPSUHVDLQIRUPDUTXHIRLXPD³LQIHOL]FRLQFLGrQFLD´ Não o medo que paralisa e gela,
RFRUWHGDiUYRUH
HVWiWXDV~ELWD
D XPD FDPSDQKD DPELHQWDO WHU VXEVWLWXtGR XPD mas
iUYRUHFHQWHQiULD
E D HPSUHVD XWLOL]DU D LPDJHP GH XPD FULDQoD QD HVVHPHGRIDVFLQDQWHHIUHPHQWHGHFXULRVLGDGH
FDPSDQKD >TXHID]
RMRYHPIHOLQRVHJXLUSDUDIUHQWHIDUHMDQGRRYHQWR
QUESTÃO 127 DRVDLUDSULPHLUDYH]GDJUXWD
$V OXWDV SRGHP VHU FODVVL¿FDGDV GH GLIHUHQWHV 0HGRTXHRIXVFDOX]
IRUPDV GH DFRUGR FRP D UHODomR HVSDFLDO HQWUH RV
RSRQHQWHVAs lutas de contato diretoVmRFDUDFWHUL]DGDV Cumplicentemente,
SHODPDQXWHQomRGRFRQWDWRGLUHWRHQWUHRVDGYHUViULRV DVIROKDVFRQWDPWHXPVHJUHGR
RV TXDLV SURFXUDP HPSXUUDU GHVHTXLOLEUDU SURMHWDU RX
YHOKRFRPRRPXQGR
LPRELOL]DURRSRQHQWH-iDVOXWDVTXHPDQWrPRDGYHUViULR
a distância VmRFDUDFWHUL]DGDVSHODPDQXWHQomRGHXPD $GROHVFHQWHROKD$YLGDpQRYD
GLVWkQFLDVHJXUDHPUHODomRDRDGYHUViULRSDUDQmRVHU
A vida é nova e anda nua
atingido pelo oponente, procurando o contato apenas no
PRPHQWRGDDSOLFDomRGHXPDWpFQLFD JROSH  ±YHVWLGDDSHQDVFRPRWHXGHVHMR
48,17$1$01DUL]GHYLGUR6mR3DXOR0RGHUQD
6HFUHWDULDGH(VWDGRGD(GXFDomR'LUHWUL]HVFXUULFXODUHVGHHGXFDomRItVLFDSDUDRVDQRV
¿QDLVGRHQVLQRIXQGDPHQWDOHSDUDRHQVLQRPpGLR&XULWLED6((' DGDSWDGR 
$R DERUGDU XPD HWDSD GR GHVHQYROYLPHQWR KXPDQR R
&RP EDVH QD FODVVL¿FDomR SUHVHQWH QR WH[WR VmR SRHPD PRELOL]D GLIHUHQWHV HVWUDWpJLDV GH FRPSRVLomR
H[HPSORVGHOXWDGHFRQWDWRGLUHWRHGHOXWDTXHPDQWpP 2 SULQFLSDO UHFXUVR H[SUHVVLYR HPSUHJDGR SDUD D
RDGYHUViULRDGLVWkQFLDUHVSHFWLYDPHQWH FRQVWUXomRGHXPDLPDJHPGDDGROHVFrQFLDpD

A MXG{HNDUDWr A KLSpUEROHGRPHGR
B MLXMtWVXHVXP{ B PHWiIRUDGDHVWiWXD
C SHUVRQL¿FDomRGDYLGD
C ER[HHkung fu
D DQWtWHVHHQWUHMXYHQWXGHHYHOKLFH
D HVJULPDHOXWDROtPSLFD
E comparaçãRHQWUHGHVHMRHQXGH]
E Muay Thai e tae kwon do

/&žGLD_&DGHUQR&,1=$3iJLQD
*CZ1625DOM16* 2016

QUESTÃO 130 $VVHTXrQFLDVWH[WXDLV³$WpRXWURGLD´H³DJRUD´DX[LOLDP


DSURJUHVVmRWHPiWLFDGRWH[WRSRLVGHOLPLWDP
3HGUDVREUHSHGUD
A RSHU¿OVRFLDOGRVHQYROYLGRVQDUHYROXomRGLJLWDO
$OJXPDV ID]HQGDV JD~FKDV DLQGD SUHVHUYDP DV B ROLPLWHHWiULRGRVSURPRWRUHVGDUHYROXomRGLJLWDO
WDLSDV PXURV GH SHGUD SDUD FHUFDU R JDGR 8P WLSR GH
FHUFD SULPLWLYD 1mR Ki QDGD TXH SUHQGD XPD SHGUD QD C RVSHUtRGRVSUpHSyVUHYROXomRGLJLWDO
RXWUDFXLGDGRVDPHQWHHPSLOKDGDVFRPDOWXUDGHDWpXP D DXUJrQFLDHDUDSLGH]GDUHYROXomRGLJLWDO
PHWUR(QJHQKDULDVLPSOHVTXHMiGXUDDQRV$PHVPD E RDOFDQFHWHUULWRULDOGDOHLWXUDGLJLWDO
técnica usada no mangueirão, uma espécie de curral
RQGHRVDQLPDLV¿FDYDPFRQ¿QDGRVjQRLWH$VWDLSDVVmR QUESTÃO 132
DWULEXtGDVDRVMHVXtWDV2REMHWLYRHUDGRPDURJDGR[XFUR
VROWRQRVFDPSRVSHORVFRORQL]DGRUHVHVSDQKyLV 2PLVWpULRGREUHJD
)(55,05HYLVWD7HUUDGD*HQWHQDEU
)DPRVR QR VHX WHPSR R PDUHFKDO 6FK|QEHUJ
8PWH[WRSRGHFRPELQDUGLIHUHQWHVIXQo}HVGHOLQJXDJHP   GLWDYD D PRGD HP /LVERD RQGH VHX QRPH IRL
([HPSOR GLVVR p Pedra sobre pedra, que se vale da DSRUWXJXHVDGR SDUD &KXPEHUJDV &RQVWD TXH HOH IRL
IXQomRUHIHUHQFLDOHGDPHWDOLQJXtVWLFD$PHWDOLQJXDJHP UHVSRQViYHOSHODSRSXODUL]DomRGRVYDVWRVELJRGHVWXIDGRV
é estabelecida QD 0HWUySROH (QWUH RV DGHSWRV HVWDULD R JRYHUQDGRU
A por tempos verbais articulados no presente e no GD SURYtQFLD GH 3HUQDPEXFR R SRUWXJXrV -HU{QLPR GH
SUHWpULWR 0HQGRQoD)XUWDGRTXHSRULVVRDTXLJDQKRXRDSHOLGRGH
&KXPEUHJDV ± YDULDQWH GR DSRUWXJXHVDGR &KXPEHUJDV
B SHODVIUDVHVVLPSOHVHUHIHUrQFLDDRGLWDGR³QmR¿FDUi
7DOYH]SRUVHUXPKRPHPQmRPXLWREHQTXLVWRQD&RO{QLD
SHGUDVREUHSHGUD´
RDSHOLGRGHXRULJHPDRDGMHWLYRxumbrega³FRLVDUXLP´
C SHOD OLQJXDJHP LPSHVVRDO H REMHWLYD PDUFDGD SHOD ³RUGLQiULD´(WDOYH]SRUVHUXPKRPHPWDPEpPGDIROLD
WHUFHLUDSHVVRD surgiu o verbo xumbregar, que inicialmente teve o sentido
D SHODGH¿QLomRGHWHUPRVFRPR³WDLSD´H³PDQJXHLUmR´ GH³HPEULDJDUVH´HGHSRLVYHLRDDGTXLULURGH³EROLQDU´
E SRUDGMHWLYRVFRPR³SULPLWLYDV´H³VLPSOHV´LQGLFDQGR ³JDUDQKDU´ 'HGXomR OyJLFD GH FRLVD UXLP D EHEHGHLUD
o ponto GHYLVWDGRDXWRU e atos libidinosos, as palavras xumbrega ou xumbregar
FKHJDUDP DRV DQRV  GR VpFXOR ;; QD IRUPD UHGX]LGD
QUESTÃO 131 brega, designando locais onde se bebe, se bolina e
VH RXYHP FDQWRUHV FDIRQDV ( R TXH LQLFLDOPHQWH HUD
VXEVWDQWLYR³P~VLFDGHEUHJD´DFDERXYLUDQGRDGMHWLYR
5HYROXomRGLJLWDOFULDDHUDGROHLWRUVXMHLWR
³P~VLFDEUHJD´±QXPDMiGLVWDQWHUHIHUrQFLDDXPFHUWR
)RLVH XPD YH] XP OHLWRU &RP D UHYROXomR GLJLWDO PDUHFKDODOHPmRFKDPDGR6FK|QEHUJ
TXHPOrSDVVDDWHUYR]QRSURFHVVRGHOHLWXUD³$WpRXWUR $5$Ò-23&5HYLVWD863QQRY DGDSWDGR 

GLDDVFUtWLFDVOLWHUiULDVHUDPH[FOXVLYLGDGHGHXPJUXSR 2 WH[WR WUDWD GDV PXGDQoDV OLQJXtVWLFDV TXH UHVXOWDUDP


IHFKDGR DVVLP FRPR HP WDQWDV RXWUDV iUHDV $JRUD QDSDODYUD³EUHJD´$RDSUHVHQWDUDVVLWXDo}HVFRWLGLDQDV
WHPRVJUXSRVTXHFRQYHUVDPWURFDPVHPDQLIHVWDPHP TXH IDYRUHFHUDP DV UHLQWHUSUHWDo}HV GR VHX VHQWLGR
tempo real, recomendam ou desaprovam, trocam ideias RULJLQDORDXWRUPRVWUDDLPSRUWkQFLDGD
com os autores, participam ativamente da construção de
A LQWHUDomR RUDO FRPR XP GRV DJHQWHV UHVSRQViYHLV
REUDVOLWHUiULDVFROHWLYDV,VVRpXPMHLWRQRYRGHSHQVDUD SHODWUDQVIRUPDomRGROp[LFRGRSRUWXJXrV
HVFULWDGHFRQVWUXLUPHPyULDHRSUySULRFRQKHFLPHQWR´
B compreensão limitada de sons e de palavras para a
DQDOLVDXPDSURIHVVRUDGHFRPXQLFDomRGD38&0*
FULDomRGHQRYDVSDODYUDVHPSRUWXJXrV
$VHFUHWiULD)DELDQD$UD~MRpXPD³OHLWRUDVXMHLWR´ C HOHLomR GH SDODYUDV IUHTXHQWHV H UHSUHVHQWDWLYDV QD
FRPR'DQLHODFKDPDHVVHVQRYRVDWRUHVGRXQLYHUVRGD IRUPDomRGROp[LFRGDOtQJXDSRUWXJXHVD
OHLWXUD /HLWRUD DVVtGXD GHVGH R ¿QDO GD DGROHVFrQFLD D LQWHUIHUrQFLD GD GRFXPHQWDomR KLVWyULFD QD
TXDQGRIRLVHGX]LGDSHODVpULHHarry Potter, só neste ano FRQVWLWXLomRGROp[LFR
Mi OHX PDLV GH  WtWXORV 6XDV OHLWXUDV QmR FRVWXPDP E UHDOL]DomRGHDo}HVGHSRUWXJXHVHVHGHEUDVLOHLURV
WHUPLQDUTXDQGRIHFKDXPOLYUR)DELDQDHVFUHYHUHVHQKDV D ¿P GH SDGURQL]DU DV YDULHGDGHV OLQJXtVWLFDV
GHWtWXORVFRPR(VWLOKDoDPHURPDQFHIDQWiVWLFRQDOLQKD OXVLWDQDV
de Crepúsculo, publicadas em um blog FRP R TXDO IRL
FRQYLGDGD D FRODERUDU ³(VFUHYHU VREUH XP OLYUR p XPD
IRUPD GH UHOrOR ( FRQYHUVDU SHVVRDO RX YLUWXDOPHQWH
FRPRXWURVOHLWRUHVWDPEpP´GHIHQGH
)$17,1,'-RUQDO3DPSXOKDQ PDLR DGDSWDGR 

/&žGLD_&DGHUQR&,1=$3iJLQD
2016 *CZ1625DOM17*
QUESTÃO 133 Nos anos de  D QHFHVVLGDGH GH PRGHUQL]DU R
%UDVLO UHÀHWLXVH QD SURSRVWD GH UHQRYDomR HVWpWLFD
GHIHQGLGD SRU DUWLVWDV PRGHUQLVWDV FRPR 5DXO %RSS 1R
SRHPD R SRVLFLRQDPHQWR IDYRUiYHO jV WUDQVIRUPDo}HV
da sociedade brasileira aparece diretamente relacionado
j H[SHULPHQWDomR QD SRHVLD $ UHODomR GLUHWD HQWUH
PRGHUQL]DomRHSURFHGLPHQWRHVWpWLFRQRSRHPDGHYHVH
jFRUUHVSRQGrQFLDHQWUH
A D GLVFXVVmR GH WHPD WpFQLFR H D IUDJPHQWDomR GD
OLQJXDJHP
B D D¿UPDomR GD PXGDQoD GRV WHPSRV H D LQRYDomR
YRFDEXODU
C DRSRVLomRjUHDOLGDGHUXUDOGRSDtVHDVLPSOL¿FDomR
GDVLQWD[H
D D DGHVmR DR ULWPR GH YLGD XUEDQR H D VXEMHWLYLGDGH
GDOLQJXDJHP
E D H[RUWDomR j DPSOD GLIXVmR GDV HVWUDGDV H D
OLEHUGDGHGRVYHUVRV

QUESTÃO 135

'$9,'-/1DSROHmRFUX]DQGRRV$OSHVÏOHRVREUHWHODFP[FP
0XVHXGH9HUVDOKHV3DULV

A pintura 1DSROHmRFUX]DQGRRV$OSHVGRDUWLVWDIUDQFrV
-DFTXHV /RXLV'DYLG SURGX]LGD HP  FRQWHPSOD DV
FDUDFWHUtVWLFDVGHXPHVWLORTXH
A XWLOL]DWpFQLFDVHVXSRUWHVDUWtVWLFRVLQRYDGRUHV
B UHÀHWHDSHUFHSomRGDSRSXODomRVREUHDUHDOLGDGH
C FDULFDWXUL]DHSLVyGLRVPDUFDQWHVGDKLVWyULDHXURSHLD
D LGHDOL]D HYHQWRV KLVWyULFRV SHOD yWLFD GH JUXSRV
A MOBILIDADE PRECISA DE
DIVERSIDADE
GRPLQDQWHV
E FRPS}H REUDV FRP EDVH QD YLVmR FUtWLFD GH DUWLVWDV
FRQVDJUDGRV
QUESTÃO 134
68*$,&'LVSRQtYHOHPZZZDFHVVLELOLGDGHQDSUDWLFDFRPEU$FHVVRHPMXQ
&RPRVHYDLGH6mR3DXORD&XULWLED 
2VWHPSRVPXGDUDP 2 WH[WR VXJHUH TXH D PRELOLGDGH p XPD TXHVWmR
FUXFLDO SDUD D YLGD QDV FLGDGHV 1HOH GHVWDFDVH D
2PXQGRFRQWHPSRUkQHRSXOVDHPULWPRVDFHOHUDGRV
necessidade de
1RYRVIDWRUHVUHYHODPFRQYHQLrQFLDGHRXWURVPpWRGRV
Surgem, no decurso dos nossos dias, motivos que nos A incorporar meios de transportes diversos para
FRQYHQFHP GH TXH FDGD PXQLFtSLR GHYH OHYDU D VpULR R YLDELOL]DURGHVORFDPHQWRXUEDQR
SUREOHPDGDFLUFXODomRURGRYLiULD B LQYHVWLUHPWUDQVSRUWHVGHEDL[RFXVWRSDUDPLQLPL]DU
3DUDIDFLOLWDUDDomRDGPLQLVWUDWLYD RVLPSDFWRVDPELHQWDLV

3DUDXPDUHYLVmRGDVVXDVSRVVLELOLGDGHVHFRQ{PLFDV C ampliar a quantidade de transportes coletivos para


DWHQGHUWRGDDSRSXODomR
5LWPRGHUXUDOL]DomR
D privilegiar meios alternativos de transporte para
&RVWXUDURSDtVFRPHVWUDGDVDOHJUHVGHVOLJDGDVGH JDUDQWLUDPRELOLGDGH
KRUiULRV/LYUHVHFKHLDVGHVROFRPRXPYHUVRPRGHUQR
E adotar medidas para evitar o uso de transportes
%23353RHVLDFRPSOHWDGH5DXO%RSS5LRGH-DQHLUR-RVp2O\PSLR
São Paulo: Edusp, 19 IUDJPHQWR 
PRWRUL]adoV

/&žGLD_&DGHUQR&,1=$3iJLQD
*CZ1625DOM18* 2016

INSTRUÇÕES PARA A REDAÇÃO


x 2UDVFXQKRGDUHGDomRGHYHVHUIHLWRQRHVSDoRDSURSULDGR
x 2WH[WRGH¿QLWLYRGHYHVHUHVFULWRjWLQWDQDIROKDSUySULDHPDWpOLQKDV
x $UHGDomRTXHDSUHVHQWDUFySLDGRVWH[WRVGD3URSRVWDGH5HGDomRRXGR&DGHUQRGH4XHVW}HVWHUiRQ~PHURGHOLQKDVFRSLDGDV
GHVFRQVLGHUDGRSDUDHIHLWRGHFRUUHomR
5HFHEHUiQRWD]HURHPTXDOTXHUGDVVLWXDo}HVH[SUHVVDVDVHJXLUDUHGDomRTXH
x WLYHUDWp VHWH OLQKDVHVFULWDVVHQGRFRQVLGHUDGD³WH[WRLQVX¿FLHQWH´
x IXJLUDRWHPDRXTXHQmRDWHQGHUDRWLSRGLVVHUWDWLYRDUJXPHQWDWLYR
x DSUHVHQWDUSURSRVWDGHLQWHUYHQomRTXHGHVUHVSHLWHRVGLUHLWRVKXPDQRV
x DSUHVHQWDUSDUWHGRWH[WRGHOLEHUDGDPHQWHGHVFRQHFWDGDGRWHPDSURSRVWR

TEXTOS MOTIVADORES
TEXTO I
8PWHUoRGRVDOLPHQWRVSURGX]LGRVQRPXQGRpGHVSHUGLoDGRDFDGDDQR±MXQWRFRPWRGDDHQHUJLDPmRGH
REUDiJXDHSURGXWRVTXtPLFRVHQYROYLGRVHPVXDSURGXomRHGHVFDUWH2%UDVLOWHPPLOK}HVGHEUDVLOHLURV
TXHHVWmRHPVLWXDomRGHLQVHJXUDQoDDOLPHQWDURTXHUHSUHVHQWDGDSRSXODomR6HJXQGRUHODWyULRGD)$2
2UJDQL]DomRGDV1Do}HV8QLGDVSDUDD$OLPHQWDomRHD$JULFXOWXUD GHPLOK}HVGHSHVVRDVRXVHMD
HPFDGDVRIUHGHIRPHQRPXQGR
'LVSRQtYHOHPZZZEDQFRGHDOLPHQWRVRUJEU$FHVVRHPPDLR DGDSWDGR 

TEXTO II
2GHVSHUGtFLRGHDOLPHQWRVQR%UDVLOFKHJDDPLOWRQHODGDVSRUGLDVHJXQGRSHVTXLVDGD(PSUHVD%UDVLOHLUDGH
3HVTXLVD$JURSHFXiULD (PEUDSD $QXDOPHQWHDTXDQWLDDFXPXODGDpVX¿FLHQWHSDUDDOLPHQWDUFHUFDGHPLOK}HV
GHSHVVRDVGLDULDPHQWH
'LVSRQtYHOHPZZZUHGHEUDVLODWXDOFRPEU$FHVVRHPGHPDLR

TEXTO III TEXTO IV


O deVSHUGtFLR GH DOLPHQWRV QDV VRFLHGDGHV ULFDV
resulta de uma combinação entre o comportamento
O CAMINHO DO DESPERDÍCIO NO BRASIL GR FRQVXPLGRU H D IDOWD GH FRPXQLFDomR DR ORQJR
GD FDGHLD GH DEDVWHFLPHQWR 2V FRQVXPLGRUHV QmR
FRQVHJXHP SODQHMDU VXDV FRPSUDV GH IRUPD H¿FD]
H SRU LVVR FRPSUDP HP H[FHVVR RX H[DJHUDP
no cumprimento das datas de validade dos
SURGXWRV 3RU RXWUR ODGR RV SDGU}HV HVWpWLFRV H GH
TXDOLGDGH OHYDP RV GLVWULEXLGRUHV D UHMHLWDU JUDQGHV
10% 50% 30% 10% TXDQWLGDGHV GH DOLPHQWRV SHUIHLWDPHQWH FRPHVWtYHLV
de todo o desperdício ocorre no manuseio ocorre nas centrais são diluídos entre 1RV SDtVHV HP GHVHQYROYLPHQWR DV JUDQGHV SHUGDV
ocorre ainda na colheita e transporte de abastecimento supermercados e SyVFROKHLWDDLQGDQDIDVHLQLFLDOGDFDGHLDDOLPHQWDU
(CEASAS) consumidores
são o principal problHPD
'LVSRQtYHOHPKWWSLQIRJUD¿FRVRJORERJORERFRP$FHVVRHPMXQ 'LVSRQtYHOHPZZZRQXYHUGHRUJEU$FHVVRHPPDLR

PROPOSTA DE REDAÇÃO
$SDUWLUGDOHLWXUDGRVWH[WRVPRWLYDGRUHVVHJXLQWHVHFRPEDVHQRVFRQKHFLPHQWRVFRQVWUXtGRVDRORQJRGHVXD
IRUPDomRUHGLMDWH[WRGLVVHUWDWLYRDUJXPHQWDWLYRHPPRGDOLGDGHHVFULWDIRUPDOGDOtQJXDSRUWXJXHVDVREUHRWHPD
³$OWHUQDWLYDVSDUDDGLPLQXLomRGRGHVSHUGtFLRGHDOLPHQWRVQR%UDVLO´DSUHVHQWDQGRSURSRVWDGHLQWHUYHQomRTXH
UHVSHLWHRVGLUHLWRVKXPDQRV6HOHFLRQHRUJDQL]HHUHODFLRQHGHIRUPDFRHUHQWHHFRHVDDUJXPHQWRVHIDWRVSDUD
GHIHVDGHVHXSRQWRGHYLVWD

/&žGLD_&DGHUQR&,1=$3iJLQD
2016 *CZ1625DOM19*
MATEMÁTICA E SUAS TECNOLOGIAS Se a estratégia deu certo, a ordenada da posição original
da bola branca era
4XHVW}HVGHD A 
B 
QUESTÃO 136 C 
D 
8PD SDUWLGD GH YROHLERO HQWUH %UDVLO H ,WiOLD IRL E 
decidida em cinco sets $V SRQWXDo}HV GR MRJR HVWmR
GHVFULWDVQDWDEHOD QUESTÃO 138

1º set 2º set 3º set 4º set 5º set 1RSURMHWRGHDUERUL]DomRGHXPDSUDoDHVWiSUHYLVWD


DFRQVWUXomRGHXPFDQWHLURFLUFXODU(VVHFDQWHLURVHUi
%UDVLO     18 FRQVWLWXtGRGHXPDiUHDFHQWUDOHGHXPDIDL[DFLUFXODUDR
,WiOLD  20    VHXUHGRUFRQIRUPHLOXVWUDD¿JXUD
Nessa partida, a mediana dos pontos obtidos por set pelo
WLPHGD,WiOLDIRLLJXDOD
A 
B 
r
C  R
D 
E 

QUESTÃO 137

(P VXD YH] GH MRJDU XP MRJDGRU SUHFLVD GDU XPD
'HVHMDVH TXH D iUHD FHQWUDO VHMD LJXDO j iUHD GD
WDFDGD QD EROD EUDQFD GH IRUPD D DFHUWDU D EROD  IDL[DFLUFXODUVRPEUHDGD
H ID]rOD FDLU HP XPD GDV FDoDSDV GH XPD PHVD GH
$UHODomRHQWUHRVUDLRVGRFDQWHLUR R HGDiUHDFHQWUDO r)
ELOKDU&RPRDERODHQFRQWUDVHHQWUHDERODEUDQFD
GHYHUiVHU
HDERODHVVHMRJDGRUDGRWDDHVWUDWpJLDGHGDUXPD
tacada na bola branca em direção a uma das laterais A R 2r
GD PHVD GH IRUPD TXH DR UHEDWHU HOD VDLD HP XPD
WUDMHWyULD UHWLOtQHD IRUPDQGR XP kQJXOR GH ƒ FRP D B R r—2
WUDMHWyULDGDWDFDGDFRQIRUPHLOXVWUDGRQD¿JXUD
r 2  2r
y C R
2

D R r 2  2r

3 9 
8 E R r
2

48(67­2
1R LQtFLR GH MDQHLUR GH XP GHWHUPLQDGR DQR XPD
IDPtOLDGHFLGLXHFRQRPL]DUSDUDDVIpULDVGHMXOKRGDTXHOH
DQRJXDUGDQGRXPDTXDQWLDSRUPrV(OHVGHFLGLUDPTXH
x HPMDQHLURJXDUGDULDP5HDSDUWLUGHIHYHUHLUR
0,5 2 3 6 JXDUGDULDPDFDGDPrVDPDLVGRTXHQRPrVDQWHULRU
4XDO IRL R WRWDO HFRQRPL]DGR HP UHDO  QR SULPHLUR
&RPHVVDHVWUDWpJLDRMRJDGRUFRQVHJXLXHQFDoDSDU semestre do ano, abandonando, por arredondamento,
DEROD&RQVLGHUHXPVLVWHPDFDUWHVLDQRGHHL[RVVREUH SRVVtYHLVFDVDVGHFLPDLVQHVVHUHVXOWDGR"
o plano da mesa, no qual o ponto de contato da bola com a
A 1 800,00
PHVDGH¿QHVXDSRVLomRQHVVHVLVWHPD$VFRRUGHQDGDV
B 2 100,00
GRSRQWRTXHUHSUHVHQWDDERODVmR  RFHQWURGD
C 
FDoDSDGHGHVWLQRWHPFRRUGHQDGDV  HDDEVFLVVD
GDERODEUDQFDpFRPRUHSUHVHQWDGRVQD¿JXUD D 
E 
07žGLD_&DGHUQR&,1=$3iJLQD
*CZ1625DOM20* 2016

QUESTÃO 140 QUESTÃO 142


Segundo o Compromisso Empresarial para Reciclagem O prédio de uma empresa tem cinco andares e,
&HPSUH  R YROXPH GH OL[R XUEDQR UHFLFODGR SDVVRX GH HP FDGD DQGDU Ki GRLV EDQKHLURV PDVFXOLQRV H GRLV
 PLOK}HV GH WRQHODGDV HP  SDUD  PLOK}HV GH IHPLQLQRV (P FDGD EDQKHLUR HVWmR LQVWDODGRV GRLV
WRQHODGDVHP1HVVHPHVPRSHUtRGRRQ~PHURGH UHFLSLHQWHVSDUDVDERQHWHOtTXLGRFRPXPDFDSDFLGDGHGH
PXQLFtSLRVFRPFROHWDVHOHWLYDSDVVRXGHSDUD P/ OLWUR FDGDXP2VUHFLSLHQWHVGRVEDQKHLURV
(VSHUDYDVH GXUDQWH HVWH SHUtRGR XP DXPHQWR GH SHOR PDVFXOLQRV VmR DEDVWHFLGRV GXDV YH]HV SRU VHPDQD
PHQRVQRYROXPHGHOL[RXUEDQRUHFLFODGRHGH H RV GRV EDQKHLURV IHPLQLQRV WUrV YH]HV SRU VHPDQD
QRQ~PHURGHPXQLFtSLRVFRPFROHWDVHOHWLYD TXDQGR HVWmR FRPSOHWDPHQWH YD]LRV 2 IRUQHFHGRU GH
'LVSRQtYHOHPKWWSUHYLVWDHSRFDJORERFRP$FHVVRHPMXO VDERQHWHOtTXLGRSDUDDHPSUHVDRIHUHFHFLQFRWLSRVGH
&RQVLGHUDQGR RV YDORUHV DSUHVHQWDGRV SDUD R SHUtRGR HPEDODJHQV,,,,,,,9H9FRPFDSDFLGDGHVGH//
GHDRVDXPHQWRVHVSHUDGRVQRYROXPHGH //H/UHVSHFWLYDPHQWH
OL[R XUEDQR UHFLFODGR H QR Q~PHUR GH PXQLFtSLRV FRP Para abastecer completamente os recipientes de
coleta seletiva
VDERQHWH OtTXLGR GRV EDQKHLURV GXUDQWH D VHPDQD D
A QmR IRUDP DWLQJLGRV SRLV R DXPHQWR QR YROXPH GH HPSUHVD SODQHMD DGTXLULU TXDWUR HPEDODJHQV GH XP
OL[R XUEDQR UHFLFODGR IRL GH  H QR Q~PHUR GH PHVPRWLSRGHIRUPDTXHQmRKDMDVREUDVGHVDERQHWH
PXQLFtSLRVFRPFROHWDVHOHWLYDIRLGH
B QmR IRUDP DWLQJLGRV SRLV R DXPHQWR QR YROXPH GH 4XHWLSRGHHPEDODJHPDHPSUHVDGHYHDGTXLULU"
OL[R XUEDQR UHFLFODGR IRL GH  H QR Q~PHUR GH
PXQLFtSLRVFRPFROHWDVHOHWLYDIRLGH A ,
C IRUDPDWLQJLGRVDSHQDVSDUFLDOPHQWHSRLVRVDXPHQWRV B ,,
QR YROXPH GH OL[R XUEDQR UHFLFODGR H QR Q~PHUR GH C ,,,
PXQLFtSLRVFRPFROHWDVHOHWLYDIRUDPGH D ,9
D IRUDPDWLQJLGRVDSHQDVSDUFLDOPHQWHSRLVRDXPHQWR
QRYROXPHGHOL[RXUEDQRUHFLFODGRIRLGHHQR E 9
Q~PHURGHPXQLFtSLRVFRPFROHWDVHOHWLYDIRLGH
E IRUDPDWLQJLGRVDSHQDVSDUFLDOPHQWHSRLVRDXPHQWR QUESTÃO 143
QRYROXPHGHOL[RXUEDQRUHFLFODGRIRLGHHQR
O quadro apresenta cinco cidades de um estado,
Q~PHURGHPXQLFtSLRVFRPFROHWDVHOHWLYDIRLGH FRP VHXV UHVSHFWLYRV Q~PHURV GH KDELWDQWHV H

QUESTÃO 141 TXDQWLGDGHGHSHVVRDVLQIHFWDGDVFRPRYtUXVGDJULSH


6DEHVHTXHRJRYHUQRGHVVHHVWDGRGHVWLQDUiUHFXUVRV
$QR DSyV DQR PXLWRV EUDVLOHLURV VmR YtWLPDV GH ¿QDQFHLURV D FDGD FLGDGH HP YDORUHV SURSRUFLRQDLV j
KRPLFtGLR QR %UDVLO 2 JUi¿FR DSUHVHQWD D TXDQWLGDGH GH SUREDELOLGDGH GH XPD SHVVRD HVFROKLGD DR DFDVR QD
KRPLFtGLRVUHJLVWUDGRVQR%UDVLOHQWUHRVDQRVH FLGDGHHVWDULQIHFWDGD
Número de homicídios
51 434

&LGDGH I II III IV V
51 043

52 000
Habitantes 180 000 100 000 110 000  
50 113

51 000
,QIHFWDGRV   9 000  11 000
49 695

4XDO GHVVDV FLGDGHV UHFHEHUi PDLRU YDORU GH UHFXUVRV


49 145

50 000
¿QDQFHLURV"
48 374

A
47 943

49 000 ,
47 707
47 578

B ,,
48 000
C ,,,
47 000 D ,9
E 9
45 360

46 000
QUESTÃO 144
45 000
2000 2001 2002 2003 2004 2005 2006 2007 2008 2009 (PXPPDSDFDUWRJUi¿FRFXMDHVFDODp
:$,6(/),6=--0DSDGDYLROrQFLDRVQRYRVSDGU}HVGDYLROrQFLDKRPLFLGDQR%UDVLO DV FLGDGHV $ H % GLVWDP HQWUH VL HP OLQKD UHWD  FP
6mR3DXOR,QVWLWXWR6DQJDUL DGDSWDGR  8PQRYRPDSDGHVVDPHVPDUHJLmRVHUiFRQVWUXtGRQD
Se o maior crescimento anual absoluto observado nessa HVFDOD
VpULHVHUHSHWLVVHGHSDUDHQWmRRQ~PHURGH 1HVVHQRYRPDSDFDUWRJUi¿FRDGLVWkQFLDHPOLQKDUHWD
KRPLFtGRVQR%UDVLODR¿QDOGHVVHSHUtRGRVHULDLJXDOD HQWUHDVFLGDGHV$H%HPFHQWtPHWURVHUiLJXDOD
A  A 
B  B 
C  C 
D  D 
E  E 
07žGLD_&DGHUQR&,1=$3iJLQD
2016 *CZ1625DOM21*
QUESTÃO 145 QUESTÃO 147
3RVVLYHOPHQWHYRFrMiWHQKDHVFXWDGRDSHUJXQWD 8PFDVDOHVHXVGRLV¿OKRVVDtUDPFRPXPFRUUHWRU
³2 TXH SHVD PDLV  NJ GH DOJRGmR RX  NJ GH de imóveis, com a intenção de comprar um lote onde
FKXPER"´ e yEYLR TXH DPERV WrP D PHVPD PDVVD IXWXUDPHQWHFRQVWUXLULDPVXDUHVLGrQFLD1RSURMHWRGD
SRUWDQWRRPHVPRSHVR2WUXTXHGHVVDSHUJXQWDpD FDVD TXH HVWD IDPtOLD WHP HP PHQWH LUmR QHFHVVLWDU
JUDQGHGLIHUHQoDGHYROXPHVTXHID]HQJDQRVDPHQWH GH XPD iUHD GH SHOR PHQRV  P2 $SyV DOJXPDV
algumas pessoas pensarem que pesa mais quem DYDOLDo}HV ¿FDUDP GH GHFLGLU HQWUH RV ORWHV  H  GD
tem maior volume, levando-as a responderem que é ¿JXUD HP IRUPD GH SDUDOHORJUDPRV FXMRV SUHoRV VmR
R DOJRGmR$ JUDQGH GLIHUHQoD GH YROXPHV GHFRUUH GD 5H5UHVSHFWLYDPHQWH
GLIHUHQoD GH GHQVLGDGH U  GRV PDWHULDLV RX VHMD D
UD]mRHQWUHVXDVPDVVDVHVHXVUHVSHFWLYRVYROXPHV 30 m
TXHSRGHVHUUHSUHVHQWDGDSHODH[SUessão: U m
v
&RQVLGHUHDVVXEVWkQFLDV$%&'H(UHSUHVHQWDGDV Lote 1 15 m
QRVLVWHPDFDUWHVLDQR YROXPH[PDVVD DVHJXLU 60°
600 B A

500 D
Lote 2 15 m
400

30 m
Massa (g)

300

200 C — 1
Use ,  H  FRPR DSUR[LPDo}HV
2 2
100 E UHVSHFWLYDPHQWHSDUDVHQ ƒ FRV ƒ H ¥ .
3DUDFRODERUDUHPQDGHFLVmRRVHQYROYLGRV¿]HUDP
0
as seguintes argumentações:
0 10 20 30 40 50 Volume
(cm3) 3DL: Devemos comprar o Lote 1, pois como uma de
suas diagonais é maior do que as diagonais do Lote 2,
$VXEVWkQFLDFRPPDLRUGHQVLGDGHp R/RWHWDPEpPWHUiPDLRUiUHD
A A 0mH: Se desconsiderarmos os preços, poderemos
B % FRPSUDUTXDOTXHUORWHSDUDH[HFXWDUQRVVRSURMHWRSRLVWHQGR
C C DPERVRPHVPRSHUtPHWURWHUmRWDPEpPDPHVPDiUHD
D D )LOKR  'HYHPRV FRPSUDU R /RWH  SRLV p R ~QLFR
E E TXHWHPiUHDVX¿FLHQWHSDUDDH[HFXomRGRSURMHWR
)LOKR: Devemos comprar o Lote 1, pois como os dois
QUESTÃO 146 lotes possuem lados de mesma medida, terão também a
PHVPDiUHDSRUpPR/RWHpPDLVEDUDWR
2EVHUYRXVHTXHWRGDVDVIRUPLJDVGHXPIRUPLJXHLUR
WUDEDOKDPGHPDQHLUDRUGHLUDHRUJDQL]DGD)RLIHLWRXP &RUUHWRU 9RFrV GHYHP FRPSUDU R /RWH  SRLV p R
H[SHULPHQWRFRPGXDVIRUPLJDVHRVUHVXOWDGRVREWLGRV TXHWHPPHQRUFXVWRSRUPHWURTXDGUDGR
IRUDP HVERoDGRV HP XP SODQR FDUWHVLDQR QR TXDO RV A pessoa que argumentou corretamente para a compra
HL[RVHVWmRJUDGXDGRVHPTXLO{PHWURV$VGXDVIRUPLJDV GRWHUUHQRIRLR D
SDUWLUDP MXQWDV GR SRQWR O, origem do plano cartesiano
A
xOy 8PD GHODV FDPLQKRX KRUL]RQWDOPHQWH SDUD R ODGR SDL
GLUHLWR D XPD YHORFLGDGH GH  NPK$ RXWUD FDPLQKRX
B PmH
YHUWLFDOPHQWHSDUDFLPDjYHORFLGDGHGHNPK
C ¿OKR
$SyV  KRUDV GH PRYLPHQWR TXDLV DV FRRUGHQDGDV D ¿OKR
FDUWHVLDQDVGDVSRVLo}HVGHFDGDIRUPLJD"
E FRUUHWRU
A  H  
B  H  
C  H  
D  H  
E  H  

07žGLD_&DGHUQR&,1=$3iJLQD
*CZ1625DOM22* 2016

QUESTÃO 148 QUESTÃO 150


2SUHVLGHQWHGHXPWLPHGHIXWHEROTXHUFRQWUDWDUXP Em um torneio interclasses de um colégio, visando
DWDFDQWHSDUDVHXHOHQFRHXPHPSUHViULROKHRIHUHFHX HVWLPXODU R DXPHQWR GR Q~PHUR GH JROV QRV MRJRV GH
FLQFR MRJDGRUHV (OH GHVHMD FRQWUDWDU R MRJDGRU TXH IXWHERODFRPLVVmRRUJDQL]DGRUDHVWDEHOHFHXDVHJXLQWH
REWHYHDPDLRUPpGLDGHJROVQRVDQRVGHD IRUPD GH FRQWDJHP GH SRQWRV SDUD FDGD SDUWLGD XPD
YLWyULD YDOH WUrV SRQWRV XP HPSDWH FRP JROV YDOH GRLV
2TXDGURDSUHVHQWDRQ~PHURGHJROVPDUFDGRVQRV pontos, um empate sem gols vale um ponto e uma derrota
DQRVGHDSRUFDGDXPGRVFLQFRMRJDGRUHV YDOH ]HUR SRQWR $SyV  MRJRV XP GRV WLPHV REWHYH
,,,,,,,9H9 como resultados cinco vitórias e sete empates, dos quais,
WUrVVHPJROV
1~PHUR 1~PHUR 1~PHUR 1~PHUR
-RJDGRU GHJROV GHJROV GHJROV GHJROV 'HDFRUGRFRPHVVHVGDGRVTXDOIRLRQ~PHURWRWDOGH
HP HP HP HP SRQWRVREWLGRVSHORWLPHFLWDGR"

, 21 21  21 A 22
B 
,, 20 21 22 22
C 
,,,  21 20 21 D 29
,9   19 18 E 
9  21   QUESTÃO 151
2SUHVLGHQWHGRWLPHGHYHFRQWUDWDURMRJDGRU
&LQFR PiTXLQDV GH FRVWXUD VmR XWLOL]DGDV HP XPD
A , FRQIHFomRGHFDOoDV2SURSULHWiULRGHVHMDFRPSUDUPDLV
B ,, XPDGHVVDVPiTXLQDVLGrQWLFDDXPDGDVMiH[LVWHQWHV
GHYHQGRHVFROKHUDTXHWLYHUDPDLRUPpGLDGHSURGXomR
C ,,,
SRU KRUD 1D WDEHOD HVWmR LQGLFDGDV DV TXDQWLGDGHV
D ,9 GH KRUDV WUDEDOKDGDV H GH FDOoDV FRQIHFFLRQDGDV SRU
E 9 FDGD XPD GDV PiTXLQDV HP GHWHUPLQDGRV SHUtRGRV
REVHUYDGRV
48(67­2
1~PHURGHFDOoDV
(PXPFDPSHRQDWRGHIXWHERODYLWyULDYDOHSRQWRVR 0iTXLQD +RUDV
FRQIHFFLRQDGDV
HPSDWHSRQWRHDGHUURWD]HURSRQWR*DQKDRFDPSHRQDWR
RWLPHTXHWLYHUPDLRUQ~PHURGHSRQWRV(PFDVRGHHPSDWH 1  
QRWRWDOGHSRQWRVRVWLPHVVmRGHFODUDGRVYHQFHGRUHV 2 210 
2VWLPHV5H6VmRRV~QLFRVFRPFKDQFHGHJDQKDU   1 020
R FDPSHRQDWR SRLV DPERV SRVVXHP  SRQWRV H HVWmR
PXLWRjIUHQWHGRVRXWURVWLPHV1RHQWDQWR5H6QmRVH   
HQIUHQWDUmRQDURGDGD¿QDO   800
2V HVSHFLDOLVWDV HP IXWHERO DUULVFDP DV VHJXLQWHV $PiTXLQDDVHUFRPSUDGDGHYHUiVHULGrQWLFDj
SUREDELOLGDGHVSDUDRVMRJRVGD~OWLPDURGDGD
A 
‡ 5WHPGHFKDQFHGHJDQKDUHGHHPSDWDU B 
‡ 6WHPGHFKDQFHGHJDQKDUHGHHPSDWDU C 
6HJXQGR DV LQIRUPDo}HV GRV HVSHFLDOLVWDV HP IXWHERO D 
TXDOpDSUREDELOLGDGHGHRWLPH5VHUR~QLFRYHQFHGRU
E 
GRFDPSHRQDWR"
A 
B 
C 
D 
E 

07žGLD_&DGHUQR&,1=$3iJLQD
2016 *CZ1625DOM23*
QUESTÃO 152 QUESTÃO 154

Um ciclista A usou uma bicicleta com rodas com 8PD SURIHVVRUD GH PDWHPiWLFD RUJDQL]RX XPD
GLkPHWURV PHGLQGR  FP H SHUFRUUHX FRP HOD  NP DWLYLGDGHDVVRFLDQGRXPiEDFRDWUrVGDGRVGHGLIHUHQWHV
8P FLFOLVWD % XVRX RXWUD ELFLFOHWD FRP URGDV FXMRV IRUPDWRV XP FXER FRP IDFHV QXPHUDGDV GH  D 
DVVRFLDGDVjKDVWH&XPRFWDHGURFRPIDFHVQXPHUDGDV
GLkPHWURVPHGLDPFPHSHUFRUUHXFRPHODNP GH  D  DVVRFLDGDV j KDVWH ' H XP GRGHFDHGUR FRP
&RQVLGHUHFRPRDSUR[LPDomRSDUDS IDFHV QXPHUDGDV GH  D  DVVRFLDGDV j KDVWH 8
,QLFLDOPHQWH DV KDVWHV GR iEDFR HQFRQWUDPVH YD]LDV
$UHODomRHQWUHRQ~PHURGHYROWDVHIHWXDGDVSHODVURGDV $VOHWUDV&'H8HVWmRDVVRFLDGDVDFHQWHQDVGH]HQDV
GDELFLFOHWDGRFLFOLVWD$HRQ~PHURGHYROWDVHIHWXDGDV H XQLGDGHV UHVSHFWLYDPHQWH $ KDVWH 80 UHSUHVHQWD
SHODVURGDVGDELFLFOHWDGRFLFOLVWD%pGDGDSRU XQLGDGHVGHPLOKDU

1 5HJUDV GR MRJR VmR MRJDGRV RV WUrV GDGRV MXQWRV


A H D FDGD MRJDGD FRORFDPVH EROLQKDV QDV KDVWHV
2 FRUUHVSRQGHQGRjVTXDQWLGDGHVDSUHVHQWDGDVQDVIDFHV
2 voltadas para cima de cada dado, respeitando a condição
B ³QXQFD GH]´ RX VHMD HP FDGD KDVWH SRGHP ¿FDU QR

Pi[LPRQRYHEROLQKDV$VVLPWRGDYH]TXHDTXDQWLGDGH
 GH EROLQKDV HP DOJXPD KDVWH IRU VXSHULRU D QRYH GH]
C GHODVVmRUHWLUDGDVGHVVDKDVWHHXPDEROLQKDpFRORFDGD

QD KDVWH LPHGLDWDPHQWH j HVTXHUGD %ROLQKDV HP
 TXDQWLGDGHVLJXDLVDRVQ~PHURVREWLGRVQDIDFHVXSHULRU
D
 GRV GDGRV QD VHJXQGD MRJDGD VmR DFUHVFHQWDGDV jV
 KDVWHV FRUUHVSRQGHQWHV TXH FRQWrP R UHVXOWDGR GD
E SULPHLUDMRJDGD
2
,QLFLDGD D DWLYLGDGH XP DOXQR MRJRX RV GDGRV GXDV
QUESTÃO 153 YH]HV1DSULPHLUDYH]DVTXDQWLGDGHVGDVIDFHVYROWDGDV
SDUDFLPDIRUDPFRORFDGDVQDVKDVWHV1HVWDMRJDGDQR
2WpFQLFRGHXPWLPHGHYROHLEROUHJLVWUDRQ~PHUR FXER QR RFWDHGUR H QR GRGHFDHGUR DV IDFHV YROWDGDV
GHMRJDGDVHGHDFHUWRVSRUDWOHWDHPFDGDIXQGDPHQWR SDUDFLPDIRUDPUHVSHFWLYDPHQWHH )LJXUD 
SDUDYHUL¿FDURVGHVHPSHQKRVGRVMRJDGRUHV3DUDTXH 1DVHJXQGDYH]RDOXQRMRJRXRVGDGRVHDGLFLRQRX
RWLPHWHQKDXPPHOKRUDSURYHLWDPHQWRQRIXQGDPHQWR DVTXDQWLGDGHVFRUUHVSRQGHQWHVQDVUHVSHFWLYDVKDVWHV
EORTXHLR HOH GHFLGH VXEVWLWXLU XP GRV MRJDGRUHV HP 2UHVXOWDGRHVWiUHSUHVHQWDGRQRiEDFRGD)LJXUD
TXDGUD SRU XP GRV TXH HVWmR QR EDQFR GH UHVHUYDV
2 FULWpULR D VHU DGRWDGR p R GH HVFROKHU R DWOHWD TXH
QR IXQGDPHQWR EORTXHLR WHQKD DSUHVHQWDGR R PDLRU
Q~PHURGHDFHUWRVHPUHODomRDRQ~PHURGHMRJDGDVGH
TXHWHQKDSDUWLFLSDGR2VUHJLVWURVGRVFLQFRDWOHWDVTXH
VHHQFRQWUDPQREDQFRGHUHVHUYDVQHVVHIXQGDPHQWR
HVWmRDSUHVHQWDGRVQRTXDGUR
)LJXUD
3DUWLFLSDomRHPEORTXHLRV
$WOHWD
1~PHURGH 1~PHURVGH
DFHUWRV MRJDGDV
, 20 
,, 10 
,,, 19 
,9  
)LJXUD
9 8 10
'HDFRUGRFRPDGHVFULomRDVIDFHVYROWDGDVSDUDFLPD
Qual dos atletas do banco de reservas o treinador deve no cubo, no octaedro e no dodecaedro, na segunda
FRORFDUHPTXDGUD" MRJDGDIRUDPUHVSHFWLYDPHQWH
A , A H
B ,, B H
C ,,, C H
D ,9 D H
E 9 E H

07žGLD_&DGHUQR&,1=$3iJLQD
*CZ1625DOM24* 2016

QUESTÃO 155 QUESTÃO 157


8PJHVVHLURTXHWUDEDOKDYDQDUHIRUPDGHXPDFDVD O quadro apresenta dados sobre viagens distintas,
OLGDYD FRP SODFDV GH JHVVR FRP IRUPDWR GH SHQWiJRQR UHDOL]DGDV FRP R PHVPR YHtFXOR SRU GLIHUHQWHV
regular quando percebeu que uma peça estava quebrada, PRWRULVWDV(PFDGDYLDJHPRYHtFXORIRLDEDVWHFLGRFRP
IDOWDQGRXPDSDUWHWULDQJXODUFRQIRUPHPRVWUDD¿JXUD FRPEXVWtYHO GH XP SUHoR GLIHUHQWH H WUDIHJRX FRP XPD
A
YHORFLGDGHPpGLDGLVWLQWD

&XVWRSRUOLWUR 'LVWkQFLD 9HORFLGDGH


E B 0RWRULVWD GHFRPEXVWtYHO SHUFRUULGD PpGLD
5 NP NPK
1 2,80  
2 2,89  
D C    
3DUD UHFRPSRU D SHoD HOH SUHFLVRX UHID]HU D SDUWH    
WULDQJXODUTXHIDOWDYDHSDUDLVVRDQRWRXDVPHGLGDVGRV  2,90  
kQJXORV[ EA '\ ED$H] AE'GRWULkQJXOR$'(
6DEHVHTXHHVVHYHtFXORWHPXPUHQGLPHQWRGHNP
As medidas x, y e ] HP JUDXV GHVVHV kQJXORV VmR
SRUOLWURGHFRPEXVWtYHOVHWUDIHJDUFRPYHORFLGDGHPpGLD
respectivamente,
DEDL[RGHNPK-iVHWUDIHJDUFRPYHORFLGDGHPpGLD
A H HQWUHNPKHNPKRUHQGLPHQWRVHUiGHNPSRU
B H OLWUR GH FRPEXVWtYHO 7UDIHJDQGR FRP YHORFLGDGH PpGLD
HQWUHNPKHNPKRUHQGLPHQWRVHUiGHNPSRU
C H OLWURGHFRPEXVWtYHOHDFLPDGHVVDYHORFLGDGHPpGLDR
D H UHQGLPHQWRFDLUiSDUDNPSRUOLWURGHFRPEXVWtYHO
E H 2PRWRULVWDTXHUHDOL]RXDYLDJHPTXHWHYHRPHQRUFXVWR
FRPFRPEXVWtYHOIRLRGHQ~PHUR
QUESTÃO 156
A 
$ YROHPLD 9  GH XP LQGLYtGXR p D TXDQWLGDGH WRWDO
B 
GHVDQJXHHPVHXVLVWHPDFLUFXODWyULR FRUDomRDUWpULDV
YHLDVHFDSLODUHV (ODp~WLOTXDQGRVHSUHWHQGHHVWLPDU C 
RQ~PHURWRWDO 1 GHKHPiFLDVGHXPDSHVVRDDTXDOp D 
REWLGDPXOWLSOLFDQGRVHDYROHPLD 9 SHODFRQFHQWUDomR &  E 
GH KHPiFLDV QR VDQJXH LVWR p 1  9 u & 1XP DGXOWR
QRUPDO HVVD FRQFHQWUDomR p GH    KHPiFLDV QUESTÃO 158
SRU P/ GH VDQJXH FRQGX]LQGR D JUDQGHV YDORUHV GH 1
8PD PDQHLUD DGHTXDGD GH LQIRUPDU HVVDV JUDQGHV 2DWRGHPHGLUFRQVLVWHHPFRPSDUDUGXDVJUDQGH]DV
TXDQWLGDGHV p XWLOL]DU D QRWDomR FLHQWt¿FD TXH FRQVLVWH GH PHVPD HVSpFLH 3DUD PHGLU FRPSULPHQWRV H[LVWHP
HPH[SUHVVDU1QDIRUPD1 Q u 10n, sendo 1 d Q  10 e GLYHUVRVVLVWHPDVGHPHGLGDV2SpDSROHJDGDHDMDUGD
QXPQ~PHURLQWHLUR SRU H[HPSOR VmR XQLGDGHV GH FRPSULPHQWR XWLOL]DGDV QR
&RQVLGHUHXPDGXOWRQRUPDOFRPYROHPLDGHP/ 5HLQR8QLGRHQRV(VWDGRV8QLGRV8PSpFRUUHVSRQGHD
1 200 metros ou dR]HSROHJDGDVHWUrVSpVVmRXPDMDUGD
KWWSSHUÀLQHFRP$FHVVRHPIHY DGDSWDGR

4XDO D TXDQWLGDGH WRWDO GH KHPiFLDV GHVVH DGXOWR HP 8PD KDVWH FRP  MDUGDV  SpV H  SROHJDGDV WHP
QRWDomRFLHQWt¿FD" FRPSULPHQWRHPPHWURPDLVSUy[LPRGH
A u 1010 A 
B u 109 B 
C u 109 C 
D u 1010 D 
E u 1011 E 

07žGLD_&DGHUQR&,1=$3iJLQD
2016 *CZ1625DOM25*
48(67­2 QUESTÃO 161
Uma empresa pretende adquirir uma nova impressora 8PDUWLVWDXWLOL]RXXPDFDL[DF~ELFDWUDQVSDUHQWHSDUD
FRP R REMHWLYR GH VXSULU XP GRV VHXV GHSDUWDPHQWRV DFRQIHFomRGHVXDREUDTXHFRQVLVWLXHPFRQVWUXLUXP
TXH WHP XPD GHPDQGD JUDQGH SRU FySLDV 3DUD LVVR SROtJRQRIMNKPQQRIRUPDWRGHXPKH[iJRQRUHJXODU
HIHWXRXVH XPD SHVTXLVD GH PHUFDGR TXH UHVXOWRX HP GLVSRVWRQRLQWHULRUGDFDL[D2VYpUWLFHVGHVVHSROtJRQR
WUrVPRGHORVGHLPSUHVVRUDGLVWLQWRVTXHVHGLIHUHQFLDP HVWmR VLWXDGRV HP SRQWRV PpGLRV GH DUHVWDV GD FDL[D
DSHQDVSHODVVHJXLQWHVFDUDFWHUtVWLFDV 8PHVERoRGDVXDREUDSRGHVHUYLVWRQD¿JXUD

,PSUHVVRUD ,PSUHVVRUD ,PSUHVVRUD


H P G
&DUDFWHUtVWLFDV
A % C
Q
&XVWRGDPiTXLQD
5 R$ 1 100,00 R$ 2 000,00
VHPFDUWXFKR E K
F
&XVWRGRFDUWXFKR R$ 80,00 5 5

&ySLDVSRUFDUWXFKR 1 000 2 000  C


I
3DUD IDFLOLWDU D WRPDGD GH GHFLVmR R GHSDUWDPHQWR N
LQIRUPRX TXH VXD GHPDQGD VHUi GH H[DWDPHQWH
FySLDV A M B
Assim, deve-se adquirir a impressora
&RQVLGHUDQGRDVGLDJRQDLVGRKH[iJRQRGLVWLQWDVGHIK,
A $RX%HPYH]GH& TXDQWDVWrPRPHVPRFRPSULPHQWRGHIK"
B %HPYH]GH$RX& A 1
C $HPYH]GH%RX& B 2
D &HPYH]GH$RX% C 
E $RX&HPYH]GH% D 8
E 9
QUESTÃO 160
8PPRWRULVWDSDUWLXGDFLGDGH$HPGLUHomRjFLGDGH QUESTÃO 162
%SRUPHLRGHXPDURGRYLDUHWLOtQHDORFDOL]DGDHPXPD
$SUHIHLWXUDGHXPDFLGDGHGHWHFWRXTXHDVJDOHULDV
SODQtFLH /i FKHJDQGR HOH SHUFHEHX TXH D GLVWkQFLD
SOXYLDLVTXHSRVVXHPVHomRWUDQVYHUVDOQDIRUPDGHXP
SHUFRUULGDQHVVHWUHFKRIRLGHNP$RFRQVXOWDUXP
TXDGUDGRGHODGRPVmRLQVX¿FLHQWHVSDUDFRPSRUWDU
PDSD FRP R DX[tOLR GH XPD UpJXD HOH YHUL¿FRX TXH D
RHVFRDPHQWRGDiJXDHPFDVRGHHQFKHQWHV3RUHVVD
GLVWkQFLD HQWUH HVVDV GXDV FLGDGHV QHVVH PDSD HUD
UD]mRHVVDVJDOHULDVIRUDPUHIRUPDGDVHSDVVDUDPDWHU
GHFP
seções quadradas de lado igual ao dobro das anteriores,
A escala desse mapa é SHUPLWLQGR XPD YD]mR GH  PV 2 FiOFXOR GD YD]mR
9 HP PV  p GDGR SHOR SURGXWR HQWUH D iUHD SRU RQGH
A 
SDVVDDiJXD HPP2 HDYHORFLGDGHGDiJXD HPPV 
B 1 : 1 000
6XSRQGRTXHDYHORFLGDGHGDiJXDQmRVHDOWHURXTXDO
C 
HUDDYD]mRPi[LPDQDVJDOHULDVDQWHVGDVUHIRUPDV"
D 1 : 100 000
A P/s
E 
B P/s
C 100 m/s
D 200 m³/s
E P/s

07žGLD_&DGHUQR&,1=$3iJLQD
*CZ1625DOM26* 2016

QUESTÃO 163
2&yGLJRGH7UkQVLWRGHFHUWRSDtVHVWDEHOHFHSHQDVSDUDTXHPFRQGX]LUYHtFXORDXWRPRWRUQDYLDS~EOLFDHVWDQGR
FRPFRQFHQWUDomRGHiOFRROQRVDQJXHLJXDORXVXSHULRUDJUDPDSRUOLWUR8PSHVTXLVDGRUPRQLWRURXXPLQGLYtGXR
TXH LQJHULX EHELGD DOFRyOLFD VRPHQWH DSyV R MDQWDU ([DPHV UHDOL]DGRV QR VDQJXH GHVVH LQGLYtGXR PRVWUDUDP TXH D
concentração QGHiOFRROQRVDQJXHGDGDHPJUDPDSRUOLWURDXPHQWRXGXUDQWHKRUDHPHLD'HSRLVGLVVRFRPHoRX
DGLPLQXLUHDWLQJLXDFRQFHQWUDomRSHUPLWLGDSDUDGLULJLUWUrVKRUDVDSyVDLQJHVWmRGHiOFRRO
8PJUi¿FRTXHSRGHUHSUHVHQWDUDUHODomRHQWUHRWHPSRDSyVDLQJHVWmRHDFRQFHQWUDomRGHiOFRROQRVDQJXHGHVVH
LQGLYtGXRp

Concentração de álcool no sangue Concentração de álcool no sangue

Q (g/L) Q (g/L)
1,0 1,0
0,9 0,9
0,8 0,8
0,7 0,7

A 0,6 D 0,6
0,5 0,5
0,4 0,4
0,3 0,3
0,2 0,2
0,1 0,1

0 1 2 3 4 5 6 7 t (h) 0 1 2 3 4 5 6 7 t (h)

Concentração de álcool no sangue Concentração de álcool no sangue

Q (g/L) Q (g/L)
1,0 1,0
0,9 0,9
0,8 0,8
0,7 0,7

B 0,6 E 0,6
0,5 0,5
0,4 0,4
0,3 0,3
0,2 0,2
0,1 0,1

0 1 2 3 4 5 6 7 t (h) 0 1 2 3 4 5 6 7 t (h)

Concentração de álcool no sangue

Q (g/L)
1,0
0,9
0,8
0,7

C 0,6
0,5
0,4
0,3
0,2
0,1

0 1 2 3 4 5 6 7 t (h)

07žGLD_&DGHUQR&,1=$3iJLQD
2016 *CZ1625DOM27*
QUESTÃO 164
3DUDDWUDLUXPDPDLRUFOLHQWHODXPDORMDGHPyYHLVIH]XPDSURPRomRRIHUHFHQGRXPGHVFRQWRGHHPDOJXQV
GHVHXVSURGXWRV
1RJUi¿FRHVWmRUHODFLRQDGDVDVTXDQWLGDGHVYHQGLGDVGHFDGDXPGRVSURGXWRVHPXPGLDGHSURPRomR

Quantidade vendida de cada produto


4

0
Cama Mesa Colchão Pia de cozinha

1RTXDGURFRQVWDPRVSUHoRVGHFDGDSURGXWRYHQGLGRMiFRPRGHVFRQWRGHRIHUHFLGRSHODORMD

0yYHO 3UHoR 5
Cama 
Mesa 
&ROFKmR 
3LDGHFR]LQKD 

4XDOIRLRYDORUWRWDOGHGHVFRQWRHPUHDLVFRQFHGLGRSHODORMDFRPDYHQGDGHVVHVSURGXWRVGXUDQWHHVVHGLDGH
SURPRomR"
A 
B 
C 
D 900,00
E 

QUESTÃO 165

2VVyOLGRVGH3ODWmRVmRSROLHGURVFRQYH[RVFXMDVIDFHVVmRWRGDVFRQJUXHQWHVDXP~QLFRSROtJRQRUHJXODUWRGRV
RVYpUWLFHVWrPRPHVPRQ~PHURGHDUHVWDVLQFLGHQWHVHFDGDDUHVWDpFRPSDUWLOKDGDSRUDSHQDVGXDVIDFHV(OHVVmR
LPSRUWDQWHVSRUH[HPSORQDFODVVL¿FDomRGDVIRUPDVGRVFULVWDLVPLQHUDLVHQRGHVHQYROYLPHQWRGHGLYHUVRVREMHWRV
&RPRWRGRSROLHGURFRQYH[RRVVyOLGRVGH3ODWmRUHVSHLWDPDUHODomRGH(XOHUV  A  F 2, em que V, A e F são os
Q~PHURVGHYpUWLFHVDUHVWDVHIDFHVGRSROLHGURUHVSHFWLYDPHQWH
(PXPFULVWDOFXMDIRUPDpDGHXPSROLHGURGH3ODWmRGHIDFHVWULDQJXODUHVTXDOpDUHODomRHQWUHRQ~PHURGH
YpUWLFHVHRQ~PHURGHIDFHV"
A 2V F 
B 2V  2F 
C 2V  F 
D 2V  F 
E 2V F 

07žGLD_&DGHUQR&,1=$3iJLQD
*CZ1625DOM28* 2016

QUESTÃO 166 QUESTÃO 168


Em alguns supermercados, é comum a venda $¿JXUDPRVWUDDSLUkPLGHGH4XpRSVWDPEpPFRQKHFLGD
GH SURGXWRV HP DWDFDGR FRP SUHoRV LQIHULRUHV DRV FRPRD*UDQGH3LUkPLGH(VVHpRPRQXPHQWRPDLVSHVDGR
KDELWXDLV8PGHVVHVVXSHUPHUFDGRVDQXQFLRXDYHQGD TXH Mi IRL FRQVWUXtGR SHOR KRPHP GD $QWLJXLGDGH 3RVVXL
GH VDERQHWHV HP FLQFR RSo}HV GH SDFRWHV GLIHUHQWHV DSUR[LPDGDPHQWHPLOK}HVGHEORFRVGHURFKDFDGDXP
Segue a descrição desses pacotes com as respectivas SHVDQGRHPPpGLDWRQHODGDV&RQVLGHUHTXHDSLUkPLGH
TXDQWLGDGHVHSUHoRV GH 4XpRSV VHMD UHJXODU VXD EDVH VHMD XP TXDGUDGR FRP
3DFRWH,XQLGDGHVSRU5 ODGRV PHGLQGR  P DV IDFHV ODWHUDLV VHMDP WULkQJXORV
LVyVFHOHVFRQJUXHQWHVHVXDVDUHVWDVODWHUDLVPHoDPP
3DFRWH,,XQLGDGHVSRU5
3DFRWH,,,XQLGDGHVSRU5
3DFRWH,9XQLGDGHVSRU5
3DFRWH9XQLGDGHVSRU5
Todos os sabonetes que compõem esses pacotes
VmRLGrQWLFRV
4XDOGHVVHVSDFRWHVRIHUHFHRPHQRUSUHoRSRUVDERQHWH"
A ,
B ,,
C ,,,
D ,9
E 9

QUESTÃO 167
$ HFRQRPLD QR FRQVXPR GH FRPEXVWtYHO p XP IDWRU 'LVSRQtYHOHPZZZPDXURZHLJHOEORJVSRWFRP$FHVVRHPQRY
LPSRUWDQWH SDUD D HVFROKD GH XP FDUUR e FRQVLGHUDGR
PDLV HFRQ{PLFR R FDUUR TXH SHUFRUUH D PDLRU GLVWkQFLD 2 YDORU PDLV DSUR[LPDGR SDUD D DOWXUD GD SLUkPLGH GH
SRUOLWURGHFRPEXVWtYHO Quéops, em metro, é
2 JUi¿FR DSUHVHQWD D GLVWkQFLD NP  H R UHVSHFWLYR A 
FRQVXPRGHJDVROLQD / GHFLQFRPRGHORVGHFDUURV B 
C 
60
D 
50 D E 
Consumo de gasolina (L)

48(67­2
40 B E
&RPSXWDGRUHV XWLOL]DP SRU SDGUmR GDGRV HP
30 IRUPDWRELQiULRHPTXHFDGDGtJLWRGHQRPLQDGRGHbit,
SRGHDVVXPLUGRLVYDORUHV RX 3DUDUHSUHVHQWDomR
20 C GH FDUDFWHUHV H RXWUDV LQIRUPDo}HV p QHFHVViULR ID]HU
XVR GH XPD VHTXrQFLD GH bits, o byte 1R SDVVDGR XP
byte HUD FRPSRVWR GH  bits em alguns computadores,
10 A PDVDWXDOPHQWHWHPVHDSDGURQL]DomRTXHRbyte é um
RFWHWR RX VHMD XPD VHTXrQFLD GH  bits (VVH SDGUmR
permite representar apenas 28LQIRUPDo}HVGLVWLQWDV
0 100 200 300 400 500 600
Distância percorrida (km)
6HXPQRYRSDGUmRIRUSURSRVWRGHPRGRTXHXPbyte
VHMDFDSD]GHUHSUHVHQWDUSHORPHQRVLQIRUPDo}HV
O carro mais econômico em relação ao consumo de GLVWLQWDVRQ~PHURGHbits em um byte deve passar de
FRPEXVWtYHOpRPRGHOR 8 para
A $ A 
B % B 
C & C 
D ' D 
E ( E 
07žGLD_&DGHUQR&,1=$3iJLQD
2016 *CZ1625DOM29*
QUESTÃO 170 QUESTÃO 172
8P VHPiIRUR p FRPSRVWR JHUDOPHQWH GH WUrV 1D UHIRUPD H HVWLOL]DomR GH XP LQVWUXPHQWR GH
FtUFXORVGHOX]HVFRORULGDV YHUPHOKRDPDUHORHYHUGH  SHUFXVVmR HP IRUPDWR FLOtQGULFR EXPER  VHUi FRODGD
$FRUYHUPHOKDLQGLFDTXHRYHtFXORGHYHHVWDUSDUDGRH XPD IDL[D GHFRUDWLYD UHWDQJXODU FRPR D LQGLFDGD QD
SHUPDQHFHUDVVLPDWpTXHDFRUYHUGHYROWHDDFHQGHU )LJXUD  VX¿FLHQWH SDUD FREULU LQWHJUDOPHQWH H VHP
VREUDWRGDDVXSHUItFLHODWHUDOGRLQVWUXPHQWR
2JUi¿FRDSUHVHQWDDYDULDomRGHYHORFLGDGHGHXP
FDUURDRORQJRGHXPSHUFXUVRGHPLQXWRVGHGXUDomR Z

GDUHVLGrQFLDGHXPDSHVVRDDWpVHXORFDOGHWUDEDOKR
Durante esse percurso, o carro parou somente nos
VHPiIRURVH[LVWHQWHVDRORQJRGHVHXWUDMHWR Y

V (km/h)

X
Figura 1

0 15 t (min) &RPR¿FDUiRLQVWUXPHQWRDSyVDFRODJHP"

(PTXDQWRVVHPiIRURVHOHSDURX"
Z
A 2
B 
C  A Y
D 
E 
X
QUESTÃO 171
(P  GH DEULO GH  RFRUUHX D H[SORVmR Z
H DIXQGDPHQWR GH XPD SODWDIRUPD GH SHWUyOHR
VHPLVVXEPHUVtYHO QR *ROIR GR 0p[LFR 2 DFLGHQWH
ocasionou um dos maiores desastres ecológicos mundiais, B Y
GHYLGRDRGHUUDPHGHP de óleo cru no mar,
SRUXPSHUtRGRGHGLDVHQWUHDEULOHMXOKRGH
)LQDOL]DGRRYD]DPHQWRSDUWHGRyOHRYD]DGRFRPHoRX
X
a ser queimado, diretamente, enquanto que outra parte
IRL UHPRYLGD SRU FROHWD DWUDYpV GH EDUFRV ¿OWUDGRUHV
$V GXDV WpFQLFDV MXQWDV UHWLUDYDP DSUR[LPDGDPHQWH Z
 P GH yOHR SRU GLD 'XUDQWH WRGR R SHUtRGR GH
UHPRomRIRUDPUHWLUDGRVQRWRWDODSHQDVP de
yOHR 3RU UHFRPHQGDomR GH DPELHQWDOLVWDV D UHWLUDGD C Y
WRWDOGHyOHRQmRGHYHULDXOWUDSDVVDUGLDV
'LVSRQtYHOHPZZZSRSXODUPHFKDQLFV$FHVVRHPIHY DGDSWDGR 

3DUD TXH WRGR R yOHR GHUUDPDGR QR *ROIR SXGHVVH X


WHU VLGR UHWLUDGR GHQWUR GR SUD]R UHFRPHQGDGR SHORV
DPELHQWDOLVWDV TXDO GHYHULD WHU VLGR D WD[D PtQLPD GH Z
UHPRomRGHyOHRHPPHWURF~ELFRGLD"
A 
D Y
B 
C 
D  X

E 
Z

E Y

07žGLD_&DGHUQR&,1=$3iJLQD
*CZ1625DOM30* 2016

QUESTÃO 173 QUESTÃO 175


8P SURGXWRU GH FDIp FRQWUDWRX XPD HPSUHVD 2 JRYHUQR GH XP HVWDGR LUi SULRUL]DU LQYHVWLPHQWRV
de consultoria para avaliar as produções de suas ¿QDQFHLURVQDiUHDGHVD~GHHPXPDGDVFLQFRFLGDGHV
GLYHUVDVID]HQGDV1RUHODWyULRHQWUHJXHFRQVWDTXHD DSUHVHQWDGDVQDWDEHOD
YDULkQFLDGDVSURGXWLYLGDGHVGDVID]HQGDVIRLLJXDOD
NJ 2KD 2(VVHSURGXWRUSUHFLVDDSUHVHQWDUHVVD 1~PHURWRWDO 1~PHURWRWDO
&LGDGH
LQIRUPDomRPDVHPRXWUDXQLGDGHGHSURGXWLYLGDGH GHKDELWDQWHV GHPpGLFRV
VDFDVKD (OH VDEH TXH D VDFD GH FDIp WHP  NJ M  
PDVWHPG~YLGDVHPGHWHUPLQDURYDORUGDYDULkQFLD X  
em sacas 2KD 2
Y 210 000 
$ YDULkQFLD GDV SURGXWLYLGDGHV GDV ID]HQGDV GH FDIp
H[SUHVVDHPVDFDV2KD2 é =  
W 108 000 
A 
Total  
B 
C  $ FLGDGH D VHU FRQWHPSODGD VHUi DTXHOD TXH
DSUHVHQWDU D PDLRU UD]mR HQWUH Q~PHUR GH KDELWDQWHV H
D 
TXDQWLGDGHGHPpGLFRV
E 
4XDOGHVVDVFLGDGHVGHYHUiVHUFRQWHPSODGD"
QUESTÃO 174 A M
B X
1D ¿JXUD HVWmR UHSUHVHQWDGDV HP XP SODQR
FDUWHVLDQRGXDVFLUFXQIHUrQFLDVC1 GHUDLRHFHQWURO1) C Y
e C2 GHUDLRHFHQWURO2), tangentes entre si, e uma reta t D =
WDQJHQWHjVGXDVFLUFXQIHUrQFias nos pontos P e Q E W
y
QUESTÃO 176
$ FREHUWXUD GH XPD WHQGD GH ORQD WHP IRUPDWR GH
XPD SLUkPLGH GH EDVH TXDGUDGD H p IRUPDGD XVDQGR
t TXDWUR WULkQJXORV LVyVFHOHV GH EDVH y $ VXVWHQWDomR
GD FREHUWXUD p IHLWD SRU XPD KDVWH GH PHGLGD x 3DUD
C1 P saber quanto de lona deve ser comprado, deve-se
FDOFXODU D iUHD GD VXSHUItFLH GD FREHUWXUD GD WHQGD

Q
x
O1 O2 R x
C2

y
$iUHDGDVXSHUItFLHGDFREHUWXUDGDWHQGDHPIXQomRGH
y e xpGDGDSHODH[SUHVVmR
Nessas condições, a equação da reta t é
y2
A 2y x2 
4
A \ ¥[ ¥
y2
¥ [¥ B 2y x2 
B \  2

C 4y ¥ x 2  y 2
C \ [

2 D y2
D \  [ 4 x2 
 4
 y2
E \  [ E 4 x2 
 2

07žGLD_&DGHUQR&,1=$3iJLQD
2016 *CZ1625DOM31*
QUESTÃO 177 48(67­2
2 SHUFHQWXDO GD SRSXODomR EUDVLOHLUD FRQHFWDGD j 8P DUTXLWHWR GHVHMD FRQVWUXLU XP MDUGLP FLUFXODU GH
LQWHUQHW DXPHQWRX QRV DQRV GH  D  &RQIRUPH  P GH GLkPHWUR 1HVVH MDUGLP XPD SDUWH GR WHUUHQR
GDGRVGR*UXSR,SVRVHVVDWHQGrQFLDGHFUHVFLPHQWRp VHUi UHVHUYDGD SDUD SHGUDV RUQDPHQWDLV (VVD SDUWH
PRVWUDGDQRJUi¿FR WHUi D IRUPD GH XP TXDGUDGR LQVFULWR QD FLUFXQIHUrQFLD
FRPR PRVWUDGR QD ¿JXUD 1D SDUWH FRPSUHHQGLGD
Brasileiros conectados à internet
HQWUH R FRQWRUQR GD FLUFXQIHUrQFLD H D SDUWH H[WHUQD
DR TXDGUDGR VHUi FRORFDGD WHUUD YHJHWDO 1HVVD SDUWH
GR MDUGLP VHUmR XVDGRV  NJ GH WHUUD SDUD FDGD Pð
48
$WHUUDYHJHWDOpFRPHUFLDOL]DGDHPVDFRVFRPH[DWRV
NJFDGD8VHFRPRYDORUDSUR[LPDGRSDUDS
População (%)

27

2007 2011 Ano

6XSRQKD TXH IRL PDQWLGD SDUD RV DQRV VHJXLQWHV D


PHVPDWD[DGHFUHVFLPHQWRUHJLVWUDGDQRSHUtRGR 2Q~PHURPtQLPRGHVDFRVGHWHUUDYHJHWDOQHFHVViULRV
SDUDFREULUDSDUWHGHVFULWDGRMDUGLPp
A estimativa para o percentual de brasileiros conectados
jLQWHUQHWHPHUDLJXDOD A 
A  B 
B  C 
C  D 
D  E 
E 
QUESTÃO 180
QUESTÃO 178
2SDGUmRLQWHUQDFLRQDO,62GH¿QHRVWDPDQKRV
Tradicionalmente uma SL]]D PpGLD GH IRUPDWR GH SDSHO XWLOL]DGRV HP TXDVH WRGRV RV SDtVHV FRP
FLUFXODUWHPGLkPHWURGHFPHpGLYLGLGDHPIDWLDV H[FHomRGRV(8$H&DQDGi2IRUPDWREDVHpXPDIROKD
LJXDLV PHVPD iUHD  8PD IDPtOLD DR VH UHXQLU SDUD R
UHWDQJXODU GH SDSHO FKDPDGD GH $ FXMDV GLPHQV}HV
MDQWDU IDUi XPD SL]]D GH IRUPDWR FLUFXODU H SUHWHQGH
VmR  FP u  FP$ SDUWLU GH HQWmR GREUDVH D
GLYLGLOD HP  IDWLDV WDPEpP LJXDLV (QWUHWDQWR HOHV
IROKDDRPHLRVHPSUHQRODGRPDLRUREWHQGRRVGHPDLV
GHVHMDP TXH FDGD IDWLD GHVVD SL]]D WHQKD R PHVPR IRUPDWRVFRQIRUPHRQ~PHURGHGREUDGXUDV2EVHUYHD
WDPDQKR PHVPD iUHD  GH FDGD IDWLD GD SL]]D média
¿JXUD$WHPRIRUPDWRGDIROKD$GREUDGDDRPHLRXPD
TXDQGRGLYLGLGDHPIDWLDVLJXDLV
YH]$WHPRIRUPDWRGDIROKD$GREUDGDDRPHLRGXDV
4XDORYDORUPDLVSUy[LPRGRUDLRFRPTXHGHYHVHUIHLWD YH]HVHDVVLPVXFHVVLYDPHQWH
a SL]]DHPFHQWtPHWURSDUDTXHHOHVFRQVLJDPGLYLGLOD
GDIRUPDSUHWHQGLGD" A2
84,1 cm

84,1 cm

84,1 cm

8VHFRPRDSUR[LPDomRSDUD—  A0 A1

A  118,9 cm 118,9 cm 118,9 cm

B  'LVSRQtYHOHPKWWSSWZLNLSHGLDRUJ
C  $FHVVRHPDEU DGDSWDGR 

D  4XDQWDV IROKDV GH WDPDQKR $ VmR REWLGDV D SDUWLU GH
E  XPDIROKD$"
A 8
B 
C 
D 128
E 

07žGLD_&DGHUQR&,1=$3iJLQD
*CZ1625DOM32*

2016

7UDQVFUHYDDVXD5HGDomRSDUDD)ROKDGH5HGDomR

10

HO
11

12

UN ÃO
C Ç
13

S A
14

A D
15

R RE
16

17

18



20

21
DA
22

23

24

25

26

27

28



30

/&žGLD_&DGHUQR&,1=$3iJLQD

You might also like